You are on page 1of 195

CBSE Term II

2022
Mathematics
Class
XI
CBSE Term II
2022
Mathematics
Class
XI
Complete Theory Covering NCERT

Case Based Questions

Short/Long Answer Type Questions

3 Practice Papers with Explanations

Author
Sagar Verma

ARIHANT PRAKASHAN (School Division Series)


CBSE Term II
2022

ARIHANT PRAKASHAN (School Division Series)

© Publisher
No part of this publication may be re-produced, stored in a retrieval system or by any means,
electronic, mechanical, photocopying, recording, scanning, web or otherwise without the written
permission of the publisher. Arihant has obtained all the information in this book from the sources
believed to be reliable and true. However, Arihant or its editors or authors or illustrators don’t take
any responsibility for the absolute accuracy of any information published and the damage or loss
suffered thereupon.
All disputes subject to Meerut (UP) jurisdiction only.

Administrative & Production Offices


Regd. Office
‘Ramchhaya’ 4577/15, Agarwal Road, Darya Ganj, New Delhi -110002
Tele: 011- 47630600, 43518550

Head Office
Kalindi, TP Nagar, Meerut (UP) - 250002, Tel: 0121-7156203, 7156204
Sales & Support Offices
Agra, Ahmedabad, Bengaluru, Bareilly, Chennai, Delhi, Guwahati,
Hyderabad, Jaipur, Jhansi, Kolkata, Lucknow, Nagpur & Pune.

ISBN : 978-93-25796-73-7
PO No : TXT-XX-XXXXXXX-X-XX
Published by Arihant Publications (India) Ltd.
For further information about the books published by Arihant, log on to
www.arihantbooks.com or e-mail at info@arihantbooks.com

Follow us on
CBSE Term II
2022

Contents
CHAPTER Trigonometric Functions -

CHAPTER Linear Inequalities -

CHAPTER Permutations and Combinations -

CHAPTER Conic Sections -

CHAPTER Introduction to Three-dimentional Geometry -

CHAPTER Derivatives -

CHAPTER Probability -

Practice Papers - -

Watch Free Learning Videos


Subscribe arihant Channel

þ Video Solutions of CBSE Sample Papers


þ Chapterwise Important MCQs
þ CBSE Updates
CBSE Term II
2022

Syllabus
One Paper Max Marks :

No. Units Marks


I. Sets and Functions Cont.
II. Algebra Cont.
III. Coordinate Geometry Cont.
IV. Calculus Cont.
V. Statistics and Probability Cont.
Total
Internal Assessment
Total

UNIT-I SETS AND FUNCTIONS CONT.


. Trigonometric Functions
Positive and negative angles. Measuring angles in radians and in degrees and
conversion from one measure to another. Definition of trigonometric functions with
the help of unit circle. Truth of the identity sin x + cos x = , for all x. Signs of
trigonometric functions. Domain and range of trigonometric functions and their
graphs. Expressing sin x±y and cos x±y in terms of sin x, sin y, cos x cos y and their
simple applications. Deducing identities like the following:
±
tan x ± tan y ± cot x cot y
tan x ± y = ± , cot x y =
tan x tan y cot y ± cot x

sina ± sinb = sin a ± b cos a±b

cosa ± cosb = cos a ± b cos a b

cosa cosb = sin a ± b sin a b

Identities related to sin x, cos x, tan x, sin x, cos x and tan x.


CBSE Term II
2022

UNIT-II ALGEBRA
. Linear Inequalities
Linear inequalities. Algebraic solutions of linear inequalities in one variable and their
representation on the number line. Graphical solution of linear inequalities in two
variables. Graphical method of finding a solution of system of linear inequalities in two
variables.
. Permutations and Combinations
Fundamental principle of counting. Factorial n. n Permutations and combinations,
formula for nPr and nCr, simple applications.

UNIT-III COORDINATE GEOMETRY


. Conic Sections
Sections of a cone: circles, ellipse, parabola, hyperbola. Standard equations and simple
properties of parabola, ellipse and hyperbola. Standard equation of a circle.
. Introduction to Three-dimensional Geometry
Coordinate axes and coordinate planes in three dimensions. Coordinates of a point.
Distance between two points and section formula.

UNIT-IV CALCULUS
. Derivatives
Derivative introduced as rate of change both as that of distance function and
geometrically. Definition of Derivative, relate it to scope of tangent of the curve,
derivative of sum, difference, product and quotient of functions. Derivatives of
polynomial and trigonometric functions.

UNIT-V STATISTICS AND PROBABILITY


. Probability
Random experiments; outcomes, sample spaces set representation . Events;
occurrence of events, not, and and or events, exhaustive events, mutually exclusive
events, Probability of an event, probability of not, and and or events.

INTERNAL ASSESSMENT MARKS


Periodic Test Marks
Mathematics Activities: Activity file record +Term end assessment of one activity Viva Marks
CBSE Circular
Acad - 51/2021, 05 July 2021

Exam Scheme Term I & II

dsUnh;
z ek/;fed f'k{kk cksMZ
(f'k{kk ea=ky;] Hkkjr ljdkj ds v/khu ,d Lok;r laxBu)
Hkkjr
CENTRAL BOARD OF SECONDARY EDUCATION
(An Autonomous Organisation under the Ministryof Education, Govt. of India)
dsUnh;
z ek/;fed f'k{kk cksMZ
(f'k{kk ea=ky;] Hkkjr ljdkj ds v/khu ,d Lok;r laxBu)

Hkkjr CENTRAL BOARD OF SECONDARY EDUCATION


(An Autonomous Organisation under the Ministryof Education, Govt. of India)

To cover this
situation, we
have given
both MCQs and
Subjective
Questions in
each Chapter.
dsUnh;
z ek/;fed f'k{kk cksMZ
(f'k{kk ea=ky;] Hkkjr ljdkj ds v/khu ,d Lok;r laxBu)

Hkkjr CENTRAL BOARD OF SECONDARY EDUCATION


(An Autonomous Organisation under the Ministryof Education, Govt. of India)
JEE NEET ADDA

This Pdf Is
Download From
www.jeeneetadda.in

Visit www.jeeneetadda.in for


More Premium Stuffs,Latest
Books,Test Papers,Lectures etc.
jeeneetadda
jeeneetadda_official
jeeneetadda

VISIT NOW !!
CBSE Term II Mathematics XI 1

CHAPTER 01

Trigonometric
Functions
In this Chapter...
l Measure of an Angle
l Trigonometric Functions
l Trigonometric Identities
l Transformation Formulae

Measure of an Angle Trigonometric Functions


There are two types of measure of angles In a right angle triangle, there are actually six possible
trigonometric ratios or functions.
Sexagesimal System (Degree Measure)
In right angled triangle ∆ABC,
One degree is divided into 60 equal parts, called minutes and C
Perpendicular (P)

1 minute is denoted by 1 ′. One minute is divided into 60


)
(H

equal parts, called second and 1 second is denoted by 1 ′′.


se
nu

Thus,1 ° = 60 ′ ; 1 ′ = 60 ′′
te
po
Hy

Circular System (Radian Measure)


A circle of radius r having an arc of length l will subtend an A θ B
angle θ radian at the centre, where Base (B)

l Length of arc Perpendicular  P  BC


θ= = or l = r θ (i) sin θ =  or  =
r Radius Hypotenuse  H AC
Relation between Degree and Radian
(ii) cos θ =
Base  or B  = AB
 
2π radian = 360 ° Hypotenuse  H AC
or π radian = 180 ° Perpendicular  P  BC
(iii) tan θ =  or  =
180 ° Base  B AB
1 rad = = 57 °16 ′ 22 ′′ (approx.)
π Base  or B AB
π (iv) cot θ =   =
Also, 1° = rad = 0. 01746 rad (approx.) Perpendicular  P  BC
180
Hypotenuse  H AC
π (v) sec θ =  or  =
Hence, Radian measure = × Degree measure Base  B  AB
180
Hypotenuse  H AC
180 (vi) cosec θ =  or  =
Degree measure = × Radian measure Perpendicular  P  BC
π
2 CBSE Term II Mathematics XI

Trigonometric Functions of Some Standard Angles


π π π π 2π 3π 5π 3π
30°   45°   60°   90°   120°   135°   150°   180° ( π ) 270°  
Angle θ 0°  6  4  3  2  3   4   6   2  360° (2π)

sin θ 1 1 3 3 1 1
0 1 0 −1 0
2 2 2 2 2 2
3 1 1 1 1 3
cosθ 1 0 − − − −1 0 1
2 2 2 2 2 2
1 1
tan θ 0 1 3 ∞ − 3 −1 − 0 ∞ 0
3 3
1 1
cotθ ∞ 3 1 0 − −1 − 3 ∞ 0 ∞
3 3
2 2
secθ 1 2 2 ∞ −2 − 2 − −1 ∞ 1
3 3
2 2
cosec θ ∞ 2 2 1 2 2 ∞ −1 ∞
3 3
Note Here, the symbol ‘∞’ is used for not defined.

Trigonometric Functions in Terms of sine Domain and Range of Trigonometric Functions


and cosine Functions T-function Domain Range
Given below are trigonometric functions defined in terms of sin θ R [ − 1, 1]
sine and cosine functions.
cosθ R [ −1,1]
1
cosec x = , x ≠ nπ, where n is any integer. tan θ π
{x : x ∈ R and x ≠ (2 n + 1) , n ∈ I R
sin x 2
1 π
sec x = , x ≠ (2 n + 1 ) , where n is any integer. cotθ {x : x ∈ R and x ≠ nπ, n ∈ I R
cos x 2
cosecθ {x : x ∈ R and x ≠ nπ, n ∈ I R − ( − 1, 1)
sin x π
tan x = , x ≠ (2 n + 1 ) , where n is any integer. secθ π
{x : x ∈ R and x ≠ (2 n + 1) , n ∈ I R − ( − 1, 1)
cos x 2 2
cos x
cot x = , x ≠ nπ, where n is any integer.
sin x Trigonometric Functions of Allied Angles
Two angles are said to be allied when their sum or difference
Trigonometric Identities π
is either zero or a multiple of π / 2. The angles − θ, ± θ,
The relation involving trigonometric functions which are true 2
for all values of the angles involved are called trigonometric π ± θ, 2π ± θ etc., are angles allied to the angle θ, if θ is
identities. measured in radians.
cos 2 θ + sin 2 θ =1;1 + tan 2 θ = sec 2 θ;1 + cot 2 θ = cosec 2θ The trigonometric functions changes at allied angles which
are given in the following table
Sign of Trigonometric Functions Allied angle/
in Different Quadrants Trigonometric sin θ cosec θ cosθ secθ tan θ cotθ
Y function
π π −θ − sin θ − cosec θ cosθ secθ − tan θ − cotθ
II Quadrant 2 < θ < π I Quadrant 0 < θ <
2 π /2 − θ cosθ secθ sin θ cosec θ cotθ tan θ
sin θ and cosec θ are All trigonometric functions
positive. π /2 + θ cosθ secθ − sin θ − cosec θ − cotθ − tan θ
are positive.
(90°+θ) and (180° – θ) (360°+θ) and (90° – θ) π −θ sin θ cosec θ − cosθ − secθ − tan θ − cotθ
180° 90°
X′ X π+θ − sin θ − cosec θ − cosθ − secθ tan θ cotθ
270° 360° 3 π /2 − θ − cosθ − secθ − sin θ − cosec θ cotθ tan θ
3π 3π
III Quadrant π < θ < IV Quadrant < θ < 2π
2 2 3 π /2 + θ − cosθ − secθ sin θ cosec θ − cotθ − tan θ
tan θ and cot θ are positive. cos θ and sec θ are positive.
2π − θ − sin θ − cosec θ cosθ sec θ − tan θ − cotθ
(180°+θ) and (270° – θ) (270°+θ) and (360° – θ)
2π + θ sin θ cosec θ cosθ sec θ tan θ cotθ
Y′
CBSE Term II Mathematics XI 3

Trigonometric Functions of Compound Angles Trigonometric Functions of Multiple Angles


Trigonometric ratios of compound angles in terms of
trigonometric ratios of constituent angles are given below. 1. cos 2 x = cos 2 x − sin 2 x = 2 cos 2 x − 1
1. cos ( A + B) = cos A cos B − sin A sin B 1 − tan 2 x
= 1 − 2 sin 2 x =
2. cos ( A − B) = cos A cos B + sin A sin B 1 + tan 2 x
2 tan x
3. sin ( A + B) = sin A cos B + cos A sin B 2. sin 2 x = 2 sin x cos x =
1 + tan 2 x
4. sin ( A − B) = sin A cos B − cos A sin B
2 tan x
tan A + tan B 3. tan 2 x =
5. tan ( A + B) = , where A, B and ( A + B) are 1 − tan 2 x
1 − tan A tan B
π 4. cos 3 x = 4 cos 3 x − 3 cos x
not odd multiples of .
2 5. sin 3 x = 3 sin x − 4 sin 3 x
tan A − tan B 3 tan x − tan 3 x
6. tan ( A − B) = , where A, B and ( A − B) are 6. tan 3 x =
1 + tan A tan B
π 1 − 3 tan 2 x
not odd multiples of .
2 Trigonometric Functions of Sub-Multiple Angles
cot A cot B − 1
7. cot ( A + B) = , where A, B and ( A + B) are 1. cos x = cos 2   − sin 2   = 2 cos 2   − 1
x x x
cot B + cot A  2  2  2
not multiples of π.
= 1 − 2 sin 2  
x
cot A cot B + 1
8. cot ( A − B) = , where A, B and ( A − B) are  2
cot B − cot A
not multiples of π. 1 − tan 2  
x
 2
Transformation Formulae cos x =
1 + tan 2  
x
In this topic, we deals with mainly two types of transformation.  2
1. Transformation of product into sum or difference. x
2 tan
2. Transformation of sum or difference into product. x x 2
2. sin x = 2 sin cos =
2 2 x
Transformation of Product into 1 + tan 2
Sum or Difference 2

2 tan  
x
1. 2 cos A cos B = cos ( A + B) + cos ( A − B)
 2
2. 2 sin A sin B = cos ( A − B) − cos ( A + B) 3. tan x =
1 − tan 2  
x
3. 2 sin A cos B = sin ( A + B) + sin ( A − B)
 2
4. 2 cos A sin B = sin ( A + B) − sin ( A − B)
Trigonometric Ratios of Some Specific Angles
Transformation of Sum or Difference
into Product 5 −1
1. sin 18 ° = cos 72 ° =
 A + B  A − B 4
1. cos A + cos B = 2 cos   ⋅ cos  
 2   2  10 + 2 5
2. cos 18 ° = sin 72 ° =
 A + B  A − B 4
2. cos A − cos B = − 2 sin   ⋅ sin  
 2   2  5 +1
 A + B  A − B 3. cos 36 ° = sin 54 ° =
3. sin A + sin B = 2 sin   ⋅ cos   4
 2   2 
10 − 2 5
 A + B  A − B 4. sin 36 ° = cos 54 ° =
4. sin A − sin B = 2 cos   ⋅ sin   4
 2   2 
4 CBSE Term II Mathematics XI

Solved Examples
Example 1. Convert the following into radians. Example 3. If sec θ = 2, where θ is an acute angle, then
(1) 25° (ii) 20°10' find the other five trigonometric functions.
Sol. (i) We know that, Sol. Given, secθ = 2, θ is an acute angle.
π
Radian measure = × Degree measure 2  H
180 secθ = Q secθ = B 
1
π 5π
∴Required radian measure = × 25 rad = radian A
180 36
(ii) Here, given degree measure have minutes. Firstly,
convert minutes into degree. P H
 1 ° (Perpendicular) (Hypotenuse)
Q 1′ =  
 60
 10  °  1  ° θ
10′ =   =  
 60  6 B B C
(Base)
 1 °
Now, total degree to convert = 20° +  
 6 Using Pythagoras theorem, we get
 120 + 1 °  121 ° H 2 = B2 + P2
=  = 
 6   6  ⇒ (2 ) 2 = (1 ) 2 + P 2
π  121
∴Required radian measure = ×  ⇒ 4 = 1 + P2
180  6 
121 π ⇒ P2 = 3 ⇒ P = + 3
= radian [Q length can’t be ‘–’ ve]
1080
B 1
Example 2. A horse is tied to a post by a rope. If the Now, cosθ = ⇒ cosθ =
H 2
horse moves along a circular path always keeping
P 3
the rope tight and describes 88m when it has traced sin θ = ⇒ sin θ =
H 2
out 54° at the centre. Find the length of the rope.
H 2
Sol. Let the post be at point P and PA be the length of the rope in cosec θ = ⇒ cosec θ =
tight position. P 3
Suppose the horse moves along the arc AB, so that P 3
tan θ = ⇒ tan θ =
∠APB = 54° and arc AB of length, l = 88 m. B 1
Let r be the length of the rope i.e. AP = r m. and
B
cotθ = ⇒ cotθ =
1
π  3π π P 3
  
Now, θ = 54° =  54 ×  = 1° = 180 radian 
 180 10 Example 4. Find the value of
θ=
l π π π π
Q
r
sin 2 + sec 2 + cos 2 + tan 2 .
8 cm 3 4 4 3
π π π π
A B Sol. sin 2 + sec 2 + cos 2 + tan 2
3 4 4 3
2 2 2 2
 π  π  π  π
= sin  + sec  + cos  + tan 
 3  4  4  3
54°
2
 3  1 
2
P =   + [ 2 ]2 +   + [ 3 ]2
3π 88  2   2
= 3 1
10 r = +2+ +3
88 × 10 4 2
⇒ r= 3 1
3π = + +5
880 4 2
⇒ r= = 93. 41 m [Q π = 3.14] 3 + 2 + 20 25
9. 42 = =
4 4
CBSE Term II Mathematics XI 5

Example 5. If tan θ + sin θ = m and tan θ − sin θ = n, Example 7. Which of the following is not correct?
show that m − n = 4 mn .
2 2 2 1
(i) sin θ = (ii) cos θ = −
3 4
Sol. LHS = m 2 − n 2
2
= (tan θ + sin θ )2 − (tan θ − sin θ )2 (iii) tan θ = 300 (iv) cosec θ =
3
= (tan θ + sin θ + tan θ − sin θ )(tan θ + sin θ − tan θ + sin θ ) Sol. 2
(i) sin θ = ∈ [ − 1,1]. Therefore, the given equation is
[Q a 2 − b 2 = ( a + b )( a − b )] 3
correct.
= (2 tan θ ) (2 sin θ )
1
⇒ m − n 2 = 4 tan θ sin θ
2
… (i) (ii) cos θ = − ∈ [ −1,1]. Therefore, the given equation is
4
RHS = 4 mn correct.
= 4 (tan θ + sin θ ) (tan θ − sin θ ) (iii) tan θ = 300 ∈ ( − ∞ , ∞ ). Therefore, the given equation is
correct.
= 4 tan 2 θ − sin 2 θ [Q ( a + b )( a − b ) = ( a 2 − b 2 )] (iv) Since, cosec θ ∉ ( − 1,1)
2 2
 1  ∴ cosec θ ≠ ⇒ cosec θ = is not correct.
= 4 sin 2 θ  − 1 3 3
 cos 2 θ 
Example 8. Evaluate
= 4 sin 2 θ (sec 2 θ − 1)
 41 π 
= 4 sin 2 θ tan 2 θ
(i) cos ( − 870 ° ) (ii) cos  
 4 
⇒ 4 mn = 4sin θ tan θ … (ii) Sol. (i) cos( − 870° ) = cos( 870° ) [Q cos( − θ ) = cos θ]
From Eqs. (i) and (ii), = cos (720° + 150° ) = cos(2 × 360° + 150° )
LHS = RHS Hence proved. = cos (150° )
[Q cos ( n × 360° + θ ) = cos θ , n ∈ Z]
5
Example 6. If cot x = − , x lies in II quadrant, then = cos (180° − 30° ) = − cos 30°
12 [Q cos(180° − θ ) = − cos θ]
find the value of other five trigonometric functions. 3
5 =−
Sol. Given, cot x = − and x lies in II quadrant. 2
12
 41π   π  π
In II-quadrant, tan θ, cos θ and secθ are negative. (ii) cos   = cos 10π +  = cos 2 π × 5 + 
 4   4  4
1 1 −12 π
∴ tan x = = = = cos [Q cos(2nπ + θ ) = cos θ, n ∈ Z]
cot x  5  5
−  4
 12  1
=
sec x = − 1 + tan 2 x 2
4 5
 12 
= − 1 + − 
2
Example 9. If sin A = and cos B = , where 0 < A and
 5 5 13
π
144 B < , then find the values of the following.
=− 1+ 2
25
(i) sin ( A + B) (ii) cos ( A + B)
169 13
=− =− [Q θ lies in II quadrant] (iii) sin ( A − B) (iv) cos ( A − B)
25 5 4 π
1 1 5 Sol. Given, sin A = , where 0 < A <
cos x = = =− 5 2
sec x  13 13
−  ∴ cos A = 1 − sin 2 A [Q A lies in Ist quadrant]
 5
2
 5
2  4 16
sin x = 1 − cos 2 x = 1 −  −  = 1−  = 1−
 13  5 25

25 144 12 9 3 5 π
= 1− = = ⇒ cos A = = and cos B = , where 0 < B <
169 169 13 25 5 13 2
1 1 13 ∴ sin B = 1 − cos 2 B [QB lies in Ist quadrant]
cosec x = = =
sin x  12  12
   5
2
25
 13  = 1 −   = 1−
 13 169
6 CBSE Term II Mathematics XI

⇒ sin B =
144 12
= π  π 
Example 12. The value of cot  + θ  cot  − θ  is
169 13 4  4 
(i) sin ( A + B ) = sin A cos B + cos A sin B
π  π 
4 5 3 12 20 36 56 Sol. Given expression, cot  + θ ⋅ cot  − θ
= × + × = + = 4  4 
5 13 5 13 65 65 65
 π   π 
(ii) cos( A + B ) = cos A cos B − sin A sin B  cot cot θ − 1  cot cot θ + 1
= 4  ⋅ 4 
3 5 4 12 15 − 48 −33
= × − × = =  cot π + cot θ   cot θ − cot π 
5 13 5 13 65 65  4   4 
(iii) sin ( A − B ) = sin A cos B − cos A sin B
 cot A cot B − 1
4 5 3 12 20 − 36 −16 Q cot ( A + B ) = cot A + cot B and
= × − × = = 
5 13 5 13 65 65
(iv) cos( A − B ) = cos A cos B + sin A sin B cot A cot B + 1 
cot ( A − B ) =
3 5 4 12 15 + 48 63 cot B − cot A 
= × + × = =
5 13 5 13 65 65  cot θ − 1   cot θ + 1
=  ⋅  =1
Example 10. Prove that sin A = cos (A − B ) + cos B
2 2 2  cot θ + 1  cot θ − 1 
− 2 cos ( A − B ) cos A cos B. π  π 
Sol. RHS = cos 2( A − B ) + cos 2 B − 2 cos( A − B ) cos A cos B
Example 13. Find the value of cos  − x  ⋅ cos  + x  .
6  6 
= cos 2 B + cos 2( A − B ) − 2 cos ( A − B ) cos A cos B π  π 
Sol. Given expression, cos − x ⋅ cos + x
= cos B + cos ( A − B ) {cos ( A − B ) − 2 cos A cos B}
2 6  6 
= cos 2 B + cos ( A − B ) {cos A cos B 1 π  π 
= 2 cos  + x ⋅ cos  − x 
+ sin A sin B − 2 cos A cos B} 2  6  6 
= cos 2 B + cos ( A − B ) [sin A sin B − cos A cos B ] 1   π  π   π  π  
= cos + x +  − x  + cos + x −  − x 
= cos 2 B + cos ( A − B ) [ − (cos A cos B − sin A sin B )] 2  6 6   6 6 
[Q2 cos A ⋅ cos B = cos( A + B ) + cos ( A − B )]
= cos 2 B − cos ( A − B ) cos ( A + B )
1 π  1 1   π 1
= cos 2 B − (cos 2 A − sin 2 B ) = cos + cos 2 x  =  + cos 2 x  Q cos 3 = 2 
2 3  2 2 
= cos 2 B + sin 2 B − cos 2 A 1 1
= + cos 2 x
= 1 − cos 2 A = sin 2 A = LHS Hence proved. 4 2
45 Example 14. Prove that
Example 11. If cos (α + β) = , sin (α − β) = and α, β
5
13 θ 9θ  7θ 
π cos θ cos − cos 3θ cos = sin   sin 4θ.
56
lie between 0 and , then prove that tan 2α = . 2 2 2 
4 33 θ 9θ
π Sol. LHS = cos θ cos − cos 3θ cos
Sol. Since, α, β lie between 0 and . 2 2
4 1 θ 9θ 
= 2 cos θ ⋅ cos − 2 cos 3θ ⋅ cos 
π π π 2 2 2 
∴ − < α − β < and 0 < α + β <
4 4 2   θ  θ 
cosθ +  + cosθ − 
⇒ cos (α − β ) and sin (α + β ) are positive. 1  2  2 
3 =  
Now, sin (α + β ) = 1 − cos 2(α + β ) = 2  9θ   9θ  
 − cos  3 θ +  − cos  3 θ − 
5
  2  2  
12
and cos (α − β ) = 1 − sin (α − β ) =
2
1 3θ θ 15 θ 3θ 
13 =  cos + cos − cos − cos 
2  2 2 2 2
sin (α + β ) 3 / 5 3
∴ tan (α + β ) = = = 1 θ 15 θ
cos (α + β ) 4 / 5 4 = cos − cos
2 2 2 
sin (α − β ) 5 / 13 5
and tan (α − β ) = = = 1  θ + 15 θ  θ − 15 θ 
cos (α − β ) 12 / 13 12 =− 2 sin   ⋅ sin  
2 4   4  
Now, tan 2α = tan [(α + β ) + (α − β )]
 x+y x−y
Q cos x − cos y = − 2 sin 2 ⋅ sin 2 
3 5
+
tan (α + β ) + tan (α − β ) 4 12 56
= = =
1 − tan (α + β )tan (α − β ) 1 − 3 × 5 33  θ
= + sin 4θ ⋅ sin  = RHS
4 12  2
Hence proved. ∴ LHS = RHS Hence proved.
CBSE Term II Mathematics XI 7

Example 15. If sin θ + sin φ = 3(cos φ − cos θ ), prove Example 17. Prove that
that sin 3 θ + sin 3φ = 0. 1° 1°
cot 7 = tan 82 = ( 3 + 2 )( 2 + 1 ).
Sol. Given, sin θ + sin φ = 3(cos φ − cos θ ) 2 2
 θ + φ  θ − φ 1°  1° 
⇒ 2 sin   cos   Sol. LHS = tan 82 = tan  90° − 7 
 2   2  2  2
 θ + φ  θ − φ 1°
= 2 3 sin   sin   = cot 7 = cot A (say)
 2   2  2

 θ + φ   θ − φ  θ − φ  where, A =7
⇒ 2 sin   cos   − 3 sin   =0
 2    2   2   2
cos A cos A (2 cos A ) 1 + cos 2 A
 θ + φ  θ − φ  θ − φ Now, cot A = = =
⇒ sin   = 0 or cos   − 3 sin   =0 sin A sin A (2 cos A ) sin 2 A
 2   2   2 
°
 θ + φ  15
⇒ sin   =0 1 + cos 2  
 2  1° 2 1 + cos 15°
∴ cot7 = =
2 ° sin 15°
 θ − φ 1  π 1   15
 = sin 2  
or tan 
 2  Q sin 0 = 0 and tan 6 = 3  2
3  
 θ + φ   θ − φ  π 1 + cos( 45°−30° )
⇒   = 0 or   = =
 2   2  6 sin( 45°−30° )
π 1 + (cos 45°⋅ cos 30° + sin 45°⋅ sin 30° )
⇒ θ = − φ or θ − φ = =
3 (sin 45°⋅ cos 30° − cos 45°⋅ sin 30° )
Here, two cases arise.  1 3 1 1
Case I When θ = − φ , then 1+  ⋅ + ⋅ 
 2 2 2 2
sin 3 θ + sin 3φ = sin 3( − φ ) + sin 3φ =
 1 3 1 1
= − sin 3φ + sin 3φ = 0  ⋅ − ⋅ 
 2 2 2 2
π
Case II When θ − φ = , then 3 θ − 3φ = π
3 = ( 2 + 1) ( 3 + 2 ) Hence proved.
⇒ 3 θ = π + 3φ
Example 18. If cos θ = cos α cos β, then prove that
sin 3θ + sin 3 φ = sin ( π + 3φ ) + sin 3 φ
θ +α θ −α β
= − sin 3φ + sin 3φ [Q sin( π + θ ) = − sin θ ] tan tan = tan 2 .
2 2 2
=0
Sol. Given, cos θ = cos α cos β
Example 16. If cos (θ + φ ) = m cos (θ − φ ), then prove cos θ
1−m ⇒ cos β =
that tan θ = cot φ. cos α
1+m β
1 − tan 2
Sol. Given, cos(θ + φ ) = m cos(θ − φ )
⇒ 2 = cos θ
cos(θ + φ ) m cos(θ − φ ) 1 β cos α
⇒ = ⇒ = 1 + tan 2
cos(θ − φ ) 1 cos(θ + φ ) m 2
2β β
Using componendo and dividendo rule, we get 1 − tan + 1 + tan 2
cos (θ − φ ) − cos (θ + φ ) 1 − m ⇒ 2 2 = cos θ + cos α
= 2β 2β cos θ − cos α
cos (θ − φ) + cos (θ + φ ) 1 + m 1 − tan − 1 − tan
2 2
 θ − φ + θ + φ  θ − φ − θ − φ
−2 sin   ⋅ sin   [applying componendo and dividendo]
 2   2  1−m
⇒ = θ + α θ − α
 θ − φ + θ + φ  θ − φ − θ − φ 1+ m 2 cos   cos  
2 cos   ⋅ cos   2  2   2 
 2   2  ⇒ =
β θ + α θ − α
sin θ ⋅ sin φ 1 − m Q sin ( − θ ) = − sin θ  − 2 tan 2 − 2 sin   sin  
⇒ = and cos ( − θ ) = cos θ  2  2   2 
cos θ ⋅ cos φ 1 + m  
1 θ + α θ − α
1−m ⇒ = cot   cot  
⇒ tan θ ⋅ tan φ = β  2   2 
1+ m tan 2
2
 1−m
⇒ tan θ =   cot φ Hence proved. 2β θ + α θ − α
 1 + m ⇒ tan = tan   tan   Hence proved.
2  2   2 
8 CBSE Term II Mathematics XI

θ 1−e φ Example 20. Prove that


Example 19. If tan = tan , then prove that
2 1+e 2 cot 42 °⋅ cot 66°⋅ cot 6°⋅ cot 78 ° = 1
cos θ − e Sol. To prove
cos φ = .
1 − e cos θ cot 42 °⋅ cot 66°⋅ cot 6°⋅ cot 78° = 1
θ 1−e φ LHS = cot 42 °⋅ cot 66°⋅ cot 6°⋅ cot 78°
Sol. Given, tan = tan cos 42 ° cos 66° cos 6° cos 78°
2 1+ e 2 = ⋅ ⋅ ⋅
sin 42 ° sin 66° sin 6° sin 78°
φ 1+ e θ
⇒ tan = tan  cos A 
2 1−e 2 Q cot A = sin A 
φ (2 cos 66° cos 6° ) (2 cos 78° cos 42 ° )
1 − tan 2 = ×
We know that, cos φ = 2 (2 sin 66°sin 6° ) (2 sin 78°sin 42 ° )
φ
1 + tan 2 cos 72 ° + cos 60° cos 120° + cos 36°
2 = ×
1+ e θ cos 60° − cos 72 ° cos 36° − cos120°
1− tan 2
1−e 2 cos( 90° − 18° ) + cos 60°
⇒ cos φ = =
1+ e θ cos 60° − cos( 90° − 18° )
1+ tan 2
1−e 2 cos( 90° + 30° ) + cos 36°
×
2θ cos 36° − cos( 90° + 30° )
(1 − e) − (1 + e)tan
⇒ cos φ = 2 sin 18° + cos 60° ( − sin 30° ) + cos 36°
= ×
2θ cos 60° − sin 18° cos 36° − ( − sin 30° )
(1 − e) + (1 + e)tan
2
cos 60° + sin 18° cos 36° − sin 30°
2θ θ = ×
(1 − e)cos − (1 + e)sin 2 cos 60° − sin 18° cos 36° + sin 30°
⇒ cos φ = 2 2
θ θ 1 5 − 1  5 + 1 1
(1 − e)cos 2 + (1 + e)sin 2  +   − 
2 2 2 4   4 2
θ θ θ θ = ×
 1 5 − 1  5 + 1 1 
cos 2 − sin 2 − e cos 2 + sin 2   −   + 
2 2  2 2
= 2 4   4 2
 2 θ θ
2   2 θ 2 θ
 cos + sin  − e cos − sin  ( 5 + 1) ( 5 − 1) 5 − 1
 2 2   2 2 = × =
cos θ − e (3 − 5) (3 + 5) 9 − 5
= Hence proved.
1 − e cos θ 4
= = 1 = RHS Hence proved.
4
CBSE Term II Mathematics XI 9

Chapter
Practice
PART 1
Objective Questions
l
Multiple Choice Questions 6. If A = cos 2 θ + sin 4 θ for all values of θ, then A
1. Which among the following figure is not a correct belongs to
measure of radian?  1 1 3 1  3 
(a) 0,  (b)  , (c)  , 1 (d)  , 1
B  3 4 4  2  4 
1 1 1  cosec 2 θ − sec 2 θ 
7. If tan θ = , then   is equal to
(a)
O 1 radian
(b) O
1 A 7  cosec 2 θ + sec 2 θ 
A –1
1 1 3 5
radian
1 1 (a) (b) (c) (d) (2 )
2 4 4
B 8. If 10 sin 4 α + 15 cos 4 α = 6. Then, the value of
B 27 cosec 6α + 8 sec 6 α is
radian 1
1 (a) 240 (b) 245 (c) 250 (d) 260
1 2
1
1
2 O 1 A 9. Which of the following is not correct?
(c) O A (d) 1 1 1
1
2
1 (a) sin θ = − (b) cosθ = 1 (c) secθ = (d) tan θ = 2005
1 radian
1
2 3
1
2 cos( π + x ) ⋅ cos( − x )
B 10. Evaluate .
π 
sin ( π − x ) ⋅ cos  + x 
2. Radian measure of 540° is ……… (in radian) 2 
14π 16π 17 π
(a) (b) 3π (c) (d) (a) cot 2 x (b) tan 2 x (c) − cot 2 x (d) − tan 2 x
5 5 5
3. Radian measure of 40°20 ′ is equal to 11. The value of sin 75° is
120π 121π 3+1 3 −1 2− 3
(a) radian (b) radian (a) (b) (c) 3 − 1 (d)
504 540 2 2 2 2 2 2
121π sin ( x + y)
(c) radian (d) None of these 12. If is equal to …A… . Here, A refers to
3 sin ( x − y)
4. The value of cos 1 ° cos 2 ° cos 3° K cos 179° is tan x + tan y
(a) tan x ⋅ tan y (b)
1 tan x − tan y
(a) (b) 0 (c) 1 (d) −1
2 (c) tan ( x + y ) (d) cot ( x − y )
−5 13. If 3 tan (θ − 15° ) = tan (θ + 15° ), where 0° < θ < 90°,
5. If cot x = and x lies in II quadrant, then choose then θ is
12 π π π π
the correct option. (a) (b) (c) (d)
6 4 3 2
13 12
(a) sec x = ± (b) tan x = m 1
5 5 14. If tan α = and tan β = , then α + β is
13 12 m+1 2m + 1
(c) sin x = − and cosec x =
12 13 equal to
−5 π π π π
(d) cos x = (a) (b) (c) (d)
13 2 3 6 4
10 CBSE Term II Mathematics XI

1 1 (v) The value of T3 , if θ = π is


15. If tan A = and tan B = , then tan(2A + B ) is equal to
2 3 (a) 1 (b) − 1
(a) 1 (b) 2 (c) 3 (d) 4 (c) 2 (d) − 2
16. cot x ⋅ cot 2 x − cot 2 x ⋅ cot 3x − cot 3x ⋅ cot x is equal to 4 5
22. Consider, sin A = and cos B = , where 0 < A and
1 5 13
(a) −1 (b) 0 (c) 1 (d)
2 π
B< .
x 9x 2
17. cos 2 x ⋅ cos − cos 3x ⋅ cos is equal to
2 2 Based on the above information, answer the
5x 5x following questions.
(a) − sin 5x ⋅ sin (b) sin 5x ⋅ sin
2 2
3x (i) The value of cos A + sin B is
(c) sin 3x ⋅ sin (d) None of these 5 99
2 (a) (b)
13 65
18. The expression cot 4x(sin 5x + sin 3x ) in simplified 20 9
form is (c) (d)
65 13
(a) 2 cos x ⋅ sin 4x (b) 2 sin x ⋅ sin 4x
(ii) The value of sin( A + B) is
(c) 2 cos x ⋅ cos 4x (d) None of these
17 29
19. The value of cos 12 ° + cos 84° + cos 156° + cos 132 ° (a)
13
(b)
65
is 56 − 13
1 1 1 (c) (d)
(a) (b) 1 (c) − (d) 65 17
2 2 8 (iii) The value of cos( A + B) is
 π  3π 11 33
20. The value of  1 + cos   1 + cos  (a) − (b) −
 8  8  19 65
 5π   7π  (c) −
19
(d) −
65
 1 + cos   1 + cos  is 11 33
 8  8 
1 1 1 1
(iv) The value of sin( A − B) is
(a) (b) (c) (d) − 16 − 39
8 6 4 2 (a) (b)
65 40
− 65 − 40
l
Case Based MCQs (c)
16
(d)
39
21. Consider Tn = sin n θ + cos n θ (v) The value of cos( A − B) is
Based on the above information, answer the 65 63
(a) (b)
following questions. 73 65
73 72
(i) The value of T3 − T5 is (c) (d)
65 65
(a) sin 2 θ ⋅ cos 2 θ(sin θ + cos θ)
(b) sin θ cos θ(sin θ − cos θ)
(c) sin θ(sin 2 θ + cos θ)
(d) cos θ( cos 2 θ + sin θ) PART 2
T3 − T5
(ii) The value of
T1
is Subjective Questions
(a) sin 2 θ (b) sin 2 θ cos 2 θ
(c) sin θ cos θ (d) tan 2 θ
l
Short Answer (SA) Type Questions
(iii) The value of T5 − T7 is 1. Convert the following into radians.
(a) sin 3 θ cos 3 θ (sin θ + cos θ) (i) 240° (ii) 520°
(b) sin θ cos θ (sin 3 θ + cos 3 θ) (iii) − 47 ° 31 ′
(c) sin 2 θ cos 2 θ (sin 3 θ + cos3 θ) 7  2π π π
(d) sin 3 θ cos 3 θ (sin 2 θ + tan 2 θ ) 2. Find the value of +  sin + cos 2 + sin 2 
4  6 6 4
T5 − T7
(iv) The value of is  π π π
T3 ⋅  sin 2 + cos 2 + cos 2  .
 3 3 4
(a) sin 2 θ cos 2 θ (b) sin 3 θ cos 3 θ
(c) sin 2 θ (d) cos 2 θ
CBSE Term II Mathematics XI 11

3. In ∆ABC , ∠B is right angled. If tan A = 1, then 17. Find the value of


show that 2 sin A cos A = 1 . (sin 75° + sin 15° ) (cos 75° + cos 15° ).
4. Find the value of cos θ , tan θ , cot θ, sec θ and 18. If m sin θ = n sin (θ + 2 α ), then prove that
3 3π m+n
cosec θ, if sin θ = − and π < θ < . tan (θ + α ) cot α = .
5 2 m−n
2
1 + tan 2 A 1 − tan A  19. Find the value of
5. Prove that =  = tan A.
2
1 + cot A
2  1 − cot A  π 3π 5π 7π
cos 4 + cos 4 + cos 4 + cos 4 .
8 8 8 8
6. Which of the following is not correct?
1 1 20. If cos α + cos β = 0 = sin α + sin β, then find the
(i) sin θ = − (ii) sec θ =
5 2 value of cos 2α + cos 2β.
(iii) tan θ = 20 21. Prove that
7. Evaluate
cot A + cot ( 60° + A ) − cot ( 60° − A ) = 3 cot 3A .
(i) tan ( − 2025 ° )
tan 3x 1
(ii) sin( − 1125 ° ) 22. Prove that never lies between and 3.
tan x 3
(iii) sin ( 930° )
3 3π
8. Evaluate 23. If tan x = , where π < x < , find the values of
4 2
cos 47 ° cos 13° − sin 47 ° sin 13° + sin 75°
x x x
α + β sin , cos and tan .
9. If a tan α + b tan β = ( a + b )tan   , where 2 2 2
 2 
24. Prove that
α ≠ β. Prove that a cos β = b cos α. 3
8 cos 3 A + cos 3 (120°+A ) + cos 3 (240°+A ) = cos 3A .
10. If θ lies in the I quadrant and cos θ = , 4
17
then find the value of l
Long Answer (LA) Type Questions
cos ( 30° + θ ) + cos( 45° − θ ) + cos (120° − θ ). 25. Find the distance from the eye at which a coin of
3 diameter 2 cm should be held, so as just to conceal
11. If cos (α − β) + cos (β − γ ) + cos ( γ − α ) = − , then
2 the full moon, whose angular diameter is 31 ′.
prove that 26. If x = sec φ − tan φ and y = cosec φ + cot φ, then show
cos α + cos β + cos γ = sin α + sin β + sin γ = 0.
that xy + x − y + 1 = 0.
12. Find the value of tan 105° and tan 15°.
27. Find the value of tan (α + β), if
π 1  3π  −5
13. If A + B = , then prove that cot α = , where α ∈  π ,  and sec β = , where
4 2  2  3
(i) (1 + tan A) (1 + tan B) = 2 π 
(ii) (cot A − 1 ) (cot B − 1 ) = 2 β ∈  , π .
2 
sin α − cos α
14. If tan θ = , then show that tan (θ + α ) tan (θ + β) tan (θ + γ )
sin α + cos α 28. If = = ,
a b c
sin α + cos α = 2 cos θ. a+b b+c
then prove that sin 2 (α − β) + sin 2 (β − γ )
15. Find the value of a−b b−c
(i) 2 cos 45 ° sin 15 ° (ii) 2 sin 15 ° cos 75 ° c+a
+ sin 2 ( γ − α ) = 0.
(iii) cos 315 °⋅ cos 75 ° c−a
cos ( A − B ) cos (C + D ) 29. Prove that
16. If + = 0, then prove that
cos ( A + B ) cos (C − D ) cos 2 A cos 3A − cos 2 A cos 7 A + cos A cos 10A
tan A tan B tan C tan D = − 1. sin 4A sin 3A − sin 2 A sin 5A + sin 4A sin 7 A
= cot 6A cot 5A.
12 CBSE Term II Mathematics XI

30. Prove that Y


n n
 cos A + cos B   sin A + sin B 
  + 
 sin A − sin B   cos A − cos B  12

  A −B
2 cot n   , n is even 4
=  2  A B
 0, n is odd. X

31. Prove that


cos A ⋅ cos 2 A ⋅ cos 2 2 A ⋅ cos 2 3 A K cos 2 n − 1 A 13

sin 2 n A
= n .
2 sin A
1 + sin A − cos A A Based on the above information, answer the
32. Prove that = tan . following questions.
1 + sin A + cos A 2
(i) Find the value of cos( A + B)
tan α + tan γ
33. If tan β = , then prove that (ii) Find the value of sin( A − B)
1 + tan α tan γ
sin 2 α + sin 2 γ (iii) Find the value of tan( A + B)
sin 2 β = . 36. In a class test of class XI, a teacher asked to
1 + sin 2 α sin 2 γ
π
34. If α and β are distinct roots of a cos θ + b sin θ = c, students to consider A + B = , where A and B are
4
2 ab acute angles.
then prove that sin (α + β) = 2 .
a + b2 Based on the above information, answer the
following questions.
l
Case Based Questions (i) Find the value of (1 + tan A)(1 + tan B)?
35. Rajiv construct two right angled triangles in the (ii) Find the value of (cot A − 1 )(cot B − 1 )?
fourth quadrant in such a way that the measure of (iii) Find the value of
4 12 sin( A + B) − cos( A + B) + tan( A + B).
triangle gives cos A = and cos B = , where
5 13

< A and B > 2 π.
2
CBSE Term II Mathematics XI 13

SOLUTIONS
Objective Questions Further, we have
 12   5  12
1. (d) In the figure (i) to (iv), OA is the initial side and OB is sin x = tan x cos x =  −  ×  −  =
the terminal side. The figures show the angles whose  5   13 13
1 1 1 13
measures are 1 radian, − 1 radian, 1 radian and 1 radian. and cosec x = =
2 2 sin x 12
B
6. (d) We have, A = cos 2 θ + sin 4 θ
1 1 = cos 2 θ + sin 2 θ sin 2 θ ≤ cos 2 θ + sin 2 θ
O 1 radian 1 A Therefore, A ≤ 1
O
A –1
1
radian Also, A = cos 2 θ + sin 4 θ = (1 − sin 2 θ ) + sin 4 θ
1 1 2
 1  1
= sin 2 θ −  + 1 − 
B  2  4
(i) (ii)
2
 1 3 3
B = sin 2 θ −  + ≥
radian 1
 2  4 4
1
1 2 3
1
1 1 A Hence, ≤ A ≤ 1.
2 O 4
O 1 A 1
1 1
2
1 radian 7. (b) Given, tan θ = ⇒ cot θ = 7
1 7
1
2
B ( cosec 2 θ − sec 2 θ )
Now,
(iii) (iv) ( cosec 2 θ + sec 2 θ )
(1 + cot 2 θ − 1 − tan 2 θ )
In figure (iv), the rotation is in clockwise direction, so its =
1 (1 + cot 2 θ + 1 + tan 2 θ )
measurement should be − 1 radian.
2 [Q cosec 2x = 1 + cot 2 x and sec 2 x = 1 + tan 2 x ]
π
2. (a) Radian measure = × Degree measure cot 2 θ − tan 2 θ
180 =
2 + cot 2 θ + tan 2 θ
π 56π 14π
Therefore, 504° = 504 × = = rad. 2
180 20 5  1 
( 7 )2 −  
3. (b) We know that, 180° = π radian  7
= 2
1  1 
Hence, 40°20′ = 40 ° 2 + ( 7 )2 +  
3  7
π 121 121π 49 − 1 7 48 3
= × rad = rad. = × = =
180 3 540 7 63 + 1 64 4
121π 8. (c) Given, 10sin 4 α + 15 cos 4 α = 6
Therefore, 40°20′ = rad.
540 ⇒ 10sin 4 α + 15 cos 4 α = 6 (sin 2 α + cos2 α )2
4. (b) Given expression, cos 1° cos 2 ° cos 3° K cos 179° [Q sin 2 x + cos 2 x = 1]
= cos 1° cos 2 °... cos 90° K cos 179° [Q cos 90° = 0] ⇒ 4sin 4 α + 9 cos 4 α − 12 sin 2 α ⋅ cos2 α = 0
=0
⇒ 4 tan 4 α + 9 − 12 tan 2 α = 0
−5
5. (d) Since, cot x = , we have ⇒ (2 tan 2 α )2 + ( 3)2 − 2 × 3 × 2 tan 2 α = 0
12
12 ⇒ (2 tan 2 α − 3)2 = 0
tan x = −
5 3
⇒ tan 2 α =
Now, sec 2 x = 1 + tan 2 x 2
144 169 ∴ 27 cosec 6 α + 8 sec 6α = cosec 6α (27 + 8 tan 6 α )
=1+
=  27 
25 25 = (1 + cot 2 α )3 27 + 8 × 
13  8
Hence, sec x = ± 3
5  2
= 1 +  ( 54)
Since, x lies in II quadrant, sec x will be negative.  3
3
13
Therefore, sec x = − , which also gives cos x = −
5  5 125
=   × 54 = × 54 = 250
5 13  3 27
14 CBSE Term II Mathematics XI

9. (c) We know that, the range of secθ is R − ( − 1, 1). m


+
1
1 m + 1 2m + 1
Hence, sec θ cannot be equal to . ⇒ tan(α + β ) =
3  m  1 
1−  
cos ( π + x ) cos ( − x ) ( − cos x ) (cos x )  m + 1  2 m + 1
10. (a) =
π  (sin x ) ( − sin x ) m (2 m + 1 ) + m + 1
sin ( π − x ) cos  + x ⇒ tan(α + β ) =
2  ( m + 1)(2 m + 1) − m
Q cos ( π + θ ) = − cos θ  2m 2 + m + m + 1
 cos ( − θ ) = cos θ  ⇒ tan(α + β ) =
  2m 2 + 2m + m + 1 − m
 sin ( π − θ ) = sin θ 
  2m 2 + 2m + 1
 π  ⇒ tan(α + β ) =
 cos  + θ = − sin θ  2m 2 + 2m + 1
 2  
⇒ tan(α + β ) = 1
cos 2 x π
= = cot 2 x ∴ α+β=
sin 2 x 4
11. (a) We have, 1 1
15. (c) Given that, tan A = and tan B =
sin 75° = sin ( 45° + 30° ) 2 3
= sin 45° cos 30° + cos 45°sin 30° tan 2 A + tan B
Now, tan(2 A + B ) = …(i)
[Qsin ( A + B ) = sin A cos B + cos A sin B] 1 − tan 2 A tan B
1 3 1 1 1
= ⋅ + ⋅ 2⋅
2 tan A 2 4
2 2 2 2 Also, tan 2 A = = =
1 − tan A 1 −
2 1 3
 1 3 1 
Q sin 30° = , cos 30° = and sin 45° = cos 45° = 4
 2 2 2  4 1 4 1 5
+ +
3 1 3+1 From Eq. (i), tan (2 A + B ) = 3 3 = 3 3 = 3 = 3
= + = 4 1 9−4 5
2 2 2 2 2 2 1− ⋅
sin ( x + y ) sin x cos y + cos x sin y 3 3 9 9
12. (b) We have, = 16. (c) We know that, cot 3x = cot (2 x + x )
sin ( x − y ) sin x cos y − cos x sin y
cot 2 x ⋅ cot x − 1
On dividing the numerator and denominator by cos x cos y , ⇒ cot 3x =
we get cot 2 x + cot x
sin ( x + y ) tan x + tan y ⇒ cot 3x ⋅ cot 2 x + cot 3x ⋅ cot x = cot 2 x ⋅ cot x − 1
=
sin ( x − y ) tan x − tan y ⇒ cot 3x ⋅ cot 2 x + cot 3x ⋅ cot x − cot 2 x ⋅ cot x = − 1
13. (b) Given that, 3 tan (θ − 15° ) = tan (θ + 15° ), which can be ⇒ cot x ⋅ cot 2 x − cot 2 x ⋅ cot 3x − cot 3x ⋅ cot x = 1
rewritten as 1 x 9x 
17. (b) We have , 2 cos 2 x cos − 2 cos cos 3x 
tan (θ + 15° ) 3 2  2 2 
=
tan (θ − 15° ) 1 1  x  x  9x 
= cos 2 x +  + cos 2 x −  − cos  + 3x
Applying componendo and dividendo, we get 2   2  2 2 
tan (θ + 15° ) + tan (θ − 15° )  9x 
=2 − cos  − 3x 
tan (θ + 15° ) − tan (θ − 15° ) 2 
sin (θ + 15° ) cos (θ − 15° ) + sin (θ − 15° ) cos (θ + 15° )
=2 
1 5x 3x 15x 3x 
sin (θ + 15° ) cos (θ − 15° ) − sin (θ − 15° ) cos (θ − 15° ) = cos 2 + cos 2 − cos 2 − cos 2 
2
sin 2 θ
⇒ = 2 i. e. sin 2 θ = 1 1 5x 15x 
sin 30° = cos − cos
2  2 2 
π
⇒ 2θ =   5x 15x   5x 15x 
+ −
2 1 2 2  sin 2 2 
π = −2 sin    
∴ θ= 2   2   2 
4     
m 1
14. (d) Given that, tan α = and tan β =  5x 
m +1 2m + 1 = − sin 5x ⋅ sin  − 
 2
tan α + tan β
Now, tan(α + β ) = 5x
1 − tan α tan β = sin 5x ⋅ sin
2
CBSE Term II Mathematics XI 15

18. (c) We have, cot 4x (sin 5x + sin 3x ) 21. (i) (a) T3 − T5 = (sin 3 θ + cos 3 θ) − (sin 5 θ + cos 5 θ)
5x + 3x 5x − 3x = sin 3 θ(1 − sin 2 θ ) + cos3 θ(1 − cos2 θ)
= cot 4x ⋅ 2 sin cos
2 2
= sin 3 θ ⋅ cos 2 θ + cos 3 θ ⋅ sin 2 θ
 C+D C −D
Q sin C + sin D = 2 sin 2 cos
2  = sin 2 θ ⋅ cos 2 θ (sin θ + cos θ)
cos 4x T3 − T5 sin 2 θ cos 2 θ(sin θ + cos θ)
= × 2 sin 4x cos x = 2 cos 4x cos x (ii) (b) =
sin 4x T1 (sin θ + cos θ)
19. (c) Given expression, = sin 2 θ ⋅ cos 2 θ
cos 12 ° + cos 84° + cos 156° + cos 132 °
(iii) (c) T5 − T7 = (sin 5 θ + cos 5 θ) − (sin 7 θ + cos 7 θ )
= cos 12 ° + cos 156° + cos 84° + cos 132 °
 12 ° + 156°  12 ° − 156° = sin 5 θ(1 − sin 2 θ) + cos5 θ (1 − cos2 θ )
= 2 cos  cos 
 2   2  = sin 5 θ ⋅ cos 2 θ + cos 5 θ ⋅ sin 2 θ
 84° + 132 °  84° − 132 ° = sin 2 θ ⋅ cos 2 θ (sin 3 θ + cos3 θ )
+ 2 cos  cos 
 2   2  T5 − T7 sin 2 θ ⋅ cos 2 θ(sin 3 θ + cos3 θ )
(iv) (a) =
= 2 cos 84° cos 72 ° + 2 cos 108° cos 24° T3 (sin 3 θ + cos 3 θ )
= 2 cos 84° cos( 90° − 18° ) + 2 cos( 90° + 18° )cos 24° = sin 2 θ ⋅ cos 2 θ
= 2 cos 84°sin 18° − 2 sin 18° cos 24° (v) (b) T3 = sin 3 θ + cos 3 θ
= 2 sin 18°(cos 84° − cos 24° ) at θ = π , T3 = (sin π)3 + (cos π)3
 84° + 24°  84° − 24°
= − 2 sin 18°⋅2 sin   sin   = ( 0) 3 + ( − 1) 3 = − 1
 2   2 
22. (i) (b) Given,
= − 4sin 18°sin 54°sin 30°
4 π
 5 − 1 1 sin A = , where 0 < A <
=−4  ⋅ cos 36° ⋅ 5 2
 4  2
∴ cos A = 1 − sin 2 A [Q A lies in 1st quadrant]
 5 + 1 1  5 − 1 −4 −1
= − ( 5 − 1)   ⋅ = −  = =  4 16
2
 4  2  8  8 2 = 1−  = 1−
 5 25
20. (a) Given expression,
9 3
 π  3π   5π   7π ⇒ cos A = =
1 + cos  1 + cos  1 + cos  1 + cos  25 5
 8  8  8  8
5
 π  3π    3π   and cos B = ,
= 1 + cos  1 + cos  1 + cos  π −  13
 8  8   8  π
where 0 < B <
  π 2
1 + cos  π −  
 8  ∴ sin B = 1 − cos 2 B [QB lies in Ist quadrant]
 2 π  2 3π 
= 1 − cos  1 − cos  2
 8  8  5 25
= 1−  = 1−
 3π  3π  13 169

Q 1 + cos  π − 8  = 1 − cos 8 and 144 12
 ⇒ sin B = =
 π π 169 13
1 + cos  π –  = 1 − cos  3 12 39 + 60 99
 8 8 ∴ cos A + sin B = + = =
π 3 π 5 13 65 65
= sin 2 sin 2
8 8 (ii) (c) sin( A + B ) = sin A cos B + cos A sin B
1 π  3π  4 5 3 12 20 36 56
= 1 − cos  1 − cos  = × + × = + =
4 4  4 5 13 5 13 65 65 65
1 (iii) (b) cos( A + B ) = cos A cos B − sin A sin B
[Q sin 2 θ = (1 − cos 2 θ ] 3 5 4 12 15 − 48 − 33
2 = × − × = =
1 π  π 5 13 5 13 65 65
= 1 − cos  1 + cos  (iv) (a) sin( A − B ) = sin A cos B − cos A sin B
4 4  4
4 5 3 12 20 − 36 − 16
1 2 π
= × − × = =
= 1 − cos  5 13 5 13 65 65
4  4
(v) (b) cos( A − B ) = cos A cos B + sin A sin B
1  1 1 3 5 4 12 15 + 48 63
= 1 −  = [Q ( a − b ) ( a + b ) = a 2 − b 2 ] = × + × = =
4  2 8 5 13 5 13 65 65
16 CBSE Term II Mathematics XI

Subjective Questions ⇒ AC = 2 k 2 ⇒ AC = k 2
1. (i) Required radian measure = π × Degree measure BC k 1
180 Now, sin A = = =
AC k 2 2
π 4π
= × 240 rad = radian AB k 1
180 3 and cos A = = =
π AC k 2 2
(ii) Required radian measure = × Degree measure ∴ LHS = 2sin A cos A
180
π 26  1   1  1
= × 520 rad = π radian =2     =2 × = 1 = RHS
180 9  2  2 2
(iii) We have,
4. We know that, cos 2 θ + sin 2 θ = 1 ⇒ cos θ = ± 1 − sin 2 θ
  30 °    1  °
– 47 ° 30′ = − 47 ° +    Q 1′ =  60  3 3π
  60    Given that,sin θ = − and π < θ < ⇒θ lies in III quadrant.
5 2
  1 °   95 °
In III quadrant, cosθ is negative.
= − 47 ° +    = −   9 4
  2  2 ∴ cos θ = − 1 − sin 2 θ = − 1 − =−
 25 5
Now, required radian measure sin θ  3  5 3
Now, tan θ = = −   × −  =
π cos θ  5  4 4
= × Degree measure
180 1 1 4
cot θ = = =
 π 95 − 19π tan θ  3 3
=− ×  radian = radian  
 180 2  72  4
2. We have, 1 1 5
sec θ = = =−
2 cos θ  4 4
π π π  1
2
 3  1 
2
− 
sin 2 + cos 2 + sin 2 =   +   +    5
6 6 4  2  2   2
1 1 5
1 3 1 1+ 3+2 6 3 cosec θ = = =−
= + + = = = sin θ  3 3
− 
4 4 2 4 4 2  5
2
π  3
2
π π  1
2
 1  1 + tan 2 A
∴ sin 2 + cos 2 + cos 2 =   +   +   5. LHS =
3 3 4  2   2  2 1 + cot 2 A
3 1 1 sin 2 A
=+ + 1+
4 4 2 cos 2 A  sin θ cos θ 
= Q tan θ = cos θ and cot θ = sin θ 
3+1+2 6 3 cos 2 A
= = = 1+
4 4 2 sin 2 A
7  2π π π   π π π cos 2 A + sin 2 A 1
∴ + sin + cos 2 + sin 2  ⋅ sin 2 + cos 2 + cos 2 
4  6 6 4  3 3 4 =
2
cos A 2
= cos A [Q sin 2 θ + cos 2 θ = 1]
7 3 3 7 9 16 sin 2 A + cos 2 A 1
⇒ + × = + = =4 sin 2 A
4 2 2 4 4 4 sin 2 A
3. Given, ∠B is right-angled in ∆ABC 1 sin 2 A sin 2 A
= × = = tan 2 A = RHS
and tan A = 1 cos A2
1 cos 2 A
BC 2
∴ tan A = = 1 ⇒ BC = AB  sin A 
AB  1 − tan A  1 −
2

Mid-term =   = cos A 
Let AB = BC = k, where k is a positive real number.  1 − cot A  cos A 
1 −
In right-angled ∆ABC,  sin A 
AC 2 = AB 2 + BC 2  sin θ cos θ 
Q tan θ = cos θ or cot θ = sin θ 
⇒ AC = AB + BC2 2

A (cos A − sin A )2
cos 2 A (cos A − sin A )2 × sin 2 A
= =
(sin A − cos A ) 2
(sin A − cos A )2 × cos 2 A
k 2
sin A
sin 2 A
= = tan 2A = RHS
B
k
C cos 2 A
∴ LHS = Mid-term = RHS Hence proved.
⇒ AC = k 2 + k 2
CBSE Term II Mathematics XI 17

6. (i) sin θ = −
1
∈ [ −1, 1]. Therefore, the given equation is  α + β
9. Given, a tan α + b tan β = ( a + b )tan   , where α ≠ β
5  2 
correct.
  α + β    α + β 
(ii) Since, sec θ ∉ ( − 1, 1) ⇒ a tan α − tan    = b tan   − tan β 
  2     2  
1
∴ sec θ ≠ ,  α + β α + β 
2 a sin α −  b sin  − β
1  2   2 
∴ sec x = is not correct. ⇒ =
2  α + β  α + β
cos α cos   cos   cos β
 2   2 
(iii) tan θ = 20 ∈ ( −∞ , ∞ ).
Therefore, the given equation is correct.  sin A sin B 
Q tan A − tan B = cos A − cos B 
7. (i) ∴ tan ( −2025° ) = − tan (2025° ) [Q tan( − θ ) = − tan θ ]  
= − tan ( 6 × 360°−135° ) = tan 135°  sin A cos B − cos A sin B sin ( A − B ) 
 = =
 cos A cos B cos A cos B 
[Q tan ( n × 360° − θ ) = − tan θ, n ∈ Z ]
= tan( 90°+45° ) = − cot 45°  α − β  α − β
a sin   b sin  
 2   2 
[Q tan ( 90°+ θ ) = − cot θ ] ⇒ =
cos α cos β
= −1
(ii) ∴ sin ( − 1125° ) = − sin(1125° ) [Q sin( − θ ) = − sin θ ]   α − β 
⇒ a cos β = b cos α Q α ≠ β , so sin   ≠ 0
  2  
= − sin(1080° + 45° )
= − sin ( 3 × 360° + 45° ) Hence proved.
= − sin 45° [Qsin ( n × 360° + θ ) = sin θ ] 10. Given that,
1 8 64
=− cosθ = ⇒ sin θ = ± 1 −
2 17 289
(iii) ∴ sin( 930° ) = sin ( 3 × 360°−150° ) = − sin 150° 289 − 64 15
⇒ sin θ = ± ⇒ sin θ = ±
[Q sin ( n × 360° − θ ) = − sin θ , n ∈ Z ] 289 17
= − sin (180° − 30° ) ⇒ sin θ =
15
[since, θ lies in Ist quadrant]
= − sin 30° [Q sin (180°−θ ) = sin θ ] 17
1 Now, cos( 30° + θ ) + cos( 45° − θ ) + cos(120° − θ )
=−
2 = cos( 30° + θ ) + cos( 45° − θ ) + cos( 90° + 30° − θ )
8. Q cos 47 ° cos 13° − sin 47 °sin 13° = cos ( 47 ° + 13° ) = cos( 30° + θ ) + cos( 45° − θ ) − sin( 30° − θ )
[Q cos A cos B − sin A sin B = cos ( A + B )] = cos 30° cos θ − sin 30°sin θ + cos 45° cos θ + sin 45°sin θ
1 − sin 30° cos θ + cos 30°sin θ
= cos 60° = …(i)
2 3 1 1 1 1 3
= cos θ − sin θ + cos θ + sin θ − cos θ + sin θ
Now, sin 75° = sin ( 45° + 30° ) 2 2 2 2 2 2
= sin 45° cos 30° + cos 45°sin 30°  3 1 1  1 1 3
= + −  cos θ +  − +  sin θ
[Qsin ( A + B ) = sin A cos B + cos A sin B]  2 2 2  2 2 2 
1 3 1 1  6 + 2 − 2 2 − 2 + 6
= ⋅ + ⋅ =  cos θ +   sin θ
2 2 2 2  2 2   2 2 
 1 3 1 
Q sin 30° = , cos 30° = and sin 45° = cos 45° =  6 + 2 − 2 8  2 − 2 + 6  15
 2 2 2  =  + 
 2 2  17  2 2  17
3 1 3+1 1
= + = …(ii) = ( 8 6 + 16 − 8 2 + 30 − 15 2 + 15 6 )
2 2 2 2 2 2 17(2 2 )
On adding Eq. (i) and Eq. (ii), we get 1
= (23 6 − 23 2 + 46)
cos 47 ° cos 13° − sin 47 °sin 13° + sin 75° 17(2 2 )
1 3+1 23 6 23 2 46
= + = − +
2 2 2 17(2 2 ) 17(2 2 ) 17(2 2 )
1 ( 3 + 1) × 2 23 3 23 23
= + = − +
2 2 2 × 2 17(2 ) 17(2 ) 17 2
1 2 ( 3 + 1) 23  3 − 1 1 
= + =  + 
2 4 17  2 2
18 CBSE Term II Mathematics XI

3 tan A + tan B
11. Given, cos (α − β ) + cos (β − γ ) + cos ( γ − α ) = − ⇒ =1
2 1 − tan A tan B
⇒ 2 cos (α − β ) + 2 cos (β − γ ) + 2 cos ( γ − α ) = −3 ⇒ tan A + tan B = 1 − tan A tan B
⇒ (2 cos α cos β + 2 cos β cos γ + 2 cos γ cos α ) ⇒ tan A + tan B + tan A tan B = 1 …(i)
+ (2 sin α sin β + 2 sin β sin γ + 2 sin γ sin α ) On adding 1 both sides of Eq. (i), we get
+1+1+1=0 1 + tan A + tan B + tan A tan B = 2
⇒ (2 cos α cos β + 2 cos β cos γ + 2 cos γ cos α ) ⇒ (1 + tan A ) + tan B (1 + tan A ) = 2
+ (2 sin α sin β + 2 sin β sin γ + 2 sin γ sin α ) ⇒ (1 + tan A ) (1 + tan B ) = 2
+ (cos 2 α + sin 2 α ) + (cos 2 β + sin 2 β ) Hence proved.
+ (cos 2 γ + sin 2 γ) = 0 (ii) On dividing both sides of Eq. (i) by tan A tan B, we get
⇒ (cos 2 α + cos 2 β + cos 2 γ + 2 cos α cos β tan A + tan B + tan A tan B 1
=
tan A tan B tan A tan B
+ 2 cos β cos γ + 2 cos γ cos α )
⇒ cot B + cot A + 1 = cot A cot B
+ (sin α + sin β + sin 2 γ + 2 sin α sin β + 2 sin β sin γ
2 2
⇒ cot A cot B − cot A − cot B = 1
+ 2 sin γ sin α ) = 0
⇒ cot A cot B − cot A − cot B + 1 = 2
⇒ (cos α + cos β + cos γ )2 + (sin α + sin β + sin γ )2 = 0
⇒ cot A (cot B − 1) − (cot B − 1) = 2
⇒ cos α + cos β + cos γ = 0 ⇒ (cot B − 1) (cot A − 1) = 2 Hence proved.
and sin α + sin β + sin γ = 0 Hence proved. sin α − cos α
14. Given that, tan θ =
12. ∴ tan 105° = tan ( 60° + 45° ) sin α + cos α
cos α (tan α − 1)
tan 60° + tan 45°  tan A + tan B  ⇒ tan θ =
= Q tan ( A + B ) = 1 − tan A tan B  cos α (tan α + 1)
1 − tan 60° tan 45°   π
tan α − tan
3+1 4  π 
= [Q tan 45° = 1 and tan 60° = 3] ⇒ tan θ = Q tan 4 = 1
1 − 3 ⋅1 π
1 + tan ⋅ tan α
4
3 + 1 (1 + 3 ) (1 + 3)
= = × [on rationalisation]  π
1 − 3 (1 − 3 ) (1 + 3) ⇒ tan θ = tan α − 
 4
( 3 + 1) 2 π π
= [Q( a − b )( a + b ) = a 2 − b 2] ⇒ θ =α − ⇒ α =θ +
1−3 4 4
3+1+2 3 4+2 3  π  π
= = ∴ sin α + cos α = sin θ +  + cos θ + 
1−3 −2  4  4
2 (2 + 3 ) π π π π
= = − (2 + 3) = sin θ ⋅ cos + cos θ ⋅ sin + cos θ ⋅ cos − sin θ ⋅ sin
−2 4 4 4 4
1 1 1 1
Now, tan 15° = tan( 60° − 45° ) = sin θ + cos θ + cos θ − sin θ
2 2 2 2
tan 60° − tan 45°
=  π π 1 
1 + tan 60°⋅ tan 45° Q sin 4 = cos 4 = 2 
 
 tan A − tan B 
Q tan( A − B ) = 1 + tan A ⋅ tan B 
2
= ⋅ cos θ = 2 cosθ
  2
3 −1 3 −1 3 −1 15. (i) ∴ 2 cos 45°sin 15°
= = ×
1 + 3 ⋅1 3+1 3 −1 = sin ( 45° + 15° ) − sin ( 45° − 15° )
[on rationalisation] [Q2 cos A sin B = sin ( A + B ) − sin ( A − B )]
( 3 − 1) 2
= sin 60° − sin 30°
= [Q( a − b ) ( a + b ) = a 2 − b 2]
3−1 3 1 3 −1  3 1
= − = Q sin 60° = and sin 30° = 
3+ 1−2 3 4−2 3 2 2 2  2 2 
= =
2 2 (ii) ∴ 2 sin 15° cos 75° = sin (15° + 75° ) + sin (15° − 75° )
2− 3
= =2 − 3 [Q 2 sin A ⋅ cos B = sin ( A + B ) + sin ( A − B )]
1
= sin 90° + sin ( − 60° )
13. π
(i) We have, A+B= = sin 90° − sin 60° [Qsin ( − θ ) = − sin θ]
4
3 2− 3
π =1− =
⇒ tan ( A + B ) = tan 2 2
4
CBSE Term II Mathematics XI 19

1
(iii) ∴ cos 315° cos 75° = (2 cos 315° cos 75° ) 1 3  1 3
=2⋅ ⋅ Q cos 45° = and cos 30° = 
2 2 2 2 2
 
1
= {cos ( 315° + 75° ) + cos ( 315° − 75°} 3 2 6
2 = × = …(ii)
[Q 2 2 2
2 cos A cos B = cos ( A + B ) + cos ( A − B )] On multiplying Eq. (i) and Eq. (ii), we get
1 6 6 6 3
= {cos 390° + cos 240°} (sin 75° + sin 15° ) (cos 75° + cos 15° ) = × = =
2 2 2 4 2
1 18. Given, m sin θ = n sin (θ + 2 α )
= {cos ( 360° + 30° ) + cos (180° + 60° )}
2 m sin (θ + 2 α )
1 ⇒ =
= {cos 30° + ( − cos 60° )} n sin θ
2 Applying componendo and dividendo rule, we get
[Q cos ( 360° + θ ) = cos θ and cos (180° + θ ) = − cos θ] m + n sin (θ + 2α ) + sin θ
1  3 1  3 1 =
=  −  m − n sin (θ + 2α ) − sin θ
Q cos 30° = and cos 60° = 
2 2 2  2 2
 θ + 2α + θ   θ + 2α − θ 
2 sin   cos  
1  3 − 1 3 −1  2   2 
=   = =
2 2  4  θ + 2α + θ   θ + 2α − θ 
2 cos   sin  
cos ( A − B ) cos ( C + D )  2   2 
16. Given, + =0
cos ( A + B ) cos ( C − D )   A + B  A − B 
Q sin A + sin B = 2 sin  2  cos  2  
cos ( A − B ) − cos ( C + D )  
⇒ =
cos ( A + B ) cos ( C − D )   A + B  A − B 
 and sin A − sin B = 2 cos   sin  
cos ( A − B ) + cos ( A + B )   2   2  

cos ( A − B ) − cos ( A + B ) 2 sin (θ + α ) cos α
=
− cos ( C + D ) + cos ( C − D ) 2 cos (θ + α ) sin α
=
−[cos ( C + D ) + cos ( C − D )] = tan (θ + α ) cot α Hence proved.
[applying componendo and dividendo rule] π 3 π 5 π 7π
19. Given expression, cos 4 + cos 4 + cos4 + cos4
cos ( A + B ) + cos ( A − B ) 8 8 8 8

cos ( A − B ) − cos ( A + B ) 4π 4 3π 4 3π  4 π
= cos + cos + cos  π −  + cos  π − 
cos ( C − D ) − cos ( C + D ) 8 8  8   8
= π 3 π 3 π π
− [cos ( C + D ) + cos ( C − D )] = cos 4 + cos 4 + cos4 + cos4
8 8 8 8
2 cos A cos B 2 sin C sin D
⇒ =−  4π 4 3π   4π  π π 
2 sin A sin B 2 cos C cos D = 2 cos + cos = 2 cos + cos 4  −  
 8 8   8  2 8 
1
⇒ = − tan C tan D  π π
tan A tan B = 2 cos 4 + sin 4 
⇒ tan A tan B tan C tan D = −1 Hence proved.  8 8
 π π
2
π π
 75° + 15°  75° − 15° = 2  cos 2 + sin 2  − 2 cos 2 sin 2 
17. ∴ sin 75° + sin 15° = 2 sin   cos   
 2   2   8 8 8 8 
  A + B  A − B   π π  π π
2
Q sin A + sin B = 2 sin  2  cos  2   = 2 1 − 2 cos 2 sin 2  = 2 − 2 sin cos 
   8 8  8 8
90° 60° 2 2
= 2 sin cos = 2 sin 45° cos 30°  2π   1  1 3
2 2 = 2 − sin  = 2 −   = 2 − =
 8  2 2 2
1 3  1 3
=2⋅ ⋅ Q sin 45° = and cos 30° =  20. Given, cos α + cos β = 0 and sin α + sin β = 0
2 2  2 2 
On squaring both equations, we get
3 2 6 (cos α + cos β )2 = 0
= × = …(i) …(i)
2 2 2
and (sin α + sin β ) = 0
2
…(ii)
 75° + 15°  75° − 15°
Now, cos 75° + cos 15° = 2 cos   cos   On subtracting Eq. (ii) from Eq. (i), we get
 2   2 
(cos α + cos β )2 − (sin α + sin β )2 = 0
  A + B  A − B 
Q cos A + cos B = 2 cos  2  cos  2   ⇒ (cos 2 α + cos 2 β + 2 cos α cos β )
  − (sin 2 α + sin 2 β + 2 sin α sin β ) = 0
 90°  60° [Q ( a + b )2 = a 2 + b 2 + 2 ab]
= 2 cos   cos   = 2 cos 45° cos 30°
 2   2 
20 CBSE Term II Mathematics XI

⇒ cos 2 α + cos 2 β + 2 cos α cos β − sin 2 α 3 3π


23. Given, tan x = , where π < x <
− sin 2 β − 2 sin α sin β = 0 4 2
1
⇒ (cos 2 α − sin 2 α ) + (cos 2 β − sin 2 β ) Q cos x = ±
+ 2 (cos α cos β − sin α sin β ) = 0 1 + tan 2 x
⇒ cos 2α + cos 2β + 2 cos (α + β ) = 0 1
∴ cos x = − [Q x lies in III quadrant]
Q cos 2 x = cos 2 x − sin 2 x and  1 + tan 2 x
 
 cos A cos B − sin A sin B = cos( A + B )  1 4 4
⇒ cos x = − =− ⇒ cos x = −
∴ cos 2α + cos 2β = −2 cos(α + β ) 1+
9 25 5
21. LHS = cot A + cot ( 60° + A ) − cot ( 60° − A ) 16
1 1 1 4
= + − 1−
tan A tan ( 60° + A ) tan ( 60° − A ) x 1 + cos x 5 =− 1
Now, cos = − =−
1 1 1 2 2 2 10
= + −
tan A tan 60°+ tan A tan 60°− tan A  3 π π x 3π 
1 − tan 60° tan A 1 + tan 60°tan A Q π < x < 2 ⇒ 2 < 2 < 4 
 
 tan A ± tan B  ⇒ cos x < 0, sin x > 0 
Q tan ( A ± B ) = 1 m tan A tan B 
  x 1 − cos x 1 + 4/ 5 9 3
sin = = = =
1 1 − 3 tan A 1 + 3 tan A 2 2 2 10 10
= + −
tan A 3 + tan A 3 − tan A x sin x / 2 3 / 10
[Q tan 60° = 3 ] and tan = = =−3
2 cos x / 2 − 1 / 10
1 8 tan A 3 − 9 tan 2 A 24. LHS = cos 3 A + cos 3 (120° + A ) + cos 3 (240° + A )
= − =
tan A 3 − tan 2 A 3 tan A − tan 3 A
3 1 
 1 − 3 tan 2 A  =  cos A + cos 3A 
=3  4 4 
 3 tan A − tan 3 A  3 1 
+  cos(120° + A ) + cos 3 (120° + A )
3  4 4 
= = 3 cot 3A = RHS Hence proved.
tan 3A 3 1 
+  cos (240° + A ) + cos 3 (240° + A )
tan 3x 4 4 
22. Let y=
tan x  3 1 
=  cos A + cos 3A 
3 tan x − tan 3 x 4 4 
⇒ y=
tan x(1 − 3 tan 2 x ) 3 1 
+  cos (120° + A ) + cos ( 360° + 3A )
3 − tan 2 x 4 4 
⇒ y=
1 − 3 tan 2 x 3 1 
+  cos (240° + A ) + cos (720° + 3A )
⇒ y − 3y tan 2 x = 3 − tan 2 x 4 4 
⇒ (1 − 3y )tan 2 x = 3 − y 3
= [cos A + cos (120° + A ) + cos (240° + A )]
3−y 4
⇒ tan 2 x =
1 − 3y 1
+ [cos 3A + cos ( 360° + 3A ) + cos ( 360° × 2 + 3A )]
3−y y−3 4
⇒ ≥ 0⇒ ≥ 0 [Q tan 2 x ≥ 0 for all x] 3
1 − 3y 3y − 1 = [cos A + cos (120° + A ) + cos (240° + A )]
Case I When y − 3 ≥ 0 and 3y − 1 > 0 4
1
∴ y ≥ 3 and y >
1
∴y≥3 + [cos 3A + cos 3A + cos 3A ]
3 4
3
Case II When y − 3 ≤ 0 and 3y − 1 < 0 = [cos A + cos(120°+ A ) + cos (240°+ A )]
1 4
∴ y ≤ 3 and y < 1
+ × 3(cos 3A )
3 4
1
⇒ y< 3 3
3 = [cos A + 2 cos(180° + A ) ⋅ cos ( −60° )] + cos 3A
4 4
 1
⇒ y ∈  − ∞ ,  ∪ [ 3, ∞ ) 3 1 3
 3 = cos A − 2 cos A ⋅  + cos 3A
4  2 4
1
Hence, y does not lie between and 3. Hence proved. 3 3 3
3 = [cos A − cos A ] + cos 3A = cos 3A
4 4 4
Hence proved.
CBSE Term II Mathematics XI 21

25. Let AB be the diameter of the moon and O be the eye of the tan α + tan β
27. We have, tan(α + β ) =
observer so that ∠AOB = 31′. Let CD = 2 cm be the 1 − tan α tan β
diameter of the coin. The full moon will be just concealed, if
 tan x + tan y 
the diameter of a coin also subtends the same angle as the Q tan ( x + y ) = 1 − tan x tan y 
diameter of the moon at O, i.e. if ∠COD = 31′.  
A 1  1 
Given, cot α = ⇒ tan α = 2 Q tan α = cot α 
2  
−5
C Also, secβ =
r 3
l
Then, tan β = ± sec 2 β − 1
O 31′
r D B 25 16  −5 
⇒ tan β = ± −1 = ± Q secβ = 3 
As, ∠COD is small, CD may be treated as the arc of a circle, 9 9
whose centre is O and radius = OC or OD. Let OC = OD = r 4
∴ tan β = ±
3
Here, l = length of an arc CD = 2 cm 4
But, tan β ≠
 1 °   1  ° 3
and θ = 31′ =  31 ×  Q 1′ =   
 60   60   π  
 Q β ∈  2 , π  and tan β is −ve in IInd quadrant
 
 31 π   π 
= ×  radian Q 1° = 180 radian  4
 60 180 ∴ tan β = −
3
l 2
Now, r= =  4
θ 31 × π 2 + − 
tan α + tan β  3
60 180 ∴ tan (α + β ) = =
2 × 60 × 180 1 − tan α tan β  −4
1 − (2 )  
=  3
31 × π
21600 × 7  −4 
 22 
Q tan α = 2 and tan β = 3 
= Q π = 7 
31 × 22
75600  4  2
= cm 2 −   
 3  3 2
341 = = =
756  8   11 11
= m [Q 100 cm = 1 m ] 1 +   
341  3  3 
= 2.22 m tan (θ + α ) tan (θ + β )
28. Given, =
26. Given that, x = sec φ − tan φ …(i) a b
y = cosec φ + cot φ a tan (θ + α )
and …(ii) ⇒ =
b tan (θ + β )
On multiplying Eqs. (i) and (ii), we get
xy = (sec φ − tan φ )( cosec φ + cot φ ) a + b tan (θ + α ) + tan (θ + β )
⇒ =
⇒ xy = sec φ ⋅ cosec φ − cosec φ ⋅ tan φ + sec φ cot φ a − b tan (θ + α ) − tan (θ + β )
− tan φ cot φ [applying componendo and dividendo rule]
1 1 a + b sin (θ + α + θ + β )
⇒ xy = sec φ ⋅ cosec φ − + −1 ⇒ =
cos φ sin φ a − b sin (θ + α − θ − β )
⇒ 1 + xy = sec φ cosec φ − sec φ + cosec φ …(iii)
From Eqs. (i) and (ii), we get  sin A cos B + cos A sin B 
 tan A + tan B cos A cos B sin ( A + B ) 
x − y = sec φ − tan φ − cosec φ − cot φ Q = = 
⇒ x − y = sec φ − cosec φ −
sin φ cos φ
−  tan A − tan B sin A cos B − cos A sin B sin ( A − B ) 
cos φ sin φ  cos A cos B 
 sin 2 φ + cos 2 φ  a + b sin (2 θ + α + β )
⇒ =
⇒ x − y = sec φ − cosec φ −   a−b sin (α − β )
 sin φ cos φ 
1  a + b
⇒ x − y = sec φ − cosec φ − ⇒   sin (α − β ) = sin (2 θ + α + β )
sin φ cos φ  a − b
⇒ x − y = sec φ − cosec φ − cosec φ sec φ  a + b
⇒  sin (α − β ) = sin(2 θ + α + β ) sin (α − β )
2

⇒ x − y = − (sec φ cosec φ − sec φ + cosec φ )  a − b


⇒ x − y = − ( xy + 1) [from Eq. (iii)]  a + b 1
⇒  sin (α − β ) = [2 sin (2 θ + α + β )sin (α − β )]
2
⇒ xy + x − y + 1 = 0 Hence proved.  a − b 2
22 CBSE Term II Mathematics XI

 a + b 1   A + B  A − B 
n
⇒  sin (α − β ) = [cos (2 θ + 2 β ) − cos (2 θ + 2α )]
2
 a − b 2  2 sin  2  cos  2  
+ 
[Q 2 sin A sin B = cos ( A − B ) − cos( A + B )]  − 2 sin  A + B  sin  A − B  
b+c 2 1   2   2  
Similarly, sin (β − γ ) = [cos (2 θ + 2 γ )
b−c 2 n n
  A − B    A − B 
– cos (2 θ + 2 β )]  cos  2    cos  2  
c+a 2 1 =  + − 
and sin ( γ − α ) = [cos (2 θ + 2 α ) − cos (2 θ + 2 γ )]  sin  A − B    sin  A − B  
c−a 2   2     2  
 a + b  b + c  A − B  A − B
 sin (α − β ) +   sin (β − γ )
2 2
Now,  ⇒ LHS = cot n   + ( − 1)n cot n  
 a − b  b − c  2   2 
 c + a Here, two cases arise.
+  sin ( γ − α )
2
 c − a Case I When n is even, then
1  A − B n  A − B
= [cos (2 θ + 2 β ) − cos (2 θ + 2 α ) + cos (2 θ + 2 γ ) cot n   + cot   [Q( − 1)n = 1, if n is even]
2  2   2 
− cos (2 θ + 2 β ) + cos (2 θ + 2 α ) − cos (2 θ + 2 γ )]
 A − B
1 = 2 cot n  
= × 0= 0  2 
2
Case II When n is odd, then
∴ LHS = RHS Hence proved.
cos 2 A cos 3A − cos 2 A cos 7 A + cos A cos 10A  A − B n  A − B
29. LHS =
cot n   − cot  
 2   2 
sin 4A sin 3A − sin 2 A sin 5A + sin 4A sin 7 A
2 cos 3A cos 2 A − 2 cos 7 A cos 2 A + 2 cos 10A cos A [Q( − 1)n = − 1, if n is odd]
=
2 sin 4A sin 3A − 2 sin 5A sin 2 A + 2 sin 7 A sin 4A =0
cos ( 3A + 2 A ) + cos ( 3A − 2 A ) − [cos (7 A + 2 A )    A − B
+ cos (7 A − 2 A )] + [cos (10A + A ) + cos (10A − A )]  2 cot n   , when n is even
Hence, LHS =   2 
= 

cos ( 4A − 3A ) − cos ( 4A + 3A ) − [cos ( 5A − 2 A )  0, when n is odd
− cos ( 5A + 2 A )] + [cos (7 A − 4A ) − cos (7 A + 4A )] 31. LHS = cos A ⋅ cos 2 A ⋅ cos 2 2 A ⋅ cos 2 3 A ... cos2 n − 1 A
 
cos 5A + cos A − cos 9A − cos 5A + cos 11A + cos 9A 1
= = [(2 sin A cos A )cos 2 A cos 2 2 A
cos A − cos 7 A − cos 3A + cos 7 A + cos 3A − cos 11A 2 sin A
cos A + cos 11A ⋅ cos 2 3 A K cos 2 n − 1 A ]
=
cos A − cos 11A 1
A + 11A   A − 11A  = [(sin 2 A cos 2 A ) ⋅ cos 2 2 A cos 2 3 A
2 cos   cos   2 sin A
 2   2 
= K cos 2 n − 1 A ]
A + 11A   11 A − A 
2 sin   sin   1
 2   2  = [(2 sin 2 A cos 2 A )cos 2 2 A ⋅ cos 2 3 A
2 2 sin A
  x + y  x − y 
Q cos x + cos y = 2 cos  2  cos  2   K cos 2 n − 1 A ]
 
1
  x + y  x − y  = [(sin 2 2 A cos 2 2 A )cos 2 3 AK cos 2 n − 1 A ]
 and cos x − cos y = − 2 sin   sin  
 2   2  2 2 sin A
 
  x + y  y − x  1
= 2 sin   sin   = [(2 sin 2 2 A cos 2 2 A )cos 2 3 A K cos 2 n − 1 A ]
  2   2   2 3 sin A

cos 6A cos 5A 1
= [Q cos( − θ ) = cos θ ] = 3
[sin 2 3 A cos 2 3 A K cos 2 n − 1 A ]
sin 6A sin 5A 2 sin A
= cot 6A cot 5A = RHS Hence proved. Proceeding in this manner, we get
1
[sin 2 n − 1 A ⋅ cos 2 n − 1 A ]
n n
 cos A + cos B   sin A + sin B  LHS = n − 1
30. LHS =   +  2 sin A
 sin A − sin B   cos A − cos B 
1
n = [2 sin 2 n − 1 A cos 2 n − 1 A ]
  A + B  A − B  n
2 sin A
2 cos  2  cos  2  
=  =
1
sin 2 n A = RHS
 2 cos  A + B  sin  A − B  
Hence proved.
2 n sin A
  2   2  
CBSE Term II Mathematics XI 23

1 + sin A − cos A 1 − cos A + sin A 2(sin 2 α + sin 2 γ )


32. LHS = = =
1 + sin A + cos A 1 + cos A + sin A 2 + cos 2(α − γ ) − cos 2(α + γ )
 A 2(sin 2 α + sin 2 γ ) sin 2 α + sin 2 γ
1 − 1 − 2 sin 2  + sin A = =
 2 2 + 2 sin 2 α sin 2 γ 1 + sin 2 α sin 2 γ
=
 2A  34. Given that, α and β are distinct roots of
1 + 2 cos − 1 + sin A
 2  a cos θ + b sin θ = c
 2θ 2θ  ∴ a cos α + b sin α = c and a cos β + b sin β = c
Q cos θ = 1 − 2 sin 2 = 2 cos 2 − 1
⇒ a cos α + b sin α = a cos β + b sin β
A A
1 − 1 + 2 sin 2 + sin A 2 sin 2 + sin A ⇒ a(cos α − cos β ) + b(sin α − sin β ) = 0
= 2 = 2  α + β  α − β
2A 2A
⇒ − 2 asin   sin  
1 + 2 cos − 1 + sin A 2 cos + sin A  2   2 
2 2
 α + β  α − β
A A A + 2 b cos   sin   =0
2 sin 2 + 2 sin cos  2   2 
2 2 2  θ θ
= Q sin θ = 2 sin cos 
A A A  2 2  α − β   α + β  α + β 
2 cos 2 + 2 sin cos ⇒ 2 sin   − a sin   + b cos   =0
2 2 2  2    2   2  
A A A  α + β (α + β )
2 sin sin + cos  ⇒ b cos   = a sin
2 2 2  2  2
=
A A A
2 cos sin + cos    α − β 
2 2 2 Q α ≠ β , therefore sin  2  ≠ 0
 
A
sin  sin θ  (α + β ) b
= 2 = tan A ⇒ tan =
A 2 Q cos θ = tan θ  2 a
cos  
(α + β ) b 2b
2 2 tan 2×
= RHS Hence proved. ∴ sin (α + β ) = 2 = a = 2a 2
2  α + β a +b
2
b
sin α sin γ 1 + tan   1+ 2
+  2  a a2
tan α + tan γ cos α cos γ
33. Given, tan β = = 2 ab
1 + tan α tan γ 1 + sin α sin γ = 2 Hence proved.
cos α cos γ a + b2
4 3π
sin α cos γ + cos α sin γ 35. Given, cos A = , where < A < 2π
= 5 2
cos α cos γ + sin α sin γ
3
sin (α + γ ) ∴ sin A = − 1 − cos 2 A = −
⇒ tan β = 5
cos (α − γ )
[Q A lies in IVth quadrant]
2 tan β 3π
Now, sin 2 β = and
12
cos B = , where < B < 2π
1 + tan 2 β 13 2
sin(α + γ ) 5
2× ∴ sin B = − 1 − cos 2 B = −
cos (α − γ ) 13
=
sin 2(α + γ ) [QB lies in IVth quadrant]
1+
cos 2(α − γ ) 4 12  3  5 
(i) cos ( A + B ) = × − −  − 
2 sin (α + γ )cos (α − γ ) 5 13  5  13
=
cos 2(α − γ ) + sin 2(α + γ ) [Q cos( A + B ) = cos A cos B − sin A sin B ]
sin 2 α + sin 2 γ 48 15 33
= = − =
cos 2(α − γ ) + sin 2(α + γ ) 65 65 65
 3 12 4  5 
[Q2sin A cos B = sin ( A + B ) + sin ( A − B )] (ii) sin ( A − B ) =  −  × − × − 
 5 13 5  13
2(sin 2 α + sin 2 γ )
= [Q sin ( A − B ) = sin A cos B − cos A sin B ]
2 cos 2(α − γ ) + 2 sin 2(α + γ ) 36 20 16
=− + =−
2(sin 2 α + sin 2 γ ) 65 65 65
=
1 + cos 2(α − γ ) + 1 − cos 2(α + γ )  3  12   4  5 
(iii) Qsin ( A + B ) =  −  ⋅   +   ⋅  − 
Q 2 cos 2 A = 1 + cos 2 A   5  13   5  13
  [Q sin( A + B ) = sin A cos B + cos A sin B]
and 2 sin A = 1 − cos 2 A 
2
24 CBSE Term II Mathematics XI

− 36 − 20 56 (ii) On dividing both sides of Eq. (i) by tan A tan B, we get


= =−
5 × 13 65 tan A + tan B + tan A tan B 1
=
56 tan A tan B tan A tan B

56 ⇒ cot B + cot A + 1 = cot A cot B
∴ tan ( A + B ) = 65 = −
33 33 ⇒ cot A cot B − cot A − cot B = 1
65 ⇒ cot A cot B − cot A − cot B + 1 = 2
 sin( A + B )  ⇒ cot A (cot B − 1) − (cot B − 1) = 2
Q tan( A + B ) = cos( A + B ) 
  ⇒ (cot B − 1) (cot A − 1) = 2
36. π π
(i) We have, A+B= (iii) Given, A + B =
4 4
π π 1
⇒ tan ( A + B ) = tan ∴ sin( A + B ) = sin = ,
4 4 2
tan A + tan B π 1
⇒ =1 cos( A + B ) = cos =
1 − tan A tan B 4 2
⇒ tan A + tan B = 1 − tan A tan B π
and tan( A + B ) = tan = 1
⇒ tan A + tan B + tan A tan B = 1 …(i) 4
On adding 1 both sides of Eq. (i), we get ∴ sin( A + B ) − cos( A + B ) + tan( A + B )
1 + tan A + tan B + tan A tan B = 2 1 1
= − + 1=1
⇒ (1 + tan A ) + tan B (1 + tan A ) = 2 2 2
⇒ (1 + tan A ) (1 + tan B ) = 2
Chapter Test
(ii) The value of sin2A is
l
Multiple Choice Questions
120 120
(a) (b)
1. Which of the following is correct? 149 169
120 120
(a) sin 1° > sin1 (b) sin 1° < sin1 (c) (d)
π 179 159
(c) sin 1° = sin1 (d) sin 1° = sin1
18° (iii) The value of cos2A is
2. If sinθ + cosec θ = 2, then sin θ + cosec θ is
2 2
(a)
119
(b)
119
equal to 169 159
119 119
(a) 1 (b) 4 (c) (d)
(c) 2 (d) None of these 149 139
(iv) The value of tan3 A is
3. Consider the information given below
2035 1035
13 (a) (b)
sec x = and x lies in IV quadrant. Find the 828 828
5 2035
(c) (d) None of these
value of sinx. 728
12 12
(a) − (b) (v) The value of sec A is
13 13
5 5 13 12
(c) − (d) (a) (b)
13 13 10 13
12 13
(c) − (d)
4. If tanθ = 3 and θ lies in III quadrant, then the 13 12
value of sin θ is
(a)
1
(b) −
1
l
Short Answer Type Questions
10 10
−3 3
6. In a circle of diameter 44 cm, the length of
(c) (d) chord is 22 cm. Find the length of minor arc
10 10
of the chord.
7. Prove the following identities.
l
Case Based MCQs (i) tan2 θ − sin2 θ = tan2 θ sin2 θ
5. Vinit and Vipul are studying together and they (ii) (1 + cotθ − cosec θ) (1 + tanθ + secθ) = 2
5
are given the information that tan A = , where 8. Find the value of
12
π sin 135° cosec 225° tan 150° cot 315°
x ∈ R and x ≠ (2n + 1) , n ∈ I .
2 + cos 135°sec225° tan 120°cot330° .
Based on the above information, answer the
following questions
l
Long Answer Type Questions
(i) The value of tan2A is 13
9. If x and y are acute angles such that cos x =
116 119 14
(a) (b) 1 π
115 120 and cos y = , then prove that x − y = − .
120 121 7 3
(c) (d)
119 120 10. Prove that
tan θ tan (60° − θ) tan (60° + θ) = tan 3 θ.

Answers
For Detailed Solutions
1. (b) 2. (c) 3. (a) 4. (c)
Scan the code
484 3
5. (i) (c) (ii) (b) (iii) (a) (iv) (a) (v) (d) 6. cm 8. 3 −
21 3
26 CBSE Term II Mathematics XI

CHAPTER 02

Linear
Inequalities
In this Chapter...
l Linear Inequality
l Solution of an Inequality
l System of Inequalities in One Variable
l Linear Inequations in Two Variables and their Graphical Solution

An inequality is said to be linear, if the variable (s) occurs in Here, end points of the interval i.e. a and b are included in the
first degree only and there is no term involving the product of interval. So, on number line, draw filled circle ( •) at a and b.
the variables. e.g. ax + b ≤ 0, ax + by + c > 0, ax ≤ 4.
Open Interval
Linear Inequality in One Variable If a and b are real numbers, such that a < b, then the set of all
A linear inequality which has only one variable, is called real numbers x, such that a < x < b, is called an open interval
linear inequality in one variable. e.g. ax + b < 0, where a ≠ 0. and is denoted by ( a , b ) or ] a , b [.
An inequality in one variable, in which degree of variable is 2, ∴ ( a , b ) = {x : a < x < b , x ∈ R}
is called quadratic inequality in one variable. On the number line, ( a , b ) may be represented as follows
e.g. ax 2 + bx + c ≥ 0, a ≠ 0 ; 3 x 2 + 2 x + 1 ≤ 0 a<x<b
−∞ ∞
a b
Linear Inequality in Two Variables
A linear inequality which have only two variables, is called Here, end points of the interval i.e. a and b are not included
linear inequality in two variables. e.g. 3 x + 11 y ≤ 0, 4 t + 3 y > 0 in the interval. So, on number line, draw open circle (o) at a
and b.
Concept of Intervals Semi-open or Semi-closed Intervals
On number line or real line, various types of infinite subsets, If a and b are real numbers, such that a < b.
known as intervals, are defined below
Then, ( a , b ] = {x : a < x ≤ b , x ∈ R}
Closed Interval
and [ a , b ) = {x : a ≤ x < b , x ∈ R}
If a and b are real numbers, such that a < b, then the set of all
are known as semi-open or semi-closed intervals.
real numbers x, such that a ≤ x ≤ b, is called a closed interval
and is denoted by [a , b]. On the number line, these intervals may be represented
as follows
∴ [ a , b ] = {x: a ≤ x ≤ b , x ∈ R} a<x≤b
−∞ ∞
On the number line, [ a , b ] may be represented as follows a (a, b] b
a≤x≤b a≤x<b
−∞ ∞ −∞ ∞
a b a [a, b) b
CBSE Term II Mathematics XI 27

Solution of an Inequality System of Inequalities in One Variable


Any solution of an inequality is the value(s) of variable(s) Two or more inequalities taken together comprise a system of
which makes it a true statement. inequalities and the solution of the system of inequalities are
We can find the solutions of an inequality by hit and trial the solutions common to all the inequalities comprising the
method but it is not very efficient because this method is time system.
consuming and sometimes not feasible. So, we solve e.g. x = 10 is the solution of the system of inequalities
inequalities with systematic technique. 4 x + 3 ≤ 91 and 2 x ≥ x + 8
Some properties or rules which are used to solve the Solution of System of Linear Inequalities
inequalities, are given below in One Variable
Addition or Subtraction We know that, the solution set of a linear inequality in one
Some number may be added (or subtracted) to (from) both variable is the set of all points on the number line satisfying
sides of an inequality i.e. if a > b, then for any number c, the given inequality.
a + c > b + c or a − c > b − c Therefore, the solution set of a system of linear inequalities in
one variable is defined as the intersection of the solution set
e.g. (i)10 > 5 ⇒10 + 7 > 5 + 7 [adding 7 both sides]
of the linear inequalities in the system.
⇒ 17 > 12, which is true.
e.g. If the solution sets of linear inequalities in the system are
(ii) −8 > − 13 ⇒ −8 − 2 > − 13 − 2 ( − ∞ , 5 ] and [ 5 , ∞ ), then the solution of system of linear
[subtracting 2 from both sides] inequalities in one variable is 5 only. Because, if we represent
⇒ −10 > − 15, which is true. the solution sets on the number line, we see that the value
which are common to both is 5 only.
Multiplication or Division
If both sides of an inequality are multiplied (or divided) by the

same positive number, then the sign of inequality remains the –∞ –3 –2 –1 0 1 2 3 4 5 6 7
same. But when both sides are multiplied (or divided) by the
same negative number, then the sign of inequality is reversed. The process of finding solution of system of linear inequalities
of different types are given below.
Let a, b and c be three real numbers, such that a > b and c ≠ 0.
a b
(i) If c > 0, then > and ac > bc.
Type I
c c When Two Separate Linear Inequalities are Given
a b
(ii) If a > b and c < 0, then < and ac < bc. If the given system of inequalities comprise by two separate
c c linear inequalities, then to solve these we use the following
Method to Solve a Linear Inequality working steps
in One Variable Step I Solve each inequality separately and obtain their
solution sets.
Step I Collect all terms involving the variable ( x) on one
Step II Represent the solution sets on a number line and
side and constant terms on other side with the help then find the values of the variable which are
of above rules and then reduce it in the form ax < b common to them.
or ax ≤ b or ax > b or ax ≥ b.
Or Find the intersection of the solution sets obtained
Step II Divide this inequality by the coefficient of variable in step I.
( x). This gives the solution set of given inequality.
Step III Write the solution set.
Type 2
cx + d
When Inequalities of the form a ≤ e ≤ b,
Representation of solution of Linear Inequality
in One Variable on Number Line where a, b, c, d ∈ R
To represent the solution of a linear inequality in one variable This type of inequalities will be formed by combining the
on a number line, use the following rules cx + d cx + d
(i) To represent x < a (or x > a) on a number line, put a inequalities a ≤ and ≤ b.
e e
circle (o) on the number a and dark the line to the left
To solve such type of inequalities, make the middle term free
(or right) of the number a.
from constant (i.e. write the given inequalities as f ≤ x ≤ g,
(ii) To represent x ≤ a (or x ≥ a) on a number line, put a where f and g are some real numbers by using the rule of
dark circle (•) on the number a and dark the line to the addition, subtraction, multiplication, division in each term of
left (or right) of the number a. given inequalities.)
28 CBSE Term II Mathematics XI

Linear Inequations in Two Variables


and their Graphical Solution
Graph of Linear Inequality in Two Variables
An inequality of the form ax + by + c > 0 or ax + by + c < 0 or
Working rule for drawing the graph of linear inequalities in
ax + by + c ≥ 0 or ax + by + c ≤ 0, where a ≠ 0 and b ≠ 0, is
called a linear inequality in two variables x and y. two variables are discussed below
Step I Consider the equation ax + by = c, in place of
The region containing all the solutions of an inequality, is
given inequality ax + by ≤ c or ax + by ≥ c or
called the solution region.
ax + by < c or ax + by > c, which represents a
straight line in XY-plane.
Concept of Half Planes
Step II Put x = 0 in the equation obtained in step I to get
The graph of ax + by + c = 0 is a straight line which divides the
the point, where the line meets Y-axis. Similarly,
cartesian plane or XY-plane into two parts. Each part is
put y = 0 to obtain a point, where the line meets
known as half plane.
the X-axis.
Types of Half Planes Step III Draw a line joining the points obtained in step II.
1. Left and right half planes A vertical line will divide the If the inequality is of the form of < or >, then
XY-plane in two parts, left half plane and right half draw dotted line to indicate that the points on the
plane. line are excluded. Otherwise, draw a thick or dark
Y line to indicate that the points on this line are
included.

Right
Step IV Take any point (preferable origin, i.e. (0, 0)), not
Left
half plane half plane lying on the line, and check whether this satisfies
X′
O X the given linear inequality or not.
Step V If the inequality is satisfied, by this point then
shade that portion of the plane which contains the
chosen point. The shaded region obtained in step V,
Y′ represents the graph of linear inequality in two
variable.
2. Lower and upper half planes A non-vertical line will
divide the XY-plane into two parts, lower half plane and Graphical Solutions of System of Linear
upper half plane. Inequalities in Two Variables
Y
lf p er

A system of linear inequalities in two variables can be


e
lan
ha Upp

solved by graphical method.


X′ X For finding the solution, we use the following steps
O
Lower
half plane Step I Draw the graph of all the given inequalities.
Step II Find the common shaded region, which satisfies
Y′ all the given linear inequalities.
Step III This common region is the required solution
3. Closed half plane A half plane in XY-plane is called a region of the system of given inequalities.
closed half plane, if the line separating the plane is also
included in the half plane. If there is no common region, then the system of
inequalities has no solution.
Therefore, the graph of a linear inequality involving sign
≤ or ≥ is always closed half plane. Inequality between Coordinate Axes
4. Open half plane A half plane in XY-plane is called an
The inequality x ≥ 0 consist of Y-axis and the plane on the
open half plane, if the line separating the plane is not
right side of Y-axis. Also, y ≥ 0 consist of X-axis and the
included in the half plane.
plane above the X-axis.
Therefore, the graph of a linear inequality involving sign
∴The inequalities x ≥ 0 and y ≥ 0, together represent the
< or > is always an open half plane.
first quadrant including the point on the axes.
CBSE Term II Mathematics XI 29

Solved Examples
Example 1. Solve 24x < 100, when Transferring the term 6x to LHS and ( − 4) to RHS,
(i) x is a natural number (ii) x is an integer. 4x − 6x < − 12 + 4
Sol. We have 24x < 100, dividing both sides by 24, ⇒ − 2x < − 8
24x 100 On dividing both sides by −2, we get
⇒ < (using rule 2) −2 x −8
24 24 ⇒ > (using rule 2)
50 25 −2 −2
⇒ x< ⇒x <
12 6 ⇒ x>4
1  1
⇒ x<4  i. e. x is less than 4 
6  6
–∞ 4 5 6 +∞
(i) When x is a natural number (only positive integer) in
this case, the solution set of the inequality is {1, 2, 3, 4}. ∴ Solution set = ( 4, ∞ ).
(ii) When x is an integer, the solution set of the given Example 5. 37 − (3x + 5) ≥ 9x − 8 (x − 3).
inequality is {……− 4, − 3, − 2 , − 1, 0, 1, 2 , 3, 4}
Sol. We have, 37 − ( 3x + 5) ≥ 9x − 8 ( x − 3)
Example 2. Solve − 12 x > 30, when ( 37 − 3x − 5) ≥ 9x − 8 x + 24
(i) x is a natural number (ii) x is an integer. ⇒ 32 − 3x ≥ x + 24
Sol. We have −12 x > 30, dividing both sides by −12 Transferring the term 24 to LHS and the term ( − 3x ) to RHS,
− 12 x 30 32 − 24 ≥ x + 3x
<
−12 − 12 ⇒ 8 ≥ 4x ⇒ 4x ≤ 8
5 On dividing both sides by 4, we get
⇒ x<−
2 4x 8
⇒ ≤ (using rule 2)
(i) When x is a natural number, then there is no solution of 4 4
given inequality. As natural numbers are positive
⇒ x ≤2
numbers and there is no positive number which is less
than negative number.
(ii) When x is an integer, the solution of given inequality is –∞ 0 1 2 +∞
{.... − 4, − 3} as there are infinite number which are less
then − .
5 ∴ Solution set = ( − ∞ , 2 ].
2
Example 6. Find the solutions of the system of
Example 3. Solve 5x − 3 < 7, when inequalities 3x − 7 < 5 + x and 11 − 5x ≤ 1 on the
(i) x is an integer (ii) x is a real number. number line.
Sol. We have, 5x − 3 < 7, adding 3 on both sides, Sol. Given, inequalities are
3x − 7 < 5 + x ... (i)
⇒ 5x − 3 + 3 < 7 + 3 (using rule 1)
and 11 − 5x ≤ 1 …(ii)
5x < 10
From inequality (i), we have
On dividing both side by 5, we get
3x − 7 < 5 + x
5x 10
< (using rule 2) or x<6 ... (iii)
5 5
⇒ x <2 Also, from inequality (ii), we have
(i) When x is an integer, the solution of the given 11 − 5 x ≤ 1
inequality is {.... , − 1, 0, 1}. or − 5 x ≤ − 10
(ii) When x is a real number, the solution of given i.e. x ≥2 …(vi)
inequality is ( − ∞ , 2 ) i.e. all the numbers lying between If we draw the graph of inequalities (iii) and (vi) on the
− ∞ and 2 but ∞ and 2 are not included as x < 2. number line, we see that the values of x, which are common
Example 4. 2(2 x + 3) − 10 < 6 (x − 2 ). to both, are shown by bold line in figure.
Sol. We have, 2 (2 x + 3) − 10 < 6 ( x − 2 )
⇒ 4x + 6 − 10 < 6x − 12
⇒ 4x − 4 < 6x − 12 –1 0 1 2 3 4 5 6 7 8 9
30 CBSE Term II Mathematics XI

Example 7. The solution of the inequality Example 10. Find the graphical solution of 3x + 4y ≤ 12.
− 8 ≤ 5x − 3 < 7 is Sol. We have the given inequality 3x + 4y ≤ 12 …(i)
Sol. In this case, we have two inequalities, − 8 ≤ 5x − 3 and
Step 1. Consider the inequation as a strict equation i.e.
5x − 3 < 7, which we will solve simultaneously. 3x + 4y = 12
We have, − 8 ≤ 5x − 3 < 7 Step 2. Find the points on the X-axis and Y-axis i.e.
or − 5 ≤ 5x < 10
x 4 0
or − 1 ≤ x < 2 i.e. x ∈ [ − 1, 2 ).
y 0 3
Example 8. In an experiment, a solution of hydrochloric Step 3. Plot the graph using the above table.
acid is to be kept between 30°C and 35° C. The
range of temperature in degree Fahrenheit, if Step 4. Take a point (0, 0) and put it in the given inequation
5 (i), we get
conversion formula is given by C = (F − 32 ), where 0 + 0 ≤ 12
9
C and F represent temperature in degree Celsius which is true, so the shaded region will be towards
and degree Fahrenheit respectively, is between the origin.
Y
…A… and …B… . Here, A and B refer to
Sol. It is given that 30 < C < 35.

3x
5 (0,3)

+
Putting C = (F − 32 ), we get

4y
=
9

12
5 X′ X
30 < (F − 32 ) < 35, O (4,0)
9
9
Multiplying to each term,
5
9 9
or × ( 30) < (F − 32 ) < × ( 35) Y'
5 5
Here, shaded region shows the inequality.
or 54 < (F − 32 ) < 63
or 86 < F < 95. Example 11. Represent the inequality y + 8 ≥ 2 x
Thus, the required range of temperature is between 86° F graphically.
and 95° F. Sol. The given inequation is y + 8 ≥ 2 x …(i)
Example 9. Find the graphical solution of the inequality Step 1. Consider the inequation as strict equation i.e.
y + 8 = 2x
2 x + y ≥ 6.
Step 2. Find the points on the X-axis and Y-axis i.e.
Sol. We have the given inequality 2 x + y ≥ 6 …(i)
Step 1. Consider the inequation as a strict equation x 0 4
i.e. 2x + y = 6 y –8 0
Step 2. Find the points on X-axis and Y-axis i.e.
Y
x 3 0 (4, 0)
y 0 6 X' X
O
2x

Step 3. Plot the graph using the above table.


8=

Step 4. Take a point ( 0, 0) and put it in the given inequation


y+

(i), we get 0 + 0 ≥ 6 which is false, so shaded region


will be away from the origin. (0,– 8)
Y

(0,6) Y'
2x+

Step 3. Plot the graph using the above table.


y=

Step 4. Take a point (0, 0) and put it in the given inequation


6

X′ X (i), we get
O (3,0)
0+ 8≥0
which is true, so the shaded region will be towards
the origin.
Y' Thus, shaded region shows the inequality.
CBSE Term II Mathematics XI 31

Example 12. Solve the following inequality graphically Step 3. Plot the graph using the above tables.
x − y ≤ 2. Step 4. Take a point (0, 0) and put it in the inequations (i)
Sol. The given inequation is x − y ≤ 2 …(i) and (ii),
0+ 0≤6
Step 1. Consider the inequation as a strict equation i.e.
0≤6 (True)
x − y =2
Step 2. Find the points on the X-axis and Y-axis i.e. So, the shaded region will be towards the origin.
and 0+ 0≥4 ⇒ 0≥4 (False)
x 2 0
So, the shaded region will be away from the origin.
y 0 –2
Thus, common shaded region shows the solution of
Y the inequalities.
(2,0) Example 14. Solve the system of equations graphically
X' X
O x + y ≥ 4, 2 x − y > 0.
2

Sol. The given system of inequalities


y=

x+y≥4
x–

…(i)
(0,–2)
2x − y > 0 …(ii)
Step 1. Consider the given inequations as strict equations.
x+y=4
Y' 2x − y = 0
Step 3. Plot the graph using the above table. Step 2. Find the points on the X-axis and Y-axis for
Step 4. Take a point ( 0, 0) and put it in the given inequation x+y=4
(i), we get x 0 4
0 − 0 ≤2 ⇒0 ≤2 y 4 0
which is true, so the shaded region will be towards
the origin. and 2x − y = 0
Thus, shaded region shows the inequality. x 0 1
Example 13. Solve the system of inequality graphically y 0 2
x + y ≤ 6, x + y ≥ 4.
Sol. The given system of inequalities
Y
x+y≤6 …(i)
x+y≥4 …(ii) (0,4)
4
Step 1. Consider the inequations as strict equations
3
i.e. x + y = 6 and x + y = 4
Step 2. Find the points on the X-axis and Y-axis for x + y = 6 2
1 (1,2)
x 0 6 (4,0)
y 6 0 X' X
O
x+

1 2 3 4
x+y=4
0

and
y=
y=

x 0 4
2x –

y 4 0
Y'
Y
(0,6)
Step 3. Plot the graph using the above tables.
Step 4. Take a point ( 0, 0) and put it in the inequation (i)
x+

0+ 0≥4 (False)
y=

(0,4)
So, the shaded region will be away from the origin.
6
x+

Take a point (1, 0) and put it in the inequation (ii)


y=

(6,0) 2 −0>0 (True)


4

X' X
O (4,0) So, the shaded region will be towards the origin.
Thus, common shaded region shows the solution of
the inequalities.
Y'
32 CBSE Term II Mathematics XI

Chapter
Practice
PART 1
Objective Questions
l
Multiple Choice Questions 9. Which of the following is the solution set of the
1. The solution set is x ( 5x − 2 ) (7 x − 3)
inequality < − ?
4 3 5
9
2 (a) ( 4, ∞ ) (b) ( −∞ , 4)
(c) [ 4, ∞ ) (d) ( −∞ , 4]
9  9 
(a) x ∈  , ∞ (b) x ∈  , ∞ 3x
2  2  10. The solution set of − 12 < 4 − ≤ 2 is
 9  9 −5
(c) x ∈− ∞ ,  (d) x ∈ −∞ ,  − 80 −10   80 10
 2  2  (a)  (b) −
 3 3 
, ,
 3 3 
2. The solution set of the inequality 4x + 3 < 6x + 7 is  − 80 − 10 
(c) , (d) None of these
(a) [ −2 , ∞ ) (b) ( −∞ , − 2 )  3 3 
(c) ( −2 , ∞ ) (d) None of these
11. If 3 ≤ 3 t − 18 ≤ 18, then which one of the following
3. If −3x + 17 < − 13, then is true?
(a) x ∈(10, ∞ ) (b) x ∈ [10, ∞ ) (a) 15 ≤ 2 t + 1 ≤ 20
(c) x ∈( − ∞ ,10] (d) x ∈[ −10,10) (b) 8 ≤ t < 12
4. The solution set of 5x − 3 < 7, where x is an (c) 8 ≤ t + 1 ≤ 13
integer is (d) 21 ≤ 3 t ≤ 24
(a) {……, − 1, 0, 1} (b) {……, − 3, − 2, − 1} 12. The solution set of 2 ≤ 3x − 4 ≤ 5 is
(c) ( − ∞ , 2 ) (d) None of these (a) [2, 3] (b) (2, 3)
5. If 3x + 8 > 2, then which of the following is true? (c) (2, ∞) (d) (− ∞, 3)
(a) x ∈ {−1, 0, 1, 2 , K}, when x is an integer 13. The solution set of the inequality 6 ≤ −3 (2 x − 4) < 12
(b) x ∈ [ −2 , ∞ ), when x is a real number is
(c) Both (a) and (b) (a) ( 0, 1] (b) [ 0, 1) (c) [ 0, 1] (d) ( 0, ∞ )
(d) None of the above 14. Consider the inequality 40x + 20y ≤ 120, where x
6. The solution set of 3(1 − x ) < 2 ( x + 4) is and y are whole numbers. Then, its solution set is
(a) [ − 1, ∞ ) (b) ( − 1, ∞ ) (a) (0, 0), (5, 5), (1, 1), (2, 2), (3, 0)
(c) ( − ∞ , − 1] (d) ( − ∞ , 1) (b) (0, 0), (0, 1), (0, 2), (0, 3), (0, 4), (0, 5), (0, 6)
7. The set of real x satisfying the inequality (c) (1, 0), (2, 0), (3, 0), (4, 0), (5, 0)
5 − 2x x (d) None of the above
≤ − 5 is
3 6 15. The length of a rectangle is three times the breadth.
(a) ( −∞ , 8) (b) ( 8, ∞ ) If the minimum perimeter of the rectangle is
(c) [ 8, ∞ ) (d) ( −∞ , 8] 160 cm, then
8. The solution set of 5x − 3 ≥ 3x − 5 is (a) breadth > 20 cm
(b) length < 20 cm
(a) ( − 1, ∞ ) (b) (1, ∞ )
(c) breadth ≥ 20 cm
(c) [ − 1, ∞ ) (d) None of these
(d) length ≤ 20 cm
CBSE Term II Mathematics XI 33

16. The marks obtained by a student of class XI in first 21. The solution set of the inequalities 2 x + y ≥ 4,
and second terminal examinations are 62 and 48, x + y ≤ 3 and 2 x − 3y ≤ 6, is
respectively. The minimum marks he should get in Y
the annual examination to have an average of atleast
60 marks, are
(a) 70 (b) 50 (c) 74 (d) 48
(a) X′ X
17. The number of pairs of consecutive odd natural O
numbers both of which are larger than 10, such that x+y
=6 =3
their sum is less than 40, is –3
y
(a) 8 (b) 6 (c) 4 (d) 3 2x Y′
2x + y = 4
18. In drilling world's deepest hole it was found that Y
the temperature T in degree Celcius, x km below
the earth's surface was given by T = 30 + 25 ( x − 3),
3 ≤ x ≤ 15. At what depth will the temperature be
(b) X′ X
between 155°C and 205°C? O
(a) 10 to 12 km (b) 8 to 10 km x+
y=
y =6 3
(c) 8 to 10 km (d) 15 to 18 km –3

2x
2x

+
19. The graphical solution of 3x − 6 ≥ 0 is

y=
Y′

4
Y Y Y

(a) X′ X (b) X′ X
O (2,0) O (2,0)
(c) X′ X
O
=6

x+
Y′ x =2 Y′ x =2 y

y=
–3
2x

3
Y′
Y Y 2x + y = 4

x+ Y
y=
(c) X′ X (d) X′ X
3
O (2,0) O (2,0)

(d) X′ X
O
Y′ x =2 Y′ x =2
6
y=
2x

20. The graphical solution of the system of linear x –3


+

2 Y′
y=

inequalities, 3x + 4y ≥ 12, y ≥ 1 and x ≥ 0, is


4

Y Y
l
Case Based MCQs
y=1 y=1 22. Shweta was teaching “method to solve a linear
(a) X′ (b) X′ X inequality in one variable” to her daughter.
X O
3x

O Step I Collect all terms involving the variable ( x) on


+4
y=

Y′ 3x + 4y = 12 Y′ one side and constant terms on other side with


12

the help of above rules and then reduce it in the


Y
Y form ax < b or ax ≤ b or ax > b or ax ≥ b.
y=1 Step II Divide this inequality by the coefficient of
y=1 variable ( x). This gives the solution set of given
(c) X′ X (d)X′ X inequality.
O O
3x

3x

Step III Write the solution set.


+4

+4
y=

y=

Y′ Y′ Based on above information, answer the following


12

12

questions.
34 CBSE Term II Mathematics XI

(i) The solution set of 24 x < 100, when x is a natural


number is PART 2
(a) {1, 2, 3, 4} (b) (1, 4)
(c) [1, 4] (d) None of these Subjective Questions
(ii) The solution set of 24 x < 100, when x is an integer is
(a) {…… − 4, − 3, − 2 , − 1, 0, 1, 2, 3, 4} l
Short Answer Type Questions
(b) ( − ∞ , 4]
(c) [ 4, ∞ ]
1. Find the solutions set of 3x − 5 < x + 7, where x is a
natural number.
(d) None of the above
5 − 3x x
(iii) The solution set of − 5 x + 25 > 0 is 2. Solve the inequality ≤ − 5.
(a) [ 5, ∞ ) (b) ( − ∞ , 5] (c) ( 5, ∞ ) (d) ( − ∞ , 5) 3 6
(iv) The solution set of 3 x − 5 < x + 7 is 7x
3. Solve the inequalities − 3 ≤ 4 − ≤ 18.
(a) (6, ∞) (b) [ 6, ∞ ) (c) ( − ∞ , 6) (d) ( − ∞ , 6] 2
x x 1 3  1
(v) The solution set of x + + < 11 is 4. Find the solution set of  x + 4 ≥ ( x − 6).
2 3 2 5  3
(a) ( − ∞ , 6] (b) ( − ∞ , 6)
(c) [6, ∞) (2 x − 1 ) ( 3x − 2 ) (2 − x )
(d) None of these 5. Find the solution set of ≤ − .
23. A manufacturing company produces certain 3 4 5
goods.The company manager used to make a data 3 (x − 2 )
6. Solve the linear inequalities − 15 < ≤ 0.
record on daily basis about the cost and revenue of 5
these goods separately. The cost and revenue 7. Solve the following system of inequalities 2 x − 3 < 7
function of a product are given by C( x ) = 20x + 4000 and 2 x > − 4. Also, represent the solution
and R( x ) = 60x + 2000, respectively, where x is the graphically on the number line.
number of goods produced and sold.
8. The sum of three consecutive integers must not be
Based on above information, answer the following more than 12. What are the integers?
questions.
9. A company manufactures cassettes. Its cost and
(i) How many goods must be sold to realise some revenue functions are C( x ) = 26000 + 30x and
profit?
R( x ) = 43x, respectively, where x is the number of
(a) x < 50 (b) x > 50 (c) x ≥ 50 (d) x ≤ 50
cassettes produced and sold in a week. How many
(ii) If the cost and revenue functions of a product are cassettes must be sold by the company to realise
given by C( x) = 3 x + 400 and R( x) = 5 x + 20 some profit?
respectively, where x is the number of items
produced by the manufacturer, then how many 10. The water acidity in a pool is considered normal,
items must be sold to realise some profit? when the average pH reading of three daily
(a) x ≤ 190 (b) x ≥ 190 (c) x < 190 (d) x > 190 measurements is between 8.2 and 8.5. If the first
two pH readings are 8.48 and 8.35, then find the
(iii) Let x and b are real numbers. If b > 0 and x < b,
then
range of pH value for the third reading that will
(a) x is always positive (b) x is always negative
result in the acidity level being normal.
(c) x is real number (d) None of these 11. In the first four examinations, each of 100 marks,
(iv) The solution set of 3 x − 5 < x + 7, when x is a whole Mohan got 94, 73, 72 and 84 marks. If a final average
number is given by greater than or equal to 80 and less than 90 is needed
(a) {0, 1, 2, 3, 4, 5} (b) ( − ∞ , 6) to obtain a final grade B in a course, then what
(c) [0, 5] (d) None of these range of marks in the fifth (last) examination will
result, if Mohan receiving grade B in the course?
(v) Graph of inequality x > 2 on the number line is
represented by 12. Find all pairs of consecutive even positive integers,
both of which are larger than 5, such that their sum
(a) – ∞ +∞
–2 –1 0 1 2 3 4 is less than 23.
(b) – ∞ +∞ 13. A solution is to be kept between 68°F and 77°F.
–2 –1 0 1 2 3 4 What is the range of temperature in degree Celsius
(C), if the Celsius/Fahrenheit (F) conversion
(c) – ∞ +∞
–2 –1 0 1 2 3 4 9
formula is given by F = C + 32 ?
(d) None of the above 5
CBSE Term II Mathematics XI 35

14. The longest side of a triangle is 3 times the shortest l


Long Answer Type Questions
side and the third side is 2 cm shorter than the longest 26. Solve the following system of inequalities and
side. If the perimeter of the triangle is atleast 61 represent the solution graphically on the number line.
cm, find the minimum length of the shortest side.
5(2 x − 7 ) − 3(2 x + 3) ≤ 0, 2 x + 19 ≤ 6x + 47
15. The longest side of a triangle is twice the shortest
27. Solve the following system of linear inequalities and
side and the third side is 2 cm longer than the
represent the solution graphically on the number line.
shortest side. If the perimeter of the triangle is
2x − 3 4x
more than 166 cm, then find the minimum length of 2(2 x + 3) − 10 < 6 ( x − 2 ) and + 6 ≥2 +
the shortest side. 4 3
16. A man wants to cut three lengths from a single 28. A solution of 9% acid is to be diluted by adding
piece of board of length 91 cm, the second length is 3% acid solution to it. The resulting mixture is to
to be 3 cm longer than the shortest and third length be more than 5% but less than 7% acid. If there is
is to be twice as long as the shortest. What are the 460 L of the 9% solution, how many litres of 3%
possible lengths for the shortest board, if third solution will have to be added?
piece is to be atleast 5 cm longer than the second? 29. A solution of 8% boric acid is to be diluted by
17. A manufacturer has 600 L of 12% solution of acid. adding 2% boric acid solution to it. The resulting
The volume of 30% acid solution must be added to mixture is to be more than 4% but less than 6% boric
it, so that acid content in the resulting mixture will acid. If we have 640 L of the 8% solution, how many
be more than 15% but less than 18%, is between litres of the 2% solution will have to be added?
…X… and …Y… . Here, X and Y refer to 30. Solve the inequalities graphically
MA 3x + 4y ≤ 60, x + 3y ≤ 30, x ≥ 0 and y ≥ 0.
18. IQ of a person is given by the formula IQ = × 100
CA 31. Find the linear inequalities for which the shaded
where, MA is the mental age and CA is chronical region in the given figure is the solution set.
age. If 80 ≤ IQ ≤ 140 for a group of 12 yr old children, Y
find the range of their mental age. (0, 8)

x=5
19. Ravi obtained 70 and 75 marks in first two unit
tests. Find the maximum marks, he should get in y=5
the third test to have an average of atleast 60 marks. (0, 4)
x+
y=
20. To receive grade A in a course, one must obtain an 8
average of 90 marks or more in five examinations X′ O X
(4, 0) x + (8, 0)
(each of 100 marks.) If Sunita’s marks in first four y=
examinations are 87, 92, 94 and 95, find minimum Y′ 4
marks that Sunita must obtain in fifth examination
to get grade A in the course. 32. Solve the following system of linear inequalities
21. Find the linear inequalities for which the shaded 3x + 2 y ≥ 24, 3x + y ≤ 15 and x ≥ 4.
region in the given figure is the solution set. 33. Show that the following system of linear
Y inequalities has no solution.
x + 2 y ≤ 3, 3x + 4y ≥ 12 , x ≥ 0 and y ≥ 1.
(0, 24) 34. The graphical solution of the inequalities
(0, 20) x + 2 y ≤ 10,
x+
y= x + y ≥ 1,
20 x − y ≤ 0,
X
O (16, 0) 3x+ (20, 0) x≥0
2y
=
48 and y≥0
35. Solve the system of inequalities graphically.
22. Solve the inequality 2 x + y > 3 graphically. x + y ≤ 5,
23. Solve the inequality 5x + 2 y ≤ 10 graphically. 4x + y ≥ 4,
24. Draw the graph of the inequality y + 8 ≥ 2 x. x + 5y ≥ 5,
x≤4
25. Solve 4x − y > 0 graphically. and y≤3
36 CBSE Term II Mathematics XI

SOLUTIONS
Objective Questions 7. (c) We have,
9 5 − 2x x
1. (b) The given graph represents all the values greater than , ≤ −5
2 3 6
9 or 2( 5 − 2 x ) ≤ x − 30
including on the real line.
2 or 10 − 4x ≤ x − 30
9 
x ∈ , ∞ or −5x ≤ − 40,
2 
i.e. x≥8
2. (c) We have, 4x + 3 < 6x + 7
Thus, all real numbers x which are greater than or equal to
or 4x − 6x < 6x + 4 − 6x 8, are the solutions of the given inequality i.e. x ∈ [ 8, ∞ ).
or −2 x < 4 or x > − 2 8. (c) We have, 5x − 3 ≥ 3x − 5
i.e. all the real numbers which are greater than −2, are the
Transferring the term 3x to LHS and the term ( − 3) to RHS,
solutions of the given inequality. Hence, the solution set is
( −2 , ∞ ). 5x − 3x ≥ − 5 + 3
3. (a) Given that, − 3x + 17 < − 13 ⇒ 2x ≥ − 2
2x 2
⇒ 3x − 17 > 13 [multiplying by −1 both sides] ⇒ ≥−
⇒ 3x > 13 + 17 [adding 17 both sides] 2 2
−2
⇒ 3x > 30 ⇒ x≥
∴ x > 10 2
4. (a) We have, 5x − 3 < 7 ⇒ x ≥ −1

On adding 3 both sides, we get –∞ –1 0 1 +∞


5x − 3 + 3 < 7 + 3
All the numbers on the right side of − 1 will be greater
⇒ 5x < 10 than it.
On dividing both sides by 5, we get ∴ Solution set is [ −1, ∞ ).
5x 10
< x ( 5 x − 2 ) (7 x − 3 )
5 5 9. (a) We have, < −
4 3 5
⇒ x <2
⇒ 15x < 16x − 4
∴ When x is an integer, the solution of the given inequality is
Transferring the term 16x to LHS.
{……, − 1, 0, 1}.
15x − 16x < − 4 ⇒ − x < − 4
5. (a) We have, 3x + 8 > 2
On multiplying by − 1 both sides, we get
On adding −8 both sides, x>4
⇒ 3x > − 6
On dividing by 3 both sides,
⇒ x > −2 –∞ 4 5 6 ∞
(i) When x is an integer, the solution of the given
inequality is {− 1, 0, 1, 2 ....}. ∴ Solution set is ( 4, ∞ ).
(ii) When x is a real number, the solution of the given 10. (a) The given inequality,
inequality is ( − 2 , ∞ ). i.e. all the numbers lying between 3x 3x
− 2 and ∞ but − 2 and ∞ are not included. − 12 < 4 − ≤ 2 ⇒ − 12 < 4 + ≤2
−5 5
6. (b) Again, we have, 3(1 − x ) < 2 ( x + 4) Adding ( − 4) to each term,
⇒ 3 − 3x < 2 x + 8 3x 3x
− 12 − 4 < 4 + − 4 ≤ 2 − 4 ⇒ − 16 < ≤ −2
Transferring the term 2x to the LHS and the term 3 to RHS, 5 5
− 3x − 2 x < 8 − 3 5
⇒ − 5x < 5 Multiplying by to each term,
3
− 5x 5
⇒ > 5 3x 5 5
− 5 −5 − 16 × < × ≤ −2 ×
3 5 3 3
−5
⇒ x> ⇒ x > −1 ⇒ −
80
<x≤−
10
5 3 3
–∞ –1 0 1 +∞  80 10   80 −10 
∴ Solution set is  − ,− or − , .
 3 3   3 3 
∴ Solution set is ( − 1, ∞ ).
CBSE Term II Mathematics XI 37

11. (c) Given, 3 ≤ 3 t − 18 ≤ 18 16. (a) Let x be the marks obtained by student in the annual
examination. Then,
Adding 18 to each term,
62 + 48 + x
3 + 18 ≤ 3 t − 18 + 18 ≤ 18 + 18 ≥ 60
3
⇒ 21 ≤ 3 t ≤ 36
or 110 + x ≥ 180
Dividing by 3 to each term,
or x ≥ 70
21 3 t 36
≤ ≤ Thus, the student must obtain a minimum of 70 marks to get
3 3 3
an average of atleast 60 marks.
⇒ 7 ≤ t ≤ 12
17. (c) Let x be the smaller of the two consecutive odd natural
Adding 1 to each term, number, so that the other one is x + 2. Then, we should have
7 + 1 ≤ t + 1 ≤ 12 + 1 x > 10 …(i)
⇒ 8 ≤ t + 1 ≤ 13 and x + ( x + 2 ) < 40 …(ii)
12. (a) The given inequality, 2 ≤ 3x − 4 ≤ 5 Solving Eq. (ii), we get
⇒ 2 + 4 ≤ 3x ≤ 5 + 4 2 x + 2 < 40
⇒ 6 ≤ 3x ≤ 9 i.e. x < 19 …(iii)
Dividing by 3 in each term, From Eqs. (i) and (iii), we get
6 3x 9
≤ ≤ 10 < x < 19
3 3 3 Since, x is an odd number, x can take the values 11, 13, 15,
⇒ 2≤x≤3 and 17. So, the required possible pairs will be
∴ Solution set is [2 , 3]. (11, 13), (13, 15), (15, 17), (17, 19)
13. (a) We have, 18. (b) Given that, T = 30 + 25 ( x − 3), 3 ≤ x ≤ 15
6 ≤ − 3(2 x − 4) < 12 or 6 ≤ − 6x + 12 < 12 According to the question, 155 < T < 205
On subtracting 12 from each term, we get ⇒ 155 < 30 + 25 ( x − 3) < 205
6 − 12 ≤ − 6x + 12 − 12 < 12 − 12 ⇒ 8 < x < 10
⇒ − 6 ≤ − 6x < 0 Hence, at the depth 8 to10 km temperature lies between
On dividing each term by − 6, we get 155°C to 205°C.
− 6 − 6x 0 19. (a) Graph of 3x − 6 = 0 is given in the figure.
≥ > Y
−6 −6 −6
[while dividing each term by the same negative number,
then sign of inequalities will get change] 4
x=2

⇒ 1≥x>0 3 II

which can be written as 0 < x ≤ 1. 2


Hence, solution set of given system of inequations is (0, 1]. 1 I
X′ X
14. (b) Given, 40x + 20y ≤ 120, x and y are whole numbers. O 1 2 3 4 5
To start with, let x = 0. Then, LHS of given inequality is
Y′
40x + 20y = 40 ( 0) + 20y = 20y
Thus, we have We select a point say (0, 0) and substituting it in given
20y ≤ 120 inequality, we see that
or y≤6 3 ( 0) − 6 ≥ 0
For x = 0, the corresponding values of y can be 0, 1, 2, 3, 4, or − 6 ≥ 0 which is false.
5, 6 only. In this case, the solutions of given inequality are Thus, the solution region is the shaded region on the right
(0, 0), (0, 1), (0, 2), (0, 3), (0, 4), (0, 5) and (0, 6). hand side of the line x = 2.
15. (c) Let breadth of rectangle be x cm. Also, all the points on the line 3x − 6 = 0 will be included in
the solution. Hence, a dark line is drawn in the solution
∴ Length of rectangle = 3x region.
Perimeter of rectangle = 2 (Length + Breadth) 20. (c) Given inequalities are
= 2 ( x + 3x ) = 8x 3x + 4y ≥ 12, …(i)
Given, perimeter ≥ 160 cm y ≥1 …(ii)
8x ≥ 160 and x≥0 …(iii)
Dividing both sides by 8, The line corresponding to (i) is
x ≥ 20 cm 3x + 4y = 12 …(iv)
38 CBSE Term II Mathematics XI

Table for 3x + 4y = 12 Y

x 4 0
y 0 3 (0,4)

Draw the graph of the line 3x + 4y = 12.


Q 3 ( 0) + 4( 0) ≥ 12 i.e. 0 ≥ 12, which is not true. (0,3)
∴Inequality (i) represent the half plane made by the line (iv),
which does not contain origin. X' X
(2,0) (3,0)
Inequality y ≥ 1 represent the half plane made by the line
–2) x+
y = 1, which does not contain origin. (Q 0 ≥ 1 is not true) (0, y=
3

2x
Y
=6

+y
y
–3
2x

=4
Y'
(0, 3)
Thus, common shaded region shows the solution of the
inequalities.
O y =1
22. (i) (a) We have, 24x < 100
X′ X
(4, 0) On dividing both sides by 24, we get
24x 100 25
3x + 4y = 12 = < ⇒x <
24 24 6
Y′
When x is a natural number, then solutions of the
Inequality x ≥ 0 represent the right side of the Y-axis. 25
inequality x < , are all natural numbers, which are
Hence, the shaded part is common to all the given 6
inequalities. 25
less than . In this case, the following values of x make
21. (d) The given system of inequalities 6
the statement true.
2x + y ≥ 4 …(i)
x = 1, 2 , 3, 4
x+y≤3 …(ii)
Hence, the solution set of inequality is {1, 2 , 3, 4}.
2 x − 3y ≤ 6 …(iii)
(ii) (a) We have, 24x < 100
Step I Consider the inequations as strict equations i.e. On dividing both sides by 24, we get
2x + y = 4 24x 100 25
x+y=3 = < ⇒x<
2 x − 3y = 6 24 24 6
When x is an integer.
Step II Find the points on the X-axis and Y-axis for,
In this case, solutions of given inequality are
2x + y = 4 K, − 4, − 3, − 2 , − 1, 0, 1, 2 , 3, 4
x 0 2 Hence, the solution set of inequality is
4 0 {K , − 4, − 3, − 2 , − 1, 0, 1, 2 , 3, 4}
y
(iii) (d) We have, − 5x + 25 > 0
x+y=3 On adding 5x both sides, we get
x 0 3 − 5x + 25 + 5x > 0 + 5x ⇒ 25 > 5x ⇒ 5x < 25
y 3 0 On dividing both sides by 5, we get
5x 25
and 2 x − 3y = 6 ⇒ < ⇒x < 5
5 5
x 0 3 Hence, the required solution set is (− ∞, 5).
y –2 0 (iv) (c) We have, 3x − 5 < x + 7
Step III Plot the graph using the above tables. ⇒ 3x − 5 + 5 < x + 7 + 5 [adding 5 both sides]
⇒ 3x < x + 12
Step IV Take a point (0, 0) and put it in the inequations (i), ⇒ 3x − x < x + 12 − x
(ii) and (iii), we get [subtracting x from both sides]
0+ 0≥4 (false)
⇒ 2 x < 12
So, the shaded region will be away from origin. 2 x 12
0+ 0≤3 (true) ⇒ < [dividing both sides by 2]
2 2
So, the shaded region will be towards origin. ⇒ x<6
0−0≤6 (true) The solution set is {x : x ∈ R and x < 6} i.e. any real
So, the shaded region will be towards origin. number less than 6. This can also be written as (− ∞, 6).
CBSE Term II Mathematics XI 39

x x Subjective Questions
(v) (b) We have, x + + < 11
2 3 1. We have, 3x − 5 < x + 7
6x + 3x + 2 x 11x ⇒ 3x − 5 + 5 < x + 7 + 5
⇒ < 11 ⇒ < 11 [adding 5 both sides]
6 6 ⇒ 3x < x + 12
6
On multiplying both sides by , we get ⇒ 3x − x < x + 12 − x [subtracting x from both sides]
11
⇒ 2 x < 12
11x 6 6
× < 11 × ⇒ x<6 2 x 12
6 11 11 ⇒ < [dividing both sides by 2]
2 2
∴ x ∈ ( − ∞ , 6)
⇒ x<6
23. (i) (b) We know that, Profit = Revenue − Cost
Now, if x is a natural number, then the solution set is
∴In order to realise some profit, revenue should be {1, 2, 3, 4, 5}.
greater than the cost. 5 − 3x x
Thus, we should have R ( x ) > C( x ) 2. We have, ≤ −5
3 6
⇒ 60x + 2000 > 20x + 4000 5 − 3x x − 30
⇒ ≤
⇒ 60x + 2000 − 20x > 20x + 4000 − 20x 3 6
[subtracting 20x from both sides] 5 − 3x x − 30
⇒ ×6≤ ×6
⇒ 40x + 2000 > 4000 3 6
⇒ 40x + 2000 − 2000 > 4000 − 2000 [multiplying both sides by 6]
[subtracting 2000 from both sides] ⇒ 2( 5 − 3x ) ≤ x − 30
⇒ 40x > 2000 ⇒ 10 − 6x ≤ x − 30
40x 2000 ⇒ 10 − 6x − 10 ≤ x − 30 − 10
⇒ > [subtracting 10 from both sides]
40 40
[dividing both sides by 40] ⇒ − 6x ≤ x − 40
⇒ − 6x − x ≤ x − 40 − x
⇒ x > 50
[subtracting x from both sides]
Hence, the manufacturer must sell more than 50 items
to realise some profit. ⇒ − 7 x ≤ − 40
(ii) (d) In order to realise some profit, we should have ⇒ 7 x ≥ 40 [multiplying by − 1 both sides]
7 x 40
R ( x ) > C( x ) ⇒ ≥ [dividing by 7 both sides]
⇒ 5x + 20 > 3x + 400 7 7
40
⇒ 5x + 20 − 3x > 3x + 400 − 3x ⇒ x≥
7
⇒ 2 x + 20 > 400
 40 
⇒ 2 x + 20 − 20 > 400 − 20 i.e. x∈ , ∞
 7 
⇒ 2 x > 380
 40 
⇒ x > 190 Hence, the required solution set is , ∞ .
 7 
(iii) (d) We have, b > 0 7x
and x<b 3. We have, − 3 ≤ 4 − ≤ 18
2
Its mean x is always less than some positive quantity.
On subtracting 4 from each term, we get
∴ x may be a real number. 7x 7x
(iv) (a) We have, 3x − 5 < x + 7 −3−4≤4− − 4 ≤ 18 − 4 ⇒ − 7 ≤ − ≤ 14
2 2
⇒ 3x − 5 + 5 < x + 7 + 5 [adding 5 both sides]  −2 
⇒ 3x < x + 12 On multiplying each term by   , we get
 7 
⇒ 3x − x < x + 12 − x
 2  −7  −2   −2 
[subtracting x from both sides] −7  −  ≥ x ×   ≥ 14 ×  
⇒ 2 x < 12  7 2  7   7 
2 x 12 [while multiplying each term by the same negative number,
⇒ < [dividing both sides by 2]
2 2 then the sign of inequalities will get change]
⇒ x<6 ⇒ 2 ≥ x ≥ − 4 or − 4 ≤ x ≤ 2 or x ∈ [ − 4, 2 ]
Now, if x is a whole number, then the solution set Hence, solution set of given system of inequations is [ − 4, 2 ].
{0, 1, 2, 3, 4, 5}. 13  1
(v) (b) Graph of x ≥ 2 on the number line is represented by 4. We have,  x + 4 ≥ ( x − 6)
2 5  3
–∞ +∞ 1  3x + 20 1
–3 –2 –1 0 1 2 3 4 ⇒   ≥ ( x − 6)
2 5  3
40 CBSE Term II Mathematics XI

1 1 7. We have the following inequalities,


⇒ ( 3x + 20) ≥ ( x − 6)
10 3 2x − 3 < 7 …(i)
30 30 2x > − 4
⇒ ( 3x + 20) ≥ ( x − 6) and …(ii)
10 3 From inequality (i), we have
[multiplying both sides by LCM (10, 3) = 30] 2x − 3 < 7
⇒ 3( 3x + 20) ≥ 10( x − 6) ⇒ 2x − 3 + 3 < 7 + 3 [adding 3 both sides]
⇒ 9x + 60 ≥ 10x − 60 ⇒ 2 x < 10
⇒ 9x + 60 − 60 ≥ 10x − 60 − 60 2 x 10
⇒ < [dividing both sides by 2]
[subtracting 60 from both sides] 2 2
⇒ 9x ≥ 10x − 120 ⇒ x<5
⇒ 9x − 10x ≥ 10x − 120 − 10x ∴The solution set is (− ∞, 5).
[subtracting 10x from both sides] From inequality (ii), we have
⇒ − x ≥ − 120 ⇒ x ≤ 120 2x > − 4
[multiplying both sides by − 1] 2x − 4
> [dividing both sides by 2]
∴ x ∈ ( − ∞ , 120] 2 2
2 x − 1 3x − 2 2 − x ⇒ x > −2
5. We have, ≤ −
3 4 5 ∴ The solution set is (− 2, ∞).
On multiplying both sides by LCM of 3, 4 and 5 i.e. 60, Now, let us draw the graphs of the solutions of both
we get inequalities on number line.
2x − 1 3x − 2 2−x
× 60 ≤ × 60 − × 60 –∞ ∞
3 4 5 –2 1 0 1 2 3 4 5
⇒ 20(2 x − 1) ≤ 15( 3x − 2 ) − 12(2 − x ) –∞ ∞
–2 –1 0 1 2 3 4 5
⇒ 40x − 20 ≤ 45x − 30 − 24 + 12 x
⇒ 40x − 20 ≤ 57 x − 54 It can be seen that the values of x, which are common to
both are lying in the interval (− 2, 5).
⇒ 40x − 20 + 20 ≤ 57 x − 54 + 20 [adding 20 both sides]
Hence, the solution set of given system of inequations is
⇒ 40x ≤ 57 x − 34 (− 2, 5) and this can be represented graphically on the
⇒ 40x − 57 x ≤ 57 x − 34 − 57 x number line as
[subtracting 57x from both sides]
–∞ –2 0 5 ∞
⇒ − 17 x ≤ − 34
−17 x −34 8. Let the three consecutive integers be x, ( x + 1) and ( x + 2 ).
⇒ ≤ [dividing both sides by 17]
17 17 Since, sum of these integers must not be more than 12, i.e.
⇒ − x ≤ −2 x + ( x + 1) + ( x + 2 ) cannot exceed 12, therefore, we have
⇒ x ≥2 [multiplying both sides by –1] ⇒ x + ( x + 1) + ( x + 2 ) ≤ 12
Thus, the solution set is [2 , ∞ ). ⇒ 3x + 3 ≤ 12
6. We have, ⇒ 3x + 3 − 3 ≤ 12 − 3
3( x − 2 ) [subtracting 3 from both sides]
− 15 < ≤0
5 ⇒ 3x ≤ 9
3( x − 2 ) ∴ x≤3 [dividing both sides by 3]
⇒ −15 × 5 < ×5≤0×5
5 9. Cost function, C( x ) = 26000 + 30x
[multiplying each term by 5]
and revenue function, R ( x ) = 43x
⇒ −75 < 3( x − 2 ) ≤ 0
For profit, R ( x ) > C( x )
−75 3( x − 2 ) 0
⇒ < ≤ [dividing each term by 3] ⇒ 26000 + 30x < 43x
3 3 3
⇒ 30x − 43x < − 26000
⇒ − 25 < x − 2 ≤ 0
⇒ − 13x < − 26000
⇒ − 25 + 2 < x − 2 + 2 ≤ 0 + 2
⇒ 13x > 26000
[adding 2 each term]
26000
⇒ − 23 < x ≤ 2 ⇒ x>
13
∴ x ∈ ( − 23, 2 ]
∴ x > 2000
Hence, solution set of the given system of inequations is Hence, more than 2000 cassettes must be produced to get
( − 23, 2 ]. profit.
CBSE Term II Mathematics XI 41

10. Given, first pH value = 8. 48 ⇒


9
68 − 32 < C < 77 − 32
and second pH value = 8. 35 5
Let third pH value be x. [subtracting 32 from each term]
9
Since, it is given that average pH value lies between 8.2 ⇒ 36 < C < 45
and 8.5. 5
8. 48 + 8. 35 + x 5 5  5 
∴ 8. 2 < < 8. 5 ⇒ 36 × < C < 45 × multiplying in each term
3 9 9  9 
16. 83 + x ⇒ 4×5<C<5×5
⇒ 8. 2 < < 8. 5
3 ⇒ 20 < C < 25
⇒ 3 × 8. 2 < 16. 83 + x < 8. 5 × 3 Hence, the required range of temperature is between 20°C
and 25°C.
⇒ 24. 6 < 16. 83 + x < 25. 5 14. Let the shortest side be x cm. Then, according to the given
⇒ 24. 6 − 16. 83 < x < 25. 5 − 16. 83 condition,
⇒ 7.77 < x < 8. 67 Longest side = 3x cm and third side = ( 3x − 2 ) cm
Thus, third pH value lies between 7.77 and 8.67. Now, perimeter of triangle ≥ 61 i.e. sum of all sides ≥ 61
11. Let x be the score obtained by Mohan in the last ⇒ x + 3x + 3x − 2 ≥ 61
examination. ⇒ 2 + 7 x − 2 ≥ 61 + 2 (using rule 1)
94 + 73 + 72 + 84 + x ⇒ 7 x ≥ 63
Then, 80 ≤ < 90
5 7 x 63
323 + x ⇒ ≥ (using rule 2)
⇒ 80 ≤ < 90 7 7
5
⇒ x≥9
⇒ 400 ≤ 323 + x < 450
∴ Minimum length of the shortest side = 9 cm
[multiplying both sides by 5] 15. Let the length of shortest side be x cm.
⇒ 400 − 323 ≤ 323 + x − 323 < 450 − 323 According to the given information,
[subtracting 323 from each term] Longest side = 2 × Shortest side
⇒ 77 ≤ x < 127 = 2x cm
Since, the upper limit is 100, therefore the required range is and third side = 2 + Shortest side
77 ≤ x ≤ 100.
= (2 + x ) cm
12. Let x be the smaller of the two positive even integers, so that
the other one is x + 2, then we should have Perimeter of triangle = x + 2 x + ( x + 2 ) = 4x + 2
x>5 …(i) According to the question,
x+2 > 5 …(ii) Perimeter > 166 cm
and their sum, x + ( x + 2 ) < 23 …(iii) ⇒ 4x + 2 > 166
From inequalities (i) and (ii), we get ⇒ 4x > 166 − 2
x>3 …(iv) ⇒ 4x > 164
164
From inequality (iii), we get ∴ x> = 41 cm
4
2 x + 2 < 23
Hence, the minimum length of shortest side be 41cm.
⇒ 2 x + 2 − 2 < 23 − 2
16. Let the length of the shortest piece be x cm, so that the
[subtracting 2 from both sides]
lengths of second and third piece are ( x + 3) cm and 2x cm,
⇒ 2 x < 21 respectively.
⇒ x < 10. 5[dividing both sides by 2] …(v) Then, x + ( x + 3) + 2 x ≤ 91 …(i)
Now, from inequalities (iv) and (v), we get
and 2 x ≥ ( x + 3) + 5 …(ii)
3 < x < 10.5
From inequality (i), we get
Since, x is an even number. So, x can take the values 4, 6, 8
and 10. 4x + 3 ≤ 91
Hence, the required possible pairs will be ( 4, 6) ( 6, 8), ( 8, 10) ⇒ 4x + 3 − 3 ≤ 91 − 3
and (10, 12 ). [subtracting 3 from both sides]
13. It is given that, 68 < F < 77 ⇒ 4x ≤ 88
9 4x 88
On putting F = C + 32, we get ⇒ ≤ [dividing both sides by 4]
5 4 4
9 ⇒ x ≤ 22
68 < C + 32 < 77 …(iii)
5
42 CBSE Term II Mathematics XI

From inequality (ii), we get ⇒ 145 + x ≥ 60 × 3


2x ≥ x + 8 ⇒ 145 + x ≥ 180
⇒ 2x − x ≥ x + 8 − x Now, transferring the term 145 to RHS,
[subtracting x from both sides] ⇒ x ≥ 180 − 145 ⇒ x ≥ 35
⇒ x≥8 …(iv) i.e. Ravi should get greater than or equal to 35 marks in
third unit test to get an average of atleast 60 marks.
From inequalities (iii) and (iv), we get
20. Let Sunita got x marks in the fifth exam.
8 ≤ x ≤ 22
∴ Average marks obtained by Sunita
Hence, the shortest piece must be atleast 8 cm long but not Sum of marks in all exams
more than 22 cm long. =
Number of exams
17. Let x L of 30% acid solution is required to be added. Then, 87 + 92 + 94 + 95 + x
Total mixture = ( x + 600) L =
5
Therefore, 30% x + 12% of 600 > 15% of ( x + 600) 368 + x
=
and 30% x + 12% of 600 < 18% of ( x + 600) 5
30x 12 15 Now, it is given that Sunita wants to obtain grade A for that
or + ( 600) > ( x + 600)
100 100 100 her average marks should be greater than or equal to 90.
30x 12 18 368 + x
and + ( 600) < ( x + 600) i.e. ≥ 90
100 100 100 5
or 30x + 7200 > 15x + 9000 ⇒ 368 + x ≥ 450
and 30x + 7200 < 18x + 10800 Transferring the term 368 to RHS,
or 15x > 1800 and 12 x < 3600 ⇒ x ≥ 450 − 368
or x > 120 and x < 300 ⇒ x ≥ 82
i.e. 120 < x < 300 i.e. Sunita should got greater than or equal to 82 marks in
fifth exam to get grade A.
∴ X = 120 and Y = 300
21. Consider the line 3x + 2 y = 48, we observe that the shaded
18. Given that, 80 ≤ IQ ≤ 140 …(i)
region and the origin are on the same side of the line.
MA
Putting IQ = × 100 in Eq. (i) 3x + 2 y = 48 and (0, 0) satisfy the linear constraint
CA 3x + 2 y ≤ 48. So, we must have one inequation as
⇒ 80 ≤
MA
× 100 ≤ 140 3x + 2 y ≤ 48.
CA Now, consider the line x + y = 20. We find that the shaded
MA region and the origin are on the same side of the line
⇒ 80 ≤ × 100 ≤ 140 (Q given that, CA = 12 yr)
12 x + y = 20 and (0, 0) satisfy the constraints x + y ≤ 20 . So,
Multiplying by 12 in each term, the second inequation is x + y ≤ 20.
MA We also notice that the shaded region is above X-axis and is
⇒ 80 × 12 ≤ × 100 × 12 ≤ 140 × 12
12 on the right side of Y-axis, so we must have x ≥ 0, y ≥ 0.
⇒ 960 ≤ MA × 100 ≤ 1680 Thus, the linear inequations corresponding to the given
Dividing by 100 in each term, solution set are 3x + 2 y ≤ 48, x + y ≤ 20 and x ≥ 0, y ≥ 0.
960 MA × 100 1680 22. We have, 2x + y > 3
⇒ ≤ ≤
100 100 100 In equation form, given inequality can be written as
⇒ 9. 6 ≤ MA ≤ 16. 8 ⇒ MA ∈[ 9. 6, 16. 8] 2x + y = 3 ...(i)
Hence, mental age should be greater than equal to 9.6 but On putting x = 0 in Eq. (i), we get
less than equal to 16.8.
2( 0) + y = 3 ⇒ y = 3
19. Let Ravi got x marks in third unit test.
∴ Line meets Y-axis at point A ( 0, 3).
∴ Average marks obtained by Ravi
Sum of marks in all tests On putting y = 0 in Eq. (i), we get
=
Number of tests 3
70 + 75 + x 145 + x 2x + 0 = 3 ⇒ x =
= = 2
3 3 3 
∴ Line meets X-axis at point B  , 0 .
Now, it is given that he wants to obtain an average of atleast 2 
60 marks.
3 
Atleast 60 marks means that the marks should be greater On joining the points A( 0, 3) and B  , 0 by dotted line, we
than or equal to 60. 2 
145 + x get the line AB. [Q given inequality has sign >, so we
i.e. ≥ 60 join the points by a dotted line]
3
CBSE Term II Mathematics XI 43

Now, take a point not lying on the line say (0, 0) to check 24. Given, inequality is y + 8 ≥ 2 x.
whether it satisfies the given inequality or not. In equation form, it can be written as
On putting x = 0 and y = 0 in given inequality, we get y + 8 = 2 x or 2 x − y = 8
2( 0) + 0 > 3 ⇒ 0 > 3 which is not correct. On putting y = 0, we get
So, we shade the portion which does not contain (0, 0) i.e. 2x − 0 = 8 ⇒ x = 4
the region above the dotted line AB. ∴ Line 2 x − y = 8 cuts the X-axis at A (4, 0).
Y
On putting x = 0, we get
2 × 0 − y = 8⇒y = − 8
∴ Line 2 x − y = 8 cuts the Y-axis at B(0, −8).
On joining the points A ( 4, 0) and B( 0, − 8) by a dark line,
3 A (0,3) we get the line AB.
[Q given inequality has sign ≥, so we draw dark line]
2 Let us check, whether O (0, 0) satisfy the inequality or not
2x

and shade the suitable region.


+

( 32 ,0 )
y=

1
On putting x = y = 0 in y + 8 ≥ 2 x, we get
3

X′ X 0 + 8 ≥2⋅0
0 1 B 2 3
⇒ 8 ≥ 0, which is true.
Y′
So, shade the region containing ( 0, 0) i.e. the region above
the line AB.
Here, the shaded region is the required solution region. Y
23. Given, inequality is 5x + 2 y ≤ 10.
O (4, 0)
Corresponding equation of line is X′ X
(0, 0) A
5x + 2 y = 10
y + 8 = 2x
On putting x = 0, we get
5 ( 0) + 2 y = 10 ⇒ y = 5 B (0, –8)
Thus, the line intersect the Y-axis at point ( 0, 5). Y′
On putting y = 0, we get
Here, the shaded region including the points on the line
5x + 2 ( 0) = 10 ⇒ x = 2 represent the graph of the inequality.
x 0 2 25. We have, 4x − y > 0
y 5 0 Its equation form is 4x − y = 0. ...(i)
On putting y = 0 in Eq. (i), we get
Thus, the line intersect the X-axis at point (2 , 0).
4x − 0 = 0 ⇒ x = 0
Join the points A ( 0, 5) and B (2 , 0) with a dark line.
On putting x = 1 in Eq. (i), we get
[Q given inequality has sign ‘≤’] 4(1) − y = 0 ⇒ y = 4
Y
x 1 0
A
5 (0,5) y 4 0
4
Thus, the line 4x − y = 0 passes through the points A ( 0, 0)
5x+

3 and B (1, 4)
2y=

2 Since, the inequality is of the form <, so we join the points


10

1 A( 0, 0) and B(1, 4) by a dotted line.


(2,0)
X′ X Y
0 1 2 B
0
y=

Y′ 4 (1, 4)
4x –

3
On putting (0, 0) in given inequality, we get
2 (1, 2)
5 × 0 + 2 × 0 ≤ 10 ⇒ 0 ≤ 10, which is true.
So, the half plane of 5x + 2 y ≤ 10 contains origin. X′
1
X
1 2
Now, shade the half plane which contain ( 0, 0).
Here, the shaded region including the points on the line Y′
represent the solution region of given inequality.
44 CBSE Term II Mathematics XI

Now, take the point, say (1, 2) not lying on the line, to check ⇒ 4x − 6x < 6x − 8 − 6x
whether it satisfies the given linear inequality or not. [subtracting 6x from both sides]
At (1, 2 ), 4(1) − 2 > 0 ⇒ 2 > 0, which is true. ⇒ −2 x < −8
∴ (1, 2) satisfies the given inequality, so we shade the region ⇒ 2 x > 8 [dividing both sides by −1, then
which contains (1, 2). Thus, shaded region (excluding all inequality sign will also change]
points on the line) gives the solution region. 2x 8
26. We have the following inequalities ⇒ > [dividing both sides by 2]
2 2
5(2 x − 7 ) − 3(2 x + 3) ≤ 0 …(i) ∴ x>4 K(iii)
and 2 x + 19 ≤ 6x + 47 …(ii) Thus, any value of x greater than 4 satisfies the inequality.
From inequality (i), we get ∴ The solution set is ( 4, ∞ ).
5 (2 x − 7 ) − 3 (2 x + 3 ) ≤ 0 The representation of solution of inequality (i) is
⇒ 10x − 35 − 6x − 9 ≤ 0 ⇒ 4x − 44 ≤ 0 x>4

⇒ 4x − 44 + 44 ≤ 44 [adding 44 both sides] 0 1 2 3 4 5
⇒ 4x ≤ 44
From inequality (ii), we get
⇒ x ≤ 11 …(iii) 2x − 3 4x
[dividing both sides by 4] + 6≥2 +
4 3
∴The solution set is (− ∞, 11]. 2 x − 3 + 24 6 + 4x
⇒ ≥
From inequality (ii), we get 4 3
2 x + 19 ≤ 6x + 47 2 x + 21 6 + 4x
⇒ ≥
⇒ 2 x + 19 − 2 x ≤ 6x + 47 − 2 x 4 3
[subtracting 2x from both sides] ⇒ 3(2 x + 21) ≥ 4( 6 + 4x )
⇒ 19 ≤ 4x + 47 [multiplying both sides by LCM ( 4, 3) = 12]
⇒ 19 − 47 ≤ 4x + 47 − 47 ⇒ 6x + 63 ≥ 24 + 16x
[subtracting 47 from both sides] ⇒ 6x + 63 − 63 ≥ 24 + 16x − 63
⇒ − 28 ≤ 4x or 4x ≥ − 28 [subtracting 63 from both sides]
4x − 28 ⇒ 6x ≥ 16x − 39
⇒ ≥ [dividing both sides by 4]
4 4 ⇒ 6x − 16x ≥ 16x − 39 − 16x
⇒ x ≥ −7 [subtracting 16x from both sides]
∴The solution set is [− 7, ∞). ⇒ −10x ≥ −39
Now, let us draw the graphs of the solutions of both ⇒ 10x ≤ 39
inequalities on number line. [multiplying both sides by −1, the inequality
sign will also change]
–∞ –7 0 11 ∞
10x 39

⇒ ≤ [dividing both sides by 10]
–∞ –7 0 11 10 10
⇒ x ≤ 3. 9 K(iv)
It can be seen that the values of x, which are common to
both are lying in the interval [− 7, 11]. Thus, any value of x less than or equal to 3. 9 satisfies the
inequality.
Hence, the solution set of given system of inequations is
[− 7, 11] and this can be represented graphically on the ∴The solution set is ( −∞ , 3. 9].
number line as Its representation on number line is,
∞ x ≤ 3.9
–∞ –7 0 11 –∞ ∞
0 3.9
27. Given, system of linear inequalities is
2(2 x + 3) − 10 < 6( x − 2 ) …(i) We see that there is no common value of x, which satisfies
2x − 3 4x both inequalites (iii) and (iv).
and + 6≥2 + K(ii)
4 3 Hence, the given system of inequalities has no solution set.
From inequality (i), we get 28. Let x L of 3% solution be added to 460 L of 9% solution of
acid.
2 (2 x + 3) − 10 < 6( x − 2 )
Then, total quantity of mixture = ( 460 + x ) L
⇒ 4x + 6 − 10 < 6x − 12
Total acid content in the ( 460 + x ) L of mixture
⇒ 4x − 4 < 6x − 12
 9 3 
⇒ 4x − 4 + 4 < 6x − 12 + 4 [adding 4 both sides] =  460 × +x× 
 100 100
⇒ 4x < 6x − 8
CBSE Term II Mathematics XI 45

It is given that acid content in the resulting mixture must be 4x 1280


⇒ > [dividing both sides by 4]
more than 5% but less than 7% acid. 4 4
Therefore, 5%of( 460 + x ) < 460 ×
9
+
3x
< 7% of ( 460 + x ) ⇒ x > 320 ...(ii)
100 100 Now, from inequalities (i) and (ii), we get
5 9 3 7 320 < x < 1280
⇒ × ( 460 + x ) < 460 × + x< × ( 460 + x )
100 100 100 100 Thus, the number of litres of the 2% boric acid solution will
⇒ 5 × ( 460 + x ) < 460 × 9 + 3x < 7 × ( 460 + x ) have to be greater than 320 L and less than 1280 L.
[multiplying by 100] 30. We have the following inequalities
⇒ 2300 + 5x < 4140 + 3x < 3220 + 7 x 3x + 4y ≤ 60 …(i)
Taking first two inequalities, 2300 + 5x < 4140 + 3x x + 3y ≤ 30 …(ii)
⇒ 5x − 3x < 4140 − 2300 x≥0 …(iii)
⇒ 2 x < 1840 and y≥0 …(iv)
1840 Take inequality (i),
⇒ x<
2 3x + 4y ≤ 60
⇒ x < 920 …(i) In equation form, it can be written as 3x + 4y = 60
Now, let us construct the following table
Taking last two inequalities,
4140 + 3x < 3220 + 7 x x 0 20
⇒ 3x − 7 x < 3220 − 4140 y 15 0
⇒ − 4x < − 920
Thus, the line intersect X-axis at A(20, 0) and Y-axis at
⇒ 4x > 920
B( 0, 15). Now, joining the points A and B by a dark line, we
920
⇒ x> get the line AB.
4 On putting (0, 0) in the given inequality, we get
⇒ x > 230 …(ii) 0 + 0 ≤ 60 ⇒ 0 ≤ 60, which is true.
Hence, the number of litres of the 3% solution of acid must So, for 3x + 4y ≤ 60, shade the half plane which contain (0, 0).
be more than 230 L and less than 920 L.
Take inequality (ii), x + 3y ≤ 30
29. Let x be the number of litres of 2% boric acid solution.
Then, total mixture = ( 640 + x ) litres In equation form, it can be written as x + 3y = 30
According to the question, Now, let us construct the following table
2% of x + 8% of 640 > 4% of (640 + x) x 0 30
2x 8 4 y 10 0
⇒ + × 640 > ( 640 + x )
100 100 100
Thus, the line intersect X-axis at C( 30, 0) and Y-axis at
⇒ 2 x + 5120 > 2560 + 4x D( 0, 10).
[multiplying both sides by 100] Now, joining the points C and D by a dark line, we get the
⇒ 2 x + 5120 − 2 x > 2560 + 4x − 2 x line CD.
On putting (0, 0) in the given inequality, we get
[subtracting 2x from both sides]
0 + 0 ≤ 30 ⇒ 0 ≤ 30, which is true.
⇒ 5120 > 2560 + 2 x
So, for x + 3y ≤ 30, shade the half plane which contain (0, 0).
⇒ 5120 − 2560 > 2560 + 2 x − 2560 Take inequalities (iii) and (iv), x ≥ 0 and y ≥ 0
[subtracting 2560 from both sides] These inequalities represent the region of first quadrant
⇒ 2560 > 2 x ⇒ 2 x < 2560 including the points on the axes. So, shade the first quadrant.
⇒ x < 1280 [dividing both sides by 2] ...(i) Y
30
Also, 2% of x + 8% of 640 < 6% of ( 640 + x )
2 8 6 20
⇒ ×x+ × 640 < × ( 640 + x ) B (0, 15)
100 100 100
⇒ 2 x + 5120 < 3840 + 6x
D(0, 10)
[multiplying both sides by 100]
C (30, 0)
⇒ 2 x + 5120 − 2 x < 3840 + 6x − 2 x X′
O A(20, 0) 3x X
[subtracting 2x from both sides] 30 x + 3y=
20 + 4y 30
Y′ =6
⇒ 5120 < 3840 + 4x 0
⇒ 5120 − 3840 < 3840 + 4x − 3840
[subtracting 3840 from both sides] Here, common shaded region represent the solution region
for system of inequalities.
⇒ 1280 < 4x or 4x > 1280
46 CBSE Term II Mathematics XI

31. Consider, the line x + y = 4. The graph of the above inequations is given below.
We observe that the shaded region and the origin lie on the Y
opposite side of this line and (0, 0) satisfies x + y ≤ 4. x=4
Therefore, we must have x + y ≥ 4, as the linear inequation 15 (0,15)
corresponding to the line x + y = 4. 14
Consider the line x + y = 8, clearly the shaded region and 13
(0,12)
origin lie on the same side of this line and (0, 0) satisfies the 12
constraints x + y ≤ 8. Therefore, we must have x + y ≤ 8, as 11
the linear inequation corresponding to the line x + y = 8. 10
Consider the line x = 5. It is clear from the graph that the 9
shaded region and origin are on the left of this line and (0, 0) 8
satisfy the constraint x ≤ 5. 7
6
Hence, x ≤ 5 is the linear inequation corresponding to x = 5.
5
Consider the line y = 5, clearly the shaded region and origin
4
are on the same side (below) of the line and (0, 0) satisfy the
3
constrain y ≤ 5.
2
Therefore, y ≤ 5 is an inequation corresponding to the line 1
y = 5. (5,0) (8,0)
X′ X
O 1 2 3 4 5 6 7 8
We also notice that the shaded region is above the X-axis and
on the right of the Y-axis i.e. shaded region is in first Y′ 3x+2y =24
quadrant. So, we must have x ≥ 0, y ≥ 0. 3x+y =15
Thus, the linear inequations comprising the given solution It is clear from the graph that there is no common region
set are corresponding to these inequality.
x + y ≥ 4, x + y ≤ 8, x ≤ 5, y ≤ 5, x ≥ 0 and y ≥ 0. Hence, the given system of inequalities have no solution.
32. Consider the inequation 3x + 2 y ≥ 24 as an equation, 33. Consider the inequation x + 2 y ≤ 3 as an equation, we have
we have x + 2y = 3
3x + 2 y = 24 ⇒ x = 3 − 2y
⇒ 2 y = 24 − 3x ⇒ 2y = 3 − x

x 0 8 4 x 3 1 0
y 0 1 1.5
y 12 0 6
Now, (0, 0) satisfy the inequation x + 2 y ≤ 3.
Hence, line 3x + y = 24 intersects coordinate axes at points So, half plane contains (0, 0) as the solution and the line
( 8 , 0) and ( 0 ,12 ). x + 2 y = 3 intersect the coordinate axis at ( 3, 0) and (0, 3/2).
Now, ( 0 , 0) does not satisfy the inequation 3x + 2 y ≥ 24. Consider the inequation 3x + 4y ≥ 12 as an equation,
Therefore, half plane of the solution set does not contains we have,
( 0 , 0 ).
3x + 4y = 12 ⇒ 4y = 12 − 3x
Consider the inequation 3x + y ≤ 15 as an equation, we have
x 0 4 2
3x + y = 15
⇒ y = 15 − 3x y 3 0 3/2

x 0 5 3 Thus, coordinate axis, intersected by the line 3x + 4y = 12 at


points (4, 0) and (0, 3).
y 15 0 6
Now, (0, 0) does not satisfy the inequation 3x + 4y ≥ 12.
Line 3x + y = 15 intersects coordinate axes at points ( 5 , 0) Therefore, half plane of the solution does not contained
and ( 0 ,15 ). (0, 0).
Now, point ( 0 , 0) satisfy the inequation 3x + y ≤ 15. Consider the inequation y ≥1 as an equation, we have
Therefore, the half plane of the solution contain origin. y = 1.
Consider the inequality x ≥ 4 as an equation, we have It represents a straight line parallel to X-axis passing through
point (0, 1).
x=4
Now, (0, 0) does not satisfy the inequation y ≥ 1.
It represents a straight line parallel to Y-axis passing through
Therefore, half plane of the solution does not contains (0, 0).
( 4 , 0). Now, point ( 0 , 0) does not satisfy the inequation x ≥ 4 .
Clearly x ≥ 0 represents the region lying on the right side of
Therefore, half plane does not contains ( 0 , 0),
Y-axis.
CBSE Term II Mathematics XI 47

The solution set of the given linear constraints will be the So, the shaded region will be towards point (2, 2).
intersection of the above region. And take a point (0, 1) and put it in the inequation (iii), we
Y get
3x+4y ≥ 12 0−1≤ 0 (true)
3 So, the shaded region will be towards point (0, 1).
(0,3)
Y
(0,1.5)
y=1
x+
(4,0) X-axis (0,5) 2y
X′ =1
–4 –3 –2 –1 O 1 2 3 4 0
(3,0) x+2y ≤ 3 (10/3, 10/3)

(0,1) (1/2, 1/2)


X′ X
0
y=

x+
Y′
x–

y=
1
It is clear from the graph the shaded portions do not have
common region.
So, solution set is null set. Y′
34. The given system of inequalities
Thus, common shaded region shows the solution of the
x + 2 y ≤ 10 …(i) inequalities.
x + y ≥1 …(ii) 35. We have,
x−y≤0 …(iii) x+y≤5 ...(i)
x ≥ 0, y ≥ 0 …(iv) 4x + y ≥ 4 ...(ii)
Step I Consider the given inequations as strict equations i.e. x + 5y ≥ 5 ...(iii)
x + 2 y = 10, x + y = 1, x − y = 0 x≤4 ...(iv)
and x = 0, y = 0 and y≤3 ...(v)
Step II Find the points on the X-axis and Y-axis for Take inequality (i), x + y ≤ 5
x + 2 y = 10 In equation form, it can be written as x + y = 5
x 0 10 Now, let us construct the following table
y 5 0 x 0 5
and x + y =1 y 5 0

x 0 1 Thus, the line x + y = 5 passes through the points A(0,5) and


y 1 0 B(5, 0).
Now, on joining the points A and B by a dark line, we get the
For x−y=0 line AB.
x 1 2 On putting x = 0 and y = 0 in inequality (i), we get
0 ≤ 5, which is true.
y 1 2
So, for inequality x + y ≤ 5, shade the half plane which
Step III Plot the graph of x + 2 y = 10,x + y = 1 and x − y = 0 contains origin.
using the above tables. Take inequality (ii), 4x + y ≥ 4
Step IV Take a point (0, 0) and put it in the inequations (i) In equation form, it can be written as 4x + y = 4
and (ii), Now, let us construct the following table
0 + 0 ≤ 10 (true) x 0 1
So, the shaded region will be towards origin. y 4 0
And 0+ 0≥1 (false)
So, the shaded region will be away from the origin. Thus, the line 4x + y = 4 passes through points C(0, 4) and
D(1, 0).
Again, take a point (2 , 2 ) and put it in the inequation (iv),
we get Now, joining the points C and D by a dark line, we get the
line CD.
2 ≥ 0, 2 ≥ 0 (true)
48 CBSE Term II Mathematics XI

On putting x = 0 and y = 0 in inequality (ii), we get For this inequality, shade the half plane which contains
origin. [Q 0 ≤ 4, is true]
4( 0) + 0 ≥ 4
Take inequality (v), y ≤ 3
⇒ 0 ≥ 4, which is false.
Linear equation corresponding to inequality (v) is y = 3. This
So, for inequality 4x + y ≥ 4, shade the half plane which does is a line parallel to X-axis at a distance of 3 units above the
not contain origin. X-axis.
Take inequality (iii), x + 5y ≥ 5 For this inequality, shade the half plane which contains
In equation form, it can be written as origin. [Q 0 ≤ 3, is true]
Y
x + 5y = 5
Now, let us construct the following table

x=4
x 0 5 A (0,5) x+
y=
y 1 0 C (0,4) 5
y=3
Thus, the line x + 5y = 5 passes through the points E(0,1)
and B (5,0). x+5
y =5
Now, joining the points E and B by a dark line, we get the (0,1) B(5, 0)
X′ E X
line EB. O

4x+
On putting x = 0 and y = 0 in inequality (iii), we get

y=4
Y′
0 ≥ 5, which is false. D
So, for inequality x + 5y ≥ 5, shade the half plane which does (1, 0)
not contain origin.
The common shaded region represent the solution region for
Take inequality (iv), x ≤ 4 system of inequalities.
Linear equation corresponding to inequality (iv) is x = 4.
This is a line parallel to Y-axis at a distance 4 units to the
right of Y-axis.
Chapter Test
l
Multiple Choice Questions
On the basis of the above information, solve the
1. The solution set of the inequality following questions.
37 − (3 x + 5) ≥ 9 x − 8(x − 3) is (i) As the shaded region represents in the first
(a) ( −∞, 2) (b) ( −∞, − 2) (c) ( −∞, 2] (d) ( −∞, − 2] quadrant, then the inequalities satisfies
(a) x ≥ 0, y ≤ 0 (b) x ≥ 0, y ≥ 0
2. The solution set of the inequalities (c) x ≤ 0, y ≤ 0 (d) x ≤ 0, y ≥ 0
6 ≤ − 3(2x − 4) < 12 is
(ii) A vertical line will divide the xy-plane in two
(a) ( −∞, 1] (b) (0, 1] (c) (0, 1] ∪ [1, ∞) (d) [1, ∞)
parts, which is
5 −3x (a) left and upper half planes
3. If − 5 ≤ ≤ 8, then
2 (b) left and right half planes
11 11 (c) right and lower half planes
(a) − < x <5 (b) − ≤ x≤5
3 3 (d) None of the above

(c) − 11≤ x ≤ 5
11
(d) − < x < 5 (iii) Line 3 x + 4 y = 18 represents the inequality in
3 the given system is
3 (x − 2) (a) 3 x + 4 y ≥ 18 (b) 3 x + 4 y > 18
4. The solution set of − 15 < ≤ 0 is (c) 3 x + 4 y ≤ 18 (d) None of these
5
(a) ( − 23, 2] (b) [− 23, 2] (iv) In the given graphical solution system, line
(c) [− 23, 2) (d) None of these 2x + 3 y = 3 represents the solution set
(a) towards the origin
5. The graph of the solutions of inequality (b) away from the origin
3x − 4 x + 1 (c) can’t say anything
≥ − 1 on number line is
2 4 (d) None of the above
(a)
–1 0 1 2 3 (v) In the given shaded region, the solution set also
(b) lies
–1 0 1 2 3 (a) on the inequalities
(c) (b) inside the inequalities
–1 0 1 2 3 (c) can’t say anything
(d) (d) None of the above
–1 0 1 2 3
l
Short Answer Type Questions
3 x + 11
6. The solution set of 7 ≤ ≤ 11 is
2 8. Find the solution set of 3 x − 5 < x + 7, when x is
 11   11   11  an integer.
(a) 1,  (b) 1, (c)  1,  (d) (1, ∞)
 3   3   3
9. Find all the pairs of consecutive odd positive
integers, both of which are smaller than 10,
l
Case Based MCQs
such that their sum is more than 11.
7. The shaded region in the given figure is the
solution set of the system of inequalities.
10. A solution is to be kept between 40°C and 45°C.
Y
What is the range of temperature in degree
9
Fahrenheit, if the conversion formula is
(0, 2 )
9
3x
+
F = C + 32?
4 4y
=1
5
=1 8
4y l
Long Answer Type Questions
x+
–7 X
X′
O 11. How many litres of water will have to be added
2x to 1125 L of the 45% solution of acid, so that the
+
=3 3y
6y = resulting mixture will contain more than 25%
x– 3
but less than 30% acid content?
Y′
12. Find the solution set of the inequalities
Answers 4 x + 3 y ≤ 60, y ≥ 2x , x ≥ 3, x and y ≥ 0.
1. (c) 2. (b) 3. (b) 4. (a) 5. (a) 6. (b)
For Detailed Solutions
7. (i) (b) (ii) (b) (iii) (c) (iv) (b) (v) (a) 8. {.... − 3, − 2, − 1, 0, 1, 2, 3, 4, 5}
Scan the code
9. (5, 7) and (7, 9) 10. 104°F and 113°F 11. more than 562.5 but less than 900 L
50 CBSE Term II Mathematics XI

CHAPTER 03

Permutations &
Combinations
In this Chapter...
l Fundamental Principles of Counting
l Factorial Notation
l Permutations
l Restricted Permutations
l Combinations

Fundamental Principles of Counting Fundamental Principle of Addition (FPA)


There are two fundamental principles of counting which are If there are two events such that they can be performed
the base of permutations and combinations. independently in m and n ways respectively, then either of
the two events can be occurred in ( m + n ) ways.
Fundamental Principle of Multiplication (FPM) In other words, if an operation can be performed in m
If an event can occur in m different ways, another event can different ways and another operation which is independent
occur in n different ways, then the total number of occurrence of the first operation , can be performed in n different ways.
of the events in the given order is m × n. Then, either of the two operations can be performed in
( m + n ) ways.
In other words, if there are two operations say E and F such
that E can be performed in m ways and associated with each Factorial Notation
way of performing operation E, operation F can be performed n Factorial
in n ways. Then, the two operations in succession can be
Many times, we come across the products of the form
performed in m × n ways.
1 × 2 , 1 × 2 × 3,1 × 2 × 3 × 4, ... . For our convenience, we use
e.g. In a school, there are 200 boys and 150 girls. The teacher a special notation instead of writing all the factors of such a
wants to select a boy and a girl to represent the school in inter product. We write
school competition.
1! =1
Here, the teacher can select a boy in 200 ways and a girl in
2! = 2 ×1
150 ways. So, by principle of multiplication, the teacher can
select a boy and a girl in 200 × 150 = 30000 ways. 3! = 3 × 2 ×1
Note The above principle can be extended to any finite number of events M M
(or operations) as stated below :
n ! = n ( n − 1 ) ( n − 2 )… 3 ⋅ 2 ⋅1
If there are n events, say E1 , E2 , … ,En such that E1 can occur in m1
ways, E2 can occur in m2 ways, E3 can occur in m3 ways and so on, Thus, the notation n ! represent the product of first n natural
then the total number of ways in which all the events can be numbers. We read this notation as ‘n factorial’ and it is also
occurred in the stated order is m1 × m2 × m3 × … × mn. denoted by n ! .
CBSE Term II Mathematics XI 51

Clearly, for a natural number n, In Particular,


n ! = n ( n − 1 )! n! n!
(i) When r = 0, then n
P0 = = =1
= n ( n − 1 ) ( n − 2 )! [provided n ≥ 2] ( n − 0 )! n !
= n ( n − 1 ) ( n − 2 ) ( n − 3 )! [provided n ≥ 3] (ii) When r = n, then n
Pn =
n!
=
n!
= n! [Q
M M ( n − n )! 0 !
0 ! = 1]
= n( n − 1 )( n − 2 )( n − 3 )… 3 ⋅ 2 ⋅1
Note Number of permutations of n different things taken all at a
e.g. 5 ! = 5 × 4 × 3 × 2 × 1 = 120 time = n!
and 8 ! = 8 × (7 )! = 8 × 7 × ( 6 !) Permutations with Repetitions
Zero Factorial When repetition of objects is allowed, then number of
permutations can be obtained with the help of following
It does not make any sense to define 0! as product of the theorem.
integers from 1 to 0. So, we define 0 ! = 1.
Theorem 2 The number of permutation of n different
Note Factorial of fractions or negative integers are not defined. n! is defined objects taken r at a time, when each may be repeated any
only for whole numbers, i.e. for non-negative integers.
number of times in each arrangement, is n r (permutation
with repetitions).
Permutations
Note The number of permutations of n different objects, all at a time,
A permutation is an arrangement of objects in a definite order. when each may be repeated any number of times in each
Arrangement can be made by taking some or all objects at a arrangement, is nn.
time. e.g. If there are three objects say A, B and C, then the
permutations of these three objects taking two at a time are AB, Permutations When all the Objects
BA, AC, CA BC, CB and the permutations of these three objects are not Distinct Objects
taking all at a time are ABC, ACB, BAC, BCA, CAB, CBA. When all the objects are not distinct, i.e. some objects are
Here, in each case, number of permutations is 6. of same kind, then we can find the number of permutations
Note that the order of arrangement is important. Because when with the help of following theorems (without proof).
the order is changed, then different permutation is obtained. Theorem 3 The number of permutations of n objects,
where p objects are of the same kind or identical and other
Permutations, When All the Objects are Distinct n!
are distinct, is given by ..
When all given objects are distinct, then we can find the p!
number of permutations with the help of following theorem Theorem 4 The number of permutations of n objects, where
Theorem 1 The number of permutations of n different objects p 1 objects are of one kind, p 2 are of second kind, … p k are
taken r at a time, where 0 < r ≤ n and the objects do not repeat, of kth kind and the rest if any, are of different kind is
is n ( n − 1 ) ( n − 2 ) ... ( n − r + 1 ), which is denoted by n Pr or n!
.
P( n , r ). p 1 ! p 2 !… p k !
n!
i.e. P ( n , r ) = n Pr = , 0≤ r ≤ n Restricted Permutations
( n − r )!
Here, we shall discuss permutations of objects under
Proof We know that
certain conditions, e.g. permutation when certain objects
P ( n , r ) = n Pr = n ( n − 1 ) ( n − 2 ) ( n − 3 ) . . . [ n − ( r − 1 )] occur together, when position of particular objects are
On multiplying numerator and denominator by fixed, when a particular object occurs in every arrangement
etc.
( n − r ) ( n − r − 1 ) ... 3 × 2 × 1, we get
n( n − 1 ) ( n − 2 ) . . . ( n − ( r – 1 ))  Some Important Results
 × ( n − r ) ( n − ( r + 1 )) . . . 3 ⋅ 2 ⋅1
  (i) If r particular things out of n different things are to
n
Pr = be together, then we count these r particular things
( n − r ) ( n − ( r + 1 )) . . . 3 ⋅ 2 ⋅1
as one things and remaining ( n − r ) things as
n( n − 1 ) ( n − 2 ) . . . 3 ⋅ 2 ⋅1
= separate things.
( n − r ) ( n − r − 1 ) . . . 3 ⋅ 2 ⋅1
Then, total number of things = ( n − r ) + 1
n! = n − r + 1.
=
( n − r )!
Since, these are different things, therefore number of
where, 0 ≤ r ≤ n permutations of these things = ( n − r + 1 )! .
52 CBSE Term II Mathematics XI

But, r particular things can also be arrange among Meaning of n C r


themselves in r! ways.
The number of combinations of n different things taken r at a
∴ Required number of permutations = ( n − r + 1 )! r ! time is meant the number of groups of r things which can be
Note If r particular things are identical, then required number of formed from n things and generally it is denoted by symbol
permutations = (n − r + 1) ! n
C r or C( n , r ).
(ii) The number of permutations of n objects taken r at a
time, when a particular object is taken in each Combinations of n Different Things Taken r at a Time
arrangement, is r ⋅ n −1 Pr −1 . The number of combinations of n distinct objects taken r at a
time is given by
(iii) The number of permutations of n objects taken r at a
n!
time, when a particular object is never taken in each n
Cr = .
arrangement, is n −1 Pr . r !( n − r )!

(iv) The number of permutations of n different objects Some Important Theorems


taken r at a time in which two specific objects always
Theorem 1 n
Pr = n
Cr ⋅ r ! , 0 < r ≤ n
occur together, is 2 ! ( r − 1 ) ⋅ n − 2Pr − 2 .
Proof The number of combinations of n distinct objects
taken r at a time is n C r . In these combinations, r things can
Combinations
be arranged among themselves in r ! ways. So, we have
The word combination means selection. r ! permutations.
Each of the different selection, which is made by taking some Therefore, the total number of permutations of n different
or all of a number of different objects at a time, irrespective of things taken r at a time is n C r × r ! , which is equal to n Pr .
their arrangements is called a combination.
∴ n
Pr = r ! × n
Cr , 0 < r ≤ n Hence proved.
e.g. The different combinations formed from three letters A,
B, C taking two at a time are AB, AC, BC. Theorem 2 n
Cr = n
Cn − r , 0 ≤ r ≤ n
Note This theorem is used to simplify the calculation when r is large.
Difference between Permutations n +1
Theorem 3 n
Cr + n
Cr − 1 = Cr
and Combinations
Proof LHS = n
Cr + n
Cr − 1
The process of selecting objects is called combination and
that of arranging objects is called permutation. n! n!
= +
If we have 4 objects A, B, C and D, the possible selection r ! ( n − r )! ( r − 1 )! ( n − r + 1 )!
(or combination) and arrangement (or permutation) of n ! ( n − r + 1) n! r
= +
3 objects out of 4 are given below. r ! ( n − r + 1 ) ( n − r )! r ( r − 1 )! ( n − r + 1 )!
This will help you to understand clearly the difference n ! ( n − r + 1) n!r
between permutations and combinations. = +
r ! ( n − r + 1) ! r ! ( n − r + 1) !
Selection Arrangement  n − r +1 + r 
↓ ↓ = n!  
Combination Permutation  r ! ( n − r + 1 )! 
( n + 1) n !
ABC ABC, ACB, BAC, BCA, CAB, CBA =
r ! ( n + 1 − r )!
ABD ABD, ADB , BAD, BDA, DAB, DBA
( n + 1 )!
ACD ACD, ADC, CAD, CDA, DAC, =
DCA
r ! ( n + 1 − r )!
n +1
BCD
= C r = RHS Hence proved.
BCD, BDC, CBD, CDB, DBC,
DCB Theorem 4 If n
Cx = n
C y , then either x = y or x + y = n.
Total 4 combinations 24 permutations
CBSE Term II Mathematics XI 53

Solved Examples
Example 1. In a certain city, all telephone numbers 1 1 x
Example 4. If + = , then find x.
have six digits, the first two digits always being 7 ! 9 ! 10 !
41 or 42 or 46 or 62 or 64. How many telephone 1 1 x
Sol. Given, + =
numbers have all six digits distinct? 7 ! 9! 10!
Sol. Suppose first two digit is 41. Then, we left with 8 digits, 1 1 x
⇒ + =
namely 0, 2, 3, 5, 6, 7, 8, 9 which can be used in constructing 7 ! 9 × 8 × 7 ! 10 × 9 × 8 × 7 !
the telephone numbers.
1  1 1  x 
Clearly, third digit can be selected in 8 ways. ⇒ 1 +  =  
7!  72  7 !  10 × 9 × 8
Fourth digit can be selected in 8 − 1 = 7 ways.
Fifth digit can be selected in 6 ways. 73 x
⇒ =
and sixth digit can be selected in 5 ways. 72 10 × 9 × 8
Now, according to fundamental principle of multiplication, 73 × 10 × 9 × 8
⇒ x=
number of telephone starting with 41 72
= 8 × 7 × 6 × 5 = 1680 = 730
Similarly, if the telephone number start with 42 or 46 or 62
or 64, the remaining 4 digits can be selected in 1680 ways.
Example 5. How many words (with or without
dictionary meaning) can be made from the letters
Hence, total number of required telephone numbers
of the word ‘MONDAY’, assuming that no letter is
= 1680 × 5 = 8400
repeated, if
Example 2. Find the number of positive integers greater (i) 4 letters are used at a time.
than 6000 and less than 7000 which are divisible (ii) all letters are used at a time.
by 5, provided that no digit is to be repeated.
(iii) all letters are used but the first is a vowel.
Sol. We know, a number will be divisible by 5, if either 0 or 5 is
at unit’s place. Sol. (i) Number of words using 4 letters out of 6 letters
∴ Number of ways to fill the unit’s place = 2 6!
= 6P4 = =6×5×4×3
Also note that, for number to be greater than 6000 and less 2!
than 7000, digit 6 should be fixed at thousand’s place. = 360
∴ Number of ways to fill the thousand’s place = 1 (ii) Number of words using all letters = 6P6 = 6!
Now, we left with 8 digits. So hundred’s place can be filled = 720
in 8 ways and ten’s place can be filled in 7 ways. (iii) Number of words starting with vowel
Thousand’s Hundred’s = Number of ways of choosing first letter ( out of O
Ten’s place Unit’s place and A) × Number of ways of arranging 5 alphabets
place place
= 2 × 5! = 2 × 120 = 240
6 0 or 5
↓ ↓ ↓
Example 6. Find the number of 4-digit numbers that
↓ can be formed with the digits 1, 2, 3, 4 and 5 in
1 way 8 ways 7 ways 2 ways
which atleast 2 digits are identical.
∴ Total number of numbers = 1 × 8 × 7 × 2 = 112 Sol. Total number of 4-digit numbers = 5 × 5 × 5 × 5
Example 3. In how many ways, two books of different = 625
languages can be selected from 10 Hindi, 5 English (as each place can be filled by anyone of the numbers
and 7 Sanskrit books? 1, 2, 3, 4 and 5)
Sol. Clearly, a book of Hindi and a book of English can be Numbers in which no two digits are identical
selected in 10 × 5 = 50 ways. = 5× 4× 3×2
A book of English and a book of Sanskrit can be selected in (i.e. repetition not allowed) = 120
5 × 7 = 35 ways.
(as 1st place can be filled in 5 different ways, 2nd place can
A book of Hindi and a book of Sanskrit can be selected in be filled in 4 different ways and so on)
10 × 7 = 70 ways.
Number of 4-digits numbers in which atleast 2 digits
Now, by fundamental principle of addition, two books of
are identical = 625 − 120
different languages can be selected in 50 + 35 + 70 = 155
ways. = 505
54 CBSE Term II Mathematics XI

Example 7. How many numbers greater than 1000000 Example 10. Prove that r ⋅ n C r = n ⋅ n − 1 C r − 1 .
can be formed, by using the digits 2, 3, 0, 3, 4, 2, 3?
n!  n n! 
Sol. Since, 1000000 is a 7-digit number and the number of digits Sol. LHS = r ⋅ nC r = r ⋅ Q C r = 
to be used is also 7. Therefore, the number to be counted r ! ( n − r )!  r ! ( n − r )! 
will be 7-digit only. r ⋅ n!
= [Q r ! = r ( r − 1)!]
Now, let us arrange the given seven digits. r ( r − 1)! ( n − r )!
7!
This can be done in = 420 ways. =
n!
…(i)
2 ! 3! ( r − 1)! ( n − r )!
But these arrangements will include those also, where 0 is at
n −1 ( n − 1)!
the million’s place. RHS = n ⋅ Cr −1 = n ⋅
( r − 1)! {( n − 1) − ( r − 1)}!
So, keeping 0 fixed at million’s place, the remaining 6 digits
6! n ⋅( n − 1)! n!
can be arranged in = 60 ways. = =
2 ! 3! ( r − 1)! ( n − r )! ( r − 1)!( n − r )!
Hence, required number of numbers = 420 − 60 = 360. [Q n( n − 1)! = n !] …(ii)
Example 8. Find the number of arrangements of the From Eqs. (i) and (ii), we get
letters of the word ‘INDEPENDENCE’. In how LHS = RHS
n −1
many of these arrangements ⇒ r ⋅ n Cr = n ⋅ Cr −1 Hence proved.
(i) do the words start with P?
Example 11. There are 10 persons named
(ii) do all the vowels always occur together? P1 , P2 , P3 ,K , P10 . Out of 10 persons, 5 persons are
(iii) do all the vowels never occur together? to be arranged in a line such that in each
Sol. The letters in the word INDEPENDENCE are arrangement P1 must occur, whereas P4 and P5 do
(I, N, N, N, D, D, E, E, E, E, P, C). not occur. Find the number of such possible
12 !
∴Total number of arrangement = arrangements.
3! × 2 ! × 4! Sol. Given that, P1 , P2 , ... ,P10 are 10 persons, out of which 5
11!
(i) Total number of words start with P = persons are to be arranged but P1 must occur, whereas P4 and
3! × 2 ! × 4! P5 never occur.
(ii) Total number of words, when all the vowels occur ∴Selection depends on only 10 − 3 = 7 persons
8! × 5!
together = [consider all vowels at one unit] As, we have already occur P1. Therefore, we have to select
3! × 2 ! × 4! only 4 persons out of 7.
(iii) Total number of ways = Total arrangements 7!
− Number of arrangements, when vowels Number of selection = 7 C 4 =
4! (7 − 4)!
12 ! 8! × 5!
occurs together = − 7! 5040
3! × 2 ! × 4! 3! × 2 ! × 4! = = = 35
4! 3! 24 × 6
Example 9. The number of words with or without
∴ Required number of arrangement of 5 persons
meaning which can be made, using all the letters
of the word ‘AGAIN’ are …A… and if these words = 35 × 5!
are written as in dictionary, then the 50th word = 35 × 120 = 4200
is …B… . Find A and B.
Example 12. We wish to select 6 persons from 8 but if
Sol. There are 5 letters in the word ‘AGAIN’, in which A appears
the person A is chosen, then B must be chosen. In
5!
2 times. Therefore, the required number of words = = 60. how many ways, can the selection be made?
2! Sol. Let us make the following cases and find the number of
To get the number of words starting with A, we fix the letter possible selection in each case.
A at the extreme left position, we then rearrange the Case I When A is chosen
remaining 4 letters taken all at a time. There will be as many
arrangements of these 4 letters taken 4 at a time as there are In this case, B must be chosen also. So, we have to choose
permutations of 4 different things taken 4 at a time. Hence, 4 persons out of 6 persons. This can be done in
the number of words starting with A = 4! = 24. Then, starting 6×5
6
C4 = 6 C2 = = 15 ways.
4!
with G, the number of words = = 12 as after placing G at 2 ×1
2!
Case II When A is not chosen
the extreme left position, we are left with the letters A, A, I
and N. Similarly, there are 12 words starting with the next In this case, we have to choose 6 persons out of 7 persons.
letter I. Total number of words so far obtained This can be done in 7 C 6 = 7 C1 = 7 ways.
= 24 + 12 + 12 = 48. Hence, total number of ways of selecting 6 persons
The 49th word is NAAGI. The 50th word is NAAIG.
= 15 + 7 = 22
CBSE Term II Mathematics XI 55

Example 13. In an examination, a question paper Hence, by first principle of counting, total number of ways
consist of 12 questions divided into two parts I and = 5C 2 × 6 C 3
II containing 5 and 7 questions, respectively. A 5×4 6×5×4
= ×
student is required to attempt 8 questions in all, 2 6
selecting atleast 3 from each part. In how many  n ( n − 1) 
Q C 2 =
n
ways, can a student select the questions? 2 
 
Sol. Student may select 8 questions, according to following and n C = n( n − 1) ( n − 2 ) 
scheme.  3
6 
I (5 Questions) II (7 Questions) = 10 × 20
(i) 3 5 = 200 ways
(ii) 4 4 Example 15. Find the number of words, with or without
(iii) 5 3 meaning, each of 2 vowels and 3 consonants can
be formed from the letters of the words
Now, the required number of ways ‘DAUGHTER’.
= 5C 3 × 7 C 5 + 5C 4 × 7 C 4 + 5C 5 × 7 C 3 Sol.
= 5C 2 × 7 C 2 + 5C1 × 7 C 3 + 5C 0 × 7 C 3 DAUGHTER

= 10 × 21 + 5 × 35 + 1 × 35
= 210 + 175 + 35 = 420.
Vowels Consonants
Hence, the total number of ways of selecting the questions A, E, U (Total 3) D, G, H, T, R (Total 5)
by the student is 420 ways.
Take Take 3
Example 14. Find the number of ways, in which 2 black 2 vowels consonants
and 3 red balls can be selected from a bag,
containing 5 black and 6 red balls. 2 vowels out of 3 vowels can be selected = 3C 2 ways
Sol. Out of 5 black balls, 2 black balls can be selected in 5 C 2 ways. 3 consonants out of 5 consonants can be selected = 5C 3 ways
Out of 6 red balls, 3 red balls can be selected in 6 C 3 ways. and these 5 letters (2 vowels and 3 consonants) can be
arranged in 5! ways.
Bag
Hence, by FPC, total number of ways
5 Black 6 Red
Take 2 balls Take 3 balls
= 3C 2 × 5C 3 × 5! = 3C1 × 5C 2 × 5!
3× 5× 4× 5× 4× 3×2 ×1
=
6
2
5 C3
C2
= 3600 ways
56 CBSE Term II Mathematics XI

Chapter
Practice
PART 1
Objective Questions
l
Multiple Choice Questions 10. The number of 4 letter words that can be formed
1. A sequence is a ternary sequence, if it contains from alphabets of the word ‘PART’, when repetition
digits 0, 1 and 2. The total number of ternary is allowed, is
(a) 24 (b) 196 (c) 1 (d) 256
sequences of length 9 which either begin with
210 or end with 210, is 11. The number of six-digit numbers all digits of which
(a) 1458 (b) 1431 (c) 729 (d) 707 are odd, is ......... .
(a) 53 (b) 54 (c) 55 (d) 56
1 1 x
2. If + = , then x is equal to 12. The number of 5-digit telephone numbers having
9 ! 10 ! 11 !
(a) 100 (b) 110 (c) 99 (d) 121
atleast one of their digits repeated is
(a) 90000 (b) 10000
3. P( n − 1 , r ) + r ⋅ P ( n − 1 , r − 1 ) equals (c) 30240 (d) 69760
(a) P( n − 1 , r + 1) (b) P( n , r − 1)
13. Find the number of permutations of the letters of
(c) P( n , r ) (d) P( n + 1 , r + 1)
the word ‘INDEPENDENCE’ is
n −1
4. If P3 : n P4 = 1 : 9, then n equals. (a) 1663400 (b) 1663300
(a) 1 (b) 8 (c) 9 (d) 16 (c) 1663200 (d) 1663100
2n
5. Find the number of 4 letter words, with or without 14. If the ratio C 3 : n C 3 is equal to 11 : 1, the value
meaning, which can be formed out of letter of word of n is
‘ROSE’. (a) 4 (b) 5 (c) 6 (d) 7
When repetition of the letters is allowed are 260. 15. 15
C8 + 15
C9 − 15
C6 − 15
C 7 is equal to ......... .
(a) 24 (b) 256 (c) 36 (d) 12 (a) 0 (b) 2 (c) 1 (d) 3
6. It is required to seat 5 men and 4 women in a row, 16. If Pr = 840 and C r = 35, then r is equal to ......... .
n n
so that the women occupy the even places. The
number of such arrangement equals. (a) 4! (b) 3! (c) 4 (d) 3
(a) 1440 (b) 2800 (c) 2880 (d) 2840 17. Three balls are drawn from a bag containing 5 red,
7. The number of ways that two different rings can 4 white and 3 black balls. The number of ways in
be worn in four fingers with atmost one in each which this can be done, if atleast 2 are red, is
finger, is (a) 70 (b) 75 (c) 78 (d) 80
(a) 8 (b) 10 (c) 11 (d) 12 18. In a football championship, 153 matches were
8. The sum of the digits in unit place of all the played. Every two teams played one match with
numbers formed with the help of 3, 4, 5 and 6 taken each other. The number of teams, participating in
all at a time, is the championship is ......... .
(a) 432 (b) 108 (c) 36 (d) 18 (a) 18 (b) 17 (c) 16 (d) 10
9. The number of numbers between 400 and 1000 that 19. The number of parallelograms that can be formed
can be made with the digits 2, 3, 4, 5, 6 and 0, when from a set of four parallel lines intersecting another
repetition of digits is not allowed, is set of three parallel lines is
(a) 40 (b) 50 (c) 60 (d) 70 (a) 6 (b) 18 (c) 12 (d) 9
CBSE Term II Mathematics XI 57

20. The number of ways in which a team of eleven 24. Republic day is a national holiday of India.
players can be selected from 22 players always It honours the date on which the constitution
including 2 of them and excluding 4 of them is of India came into effect on 26 January 1950
(a) 16 C11 (b) 16 C 5 replacing the Government of India Act (1935) as
(c) 16 C 9 (d) 20
C9 the governing document of India and thus, turning
the nation into a newly formed republic.
21. The number of ways in which we can choose a
committee from four men and six women, so that Answer the following question, which are based on
the committee includes atleast two men and exactly the word “REPUBLIC”.
twice as many women as men is (i) Find the number of arrangements of the letters of
(a) 94 (b) 126 the word ‘REPUBLIC’.
(c) 128 (d) None of these (a) 40300 (b) 30420
(c) 40320 (d) 40400
22. The number of diagonals of n-sided polygon is
(a) n C 2 + n (b) n C 2 − n (ii) How many arrangements start with a vowel?
(a) 12015 (b) 15120
(c) n ⋅ C 2
n
(d) C 2 − 2 n
n
(a) 12018 (d) 15100
l
Case Based MCQs (iii) Which concept is used for finding the
arrangements start with a vowel?
23. Five students Ajay, Shyam, Yojana, Rahul and (a) Permutation (b) FPM
Akansha are sitting in a play ground in a line. (c) Combination (d) FPA
(iv) If the number of arrangements of the letters of the
word ‘REPUBLIC’ is abcde, the ( a + b + c + d + e )
is
(a) 10 (b) 9
(c) 8 (d) 15
(v) If the number of arrangements start with a vowel is
abcde, then ( a + b ) − ( d + e ) is
(a) 2 (b) 3
(c) 4 (d) 5

Based on the above information, answer the


PART 2
following questions.
(i) Total number of ways of sitting arrangement of
Subjective Questions
five students is
(a) 120 (b) 60
l
Short Answer Type Questions
(c) 24 (d) None of these 1. A code word is to consist of two distinct English
(ii) Total number of arrangement of sitting, if Ajay alphabets followed by two distinct numbers
and Yojana sit together, is between 1 and 9. e.g. CA23 is code word.
(a) 60 (b) 48 (i) How many such code words are there?
(c) 72 (d) 120 (ii) How many of them end with an even integer?
(iii) Total number of arrangement ‘Yojana and Rahul
2. In how many ways can 3 prizes be distributed
sitting at extreme position’ is
among 4 boys, when
(a) 24 (b) 36
(c) 48 (d) 12 (i) no boy gets more than one prize?
(iv) Total number of arrangement, if shyam is sitting in (ii) a boy may get any number of prizes?
the middle, is (iii) no boy gets all the prizes?
(a) 24 (b) 12 3. Find the number of numbers divisible by 3 that can
(c) 6 (d) 36
be formed by four different even digits.
(v) Total number of arrangement sitting Yojana and n! n!
Rahul not sit together, is 4. If and are in the ratio 2 : 1,
(a) 72 (b) 120 2 ! ( n − 2 )! 4 ! ( n − 4)!
(c) 60 (d) 144 then find the value of n.
58 CBSE Term II Mathematics XI

5. Find the value of 22. If n = m


C 2 , then find the value of n C 2 .
1
P1 + 2 ⋅ 2 P2 + 3 ⋅ 3P3 + . . . . . + n ⋅ n Pn . 23. Let Tn denote the number of triangles which can be
6. If P(2 n − 1 , n ) : P(2 n + 1 , n − 1 ) = 22 : 7, find n. formed using the vertices of a regular polygon of n
sides. If Tn + 1 − Tn = 21, then find the value of n.
7. If 5 Pr = 2 ⋅ 6Pr − 1 , then find the value of r. n +2 n −2
24. If C8 : P4 = 57 : 16, then find the value of n.
8. In a class of 10 students there are 3 girls A, B and C. 5
Find the number of different ways they can be 25. Find the value of the expression 47 C 4 + ∑ 52 − j
C 3.
arranged in a row such that no two of three girls are j =1
consecutive.
26. From a class of 25 students, 10 are to be chosen
9. How many number lying between 999 and 10000 for an excursion party. There are 3 students, who
can be formed with the help of the digits 0, 2, 3, 6, decide that either all of them will join or none of
7, 8, when the digits are not repeated? them will join. Find the number of ways the
10. How many number of words which can be formed excursion party can be chosen.
out of the letters of the word ‘ARTICLE’, so that 27. In an examination, Yamini has to select 4 questions
vowels occupy the even place? from each part. There are 6, 7 and 8 questions in
11. How many 4-digit numbers that can be formed with part I, part II and part III, respectively. What is the
the digits 1, 2, 3, 4, 5, 6, when a digit may be number of possible combinations in which she can
repeated any number of times in any arrangement? choose the questions?
12. In how many different ways, the letters of the word 28. A bag contains six white marbles and five red
‘ALGEBRA’ can be arranged in a row, if marbles. Find the number of ways in which four
(i) the two A’s are together? marbles can be drawn from the bag, if
(ii) the two A’s are not together? (i) they can be of any colour.
13. Find the number of positive integers greater than (ii) two must be white and two red.
3000 and less than 5000 which are divisible by 5, (iii) they must all be of the same colour.
provided that no digit is to be repeated. 29. A group consists of 4 girls and 7 boys. In how many
14. Three men have 4 coats, 5 waistcoats and 6 caps. ways can a team of 5 members be selected, if the
In how many ways, can they wear them? team has
15. Find the number of different words that can be (i) no girls.
formed from the letters of the word ‘TRIANGLE’, (ii) atleast one boy and one girl.
so that no vowels are together. (iii) atleast three girls.
16. Find the number of ways in which 5 boys and 30. Determine the number of 5 cards combinations out
5 girls be seated in a row, so that of a deck of 52 cards, if atleast one of the 5 cards
has to be a king.
(i) no two girls sit together.
(ii) boys and girls sit alternatively. 31. A candidate is required to answer 7 questions out of
12 questions, which are divided into two groups,
17. Determine the number of natural numbers smaller each containing 6 questions. He is not permitted to
than 10 4 , in the decimal notation of which all the attempt more than 5 questions from either group.
digits are distinct. Find the number of different ways of doing
18. Find the sum of all the numbers that can be formed questions.
with the digits 2, 3, 4, 5 taken all at a time. 32. A committee of 6 is to be chosen from 10 men
19. Find the number of numbers greater than 1000000 and 7 women, so as to contain atleast 3 men and
that can be formed by using the digits 1,2,0,2,4,2,4. 2 women. In how many different ways can this be
done, if two particular women refuse to serve on
20. How many different words can be formed by using
the same committee?
all the letters of the word ‘ALLAHABAD’? In how
many of them, vowels occupy the even position? 33. Find the number of ways of selecting a cricket team
of eleven from 17 players in which only 6 players
21. A polygon has 44 diagonal. Find the number of its
can bowl, if each cricket team of 11 must include
sides.
exactly 4 bowlers.
CBSE Term II Mathematics XI 59

34. The greatest possible number of points of 43. A committee of 7 has to be formed from 9 boys and
intersection of 8 straight lines and 4 circles equals 4 girls.
…A… . Find A. (i) Find the number of ways this can be done, if
35. Using the digits 1, 2, 3, 4, 5, 6, 7 a number of committee consists of exactly 3 girls.
4 different digits is formed. Find how many numbers (ii) Find the number of ways this can be done, if
are formed? Further, find how many of these are committee consists of atmost 3 girls.
(i) exactly divisible by 2? l
Case Based Questions
(ii) exactly divisible by 25?
(iii) exactly divisible by 4? 44. 5 boys and 5 girls are sitting in a music room in a
row. Their sitting positions can be anything.
l
Long Answer Type Questions
36. Find the number of different signals that can be
generated by arranging atleast 2 flags in order (one
below the other ) on a vertical staff, if five different Based on the above information, answer the
flags are available. following questions.
37. How many different 4-digit numbers can be formed (i) Find the number of ways in which all the girls sit
from the digits 2, 3, 4 and 6, if each digit is used together.
only once in a number? Further, how many of these (ii) Find the number of ways in which all the girls and
numbers all the boys sit together.
(i) end at 4? (iii) Find the number of ways in which all the girls are
(ii) end at 3? never sit together.
(iii) end at 3 or 4? 45. On Diwali festival, few people are playing cards.
38. Find the rank of the word ‘SUCCESS’ in the dictionary One person choose 4 cards from a pack of
if all the words are written as in a dictionary. 52 playing cards.
39. Find the number of distinct permutations of the
letters in ‘MISSISSIPPI’ such that four I's do not
come together.
40. The number of ways, the letters of the word
‘PERMUTATIONS’, can be arranged, if
(i) words start with P and end with S.
(ii) vowels are all together.
(iii) there are always 4 letters between P and S.
41. If n − 1 C r : n C r : n + 1 C r = 6 : 9 : 13, then find the
values of n and r.
42. There are 10 professors and 20 students out of
whom a committee of 2 professors and 3 students is
Based on the above information, answer the
to be formed. Find the number of ways in which
following questions
this can be done. Further find in how many of these
committees (i) Find the number of ways choosing these 4 cards
such that they are of the same suit.
(i) a particular professor is included?
(ii) Find the number of ways choosing these 4 cards
(ii) a particular student is included?
such that two are Red and two are Black cards.
(iii) a particular student is excluded?
(iii) Find the number of ways choosing these 4 cards
such that they are face cards.
60 CBSE Term II Mathematics XI

SOLUTIONS
Objective Questions Remaining 5 seats can be filled by 5 men in 5! ways.
1. (b) Since, each digit in a ternary sequence of length 9 can be Hence, by FPC, total number of ways
filled in 3 ways. = 5! × 4!
Therefore, the number of nine-digit ternary sequence = 5 × 4 × 3 ×2 ×1 × 4 × 3×2 ×1
beginning with 210 is 36 = 120 × 24 = 2880 ways
and number of nine-digit ternary sequence ending with 210 7. (d) Clearly, required number of ways is same as number
is also 36 . of permutations (or arrangements) of 4 different things taken
The number of ternary sequence of 9 digits which begin and 2 at a time.
end with 210 is 33. So, required number of ways = 4P2
∴ Required number of such numbers = 36 + 36 − 33 = 1431 4! 4 × 3 × 2 !
= = = 12
1 1 x 2! 2!
2. (d) We have, + =
9! 10! 11! 8. (b) If we fixed 3 at units place.
1 1 x Total possible number is 3! i.e. 6.
⇒ + =
9! 10 × 9! 11 × 10 × 9! Sum of the digits in unit place of all these numbers = 3! × 3
1 x 11 x Similarly, if we fixed 4, 5 and 6 at units place, in each case
⇒ 1+ = ⇒ =
10 110 10 110 total possible numbers are 3!.
⇒ x = 121 Required sum of unit digits of all such numbers
3. (c) We have, P( n − 1, r ) + r ⋅ P ( n − 1, r − 1) = ( 3 + 4 + 5 + 6) × 3 !
( n − 1)! ( n − 1)! = 18 × 3! = 18 × 6 = 108
= + r⋅ 9. (c) Given, digits are 2, 3, 4, 5, 6 and 0.
( n − 1 − r )! [( n − 1) − ( r − 1)] !
( n − 1)! ( n − 1)! Any number between 400 and 1000 will be of three digits.
= + r⋅ Since, the number should be greater than 400.
( n − r − 1)! ( n − r )!
 Therefore, hundred’s place can be filled up by anyone of the
( n − 1)! r 
= 1 +  digits 4, 5 and 6 in ‘3 ways’.
( n − r − 1)! n −r
Now, remaining two places can be filled up by the remaining
( n − 1)! n
= . five digits in 5P2 ways.
( n − r − 1)! n − r
5!
n! Hence, required numbers = 3 × 5P2 = 3 × = 60
= = P( n , r ) 3!
( n − r )!
n −1 10. (d) Number of 4 letter words that can be formed, when
P3 1
4. (c) Here, n
= repetition is allowed = 44 = 256
P4 9
11. (d) Among the digits 0, 1, 2, 3, 4, 5, 6, 7, 8, 9, clearly 1, 3, 5, 7
( n − 1) ( n − 2 ) ( n − 3) 1
⇒ = and 9 are odd.
n ( n − 1) ( n − 2 ) ( n − 3) 9
∴ Number of six-digit numbers = 5 × 5 × 5 × 5 × 5 × 5 = 56
1 1
⇒ = 12. (d) If all the digits repeated, then number of 5 digit telephone
n 9 numbers can be formed in 10 5 ways and if no digit repeated,
⇒ n=9 then 5-digit telephone numbers can be formed in 10 P5 ways.
5. (a) Number of 4 letter words (repetition not allowed) 10!
∴ Required number of ways = 10 5 − 10P5 = 100000 −
5!
The first place can be filled in 4 different ways by anyone of = 100000 − 10 × 9 × 8 × 7 × 6
the 4 letters R, O, S, E. Following which the second, third = 100000 − 30240 = 69760
and the fourth place may be filled in 3, 2, 1 ways, 13. (c) Here, we have 12 letters of which 3 are N’s, 4 are E’s,
respectively. 2 are D’s and rest are different.
= 4× 3×2 ×1 12 !
∴ Required number of permutations =
= 24 (multiplication principle) 3! 4! 2 !
6. (c) Total number of persons = 9 [here, n = 12 , p1 = 3, p2 = 4 and p3 = 2]
12 × 11 × 10 × 9 × 8 × 7 × 6 × 5 × 4!
2 4 6 8 =
1 × 3 × 5 × 7 × 9 6 × 2 × 4!
There are 4 even numbers and 4 women want to sit, this can = 11 × 10 × 9 × 8 × 7 × 6 × 5
be done in 4! ways. = 1663200
CBSE Term II Mathematics XI 61

14. (c) We have, 2n


C 3 : n C 3 = 11 : 1 20. (c) Total number of players = 22
2n
C 3 11 We have to select a team of 11 players. Selection of 11
⇒ n
= players, when 2 of them is always included and 4 are never
C3 1
included.
2 n(2 n − 1)(2 n − 2 )
Total number of players = 22 − 2 − 4 = 16
3⋅2 ⋅1 11
⇒ = ∴ Required number of selections = 16 C 9
n( n − 1)( n − 2 ) 1
3⋅2 ⋅1 21. (a)Q Number of men = 4
2 n(2 n − 1)2( n − 1) 3⋅2 ⋅1 11 and number of women = 6
⇒ × =
3⋅2 ⋅1 n( n − 1)( n − 2 ) 1 It is given that committee includes two men and exactly
4(2 n − 1) 11 twice as many women as men.
⇒ = Thus, possible selection is given in following table.
n −2 1
⇒ 8n − 4 = 11n − 22 Men Women
⇒ 18 = 3n ⇒ n = 6 2 4
Hence, the value of n is 6. 3 6
15. (a) 15
C 8 + 15C 9 − 15C 6 − 15C 7 Required number of committee formed
= 15C15 − 8 + 15C15 − 9 − 15C 6 − 15C 7 [Q n C r = n C n − r ] = 4C 2 × 6 C 4 + 4C 3 × 6 C 6
= 15C 7 + 15C 6 − 15C 6 − 15C 7 = 6 × 15 + 4 × 1 = 94
=0 22. (b) In n-sided polygon, the number of vertices = n
16. (c) Given that, n Pr = 840 and n
C r = 35 ∴ Number of lines that can be formed using n points = n C 2.
Q n
Pr = n C r ⋅ r ! Out of these, n C 2 lines, n lines form the polygon.
⇒ 840 = 35 × r! ∴ Number of diagonals = n C 2 – n
840 (i) (a) We have five student.
⇒ r! = = 24 23.
35 ∴ Total number of arrangement is 5! = 120
⇒ r! = 4 × 3 × 2 × 1 (ii) (b) Ajay and Yojana sit together.
⇒ r! = 4! Total number of arrangement = 4! × 2 ! = 24 × 2 = 48
∴ r=4 (iii) (d) Total number of arrangement of Yojana and Rahul
17. (d) Required number of ways = 5C 2 × 7 C1 + 5C 3 sitting in extreme position is 2 ! × 3! = 2 × 6 = 12
[since, atleast two red] (iv) (a) Number of arrangement of Shyam in middle is
= 10 × 7 + 10 4! = 24
= 70 + 10 = 80 (v) (a) Number of arrangement Yojana and Rahul not sit
18. (a) Let the number of team participating in championship together, is
4!
be n. × 3! = 72 or 5! − 4! × 2 ! = 72
Since, it is given that every two teams played one match 2!
with each other. 24. (i) (c) The letters in the word ‘REPUBLIC’ are all distinct.
∴ Total match played = n C 2 There are 8 letters in the given word. So, the number
of arrangements are 8! i.e. 40320.
According to the question, (ii) (b) The vowels in a given word are ‘E, I, U’. If we start
n
C 2 = 153 a word from vowel, we can choose 1 vowel from 3
n ( n − 1) vowels in 3 C1 ways. Further, remaining 7 letters can be
⇒ = 153
2 arranged in 7! ways.
⇒ n 2 − n = 306 ∴ Total number of arrangements start with a vowel
= 3C1 × 7 ! = 3 × 5040 = 15120
⇒ n 2 − n − 306 = 0
(iii) (c) Combination
⇒ ( n − 18) ( n + 17 ) = 0 (iv) (b) Since, number of arrangements are 40320.
⇒ n = 18, − 17 [inadmissible] On comparing, we get
∴ n = 18 a = 4, b = 0, c = 3, d = 2 , e = 0
19. (b) To form parallelogram, we required a pair of line from So, a + b + c + d + e = 4 + 0 + 3 + 2 + 0 = 9
a set of 4 lines and another pair of line from another set of (v) (c) Since, number of arrangements are 15120
3 lines. On comparing, we get
∴ Required number of parallelograms = 4C 2 × 3C 2 a = 1, b = 5, c = 1, d = 2 , e = 0
= 6 × 3 = 18 ∴ ( a + b ) − ( d + e ) = (1 + 5 ) − (2 + 0 ) = 6 − 2 = 4
62 CBSE Term II Mathematics XI

Subjective Questions n! n!
4. We have, : =2:1
1. (i) There are 26 English alphabets, we have to choose 2 ! ( n − 2 )! 4! ( n − 4)!
2 distinct alphabets. n! 4! ( n − 4)! 2
1st alphabet can be selected in 26 ways. ⇒ × =
2 ! ( n − 2 )! n! 1
2nd alphabet can be selected in 25 ways.
4! ( n − 4)!
Again, out of 9 digits (1 to 9), ⇒ =2
2 ! ( n − 2 )!
1st number can be selected in 9 ways.
4 × 3 × 2 ! × ( n − 4)!
2nd number can be selected in 8 ways. ⇒ =2
Thus, by the fundamental principle of multiplication, 2 ! × ( n − 2 ) ( n − 3) ( n − 4)!
the number of distinct codes 12
⇒ =2
= 26 × 25 × 9 × 8 = 46800 ( n − 2 ) ( n − 3)
(ii) As above, two distinct alphabets can be selected in ⇒ 12 = 2 ( n − 2 ) ( n − 3)
26 × 25 ways.
We have, in all 1, 2, 3, 4, 5, 6, 7, 8 and 9 digits, unit ⇒ 12 = 2 ( n 2 − 5n + 6) ⇒ 6 = n 2 − 5n + 6
place can be filled up in 4 ways (by 2, 4, 6, 8). Ten’s ⇒ n 2 − 5n = 0 ⇒ n ( n − 5) = 0
place can be filled up in 8 ways.
⇒ n = 0 or 5
[since, one of the digits is already used]
Thus, the number of such codes = 26 × 25 × 8 × 4 When n = 0, then ( n − 2 )! and ( n − 4)! are not defined, so
rejecting n = 0.
= 20800
∴ n=5
2. (i) The first prize can be given in 4 ways. The 2nd prize
can be given in 3 ways because the boy who got the 5. Here, nth term, t n = n ⋅ n Pn
first prize, cannot receive the other prize. = n ⋅ n!
The 3rd prize, can be given to anyone of the remaining = ( n + 1 − 1 )n !
2 boys in 2 ways. = ( n + 1 )n ! − n !
∴ Total number of ways in which all the prizes can be t n = ( n + 1)! − n !
given = 4 × 3 × 2 = 24 ∴ t1 = 2 ! − 1!
(ii) The first prize can be given in 4 ways. t 2 = 3! − 2 !
The 2nd prize can also be given in 4 ways, because it t 3 = 4! − 3!
may be obtained by the boy who has already received a
M M M
prize.
t n = ( n + 1)! − n !
Similarly, 3rd prize can be given in 4 ways.
∴ Required sum = ( n + 1)! − 1!
Hence, the number of ways in which all the prizes can
be given = 4 × 4 × 4 = 43 = 64 = n + 1Pn + 1 − 1
2n − 1 2n + 1
(iii) If any one of the 4 boys may get all the prizes, then 6. We have, Pn : Pn − 1 = 22 : 7
the number of ways in which all the 3 prizes given to (2 n − 1)!
a boy = 4 (2 n − 1 − n )! 22
So, the number of ways in which no boy get all the ∴ =
(2 n + 1)! 7
prizes = 64 − 4 = 60.
(2 n + 1 − n + 1)!
3. Possible even digits are 2, 4, 6, 8, 0.
(2 n − 1)! ( n + 2 )! 22
Case I Number has digits 4, 6, 8, 0 × =
( n − 1)! (2 n + 1)! 7
(here, sum of digits is 18, divisible by 3)
(2 n − 1)! ( n + 2 )( n + 1)n ( n − 1)! 22
⇒ × =
( n − 1)! (2 n + 1)(2 n )(2 n − 1)! 7
Number of arrangements = 3 × 3! ( n + 2 )( n + 1)n 22 ( n + 1)( n + 2 ) 22
⇒ = ⇒ =
=3×6 (2 n + 1)(2 n ) 7 2 (2 n + 1 ) 7
= 18 ⇒ 7( n 2 + 3n + 2 ) = 44(2 n + 1)
(Ist place can be filled using 4, 6 or 8)
⇒ 7 n 2 + 21n + 14 = 88n + 44
Case II Number has digits 2, 4, 6, 0
⇒ 7 n 2 − 67 n − 30 = 0
(here, sum of digits is 12, divisible by 3)
⇒ 7 n 2 − 70n + 3n − 30 = 0
⇒ 7 n ( n − 10) + 3 ( n − 10) = 0
Ist place cannot be filled by 0.
⇒ ( n − 10)(7 n + 3) = 0
Number of arrangements = 3 × 3! −3
= 18 ⇒ n = 10 or n =
7
∴ Number of numbers = 18 + 18 = 36 ∴ n = 10 [Q n can’t be negative]
CBSE Term II Mathematics XI 63

5
Pr 12. There are 7 letters in the word ‘ALGEBRA’. In these letters,
7. Here, =2
6
Pr − 1 ‘A’ occurs twice and rest all are different.
5! (i) Since, two A’s are always together, so let us consider
two A’s as one letter.
( 5 − r )!
⇒ =2 AA , L, G, E, B, R
6!
Now, we have 6 letters, which can be arranged in
[ 6 − ( r − 1)]! 6
P6 = 6! ways
5! (7 − r )!
⇒ × =2 Hence, required number of ways = 6! = 720
( 5 − r )! 6!
[Q two A’s can be arranged among themselves
5! (7 − r )( 6 − r )( 5 − r )!
⇒ × =2 in one way only]
( 5 − r )! 6 × 5! (ii) Required number of ways = Number of all possible
(7 − r )( 6 − r ) arrangements of the letters of the word ‘ALGEBRA’
⇒ =2
6 − Number of arrangements in
⇒ 42 − 13 r + r 2 = 12 which two A’s are together
⇒ r 2 − 13 r + 30 = 0 7! 7 
= − 6! = 6!  − 1
⇒ ( r − 10)( r − 3) = 0 2! 2 
⇒ r = 10, 3 5
= 720 × = 360 × 5 = 1800
(r = 10 rejected as 5P10 and 6 P9 are not defined) 2
∴ r=3 13. We know that number is divisible by 5, if at the unit’s place
of the number is 0 or 5.
8. X B1 X B 2 X B 3 X B 4 X B 5 X B 6 X B 7 X
We have to form 4-digit number. So, the unit’s place can be
First, we arrange 7 boys in 7! ways. filled in two ways (by either 0 or 5).
Now, for girls, we have eight places and 3 girls need to be Since, the number is greater than 3000 and less than 5000, the
arranged. This can be done in 8P3 ways. thousand’s place can be filled in two ways (by either 3 or 4).
Total ways = 7 ! × 8P3 Now, we left with 8 digits, so hundred’s place can be filled in
7 ! × 8! 8 way and ten’s place can be filled in 7 ways.
=
( 8 − 3)! Thousand’s Unit’s
7 ! × 8! place Hundred’s Ten’s place
= place place
5! 3 or 4 0 or 5
= 7 ! × 336 ↓ ↓ ↓ ↓
9. The number of 4-digit numbers formed by digits 2 ways 8 ways 7 ways 2 ways
0, 2, 3, 6, 7, 8 is 6 P4 = 360.
But here those numbers are also involved which begin ∴ Total number of numbers = 2 × 8 × 7 × 2 = 224
from 0. So, we take those numbers as three-digit numbers. 14. Here, all the 4 coats are distinct, all 5 waistcoats are distinct
and all 6 caps are also distinct.
Taking initial digit 0, the number of ways to fill remaining
3 places from five digits 2, 3, 6, 7, 8 are 5P3 = 60. So, number of ways in which three men can wear 4 coats will
be equal to the number of arrangements of 4 different things
So, the required numbers = 360 − 60 = 300 taken 3 at a time.
10. Total number of letters in the word ‘ARTICLE’ is 7, out of ∴ Number of ways in which three men can wear 4 coats
which A, E, I are vowels and R, T, C, L are consonants. 4!
= 4P3 = = 4 × 3 × 2 = 24
Since, it is given that vowels occupy even place, therefore ( 4 − 3) !
the arrangement of vowel, consonant can be understand with
the help of following diagram. Similarly, number of ways in which three men can wear
5! 5 × 4 × 3 × 2!
1 2 3 4 5 6 7 5 waistcoats = 5P3 = = = 5 × 4 × 3 = 60
( 5 − 3) ! 2!
Now, vowels can be placed at 2, 4 and 6th position.
and number of ways in which three men can wear 6 caps
Therefore, number of arrangement = 3P3 = 3! = 6 ways 6! 6 × 5 × 4 × 3!
= 6P3 = =
and consonants can be placed at 1, 3, 5 and 7th position. ( 6 − 3) ! 3!
Therefore, number of arrangement = 4P4 = 4! = 24 = 6 × 5 × 4 = 120
∴ Total number of words = 6 × 24 = 144 Since, all three men wear coat, waistcoat and cap, so by the
11. Required number of numbers is same as number of fundamental principal of multiplication,
permutation of 6 different things taken 4 at a time, when a Required number of ways = 4P3 × 5P3 × 6P3
digit may be repeated any number of times.
= 24 × 60 × 120 = 172800
Hence, required number of numbers = 64 = 1296.
64 CBSE Term II Mathematics XI

15. Number of letters in the word ‘TRIANGLE’ = 8, out of Consider the digits in the unit’s places in all these numbers.
which 5 are consonants and 3 are vowels. Each of the digit 2, 3, 4, 5 occurs in 3! ( = 6) times in the
If vowels are not together, then we have following unit’s place.
arrangement. So, total for the digits in the unit’s place in all the numbers
= (2 + 3 + 4 + 5) × 3! = 84.
V C V C V C V C V C V
Since, each of the digits 2, 3, 4, 5 occurs 3! times in any one
Consonants can be arranged in = 5! = 120 ways and vowels of the remaining places.
can occupy at 6 places. So, the sum of the digits in the ten’s, hundred’s, thousand’s
The 3 vowels can be arranged at 6 place in 6 P3ways places in all the numbers = (2 + 3 + 4 + 5) × 3! = 84
6! 6! 6 × 5 × 4 × 3!
= = = = 120 ∴ Sum of all the numbers = 84 (100 + 101 + 102 + 103 )
( 6 − 3)! 3! 3!
= 84 (1 + 10 + 100 + 1000)
Total number of arrangement = 120 × 120 = 14400
= 84 (1111)
16. We have 5 boys and 5 girls. = 93324
(i) Since, no two girls sit together, therefore the possible 19. Since, 1000000 is a 7-digit number and the number of digits
choices for girls are the places marked as ‘×’. to be used is also 7. Therefore, the numbers to be counted
× B1 × B 2 × B 3 × B 4 × B 5 × will be 7 digit only. Also, the numbers have to be greater than
Clearly, the girls can be arranged in 6P5 ways and the 1000000, so they can begin either with 1, 2 or 4.
boys can be arranged in 5! ways. The number of numbers beginning with
Hence, by fundamental principle of multiplication, 6! 4×5×6
1= = = 60 as when 1 is fixed at the extreme
required number of ways = 6P5 × 5! 3! 2 ! 2
6! left position, the remaining digits to be rearranged will be 0,
= × 5! = 6! × 5! = 86400
( 6 − 5)! 2, 2, 2, 4, 4 in which there are 3, 2’s and 2, 4’s.
6! 3×4×5×6
(ii) Let us first seat the 5 girls (or 5 boys). This can be done Total numbers begining with 2 = = = 180
in 5! ways. 2! 2! 2
Now, for each such arrangement, the 5 boys can be 6!
and total numbers begining with 4 = = 4 × 5 × 6 = 120
seated only at the cross marked place as shown below. 3!
(I) × G1 × G 2 × G 3 × G 4 × G 5 Therefore, the required number of numbers
(II) G1 × G 2 × G 3 × G 4 × G 5 ×
= 60 + 180 + 120 = 360.
In case I, 5 boys can be seated in 5P5 = 5! ways
20. There are 9 letters in the word ‘ALLAHABAD’. In these
Thus, number of ways of seating = 5! × 5! = ( 5!)2 letters, A occurs 4 times, L occurs twice and rest all are
Similarly, in case II, number of ways of seating = ( 5!)2 different.
Hence, required number of ways = ( 5!)2 + ( 5!)2 9!
∴Total number of words = = 7560
= 2 ⋅ ( 5!)2 = 28800 4! 2 !

17. The required natural numbers consist of number of 4-digit, Now, consider the case, where vowels occupy the even
3-digit, 2-digit and 1-digit. position.
Clearly, total number of 4-digit natural numbers = 9 × 9P3 Since, the word ‘ALLAHABAD’ contains 4 vowels, A, A, A
[Q in thousand place, 0 does not exist] and A, and there are 4 even places, therefore 4 vowels can
be arranged in 4 even places.
Total number of 3-digit natural numbers = 9 × 9P2
This can be done in
Total number of 2-digit natural numbers = 9 × 9P1 1 1
4
P4 × = 4! × = 1 way.
and total number of 1-digit natural numbers = 9 4! 4!
Hence, required number of natural numbers Now, we left with 5 places and 5 letters, which can be
= 9 × 9P3 + 9 × 9P2 + 9 × 9P1 + 9 1 5!
arranged in 5P5 × = ways. [Q out of 5 letters, 2 are alike]
= 9( 9 P3 + 9P2 + 9P1 + 1) 2! 2!
 9! 9! 9!  Hence, the number of words in which vowels occupy the
= 9 + + + 1
 6! 7 ! 8!  even position =
5!
= 60.
= 9 ( 9 × 8 × 7 + 9 × 8 + 9 + 1) 2!
= 9( 504 + 72 + 10) 21. Let the number of sides of a polygon = n
= 9 × 586 = 5274 Number of diagonal = Number of line segment joining any
two vertices of polygon − Number of sides
18. Total number of numbers formed with the digits 2, 3, 4, 5
taken all at a time = n C2 − n
= Number of arrangement of 4 digits, taken all at a time n ( n − 1) n ( n − 3)
= −n =
= 4P4 = 24 2 2
CBSE Term II Mathematics XI 65

n ( n − 3) 5
= 44
Now,
2
25. We have, 47
C4 + ∑ 52 − j C3
j =1
⇒ n 2 − 3n − 88 = 0 52 − 1 52 − 2 52 − 3 52 − 4 52 − 5
= 47
C4 + C3 + C3 + C3 + C3 + C3
⇒ ( n − 11) ( n + 8) = 0 ⇒ n = 11
= 47
C4 + 51
C3 + 50
C3 + 49
C3 + 48
C3 + 47
C3
or n = − 8 rejected.
22. We have, = ( C3 +
47 47
C4 ) + 48
C3 + 49
C3 + 50
C3 + 51
C3
m ( m − 1) n +1
= 48
C4 + 48
C3 + 49
C3 + 50
C3 + 51
C 3[Q C r −1 + n C r =
n
Cr ]
n= m
C2 =
2 = 49
C4 + 49
C3 + 50
C3 + 51
C3
n ( n − 1) m ( m − 1)  m ( m − 1) 
∴ n
C2 = =  − 1 = 50
C4 + 50
C3 + 51
C3
2 4  2 
m ( m − 1) ( m + 1) ( m − 2 )  = 51
C4 + 51
C3 = 52
C4
=  
4  2  52 × 51 × 50 × 49
=
( m + 1) m ( m − 1) ( m − 2 ) 4× 3×2 ×1
= 3⋅
4× 3×2 ×1 = 13 × 17 × 25 × 49
= 3⋅ m +1
C 4. = 270725
23. We have, 26. There are two possibilities.
(a) If all the three students will go in party, then we have
Tn = n C 3
to select 7 students out of 22 students.
∴ Tn + 1 − Tn = 21 (b) If all of three students will not go for party then, we
n +1
⇒ C 3 − n C 3 = 21 have to select 10 students out of 22 students.
⇒ n
C 2 + n C 3 − n C 3 = 21 [Q n C r + n C r − 1 = n +1
Cr ] Now, 7 students out of 22 students can be selected
in 22 C 7 ways.
⇒ n
C 2 = 21 22
10 students out of 22 students can be selected in C10
n ( n − 1)
= 21 ways.
2 ×1
∴ Total number of ways = 22C 7 + 22C10
n 2 − n = 42 27.
n 2 − n − 42 = 0 Name of parts and number of Number of questions
n 2 − 7 n + 6n − 42 = 0 questions in the parts selected by Yamini
Part I = 6 4
( n − 7 ) ( n + 6) = 0
∴ n =7 Part II = 7 4
24. We have, Part III = 8 4
n + 2 n −2
C8 : P4 = 57 : 16 Clearly, total number of ways of selection
(n + 2) ! = Number of ways of selecting 4 questions from 6 questions
n + 2
C 8 57 8 ! ( n + 2 − 8) ! 57 × Number of ways of selecting 4 questions from 7 questions
⇒ = ⇒ = × Number of ways of selecting 4 questions from 8 questions
n −2
P4 16 (n − 2) ! 16
( n − 2 − 4) ! = 6 C4 × 7 C4 ×8 C4
(n + 2) ! ( n − 6) ! 57 = 6 C 2 × 7 C 3 × 8C 4 [Q n C r = n C n − r ]
⇒ × =
8! ( n − 6)! ( n − 2 ) ! 16 6× 5 7 × 6× 5 8×7 × 6× 5
= × ×
( n + 2 ) ( n + 1) n ( n − 1) ( n − 2 )! 1 57 2 ×1 3×2 ×1 4× 3×2 ×1
⇒ × =
8! ( n − 2 ) ! 16
= 15 × 35 × 70 = 36750 ways
57 Hence, the total number of possible combination in which
⇒ ( n + 2 ) ( n + 1) n ( n − 1) = × 8!
16 she can choose the questions is 36750 ways.
19 × 3 28. Total number of marbles = 6 white + 5 red = 11 marbles
= × 8×7 × 6× 5× 4× 3×2 ×1
16 (i) If they can be of any colour means, we have to select 4
⇒ ( n − 1) n( n + 1) ( n + 2 ) = 19 × 3 × 7 × 6 × 5 × 4 × 3 marbles out of 11.
⇒ ( n − 1) n ( n + 1) ( n + 2 ) = 19 × ( 3 × 7 ) × ( 6 × 3) × ( 5 × 4) ∴ Required number of ways = 11C 4
⇒ ( n − 1) n ( n + 1) ( n + 2 ) = 18 × 19 × 20 × 21 (ii) If two must be white, then selection will be 6 C 2 and
On comparing both sides, we get two must be red, then selection will be 5 C 2.
n − 1 = 18 ⇒ n = 19 ∴ Required number of ways = 6 C 2 × 5C 2
66 CBSE Term II Mathematics XI

(iii) If they all must be of same colour, then selection of 32. Q Total number of men = 10
4 white marbles out of 6 = 6 C 4 and total number of women = 7
and selection of 4 red marbles out of 5 = 5C 4 We have to form a committee containing atleast 3 men and
∴ Required number of ways = 6 C 4 + 5C 4 2 women.
Number of ways = 10 C 3 × 7 C 3 +10 C 4 × 7 C 2
29. Number of girls = 4 and number of boys = 7
We have to select a team of 5 members provided that If two particular women to be always there .
(i) team having no girls. ∴ Number of ways = 10 C 4 × 5C 0 + 10 C 3 × 5C1
7! 7×6 Total number of committee when two particular women are
∴ Required selection = 7 C 5 = = = 21
5! 2 ! 2 never together
(ii) atleast one boy and one girl = Total − Together
∴ Required selection = (10 C 3 × 7 C 3 + 10 C 4 × 7 C 2 ) − (10 C 4 × 5C 0 + 10 C 3 × 5C1 )
= 7 C1 × 4C 4 + 7 C 2 × 4C 3 + 7 C 3 × 4C 2+ 7 C 4 × 4C1 = (120 × 35 + 210 × 21) − (210 + 120 × 5)
= 7 × 1 + 21 × 4 + 35 × 6 + 35 × 4 = 4200 + 4410 − (210 + 600)
= 7 + 84 + 210 + 140 = 8610 − 810 = 7800
= 441
33.
(iii) when atleast three girls are included Cricket team
= 4C 3 × 7 C 2 + 4C 4 × 7 C1
= 4 × 21 + 7 = 84 + 7 = 91 Total 17 players
30. Selection of 5 cards can be done in the following ways.
Case I II III IV
6 Bowlers 11 Batsmen
King 1 2 3 4
Other cards 4 3 2 1 We have to select 11 players including exactly 4 bowlers.
Hence, 4 bowlers will be selected from 6 bowlers and
The number of combinations in case I, II, III and IV are remaining 7 players will be selected from 11 batsmen.
respectively Now, 4 bowlers out of 6 bowlers can be selected in 6 C 4 ways.
4
C1 × 48C 4, 4 C 2 × 48C 3, 4 C 3 × 48C 2 and 4 C 4 × 48C1
7 players out of 11 players can be selected in 11 C 7 ways.
Hence, required number of combinations
Hence, by FPC, total number of ways selecting the eleven
= 4C1 × 48C 4 + 4C 2 × 48C 3 + 4C 3 × 48C 2 + 4C 4 × 48C1
players
48 × 47 × 46 × 45 4 × 3
=4× + = 6 C 4 × 11C 7 = 6 C 2 × 11C 4 [Q n C r = n C n − r ]
4× 3×2 ×1 2 ×1
48 × 47 × 46 48 × 47 6 × 5 × 11 × 10 × 9 × 8
× + 4× + 1 × 48 =
3×2 ×1 2 ×1 2 × 24
= 8 × 47 × 46 × 45 + 48 × 47 × 46 + 2 × 48 × 47 + 48  n n ( n − 1) ( n − 2 ) ( n − 3) 
Q C 4 = 
= 778320 + 103776 + 4512 + 48  24 
= 886656 = 5 × 11 × 10 × 9
31. Since, candidate cannot attempt more than 5 questions from = 55 × 90
either group. = 4950 ways
Thus, he is able to attempt minimum two questions from 34. Maximum points of intersection of two circles =2
either group. ∴ Maximum point of intersection of 4 circle
The number of questions attempted from each group is 12
= 4C 2 × 2 = × 2 = 12
given in following table. 2 ×1
Group I 5 4 3 2 Two lines can intersect at only one point.
Group II 2 3 4 5 ∴ Number of points of intersection of 8 lines
8×7
Since, each group have 6 questions and total attempted = 8C 2 = = 28
2 ×1
7 questions.
A line can intersect circle at two points.
∴ Total number of possible ways
∴ Number of points of intersection of 4 circles and 8 lines
= 6 C5 × 6 C2 + 6 C4 × 6 C3 + 6 C3 × 6 C4 + 6 C2 × 6 C5
= 4C1 × 8C1 × 2
= 2 [ 6 C5 × 6 C2 + 6 C4 × 6 C3 ] = 4 × 8 × 2 = 64
= 2 [ 6 × 15 + 15 × 20] ∴ Maximum points of intersection = 12 + 28 + 64 = 104
= 2 [ 90 + 300] Hence, A = 104.
= 2 × 390 = 780
CBSE Term II Mathematics XI 67

35. Number of four different digits formed = 7 C 4 38. Number of words beginning with C =
6!
= 120
(i) Even number should be in unit’s place and remaining 3!
any three digits, any different number can be placed. C CESSSU
∴Total number of ways = 6P3 × 3C1 = 6 ⋅ 5 ⋅ 4 ⋅ 3 = 360. 6!
Number of words beginning with E = = 60
(ii) Any four digit number is divisible by 25, if last two 3! 2 !
digits should be divisible by 25 i.e. 25, 75. E SSSUCC
Then, total numbers which are divisible by 5!
Number of words beginning with SC = = 60
25 = 5 × 4 × 2 = 40. 2!
(iii) Any number is divisible by 4, if last two digits is SC UCESS
divisible by 4 i.e. last digits can be 12, 16, 24, 32, 36, 52, 5!
56, 64, 72, 76. Number of words beginning with SE= = 30
2 !2 !
∴Total number of ways = 5P2 × 10 C1 SE UCC SS
= 5 × 4 × 10 = 200 5!
Number of words beginning with SS = = 60
36. We know, if an event E can occur in m different ways and 2!
another event F can occur in n different ways, then both events SS SUCCE
can occur together in ( m + n ) ways. [addition principle]
First words beginning with SU is SUCCESS
A signal can consist of either 2 flags, 3 flags 4 or 5 flags.
∴ The rank of the word SUCCESS is
Number of signals using 2 flags = 5 × 4 = 20
= 120 + 60 + 60 + 30 + 60 + 1 = 331
(for upper flag in the signal, we have to choose
39. The word ‘MISSISSIPPI’ has 11 letters in which
from 5 different flags and for lower, we can
choose from 4 different flags) M → 1 time
Similarly, I → 4 times
S → 4 times
Number of signals using 3 flags = 5 × 4 × 3 = 60
P → 2 times
Number of signals using 4 flags = 5 × 4 × 3 × 2 = 120
The number of permutations of the word ‘MISSISSIPPI’ in
Number of signals using 5 flags = 5 × 4 × 3 × 2 × 1 = 120 which 4 I’s, 4 S’s and 2 P’s are alike
Hence, the required number of signals 11!
= …(i)
= 20 + 60 + 120 + 120 = 320 4! 4! 2 !
[using addition principle] If all the I’s are together, then it will be considered as one
Note Conceptual mix of addition and multiplication letter together with remaining 7 letters (including 4I’s) will
principle. be considered as 8 letters. So, the number of permutations is
37. The number of 4-digit numbers formed by the digits 8!
.
2, 3, 4 and 6 4! 2 ! (QP is repeated twice and S is repeated four times)
4! 4!  n n!  Hence, total number of required arrangements
= P ( 4, 4) = = Q Pr =  11! 8!
( 4 − 4)! 0!  ( n − r )!  = −
4! 4! 2 ! 4! 2 !
4!
= = 4 × 3 × 2 × 1 = 24 [Q 0! = 1] 11 × 10 × 9 × 8 × 7 × 6 × 5 × 4!
1 =
4! × 4 × 3 × 2 × 1 × 2 × 1
(i) In this case, ‘4’ is fixed at the unit’s place. Therefore,
8 × 7 × 6 × 5 × 4!
remaining 3-digits can be 2, 3 and 6. Hence, the −
required number of 4-digits number 4! × 2 × 1
= P ( 3, 3) = 34650 − 840 = 33810
3!  n n!  40. The word ‘PERMUTATIONS’ has
= Q Pr = 
( 3 − 3)!  ( n − r )!  P — 1 time
3! E — 1 time
= = 3×2 ×1= 6 …(i) R — 1 time
0!
(ii) In this case, ‘3’ is fixed at unit’s place. Therefore, M — 1 time
remaining 3-digits can be 2, 4 and 6. Hence, the U — 1 time
required number of 4-digits numbers = P ( 3, 3) T — 2 times
3! A — 1 time
= =6 …(ii)
0! I — 1 time
(iii) The number of 4-digit numbers ending with either O — 1 time
3 or 4 N — 1 time
= 6 + 6 = 12 [from Eqs. (i) and (ii)] S — 1 time
68 CBSE Term II Mathematics XI

(i) Word start with P and end with S i.e. ( n − 1)! ( n − r ) [ n − ( r + 1)]! 2
⇒ × =
P S [ n − ( r + 1)]! n( n − 1)! 3
n−r 2
First and last position are already filled by P and S, ⇒ =
respectively. n 3
10! ⇒ 3n − 3r = 2 n
Remaining 10 positions can be filled in ways
2! ⇒ n − 3r = 0 …(i)
10 × 9 × 8 × 7 × 6 × 5 × 4 × 3 × 2 ! n
Cr 9
= Also, we have n +1
=
2! C r 13
= 720 × 7 × 6 × 5 × 4 × 3 = 1814400 n! r ! ( n + 1 − r )! 9
(ii) If all the vowels are together, then it will be considered ⇒ × =
r ! ( n − r )! ( n + 1)! 13
as one letter, i.e. (A, E, I, O, U) together with remaining
7 letters and 1 vowel (including 5 vowels) will be n! [ n − ( r − 1)]! 9
⇒ × =
considered as 8 letters and 5 vowels can be arranged in ( n − r )! ( n + 1 )n ! 13
5! ways. [ n − ( r − 1)] ( n − r )! 9
5! × 8! ⇒ =
∴ Required number of permutations = ( n − r )!( n + 1) 13
2!
n−r+1 9
(Q T is repeated twice) ⇒ =
5 × 4 × 3 × 2 × 1 × 8 × 7 × 6 × 5 × 4 × 3 × 2 ×1 n+1 13
=
2 ×1 ⇒ 13n − 13r + 13 = 9n + 9
= 2419200 ⇒ 4n − 13r = − 4 …(ii)
(iii) Total number of letters to be arranged = 12 On solving Eqs. (i) and (ii), we get
There are always 4 letters between P and S i.e. n = 12 and r = 4
1 2 3 4 5 6 7 8 9 10 11 12 42. Clearly, for forming a committee of 2 professors and
Ist ways P S
3 students, we have to select 2 professors out of
10 professors and 3 students out of 20 students.
IInd ways P S This can be done in 10 C 2 × 20 C 3 ways.
IIIrd ways P S
Thus, required number of ways
IV ways P S 10 × 9 20 × 19 × 18
Vth ways = ×
P S 2 ×1 3×2 ×1
VIth ways P S = 45 × 60 × 19 = 51300
VIIth ways P S (i) When a particular professor is included, then we have
to select 1 professor out of 9 professors and 3 students
Hence, P and S having 4 places between them can be
out of 20 students.
filled in 7 ways.
This can be done in 9 C1 × 20 C 3 ways.
Hence, P and S or S and P can be filled in7 + 7 = 14 ways.
Thus, required number of ways
Remaining 10 letters (in which T repeated twice) can 20 × 19 × 18
10! =9×
be filled in ways. 3×2 ×1
2!
= 9 × 60 × 19 = 10260
Hence, by FPC, total number of ways
(ii) When a particular student is included, then we have to
10!
= 14 × select 2 professors out of 10 professors and 2 students
2! out of 19 students.
14 × 10 × 9 × 8 × 7 × 6 × 5 × 4 × 3 × 2 × 1
= This can be done in 10 C 2 × 19 C 2.
2
Thus, required number of ways
14 × 3628800
= = 25401600 10 × 9 19 × 18
2 = ×
2 ×1 2 ×1
41. We have,
n −1
Cr 6 = 45 × 171 = 7695
n
= (iii) When a particular student is excluded, then we have to
Cr 9
select 2 professors out of 10 professors and 3 students out
( n − 1)!
of 19 students. This can be done in 10 C 2 × 19 C 3 ways.
r ! ( n − 1 − r )! 2
⇒ = Thus, required number of ways
n! 3
r ! ( n − r )! 10 × 9 19 × 18 × 17
= ×
( n − 1)! ( n − r )! 2 2 ×1 3×2 ×1
⇒ × = = 45 × 969 = 43605
[ n − ( r + 1)]! n! 3
JEE NEET ADDA

This Pdf Is
Download From
www.jeeneetadda.in

Visit www.jeeneetadda.in for


More Premium Stuffs,Latest
Books,Test Papers,Lectures etc.
jeeneetadda
jeeneetadda_official
jeeneetadda

VISIT NOW !!
CBSE Term II Mathematics XI 69

43. (i) Out of 7 persons, we want to select exactly 3 girls i. e. , 9 × 8 ×7 × 6   9 × 8 × 7 × 6 4 × 3


= × 4 +  × 
remaining 4 person will be boys.  24   24 2 
9 × 8 ×7  9 × 8 
Group + × 4 +  × 1
 6   2 
= 504 + 756 + 336 + 36
9 Boys
= 1632 ways
4 Girls
44. Given,
Take 4 boys Take 3 girls
Total number of boys = 5
(9C4 ways) (4C3 ways) Total number of girls = 5
4 boys out of 9 boys can be selected in 9 C 4 ways. (i) Let take all the girls together as one unit.
Now we have to arrange 6 units.
3 girls out of 4 girls can be selected in 4 C 3 ways.
∴ Number of ways to arrange 6 units = 6!
Hence, by FPC, total number of ways for making the
Number of possible arrangements of 5 girls = 5!
committee
Therefore, the required number of arrangements
= 9 C 4 × 4C 3
= 6! × 5!
9× 8×7 × 6 4
= × C1 (Q n C r = n C n − r ) (ii) Let’s take all the girls together as one unit and all the
24 boys together as another unit.
9× 8×7 × 6× 4 ∴ Number of way to arrange 2 units = 2!
= (
24 Number of possible arrangements of 5 girls = 5!
Q C1 = n )
n
Number of possible arrangements of 5 boys = 5!
= 9× 8×7 Therefore, the required number of arrangements.
= 72 × 7 = 2 ! × 5! × 5!
= 504 ways (iii) Required number of ways = Number of all possible
(ii) Here, we have to select atmost 3 girls i.e. there are arrangements of 5 boys and 5 girls − Number of
following four possibilities arrangements in which all girls sit together.
= 10! − 6! × 5!
9 Boys 4 Girls 9 Boys 4 Girls 45. Number of ways of choosing 4 cards = 52C 4
Take Take Take Take = 270725
4 boys 3 girls or 5 boys 2 girls
(i) There are 4 suits and each suit have 13 cards. So, from
(9C4 ways) (4C3 ways) (9C5 ways) (4C2 ways) each suit, 4 cards can be choosed in 13 C 4 way
separately.
9 Boys 4 Girls 9 Boys 4 Girls
or ∴Required number of ways = 13C 4 + 13C 4 + 13C 4 + 13C 4
Take Take
or
Take Take = 4 ⋅ 13C 4
6 boys 1 girl 7 boys 0 girls
= 2860
(9C6 ways) (4C1 ways) (9C7 ways) (4C0 ways) (ii) There are 26 red cards and 26 black cards.
Hence, total number of ways (as these events are Two cards are choosed from red and two from black,
dependent to each other) ∴Required number of ways = 26 C 2 × 26 C 2
= ( 9 C 4 × 4C 3 ) + ( 9 C 5 × 4C 2 ) + ( 9 C 6 × 4C1 ) = 105625
+ ( C7 × C0 )
9 4 (iii) There are 12 face cards and we have to select 4 cards
out of these.
= ( C 4 × C1 ) + ( C 4 × C 2 )
9 4 9 4
∴Required number of ways = 12C 4
+ ( 9 C 3 × 4C1 ) + ( 9 C 2 × 4C 0 )
= 495
(Q n C r = n C n − r )
Chapter Test
(c) F.P.M.
l
Multiple Choice Questions (d) F.P.A.

1. If (n + 1)! = 12 × (n − 1)!, then the value of n is (iii) In how many ways exactly 3 bowlers must
include out of 11 players.
equal to (a) 3950 (b) 4950
(a) 4 (b) 3 (c) 5950 (d) 6950
(c) 16 (d) 11
(iv) If the number of ways of selecting exactly
1 1 x
2. If + = , then x is equal to 3 bowlers must include out of 11 players is abcd,
6! 7 ! 8! then value of (a + b + c + d) is
(a) 56 (b) 64 (a) 10 (b) 18
(c) 36 (d) 24 (c) 20 (d) 25
3. The number of 3 letter words, with or without (v) If the number of ways of selecting exactly
meaning which can be formed out of letters of 4 bowlers must include out of players is abcd,
the word ‘NUMBER’ and the repetition of then value of (a + b + c + d) is
letters is not allowed, is (a) 18 (b) 20
(a) 120 (b) 96 (c) 22 (d) 24
(c) 60 (d) 24

4. The number of possible outcomes when a coin


l
Short Answer Type Questions
is tossed 6 times, is 7. How many 3-digit numbers can be formed
(a) 36 (b) 64
from the digits 1, 2, 3, 4 and 5 assuming
(c) 12 (d) 32
(i) repetition of digits allowed?
5. A box contains 2 white balls, 3 black balls and
4 red balls. The number of ways three balls be (ii) repetition of digits not allowed?
drawn from the box, if atleast one black ball is 8. Find the value of 6 Pr , if 5Pr = 2 × 6 Pr − 1.
to be included in the draw is ......... .
(a) 48 (b) 60 9. How many different signals can be made by
(c) 62 (d) 64 using any number of flags from 6 flags of
different colours?
l
Case Based MCQs
6. In the BCCI board, Jagmohan Dalmia is a
l
Long Answer Type Questions
Cricket team selector. He selected a cricket 10. How many numbers are there between
team from 17 players in which only 5 players 100 and 1000 which have exactly one of their
can bowl. Then, answer the following question digits as 7?
which are based on above it.
11. A committee of 10 is to be formed from 8
(i) In how many ways exactly 4 bowlers must gentlemen and 9 ladies. In how many ways this
include out of 11 players. can be done, if atleast five ladies have to be
(a) 2960 (b) 3960 included in a committee? In how many of these
(c) 4960 (d) 5960
committees
(ii) Which concept is used for finding the number
(i) the ladies are in majority?
of required players?
(a) Permutation (ii) the gentlemen are in majority?
(b) Combination

Answers
For Detailed Solutions
1. (b) 2. (b) 3. (a) 4. (b) 5. (d)
Scan the code
6. (i) (b) (ii) (b) (iii) (b) (iv) (b) (v) (a) 7. (i) 125 (ii) 60
8. 120 9. 1956 10. 225 11. (i) 8156 (ii) 0
CBSE Term II Mathematics XI 71

CHAPTER 04

Conic Sections

In this Chapter...
l Circle
l Parabola
l Ellipse
l Hyperbola

Circle Thus, the circles ( x − h ) 2 + ( y − k ) 2 = r12


A circle is defined as the locus of a point in a plane, which and ( x − h ) 2 + ( y − k ) 2 = r 22 , r1 ≠ r 2 are concentric circles.
moves in a plane such that its distance from a fixed point in
that plane is always constant.
Centre The fixed point C is called the centre of the circle. r1 r2
Radius The constant distance ( CP ) from the centre ( C) to
a point on the circle, is called radius ( r ). C (h,k)

Standard Equation of a Circle


Let C ( h , k ) be the centre of the circle, P ( x, y ) be any point on
the circumference of the circle and r be the radius of the circle. Diameter Form of the Equation of a Circle
Then, equation of circle in standard form is Let ( x1 , y 1 ) and ( x 2 , y 2 ) be the end points of the diameter of
(x − h) + (y − k ) = r
2 2 2 a circle. Then, equation of circle drawn on the diameter is

Y ( x − x1 )( x − x 2 ) + ( y − y 1 )( y − y 2 ) = 0
P(x, y)
r
Position of a Point with respect to Circle
C (h, k) Let S ≡ (x − h)2 + (y − k )2 = r 2
be the equation of the circle and P ( x1 , y 1 ) be any point in
X′ X
the plane of the circle, then
Y′
S1 ≡ ( x1 − h ) 2 + ( y 1 − k ) 2 = r 2
This equation is also known as central form of the equation
of a circle. Case I If S1 > 0, then the point lies outside the circle.
Concentric Circles Case II If S1 = 0, then the point lies on the circle.
Two circles having the same centre C ( h , k ) but different radii Case III If S1 < 0, then the point lies inside the circle.
r1 and r 2 , are called concentric circles.
72 CBSE Term II Mathematics XI

Parabola (v) Latusrectum of a parabola is a line segment through


focus and perpendicular to the axis of the parabola,
A parabola is the locus of a point which moves in a plane such whose end points lie on the parabola i.e. LL ′ = 4 a.
that its distance from a fixed point is always equal to its
distance from a fixed straight line in the same plane. (vi) Coordinates of latusrectum = ( a , ± 2 a )
(vii) Equation of latusrectum is x = a or x − a = 0.
P2
(viii) It is symmetrical about X-axis.
(Fixed straight line)

B2
P1
B1 Left Handed Parabola
F (Fixed point) If parabola opens towards left side, then it is called left
handed parabola. The equation of the left handed parabola is
of the form
y 2 = − 4 ax, a > 0
From the figure, P1F = P1B1 and P2F = P2B 2 Some important terms related to this parabola are
Here, the fixed line is called the directrix and the fixed point Y
is called focus of the parabola. A line through the focus and
perpendicular to the directix is called the axis of the parabola L P(x, y)
and point of intersection of parabola with the axis is called the M

x=a
vertex of the parabola. O
X′ X
(0,0)
F(–a,0)
Fixed straight line
L′
A B
Fixed point
Y′

(i) Vertex is O ( 0 , 0 ).
(ii) Axis is the line y = 0.
Standard Equations of Parabola (iii) Focus is F( − a , 0 ).
If the equation of the parabola having vertex at origin and the (iv) Directrix is the line x − a = 0.
axis of symmetry is either X-axis or Y-axis, then the equations (v) Length of the latusrectum is 4a.
of parabola is said to be in simplest form. (vi) Coordinates of latusrectum = ( − a , ± 2 a )
There are four standard equations of parabola which are (vii) Equation of latusrectum is x = − a or x + a = 0.
given below.
(viii) It is symmetrical about X-axis.
Right Handed Parabola
Upward Parabola
If parabola opens towards right side, then it is called right
handed parabola. The equation of right handed parabola is of If parabola opens upward, then it is called upward parabola.
the form y 2 = 4 ax, a > 0 The equation of the upward parabola is of the form
Some important terms related to this parabola are x 2 = 4 ay , a > 0
Y
Some important terms related to this parabola are
P(x,y)
M L Y

O
X′ X
(0,0) F (a, 0) F(0,a)
L′ L
x+a=0

L′ y–a=0 P(x,y)

Y′ X′ X
O (0,0)
(i) Vertex is O (0, 0).
(ii) Axis is the line y = 0. y+a=0 M
(iii) Focus is F (a, 0).
(iv) Directrix is the line x + a = 0. Y′
CBSE Term II Mathematics XI 73

(i) Focus is F( 0 , a ). Y

(ii) Vertex is O( 0 , 0 ). y–a=0

(iii) Directrix is the line y + a = 0.


O (0,0)
(iv) Axis is the line x = 0. X′ X

(v) Length of latusrectum is 4a. P(x,y)

(vi) Coordinates of latusrectum = ( ± 2 a , a ) L′


y+a=0 F(0,–a)
L

(vii) Equation of latusrectum is y = a or y − a = 0.


(viii) It is symmetrical about Y-axis. Y′

(i) Focus is F( 0 , − a ). (ii) Vertex is O( 0 , 0 ).


Downward Parabola
(iii) Directrix is the line y − a = 0. (iv) Axis is the line x = 0.
If parabola opens downwards, it is called downward parabola. (v) Length of latusrectum is 4a.
The equation of downward parabola is of the form
(vi) Coordinates of latusrectum = ( ± 2 a , ( − a ))
x 2 = − 4 ay, a > 0 (vii) Equation of latusrectum is y = − a or y + a = 0.
Some important terms related to this parabola are (viii) It is symmetrical about Y-axis.

Table for different types of Parabola


Coordinates of
Parabola Vertex Focus Latusrectum Axis Directrix Symmetry
latusrectum
y 2 = 4ax ( 0, 0) ( a, 0) 4a (a, ± 2a) y=0 x=−a X-axis
y = − 4ax
2
( 0, 0) (− a, 0) 4a (− a, ± 2a) y=0 x=a X-axis
x = 4ay
2
( 0, 0) (0, a) 4a ( ± 2a , a ) x=0 y=−a Y-axis
x = − 4ay
2
( 0, 0) (0, − a) 4a ( ± 2a , − a ) x=0 y=a Y-axis

Ellipse l
Centre The mid-point of the line segment joining the foci,
An ellipse is the locus of a point in a plane which moves in is called centre of ellipse.
the plane in such a way that the ratio of its distance from a l
Major Axis The line segment through the foci of the
fixed point (called focus) in the same plane to its distance ellipse, is called the major axis. The length of major axis is
from a fixed straight line (called directrix) is always constant, denoted by 2a.
which is always less than unity. l
Minor Axis The line segment through the centre and
The constant ratio is denoted by e and is called the perpendicular to the major axis is called minor axis. The
eccentricity of the ellipse. length of minor axis is denoted by 2b.
In the given figure, F is the focus, AA′ is the directrix and P is 2a
any point on ellipse. Major axis
A′ C
Minor
axis
c c
P(x,y) A B 2b
M O
Vertex F1 F2 Vertex
Directrix

Latusrectum D
F(Focus) Latusrectum

A
l
Vertices The end points of the major axis, are called
vertices of the ellipse.
Then by definition,
PF
l
Eccentricity The eccentricity of ellipse is the ratio of the
= e, 0 < e < 1 ⇒ PF = e ⋅ PM distance from the centre of the ellipse to one of the foci and
PM to one of the vertices of the ellipse. It is denoted by e.
Terms Related to an Ellipse c
Thus, e = ⇒ c = ae
l
Focus Two fixed points are called the foci (plural of focus) a
c
of the ellipse and denoted by F1 and F 2 . The distance Since, c< a ⇒ < 1 ⇒ e < 1
between two foci F1 and F 2 is 2c. a
74 CBSE Term II Mathematics XI

l
Latusrectum Latusrectum of an ellipse is a line segment Some important terms related to vertical ellipse are
perpendicular to the major axis through any of the foci and (i) Centre is O( 0 , 0 ).
whose end points lie on the ellipse. (ii) Vertices are C ( 0 , a ) and D ( 0 , − a ).
2b2
Thus, the length of latusrectum = 2 l = (iii) Foci are F1 (0, c) and F 2 (0, − c).
a
(iv) Length of the major axis, CD = 2 a
Standard Equation of an Ellipse (v) Length of the minor axis, AB = 2 b
x2 y2 a2 − b2
The standard equation of ellipse is 2
+ 2
= 1 , a > b, having (vi) Eccentricity, e =
c
=
a b a a
centre and major axis lies on X-axis and minor axis lies on
(vii) Equation of major axis, X = 0
Y-axis. It is also called horizontal ellipse.
(viii) Equation of minor axis, Y = 0
Y
2b2
C (0,b) (ix) Length of latusrectum =
a
X′
A B
X  b2 
O (x) Coordinates of latusrectum =  ± , ± c
(–a,0) (a,0)  a 
D (0,–b)
Y′ Comparison between two Standard Ellipse
Some important terms related to horizontal ellipse are Horizontal ellipse Vertical ellipse
S. Terms x2
y2 x2 y2
(i) Centre is O (0, 0). + = 1 , a > b > 0 + 2 = 1, a > b > 0
No. 2
(ii) Vertices are ( ± a , 0 ). a2 b 2 b a

(iii) Foci are F1 ( − c, 0 ) and F 2 ( c, 0 ), where c = a 2 − b 2 . Y Y

(iv) Length of the major axis, AB = 2 a b


a
X′ X
(v) Length of the minor axis, CD = 2 b 1. Shape O a X′
O b
X

c a −b 2 2
(vi) Eccentricity, e = = Y′
a a Y′
(vii) Equation of major axis, Y = 0 2. Centre O( 0, 0) O( 0, 0)
(viii) Equation of minor axis, X = 0 3. Vertices (± a, 0) ( 0, ± a )
2b2
(ix) Length of latusrectum = 4. Major axis 2a 2a
a
 b2  5. Minor axis 2b 2b
(x) Vertices of latusrectum =  ± c, ± 
 a 6. Value of c c = a 2 − b2 c = a 2 − b2
(xi) Focal length = 2 c 7. Equation of y=0 x=0
major axis
Other form of an Ellipse
8. Equation of x=0 y=0
x2 y2 minor axis
The another form of the equation of an ellipse is + = 1,
b2 a2 a2 a2
9. Directrices x=± y=±
a > b, having centre at origin and major axis lies on Y-axis and c c
minor axis lies on X-axis. It is also called vertical ellipse.
Y 10. Foci (± c, 0) (0, ± c)
Z M
c b2 c b2
C(0, a) 11. Eccentricity e= = 1− 2 e= = 1− 2
a a a a
L′ L 12. Length of 2 b2 2 b2
F1
(0,ae) latusrectum a a
X′
O
X 13. Coordinates of  b2   b2 
(–b, 0)A B(b, 0)
latusrectum  ± c, ±  ± , ± c
 a  a 
L′1 F2 L1
(0,–ae) 14. Focal radius 1 | = ( a − ex1 ) and
|FP 1 | = ( b − ey1 ) and
|FP
D(0, –a) |F2P| = ( a + ex1 ) |F2P| = ( b + e y1 )

Y′ Z′ M′ 15. Focal distance 2c 2c


CBSE Term II Mathematics XI 75

c
Hyperbola ∴ e= and c > a
a
A hyperbola, is the locus of a point in a plane which moves in
c
the plane in such a way that the ratio of its distance from a ⇒ >1
fixed point (called focus) in the same plane to its distance a
from a fixed straight line (called directrix) is always constant, ⇒ e >1
which is always greater than unity.
l
Directrix Directrix is a line perpendicular to the
The constant ratio is denoted by e and is called the
eccentricity of the hyperbola. a2
transverse axis and cuts it at a distance of from the
In the given figure, F is the focus, AA′ is the directrix and P is c
any point on hyperbola. centre.
A′ a2
i.e. x=±
c
M P(x, y) a2
or y=±
c
Directrix

F (Focus)
l
Latusrectum Latusrectum of a hyperbola is a line segment
perpendicular to the transverse axis through any of the foci
A
and whose end points lie on the hyperbola.
2b2
Then by definition, Thus, length of latusrectum = 2 l =
PF a
= e, where e > 1
PM Standard Equation of Hyperbola
⇒ PF = e ⋅ PM x2 y2
Standard equation of hyperbola is the form of = 1, −
Terms Related to Hyperbola a2 b2
l
Focus The two fixed points are called the foci of the whose X-axis as transverse axis and Y-axis as conjugate axis.
hyperbola and denoted by F1 and F 2 . The distance Y
Conjugate axis
between two foci F1 and F 2 is 2c.
(a,0)
Vertices Conjugate axis (c,0)
A
X′ X
(– c,0) F2 B O F1
Latusrectum (–a,0)
Transverse
Transverse axis
(Focus) F2 axis Y′
F1 2a
Latusrectum
2c Some important terms related to standard hyperbola are
(i) Vertices are A( a , 0 ) and B′ ( − a , 0 ).
Directrices Directrices
(ii) Centre is O(0, 0).
l
Centre The mid-point of the line segment joining the foci, (iii) Length of transverse axis is 2 a.
is called centre of hyberbola. (iv) Length of conjugate axis is 2 b.
l
Transverse axis The line through the foci, is called the (v) Foci are F1 ( c, 0 ) and F 2 ( − c, 0 ).
transverse axis.
a2 a2
l
Conjugate axis The line through the centre and (vi) Directrices are x = and x = − .
c c
perpendicular to the transverse axis is called the conjugate
axis. a2 + b2
(vii) Eccentricity, e = or b 2 = a 2 ( e 2 − 1 )
l
Vertices The points at which the hyperbola intersects a2
the transverse axis are called the vertices of the hyperbola. 2b2
(viii) Length of latusrectum =
The distance between two vertices is 2a. a
Eccentricity Eccentricity of the hyperbola is the ratio of  b2 
(ix) Coordinates of latusrectum =  ± ae , ± 
l

the distance of anyone focus from the centre and the  a


distance of anyone vertex from the centre and it is denoted
(x) Focal length = 2 ae
by e.
76 CBSE Term II Mathematics XI

Conjugate Hyperbola  b2 
(ix) Coordinate of latusrectum =  ± , ± ae
y2 x2  a 
The equation of the hyperbola of the form = 1 is −
a2 b2 (x) Focal length = 2 ae
called conjugate hyperbola, whose X-axis as conjugate axis
and Y-axis as transverse axis. Comparison between two Standard Hyperbola
Y Conjugate
Transverse S.No. Terms Hyperbola
axis hyperbola
F1
Equation of x2 y 2 y 2 x2
hyperbola − =1 − =1
a 2 b2 a 2 b2
A
O Centre 1. Shape Y Y
X′ X
Conjugate axis

B Vertex
X′ X X′ X

F2
Y′ Y′
Y′
2. Transverse 2a 2a
Some important terms related to conjugate hyperbola are axis
(i) Vertices are A( 0 , a ) and B( 0 , − a ). 3. Conjugate 2b 2b
(ii) Centre is O (0, 0). axis

(iii) Transverse axis is 2 a. 4. Value of c c = a 2 + b2 c = a 2 + b2


(iv) Conjugate axis is 2 b.
5. Vertices ( ± a , 0) ( 0, ± a )
(v) Foci are ( 0 , ± c).
a2 a
a2 −a 2 6. Directrices a2 a y=± or ±
(vi) Directrices are y = and y = . x=± or ± c e
c c c e
7. Foci ( ± ae, 0) or ( ± c , 0) ( 0, ± ae) or ( 0, ± c )
b2
(vii) Eccentricity, e = 1 +
a2 Eccentricity b2 c b2 c
8. e= 1+ or e= 1+ or
or b 2 = a 2 ( e 2 − 1) a2 a a2 a

2b2 9. Length of 2 b2 2 b2
(viii) Length of latusrectum = latusrectum a a
a
CBSE Term II Mathematics XI 77

Solved Examples
Example 1. Find the equation of the circle with Example 4. Find the equation of circle whose centre is
(i) centre = (2, 3) and radius = 5 units (2, 0) and touches Y-axis.
Sol. Given, centre ( h , k ) = (2 , 0) and circle touches Y-axis.
(ii) centre = ( − a , − b ) and radius = a 2 − b 2 units
∴ Radius ( r ) = x-coordinate of centre = 2
[NCERT]
So, the equation of circle is
Sol. (i) Given, centre is (2, 3).
( x − 2 ) 2 + ( y − 0 ) 2 = (2 ) 2 [Q( x − h )2 + ( y − k )2 = r 2]
∴ h = 2, k = 3 and radius ( r ) = 5 units
On putting these values in equation of circle which is the required equation of circle.
( x − h )2 + ( y − k )2 = r 2, we get Example 5. Find the centre and radius of each of the
( x − 2 ) + ( y − 3) = 5
2 2 2 following circle.
which is the required equation of circle. (i) 3 x 2 + 3 y 2 = 27 (ii) x 2 + y 2 − 6 x + 5 y − 8 = 0
(ii) Given centre is ( − a , − b ). Sol. (i) Given, Equation of circle is 3x 2 + 3y 2 = 27.
∴ h = − a, k = − b and radius ( r ) = a − b units 2 2 or x 2 + y 2 = 9 [dividing both sides by 3]
On putting these values in equation of circle ⇒ ( x − 0) 2 + ( y − 0) 2 = 9
( x − h )2 + ( y − k )2 = r 2, we get ⇒ ( x − 0) 2 + ( y − 0) 2 = 3 2 …(i)
On comparing Eq. (i) with standard form of circle i.e.
[ x − ( − a )]2 + [ y − ( − b )]2 = ( a 2 − b 2 )2
( x − h )2 + ( y − k )2 = r 2, we get
⇒ ( x + a )2 + ( y + b )2 = a 2 − b 2
Centre ( h , k ) = ( 0, 0) and radius ( r ) = 3
which is the required equation of circle. (ii) Given, equation of circle is
Example 2. Find the equation of circle whose centre is x 2 + y 2 − 6x + 5y − 8 = 0
(1, 2) and which passes through the point (4, 6). ⇒ ( x 2 − 6x ) + ( y 2 + 5y ) = 8
Sol. Coordinates of centre of given circle is (1, 2) and it passes
  5 
2
 5
2
through the point (4, 6). ⇒ ( x 2 − 6x + 32 ) + y 2 + 5y +    = 32+   + 8
 2  2
Then, radius of the circle is equal to the distance from the  
centre to a point on a circle. 2
 5
[adding 32 and   on both sides]
∴ Radius of circle = (1 − 4)2 + (2 − 6)2  2
2
= ( − 3) 2 + ( − 4) 2  5 93
⇒ ( x − 3) 2 +  y +  = …(ii)
 2 4
= 9 + 16 = 5 units
On comparing Eq. (ii) with standard form of circle i.e.
[by distance formula, distance = ( x 2 − x1 )2 + ( y 2 − y1 )2 ] ( x − h )2 + ( y − k )2 = r 2, we get
Hence, the required equation of the circle is 5 93
( x − 1) 2 + ( y − 2 ) 2 = ( 5) 2 h = 3, k = − and r = units
2 2
Example 3. Find the equation of the circle whose Example 6. Find the equation of a circle passing
centre is ( a, b ) and passes through the origin. through the point (7, 3) having radius 3 units and
Sol. We know that, circle passes through the origin, so radius of whose centre lies on the line y = x − 1.
circle will be equal to the distance between point ( a , b ) and Sol. Let ( h , k ) be the centre of circle.
origin.
∴ ( h , k ) lies on the line y = x − 1, then k = h − 1
∴ Radius of circle = Distance between points (0, 0) and ( a , b ) Therefore, the equation of circle is
= ( 0 − a ) 2 + ( 0 − b )2 = a 2 + b 2 ( x − h )2 + [ y − ( h − 1)]2 = 32 …(i)
[by distance formula] Since, the circle passes through the point (7, 3).
Q Centre ( h , k ) = ( a , b ) ∴ (7 − h ) 2 + ( 3 − h + 1 ) 2 = 9
On putting these values in equation of circle ⇒ (7 − h ) 2 + ( 4 − h ) 2 = 9
( x − h )2 + ( y − k )2 = r 2, we get ⇒ 49 − 14h + h 2 + 16 − 8h + h 2 = 9
( x − a ) + ( y − b) = ( a + b )
2 2 2 2 2
⇒ h 2 − 11h + 28 = 0
which is the required equation of circle. ⇒ ( h − 7 )( h − 4) = 0 ⇒ h = 7 or h = 4
78 CBSE Term II Mathematics XI

When h = 7, then k = 7 − 1 = 6 and when h = 4, then Example 9. Find the length of the line segment joining
k = 4 −1 = 3
the vertex of the parabola y 2 = 4ax and a point on
Hence, the required equation of circles are
the parabola, where the line segment makes an
( x − 7 )2 + ( y − 6)2 = 32 or ( x − 4)2 + ( y − 3)2 = 32
angle θ to the X-axis.
Example 7. Does the point (−2.5, 3.5) lie inside, outside Sol. Let equation of parabola be
or on the circle x + y = 25?
2 2 y 2 = 4ax …(i)
Sol. Given, equation of circle is and its vertex is (0, 0).
S1 ≡ x + y = 25
2 2 Again, let any point on the parabola be P( h , k ).
On putting the values x = h and y = k in Eq. (i), we get
or S1 ≡ x 2 + y 2 − 25 = 0 …(i)
k 2 = 4ah …(ii)
Now, put x = − 2.5 and y = 3.5 in Eq. (i), we get
Let OP( = l ) be the line segment joining the vertex and
S1 = ( − 2.5)2 + ( 3.5)2 − 25
point P. Also, it makes an angle θ with the X-axis.
= 6.25 + 12.25 − 25
Y P(h, k )
= 18.5 − 25 = − 6.5 < 0
Since, S1 < 0, so the given point ( − 2.5, 3.5) lies inside the l k
circle. θ
X′ X
Example 8. Find the vertex, axis, focus, directrix and O h A
length of latusrectum of parabola y 2 − 8 y − x + 19 = 0.
Sol. Given equation is y 2 − 8y − x + 19 = 0 Y′
⇒ y 2 − 8y + 16 = x − 19 + 16
In right angled ∆OAP,
⇒ ( y − 4) 2 = x − 3 …(i) PA k
sin θ = ⇒ sin θ = ⇒ k = l sin θ
Let y − 4 = Y and x − 3 = X …(ii) OP l
Then, Eq. (i) becomes Y 2 = X …(iii) OA h
and cos θ = ⇒ cos θ = ⇒ h = l cos θ
Now, from Eq. (iii), coordinates of vertex are OP l
X = 0 and Y = 0 On substituting the values of h and k in Eq. (ii), we get
⇒ x − 3 = 0 and y − 4 = 0 l 2 sin 2 θ = 4al cos θ
[putting the values from Eq. (ii)] 4a cos θ
⇒ x = 3 and y = 4 ⇒ l =
sin 2 θ
The equation of axis of parabola (iii) is Y = 0. which is the required length.
⇒ y −4=0 ⇒ y =4
Example 10. If a parabolic reflector is 20 cm in
On comparing Eq. (iii) with Y 2 = 4aX , we get
diameter and 5 cm deep. Find the focus.
1
4a = 1 ⇒ a = Sol. Let POQ be the parabolic reflector which is 20 cm in
4 diameter and 5 cm deep.
Coordinates of focus of parabola (iii) are P(5, 10)
Y
1
X = a, Y = 0 ⇒ x − 3 = , y − 4 = 0
4
1 13 10 cm 20 cm
5 cm
⇒ x = + 3, y = 4 ⇒ x = and y = 4
4 4 X′ X
O R
Equation of directrix of parabola (iii) is 10 cm
1
X =−a ⇒ x−3=−
4 Y′ Q
1 11
⇒ x=− +3 ⇒ x= Then, PQ = 20 cm and OR = 5 cm, where R is the mid-point
4 4 of PQ. We take OX as X-axis and OY as Y-axis. The equation
1 of parabola may be taken as y 2 = 4ax. Since, the point
Length of latusrectum = |4a| = 4 ⋅ = 1 unit
4 P( 5, 10) lies on the parabola.
Hence, for given parabola vertex ≡ ( 3, 4), axis ≡ y = 4, ∴ 102 = 4a( 5) ⇒ a = 5
 13  11 Therefore, the coordinate of the focus are (a, 0) i.e. (5, 0).
focus ≡  , 4 , directrix ≡ x = and length of latusrectum
 4  4 Hence, the focus is the mid-point of the given diameter.
= 1 unit.
CBSE Term II Mathematics XI 79

Example 11. An equilateral triangle is inscribed in the Example 13. Find the equation of the ellipse, if foci are
parabola y = 4ax, where one vertex is at the vertex
2
( ± 3, 0) and a = 4. [NCERT]
of the parabola. Find the length of the side of the Sol. Given, foci are on X-axis. So, the major axis will be along the
triangle. X-axis. So, the equation of ellipse is of the form
x2 y 2
Sol. First, we draw the parabola in the positive side of X-axis and + = 1, a > b …(i)
inside that, draw an equilateral ∆OAB. a 2 b2
Let OB = l = OA = AB ⇒ ∠BOA = 60° ⇒ ∠BOP = 30° Also, we have a = 4 and c = 3
PB 1 PB l Q b2 = a 2 − c2 ⇒ b = a 2 − c2
In ∆BOP,sin 30° = ⇒ = ⇒ PB =
OB 2 l 2 = 16 − 9 = 7
OP 3 OP l 3 On putting the value of a and b in Eq. (i), we get
andcos 30° = ⇒ = ⇒ OP = x2 y 2
OB 2 l 2 + =1
16 7
Y
B x2 y 2
Hence, the required equation of an ellipse is + = 1.
16 7
l Example 14. Find the equation of the ellipse, if length
X′
30° P
X
of major axis is 26 units and foci (± 5, 0). [NCERT]
O 60° Sol. We have, foci (±5, 0) which are on X-axis, so the equation of
ellipse is of the form
x2 y 2
+ = 1, a > b …(i)
A a 2 b2
Y′
Since, foci ≡ (± c, 0) = (± 5, 0) ⇒ c = 5
 l 3 l Length of major axis = 2 a = 26 ⇒ a = 13
∴ Coordinates of B = ( OP , PB ) =  ,  will satisfy We know that, c2 = a 2 − b2
 2 2
y 2 = 4ax ⇒ b = a 2 − c 2 = 169 − 25 = 144
2 ∴ b = 12
 l 4a × l 3
i.e.   = On putting the value of a and b in Eq. (i), we get
 2 2 x2 y2
+ =1
l 2 4a l 3 169 144
⇒ =
4 2 x2 y2
Hence, the required equation of an ellipse is + =1
⇒ l = 8 3a 169 144

Hence, the length of side of the triangle is 8 3a units. Example 15. Find the equation of the ellipse having
length of minor axis is 16 units and foci ( 0, ± 6).
Example 12. Find the equation of the ellipse whose [NCERT]
axes are along the coordinate axes, vertices are Sol. Length of minor axis, 2 b = 16 ⇒ b = 8 units
( ± 5, 0) and foci at ( ± 4, 0). [NCERT]
Foci are ( 0, ± 6) ⇒ c = 6
Sol. Given, vertices ≡ ( ± 5, 0) and foci ≡ ( ±4,0). ∴ a 2 = b 2 + c 2 = ( 8)2 + ( 6)2 = 64 + 36 ⇒ a 2 = 100
Q y-coordinate of vertices are zero. So, let equation of ellipse Here, coefficient of x in foci is zero. So, major axis is on
x2 y 2 Y-axis. Then, equation of ellipse is
be 2 + 2 = 1, a > b
a b x2 y2
+ =1
Now, vertices ≡ ( ± a , 0) = ( ± 5, 0) 64 100
∴ a = 5 and foci ≡ ( ± a e,0) = ( ± 4, 0) Example 16. Find the equation of the ellipse, if the
∴ ae = 4 ends of major axis are ( ± 3, 0) and ends of minor axis
⇒ e= =
4 4
[Q a = 5]
are ( 0, ± 2 ). [NCERT]
a 5 Sol. Since, the vertices are on X-axis and, therefore the equation
 16 x2 y 2
Now, b 2 = a 2(1 − e2 ) = 251 −  is of the form 2 + 2 = 1, a > b
 25 a b
(25 − 16) Since, major and minor axes are ( ± 3, 0) and ( 0, ± 2 ) ,
= 25 =9 respectively.
25
∴ a = 3 and b = 2
x2 y 2
Hence, the required equation of ellipse is + = 1. x2 y 2
25 9 Hence, the required equation of an ellipse is + = 1.
9 4
80 CBSE Term II Mathematics XI

Y (0, a)
Example 17. Find the equation of ellipse, if it satisfies
the condition b = 3 and c = 4, centre at origin, foci
on the X-axis.
(–b,0) (b,0)
Sol. Given, foci lies on X-axis. So, the equation of ellipse will be X′ X
of the form
x2 y 2
+ = 1, a > b ...(i)
a 2 b2 (0,– a)
Y′
Also given that, b = 3 and c = 4
∴ c 2 = a 2 − b 2 ⇒ ( 4)2 = a 2 − 9 ⇒ 16 = a 2 − 9 Thus, the equation of ellipse is passing through the points
(3, 2) and (1, 6). So, the point (3, 2) lies on Eq. (i), which gives
⇒ a = 16 + 9 ⇒ a = 25
2 2
9 4
+ =1 …(ii)
On putting the values of a 2 = 25 and b 2 = 9 in Eq. (i), we get b2 a 2
x2 y 2 Also, the point (1, 6) lies on Eq. (i).
+ =1 1 36 9 324
25 9 ∴ + =1 ⇒ 2 + 2 = 9
which is the required equation of ellipse. b2 a 2 b a
[multiplying by 9 on both sides] …(iii)
Example 18. Find the equation of the ellipse, whose On subtracting Eq. (ii) from Eq. (iii), we get
distance between directrices is 5 units and distance 320 320
between foci is 4 units. = 8 ⇒ a2 = = 40
a2 8
Sol. Let the equation of ellipse be
On putting the value of a 2 in Eq. (ii), we get
x2 y 2 9 4 9 1 9
+ = 1, a > b + =1 ⇒ 2 =1− = ⇒ b 2 = 10
a 2 b2 b 2 40 b 10 10
Then, equation of directrices are On putting the values of a and b in Eq. (i), we get
a2 a2 a x2 y 2
x=± =± =± + =1
c ae e 10 40
a a which is the required equation of ellipse.
i.e. x = and x = −
e e
2a
Example 20. Draw the shape of the hyperbola
∴ Distance between directrices = =5
e x 2 y2
2a
− = 1 and find their shape, centre, transverse
∴ e= 49 9
5 axis, conjugate axis, value of c, vertices, directrices
Also, its foci are ( ± c , 0) or ( ± ae, 0). and foci.
∴ Distance between foci = 2 ae = 4 x2 y 2
Sol. We have, equation of hyperbola − =1
⇒ ae = 2 49 9
2a  2a  x2 y 2
⇒ a   = 2 ⇒ a2 = 5 Q put e = On comparing it with − = 1, we get
 5  5  a 2 b2
2 5 2 a = 7 and b = 3
∴ e= = Shape of hyperbola is standard hyperbola, because
5 5
coefficient of y 2 is negative.
and b = a (1 − e )
2 2 2
N Y M
(7,0) Directrix
(–7,0) Directrix

 4 1
⇒ b 2 = 5 1 −  = 5 × [Q put a 2 = 5]
 5 5
∴ b2 = 1 XN X
(– √58, 0) O (√58, 0)
Hence, the required equation of ellipse is
x2 y 2
+ =1 [Q put a 2 = 5 and b 2 = 1]
5 1 NN YN MN
or x + 5y = 5
2 2 (i) Centre (0, 0)
(ii) Length of transverse axis = 2 a = 2 × 7 = 14 units
Example 19. Find the equation of the ellipse, if the (iii) Length of conjugate axis = 2 b = 2 × 3 = 6 units
centre is at (0, 0), major axis on the Y-axis and (iv) Value of c = a 2 + b 2 = 49 + 9 = 58
passing through the points (3, 2) and (1, 6). [NCERT] (v) Vertices = ( ± a , 0) = ( ± 7 , 0)
Sol. Since, major axis is along Y-axis, so equation of ellipse is a2 49
(vi) Directrices, x = ± =±
x2 y 2 c 58
+ = 1, a > b …(i)
b2 a 2 (vii) Foci = ( ± c , 0) = ( ± 58, 0)
CBSE Term II Mathematics XI 81

Example 21. Find the equation of the hyperbola, whose Example 23. Find the equation of the hyperbola whose
vertices are ( 0, ± 5) and foci ( 0, ± 8 ). [NCERT] foci are at (0, ± 6) and length of whose conjugate
Sol. We have, vertices ( 0, ± a ) = ( 0, ± 5) ⇒ a = 5 axis is 2 11 units.
and foci ( 0, ± c ) = ( 0, ± 8) ⇒ c = 8 Sol. We have, foci of the hyperbola lies on Y-axis.
We know that, c 2 = a 2 + b 2 ⇒ 64 = 25 + b 2 So, the equation of hyperbola is
⇒ b 2 = 64 − 25 ⇒ b 2 = 39 y 2 x2
− =1
a 2 b2
Here, the foci and vertices lie on Y-axis, therefore the
equation of hyperbola is of the form Foci ≡ ( 0, ± c ) = ( 0, ± 6) ⇒ c = 6
y 2 x2 Length of its conjugate axis = 2b
− =1 ∴ 2 b = 2 11 ⇒ b = 11
a 2 b2
y 2 x2 Now, c2 = a 2 + b2
i.e. − =1
25 39 ⇒ 62 = a 2 + ( 11 )2 [Q c = 6 and b = 11 ]
which is the required equation of hyperbola. ⇒ a = 36 − 11 = 25
2

Example 22. Find the equation of hyperbola, when foci Thus, a 2 = 25 and b 2 = 11
are at ( ± 5, 0) and transverse axis is of length 8 units. Hence, the required equation of hyperbola is
[NCERT]
y 2 x2
Sol. Here, foci are at ( ± 5, 0). − =1
25 11
∴ ( ± c , 0) = ( ± 5, 0)
Example 24. Find the equation of the locus of all points
⇒ c=5
such that difference of their distance from (4, 0) and
and length of transverse axis = 2 a = 8
(− 4, 0) is always to 2 units.
⇒ a = 4 units Sol. We have, foci of the hydperbola lies on X-axis. So, the
Conjugate axis equation of hyperbola is
x2 y 2
− =1 …(i)
a 2 b2
Transverse axis
(–5, 0) (5, 0) Let the given points be F1 (4, 0) and F2( − 4, 0).

1 2 = ( 4 + 4) + ( 0 − 0) = 8
2 2
Now, FF
Also, we know that, c 2 = a 2 + b 2 We know that, 1 2 = 2c
FF
⇒ 25 = 16 + b 2
[Q a = 4 and c = 5] ∴ 2c = 8 ⇒ c = 4
Let P( x , y ) be any point on the hyperbola.
⇒ b =9
2
∴ |PF1 − PF2| = 2
Since, the foci lie on X-axis. Now, |PF1 − PF2| = 2 a
Therefore, the equation of hyperbola is of the form ∴ 2a = 2 ⇒ a = 1
x2 y 2 Also, c2 = a 2 + b2
− =1 …(i)
a 2 b2 ⇒ 16 = 1 + b 2 ⇒ b 2 = 15 [Q c = 4 and a = 1]
On putting the values of a 2and b 2 in Eq. (i), we get On putting the values of a = 1 and b = 15 in Eq. (i), we get
2 2

x2 y 2 x2 y 2
− =1 − = 1 i.e. 15x 2 − y 2 = 15
16 9 1 15
which is the required equation of hyperbola. which is the required equation of hyperbola.
82 CBSE Term II Mathematics XI

Chapter
Practice
PART 1
Objective Questions
l
Multiple Choice Questions (a) y 2 =
25
x (b) x 2 =
25
y
2 2
1. The equation of the circle with centre ( − 3, 2 ) and 5 5
(c) y 2 = x (d) x 2 = y
radius 4 units, is 2 2
(a) ( x − 3)2 + ( y − 2 )2 = 16 (b) ( x + 3)2 + ( y + 2 )2 = 16 7. If the focus of a parabola is (0, − 3) and its directrix
(c) ( x − 3)2 + ( y + 2 )2 = 16 (d) ( x + 3)2 + ( y − 2 )2 = 16 is y = 3, then its equation is
(a) x 2 = − 12 y (b) x 2 = 12 y
2. If the equation of circle is x 2 + y 2 − 2 x + 4y − 8 = 0,
(c) y 2 = − 12 x (d) y 2 = 12 x
then its centre (C ) and radius (r) respectively are
(a) (–2, 4) and 13 units (b) (–1, 2) and 13 units 8. If the parabola y 2 = 4ax passes through the point
(c) (1, –2) and 13 units (d) (2, –4) and 13 units (3, 2), then the length of its latusrectum is
2 4
3. The equation of the circle having centre at ( − 3, 5) (a) units (b) units
3 3
and touching the line 7 x − 8 y + 8 = 0 is 1
2 (c) units (d) 4 units
 53  3
(a) ( x + 3) + ( y − 5) = 
2 2

 113  9. If the focal distance of a point on the parabola
2
 53 
(b) ( x − 3)2 + ( y − 5)2 =  y 2 = 12 x is 4 units, then the abscissa of that point is

 113  (a) 5 (b) 1 (c) 2 (d) − 1
2
 53  10. In an ellipse, if ends of major axis are ( 0, ± 5 ) and
(c) ( x + 3)2 + ( y − 5)2 =  
 113
2
ends of minor axis are ( ±1 , 0), then the equation of
 53  ellipse is given by
(d) ( x − 3)2 + ( y + 5)2 =  
 113
x2 y 2 x2 y 2
(a) + =1 (b) + =1
4. If the equation of parabola is y = 12 x, then2 1 5 2 5
(a) focus = ( 3 , 0) x2 y 2 x2 y 2
(c) + =1 (d) + =1
(b) length of latusrectum is 12 units 5 1 5 5
(c) directrix is x = − 3 and axis of symmetry is X-axis 11. In an ellipse, if length of major axis is 26 units
(d) All of the above and foci are ( ± 5, 0), then the equation of ellipse is
5. If parabola is passing through (2, 3), vertex (0, 0) given by
and axis is along X-axis, then the equation of x2 y2 x2 y2
(a) + =1 (b) + =1
parabola is 144 169 169 144
9 9
(a) y 2 = x (b) x 2 = y x2 y2
2 2 (c) − =1 (d) None of these
3 3 169 144
(c) y 2 = x (d) x 2 = y
2 2 12. If the foci and vertices of an ellipse be ( ±1 , 0) and
6. If parabola is passing through (5, 2), vertex (0, 0) ( ±2 , 0) respectively, then the minor axis of the
and symmetric with respect to Y-axis, then the ellipse is
equation of parabola is (a) 2 5 units (b) 2 units (c) 4 units (d) 2 3 units
CBSE Term II Mathematics XI 83

1 x2 y 2 x2 y 2
13. The length of the latusrectum of an ellipse is unit (a) − =1 (b) − =1
3 64 36 16 36
of the major axis. Its eccentricity is y 2 x2 y 2 x2
4
(c) − =1 (d) − =1
2 2 5×4×3  3 64 36 36 64
(a) (b) (c) (d)  
3 3 73  4 3
21. The equation of the hyperbola with eccentricity
14. The length of the latusrectum of the ellipse 2
and foci at ( ± 2 , 0) is
3x 2 + y 2 = 12 is
x2 y 2 4 x2 y 2 4
4 (a) − = (b) − =
(a) 4 units (b) 3 units (c) 8 units (d) units 4 5 9 9 9 9
3
x2 y 2
2 (c) − =1 (d) None of these
15. The equation of ellipse whose eccentricity is , 4 9
3
latusrectum is 5 units and the centre is (0, 0) is l
Case Based MCQs
given by
4x 2 4y 2 4x 2 4y 2
22. A man running on a race course notices that sum of
(a) + =1 (b) + =1 its distances from two flag posts from him is always
45 81 81 45
10 m and the distance between the flag posts is 8 m.
4x 2 y 2
(c) + =1 (d) None of these He notes that he can read the messages of value
81 45
system ‘Honesty’ and ‘Respect for other’ on the
x 2 y2 poles which ever side he moves, then answer the
16. If the equation of ellipse is + = 1, then which
49 36 following questions which are based on above it.
of the following is not correct? (i) The path traced by the man will be
(a) Foci = ( ± 13 , 0) and vertices = ( ± 7 , 0) (a) an ellipse (b) a parabola
(b) Length of major axis is 14 units and length of minor axis (c) a hyperbola (d) a circle
is 12 units (ii) Value of a for the standard equation of path is
13
(c) Eccentricity is (a) 10 (b) 5 (c) 15 (d) 20
7
72 (iii) Value of b for the standard equation of path is
(d) Length of latusrectum is units (a) 3 (b) 4 (c) 5 (d) 6
7
(iv) Equation of path is
17. The equation of the hyperbola whose conjugate axis
x2 y 2 x2 y 2
is 5 and the distance between the foci is 13 units, is (a) + =1 (b) + =1
x 2 4y 2 4x 2 y 2 9 25 16 25
(a) − =1 (b) − =1 x2 y 2
36 25 25 36 (c) + =1 (d) x 2 + y 2 = 1
x2 y 2 25 9
(c) − =1 (d) None of these
36 36 (v) Value of (2 a + b ) is
(a) 10 (b) 13 (c) 15 (d) 18
18. The distance between the foci of a hyperbola is
16 units and its eccentricity is 2. Its equation is 23. Due to heavy storm, an electric wire got bent as
x y2 2
x2
y 2 shown in figure. It followed a mathematical shape.
(a) − =1 (b) − =1 Answer the following question below.
32 32 4 9
x2 y 2 Y
(c) − =1 (d) None of these
32 9
19. If the vertices of hyperbola are ( ±7 , 0) and its (0, 2)

4 11 2
eccentricity is , then the equation of hyperbola is X′
( –3, 0) (3, 0)
X
3 –2 –1 –1
x2 y2 x 2 9y 2 (0, –2)
(a) − =1 (b) − =1
49 343 49 343
2 2 2 2
9y x x 9y
(c) − =1 (d) − =1 Y′
343 49 343 49
20. The equation of the hyperbola with vertices at (i) Name the shape in which the wire is bent
(a) circle (b) parabola
5
( 0, ± 6) and eccentricity is (c) ellipse (d) hyperbola
3
84 CBSE Term II Mathematics XI

(ii) The equation of the shape of curve is 2 at


6. Show that the points ( x , y) given by x = and
x2 y 2 1 + t2
(a) + =1
9 4 a(1 − t 2 )
y= lies on a circle for all real values of t
x2 y 2 1 + t2
(b) + =1
4 9 such that − 1 ≤ t ≤ 1, where a is any given real
x2 y 2 numbers.
(c) − =1
9 4 7. Find the equation of the circle, whose centre is
(d) None of the above (2 , − 3) and passing through the intersection of the
(iii) The eccentricity of the given shape is lines 3x − 2 y = 1 and 4x + y = 27.
2 x
(a) (b) 8. Find the equation of a circle whose diameter
3 3 are 2 x − 3y + 12 = 0 and x + 4y − 5 = 0 and area is
5 5 154 sq units.
(c) (d)
3 4
9. Find the equation of the circle which passes
(iv) The length of the latusrectum of the shape is through the points (2 ,−2 ) and ( 3 , 4) and whose
(a) 9 units centre lies on the line x + y = 2.
8
(b) units 10. The equation of the circle passing through the
3
4
points (4, 1) and (6, 5) and whose centre lies on the
(c) units line 4x + y = 16.
3
(d) None of the above 11. Find the equation of circle whose centre is (3, − 1)
(v) Foci of the shape is and which cuts off a chord of length 6 units on the
(a) ( ± 3, 0) (b) ( ± 5 , 0) line 2 x − 5y + 18 = 0.
(c) ( 0, ± 2 ) (d) ( 0, ± 5) 12. Find the focus and directrix of the parabola
3x 2 + 12 x + 8 y = 0.
13. Find the coordinates of a point on the parabola
PART 2 y 2 = 8 x, whose focal distance is 4 units.

Subjective Questions 14. Find the equation of the parabola, whose focus
is the point (4, 0) and whose directrix is x = − 4.
Also, find the length of latusrectum.
l
Short Answer Type Questions 15. The relationship between semi-major axis (a),
1. Find the equation of a circle which passes through semi-minor axis (b) and the distance of focus from
(3, 6) and touches the axes. the centre ( c ) of the ellipse.
2. Find the equation of a circle which touches both 16. Prove that the line lx + my + n = 0 will touch the
the axes and the line 3x − 4y + 8 = 0 and lies in third parabola y 2 = 4ax, if ln = am 2 .
quadrant.
17. Find the area of the triangle formed by the lines
3. A circle of radius 2 units lies in the first quadrant joining the vertex of the parabola x 2 = − 36y to the
and touches both the axes of coordinates. Find the
ends of the latusrectum.
equation of the circle with centre at ( 6, 5) and
touching the above circle externally. 18. An arch is in the form of a parabola with its axis
vertical. The arch is 10 m high and 5m wide at the
4. The abscissa of two points A and B are the roots of
base. It is ‘k’ m wide when it is 2m from the vertex
the equation x 2 + 2 ax − b 2 = 0 and their ordinates of the parabola. Then, find the value of k.
are the roots of the equation x 2 + 2 px − q 2 = 0. Find
19. A beam is supported at its ends by supports which
the equation and the radius of the circle with AB as are 12 m apart. Since, the load is concentrated at its
diameter. centre, there is deflection of 3 cm at the centre and
5. The number of equation of the circle with radius the deflected beam is in the shape of a parabola.
5 units whose centre lies on X-axis and passes What is the distance between deflection of 1cm and
through the point (2, 3). the centre.
CBSE Term II Mathematics XI 85

20. Find the coordinates of the foci, the vertices, the 32. A rod of length 12 cm moves with its ends always
length of major axis, the minor axis, the eccentricity touching the coordinate axes. If point P is on the
and the length of the latusrectum of the ellipse rod, which is 3 cm from the end in contact with the
x 2 y2 X-axis, then find the equation of locus of point P.
+ = 1.
4 25 33. Find the foci, vertices, eccentricity and latus
21. Find the equation of ellipse, whose axes are along rectum of the equation 5y 2 − 9x 2 = 36.
the axes of coordinates and centre at the origin and 34. Find the equation of the hyperbola whose
axes whose latusrectum is 8 units and eccentricity 3
1 eccentricity is and foci are (± 2, 0).
is . 2
2 35. Find the equation of hyperbola, if vertices are at
22. Find the equation of an ellipse whose foci are 4
1 (± 7, 0) and e = .
(± 4, 0) and the eccentricity is . 3
3
36. If e and e′ are the eccentricities of the hyperbola
23. If the major axis of the ellipse is on the X-axis and it x 2 y2
passes through the points (4, 3) and (6, 2) and the − = 1 and its conjugate hyperbola, then prove
a2 b2
centre of the ellipse is at (0, 0), then find the 1 1
equation of ellipse. that 2 + 2 = 1.
e e′
24. Find the equation of the ellipse, whose foci are
(± 3, 0) and passing through (4, 1). 37. Find the equation of hyperbola whose foci are
(0, ± 12) and the length of latusrectum is 36 units.
25. Find the equation of the ellipse whose centre
y2 x 2
lies at origin, major axis lies on the X-axis, the 38. Draw the shape of the hyperbola − = 1 and
1 9 27
eccentricity is and the length of the latusrectum find their centre, transverse axis, conjugate axis,
3 value of c, vertices, directrices, foci, eccentricity
4
is units. and latusrectum.
9
39. Find the eccentricity of the hyperbola whose
5 latusrectum is 8 units and conjugate axis is equal to
26. If the eccentricity of an ellipse is and the distance
8 half of the distance between the foci.
between its foci is 10 units, then find latusrectum of 40. If the vertices of hyperbola are ( ± 6, 0) and are of
the ellipse. the directrices is x = 4, then find the equation of
27. Find the distance between the directrices of ellipse hyperbola.
x 2 y2 41. Find the length of transverse axis of the hyperbola
+ = 1.
36 20 3x 2 − 4y 2 = 32.
28. Find the equation of ellipse having major and minor
axes along X and Y-axes respectively, the distance
l
Long Answer Type Questions
between whose foci is 8 units and the distance 42. If the circle passes through the point (2,3), then
between the directrices is 18 units. find the equation of the circle whose radius is
29. Let the centre of an ellipse is at (0, 0). If major axis 5 units and centre lies on X-axis.
is on the Y-axis and ellipse passes through the points 43. Find the equation of the circle passing through the
(3, 2) and (1, 6), then find the equation of ellipse. points (1 , − 2 ) and ( 4, − 3) and whose centre lies on
the line 3x + 4y = 7.
30. Find the equation of an ellipse whose axis lie along
the coordinate axes, which passes through the point 44. The equation of the circle passing through the
points (2, 3) and (–1, 1) and whose centre is on the
( −3, 1 ) and has eccentricity equal to 2 / 5. 2 2
 a  b
31. If the latusrectum of an ellipse is equal to half of line x − 3y − 11 = 0, is  x +  +  y +  = d, then
 2  2
minor axis, then find its eccentricity.
find the values of a, b and d.
86 CBSE Term II Mathematics XI

45. Find the axis, vertex, directrix and length of 48. Find the foci ( f ), vertices ( v), length of major axis
latusrectum of the parabola (L 1 ), length of minor axis (L 2 ), eccentricity ( e) and
9y 2 − 16x − 12 y − 57 = 0. length of latusrectum ( l ). When the equation of the
x2 y2
46. The cable of a uniformly loaded suspension bridge curve is + = 1.
hangs in the form of a parabola. The roadway which 100 400
is horizontal and 100 m long is supported by vertical 49. An arch is the form of a semi-ellipse. It is 8 m wide
wires attached to the cable, the longest wire being and 2 m high of the centre. Find the height of the
30 m and the shortest being 6 m. Find the length of arch at a point 1.5 m from one end.
a supporting wire attached to the roadway 18 m 50. Draw the shape of the hyperbola 5y 2 − 9x 2 = 36
from the middle.
and find its centre, transverse axis, conjugate axis,
x 2 y2 value of c, vertices, directrices, foci, eccentricity
47. Draw the shape of the ellipse + = 1 and find
36 16 and length of latusrectum.
their major axis, minor axis, value of c, vertices, 51. Foci ( f ) and length of latustrectum ( l ) of hyperbola
directrices, foci, eccentricity and length of
latusrectum. is given by f ( ± 3 5 , 0) and l(8 ) respectively. Find
the equation of hyperbola.

SOLUTIONS
Objective Questions Here, 4a = 12
1. (d) Here, h = − 3, k = 2 and r = 4. ⇒ a=3
Therefore, the equation of the required circle is ∴ Focus = ( a , 0) = ( 3, 0)
( x + 3)2 + ( y − 2 )2 = 16 Axis = X-axis
2. (c) We write the given equation in the standard form as Directrix, x = − a
( x 2 − 2 x ) + ( y 2 + 4y ) = 8 ⇒ x = −3
Now, completing the squares, we get ∴ Length of latusrectum = 4a = 4 × 3 = 12 units
5. (a) Given, vertex = ( 0, 0)
( x 2 − 2 x + 1) + ( y 2 + 4y + 4) = 8 + 1 + 4
Point = (2 , 3) and axis = X-axis
( x − 1)2 + ( y + 2 )2 = 13
Since, point (2 , 3) lies in first quadrant and axis is X-axis.
Comparing it with the standard form of the equation of the Hence, equation of parabola will be of the form y 2 = 4ax,
circle, we see that the centre of the circle is (1, − 2 ) and
which passes through (2, 3) i.e.
radius is 13 units.
Put x = 2 and y = 3 in y 2 = 4ax
3. (a) The perpendicular distance from centre ( − 3, 5) to the
9
7( − 3) − 8( 5) + 8 ⇒ ( 3 ) 2 = 4 a × (2 ) ⇒ a =
line is, d = 8
(7 ) 2 + ( − 8 ) 2 Hence, required equation of parabola is
− 21 − 40 + 8 − 53 53  9 9
= = = units y2 = 4   x ⇒ y2 = x
49 + 64 113 113  8 2
6. (b) Given, vertex = ( 0, 0)

C(–3, 5) Point = ( 5, 2 )
Since, point ( 5, 2 ) lies in first quadrant and axis is Y-axis.
Therefore, parabola is symmetric with respect to Y-axis.
A B Hence, equation of parabola will be of the form x 2 = 4ay,
P
which passes through (5, 2) i.e.
So, the required equation of the circle is Put x = 5, y = 2 in x 2 = 4ay
2
 53  25
( x + 3) 2 + ( y − 5) 2 =   ∴ ( 5) 2 = 4a × 2 ⇒ a =
 113  8
4. (d) Given, equation of parabola is y 2 = 12 x Hence, required equation of parabola is
which is of the form y 2 = 4ax 25 25
x2 = 4 × y ⇒ x2 = y
i.e. focus lies on the positive direction of X-axis. 8 2
CBSE Term II Mathematics XI 87

7. (a) Given that, focus of parabola at F ( 0, − 3) and equation of 12. (d) Given that, ae = 1, a = 2 and e =
1
directrix is y = 3. 2
Let any point on the parabola is P ( x , y ).  1  b2 
⇒ b = 4 1 −  = 3 Q e = 1 − 2 
Then, PF = y − 3  4  a 
⇒ ( x − 0) 2 + ( y + 3) 2 = y − 3
Hence, minor axis 2 3 units.
⇒ x 2 + y 2 + 6y + 9 = y 2 − 6y + 9 13. (b) Here, Length of Latusrectum = 1/3 unit (major axis)
⇒ x 2 + 12 y = 0 ⇒ x 2 = − 12 y 2 b2 2 a
∴ =
8. (b) Given that, parabola is a 3
y 2 = 4ax ... (i) ⇒ a 2 = 3 b 2 = 3 a 2 (1 − e 2 )
∴ Length of latusrectum = 4a 
2 b2 
Since, the parabola passes through the point ( 3, 2 ). ⇒ e= Q e = 1 − 2 
3  a 
Then, 4 = 4 a ( 3) ⇒ a = 1 / 3
∴ 4 a = 4 / 3 units 14. (d) Given, equation of ellipse is
9. (b) Given parabola is y 2 = 12 x. 3x 2 + y 2 = 12
Here, 4a = 12 ⇒ a = 3 x2 y 2
⇒ + =1
∴ Focus, F = ( 3, 0) 4 12
Let P( x , y ) be any point on the parabola, then PF = 4 units ∴ b2 = 4 ⇒ b = 2
⇒ ( x − 3) 2 + ( y − 0) 2 = ( 4) 2 and a 2 = 12 ⇒ a = 2 3
⇒ x + 9 − 6x + 12 x = 16
2
[Q y = 12 x ]
2
Q a>b
⇒ x + 6x − 7 = 0 ⇒ x = 1
2
[Q x ≠ − 7 ] 2×4 4 2 b2
∴ Length of latusrectum = = = units.
⇒ x + 7x − x − 7 = 0
2
2 3 3 a
[by splitting the middle term] 15. (b) Given that, e = 2 / 3 and latusrectum = 5 units
⇒ x (x + 7) − 1 (x + 7) = 0 2 b2 5a
⇒ ( x + 7 ) ( x − 1) = 0 i.e. = 5 ⇒ b2 =
a 2
10. (a) Since, ends of major axis ( 0, ± 5 ) are along Y-axis and We know that, b 2 = a 2 (1 − e 2 )
minor axis ( ± 1, 0) are along X-axis. Hence, equation of
5a  4
ellipse will be of the form ⇒ = a 2 1 − 
2  9
x2 y 2
+ =1 …(i) 5 5a 81
b2 a 2 ⇒ = ⇒ a = 9 / 2 ⇒ a2 =
where, a= 5 2 9 4
5 × 9 45
and b =1 ⇒ b =
2
=
2 ×2 4
x2 y2
Hence, Eq. (i) becomes 2 + =1
1 ( 5 )2 4x 2 4y 2
So, the required equation of the ellipse is + = 1.
81 45
x2 y 2
or + =1 x2 y 2
1 5 16. (c) Given, equation of ellipse is + =1
49 36
11. (b) Since, foci ( ± 5, 0) lie on X-axis, as y-coordinate is zero.
x2 y2
Hence, equation of ellipse will be of the form Q Denominator of is larger than the denominator of .
49 36
x2 y 2
+ =1 …(i) ∴ The major axis is along X-axis.
a 2 b2
x2 y 2
Given that, length of major axis 2 a = 26 On comparing given equation with 2 + 2 = 1,we get
a b
⇒ a = 13 and c = 5
a 2 = 49 and b 2 = 36 ⇒ a = 7 and b = 6
Q c2 = a 2 − b2
Here, a and b are lengths, so we take only positive sign.
⇒ ( 5)2 = (13)2 − b 2
Now, c 2 = a 2 − b 2 = 49 − 36 = 13
⇒ 25 = 169 − b 2
⇒ c = 13
⇒ b 2 = 169 − 25 = 144 ⇒ b = 12
Here, major axis is along X-axis. So,
Put the values of a and b in Eq. (i), we get
∴ Foci = ( ± c , 0) = ( ± 13 , 0)
x2 y2
+ =1 Vertices = ( ± a , 0) = ( ± 7 , 0)
169 144
88 CBSE Term II Mathematics XI

Length of major axis = 2 a = 2 × 7 = 14 units 784


= 49 + b 2
Length of minor axis = 2 b = 2 × 6 = 12 units 9
c 13 784 49
Eccentricity, e = = ⇒ b2 = −
a 7 9 1
2 b 2 2 × 36 72 784 − 441
∴ Length of latusrectum = = = units ⇒ b2 =
a 7 7 9
343
17. (a) Given, conjugate axis is 5 and distance between foci is ⇒ b =
2

13 units 9
⇒ 2 b = 5 and 2 ae = 13. 343
Put a = 49 and b =
2 2
in Eq. (i), we get
Hence, equation of the hyperbola will be of the form 9
x2 y 2 x2 y2 x 2 9y 2
− =1 …(i) − = 1 or − =1
49 343 49 343
a 2 b2
9
Now, also we know for hyperbola
y 2 x2
25 (13)2 2 20. (d) Let the equation of hyperbola be − =1
∴ b 2 = a 2( e2 − 1) ⇒ = ( e − 1) a 2 b2
4 4 e2
Then, vertices = ( 0, ± a ) = ( 0, ± 6)
25 1 169 13 5
⇒ =1− 2 or e2 = ⇒ e= a = 6 and e =
169 e 144 12 3
5
or a = 6 and b = b2 25 b2
2 ∴ e= 1+ ⇒ = 1 +
a2 9 36
Put the value of a and b in Eq. (i), we get
25 − 9 b 2 b2
x2 y 2 x 2 4y 2 ⇒ = ⇒ 16 = ⇒ b 2 = 64
− =1 ⇒ − =1 9 36 4
36 25 36 25
4 So, the required equation of hyperbola is,
18. (a) Given that, distance between the foci of hyperbola is y 2 x2
− =1
16 units. 36 64
Equation of hyperbola will be of the form 21. (a) Given that, eccentricity of the hyperbola,
x2 y 2 e = 3/2
− =1 …(i)
a 2 b2 and foci = ( ± ae, 0) = ( ± 2 , 0)
Hence, 2 a e = 16 ⇒ a e = 8 Q ae = 2
and e= 2 3 4
⇒ a × =2 ⇒a =
Now, 2 a=8 2 3
∴ b 2 = a 2 ( e 2 − 1)
⇒ a=4 2
16  9  16  5 20
⇒ a 2 = 32 ⇒ b2 =  − 1 ⇒ b =
2
  ⇒b =
2
9 4  9  4 9
∴ b 2 = a 2 ( e2 − 1 )
So, the required equation of the hyperbola is
⇒ b 2 = 32 (2 − 1 )
x2 y 2 x2 y 2 4
⇒ b = 32
2 − =1 ⇒ − =
16 20 4 5 9
Put the value of a 2 and b 2 in Eq. (i), we get 9 9
x2 y 2 22. (i) (a) Since, sum of distances of any point on race course
∴ − =1
32 32 from two fixed points is always constant, so path traced
19. (b) Since, vertices ( ± 7 , 0) lie on X-axis, as y-coordinate is by the man will be ellipse.
Y
zero. Hence, equation of hyperbola will be of the form
x2 y 2
− =1 …(i)
a 2 b2 P(Man)

where, it is given that vertices ( ± 7 , 0) = ( ± a , 0)


X′ X
4 S′ S
a = 7 and e =
3
∴ c = ae Flag pasts
4 28
⇒ c =7 × = Y′
3 3
(ii) (b) Given that, SP + S′P = 10 i.e. 2 a = 10 ⇒ a = 5
Also, c2 = a 2 + b2
CBSE Term II Mathematics XI 89

(iii) (a) Since, the coordinate ofS andS′ are( c , 0) and( −c , 0). 2. Let a be the radius of the circle, then ( − a , − a ) will be
Therefore, the distance between S and S′ is centre of the circle. [Q circle lies in third quadrant]
2c = 8 ⇒ c = 4 Also, the line 3x − 4y + 8 = 0 touches the circle. Therefore,
Q c2 = a 2 − b2 [Q a = 5] the perpendicular distance from centre to the given line is
⇒ 16 = 25 − b 2 the radius of the circle.
⇒ b 2 = 25 − 16 = 9 3( − a ) − 4( − a ) + 8 − 3a + 4a + 8
∴ a= ⇒ a=
⇒ b=3 3 +42 2 5
(iv) (c)Q a = 5 and b = 3
⇒ 5a = a + 8 ⇒ a = 2
x2 y 2
∴ + =1 Hence, the required equation of the circle is
25 9
( x + 2 )2 + ( y + 2 )2 = 2 2
(v) (b) Here, a = 5 and b = 3
∴ 2 a + b = 2 ( 5) + 3 = 13 3. Given, AM = AL = AC = 2 units
23. (i) (c) Clearly, the shape of figure represent ellipse. and points A ≡ (2 , 2 ), B ≡ ( 6, 5)
(ii) (a) From the figure, we have ∴ Distance AB = (2 − 6)2 + (2 − 5)2
a = 3 and b = 2
= ( − 4)2 + ( − 3)2 = 16 + 9 = 5 units
∴ Equation of ellipse is
x2 y 2 x2 y 2 Then, distance BC = AB − AC = 5 − 2 = 3 units
2
+ 2 =1 ⇒ + =1 Y
3 2 9 4
(iii) (c) Eccentricity of ellipse
b2 4 5
∴ e= 1− = 1− =
a2 9 3 B(6,5)
(iv) (b) Length of latus rectum of an ellipse is r2
2 b 2 2 (2 ) 2 8 r1 C
∴ = = units
a 3 3 2 A
M (2,2)
(v) (b) Foci of the ellipse is ( ± c , 0)
2
∴ c = a 2 − b2 O X
L
c= 9−4 = 5 Hence, the equation of required circle whose centre is ( 6, 5)
⇒ Foci ≡ ( ± 5 , 0) and radius is 3 units, will be
( x − 6) 2 + ( y − 5) 2 = 3 2
Subjective Questions
4. Given equations are
1. Let centre of the circle be ( a , a ), then equation of the circle
x 2 + 2 ax − b 2 = 0 and x 2 + 2 px − q2 = 0.
is ( x − a )2 + ( y − a )2 = a 2.
Let the roots of x 2 + 2 ax − b 2 = 0 be x1 and x 2.
Then, x1 + x 2 = − 2 a and x1x 2 = − b 2
C (a, a)
and the roots of x 2 + 2 px − q2 = 0 are y1 and y 2.
Then, y1 + y 2 = − 2 p and y1y 2 = − q 2
Let A ≡ ( x1 , y1 ) and B ≡ ( x 2 , y 2 )
Now, equation of circle whose diameter is AB, will be
( x − x1 )( x − x 2 ) + ( y − y1 )( y − y 2 ) = 0
Since, the point (3, 6) lies on this circle, then ⇒ x 2 − ( x1 + x 2 )x + x1x 2 + y 2− ( y1 + y 2 )y + y1y 2 = 0
(3 − a) 2 + ( 6 − a )2 = a 2
On substituting the values of x1 + x 2, x1x 2, y1 + y 2 and y1y 2,
⇒ a 2 + 9 − 6 a + 36 − 12 a + a 2 = a 2 we get
⇒ a 2 − 18 a + 45 = 0 x 2 + 2 ax − b 2 + y 2 + 2 py − q2 = 0
⇒ a 2 − 15 a − 3 a + 45 = 0 ⇒ x 2 + y 2 + 2 ax + 2 py − b 2 − q2 = 0
⇒ a ( a − 15) − 3 ( a − 15) = 0 which is the required equation of circle.
⇒ ( a − 3) ( a − 15) = 0 ⇒ x 2 + 2 ax + a 2 + y 2 + 2 py + p2 − b 2 − q2 − a 2 − p2 = 0
⇒ a = 3 or 15
So, the equation of circle is ⇒ ( x + a ) 2 + ( y + p ) 2 = ( a 2 + b 2 + p 2 + q 2 )2
( x − 3) 2 + ( y − 3) 2 = 9 On comparing to standard equation of circle, we get
or ( x − 15)2 + ( y − 15)2 = 152
radius = a 2 + b 2 + p2 + q2 units
90 CBSE Term II Mathematics XI

5. Let centre of circle be ( h , 0), as centre of circle is on X-axis, x = 5 and y = 7


so y-coordinate will be zero. So, the coordinates of P are ( 5, 7 ).
The distance between any point on the circle from the
centre = Radius 3x–2y=1
4x+y
∴ ( x 2 − x1 )2 + ( y 2 − y1 )2 = r P =27

⇒ ( h − 2 ) 2 + ( 0 − 3) 2 = 5 r
C (2,–3)
(Q x1 = 2 , y1 = 3, x 2 = h and y 2 = 0)
⇒ h 2 + 4 − 4h + 9 = 25 (squaring both sides)
(2, 3)
5 Given, coordinates of centre C are (2 , − 3).
X′ X ∴ CP = Radius
(h, 0)
⇒ ( 5 − 2 )2 + (7 + 3)2 = r ⇒ 9 + 100 = r
⇒ r = 109 units
⇒ h 2 − 4h + 13 − 25 = 0
Hence, the required equation of circle is
⇒ h 2 − 4h − 12 = 0
( x − 2 )2 + ( y + 3)2 = ( 109 )2
⇒ h 2 − 6h + 2 h − 12 = 0
8. Given, equations of diameter of circles are
⇒ h ( h − 6) + 2 ( h − 6) = 0
2 x − 3y + 12 = 0 and x + 4y − 5 = 0
⇒ ( h − 6) ( h + 2 ) = 0 On solving these equations, we get x = − 3 and y = 2
⇒ h − 6 = 0 or h + 2 = 0 We know that, centre is the point of intersection of diameter.
⇒ h = 6 or − 2 ∴ Coordinates of centre are (− 3, 2).
∴ Centre is ( 6, 0) or ( − 2 , 0). Let r be the radius of circle.
Hence, equation of circle by using Then, area of circle is 154 sq units
( x − h )2 + ( y − k )2 = r 2 22 154 × 7
⇒ 154 = × r2 ⇒ r2 = = 49
7 22
When ( h , k ) = ( 6, 0) and r = 5, is
[Q area of circle = πr 2]
( x − 6) 2 + ( y − 0) 2 = 5 2
∴ r = 7 units [Q r > 0]
and when ( h , k ) = ( − 2 , 0) and r = 5, is Hence, the required equation of circle is
⇒ ( x + 2 ) 2 + ( y − 0) 2 = 5 2 ( x + 3) 2 + ( y − 2 ) 2 = 7 2
2 at a (1 − t ) 2
6. Given, x= and y = 9. Let the equation of circle with centre ( h , k ) and radius r be
1 + t2 1 + t2 (x − h ) 2 + (y − k) 2 = r2 …(i)
On squaring, we get Since, circle passes through the points (2 ,−2 ) and ( 3, 4), so
2
 a (1 − t 2 )
2 the points (2 , − 2 ) and (3, 4) will lie on Eq. (i).
 2 at 
x2 =   and y 2
=   ∴ (2 − h ) 2 + ( − 2 − k ) 2 = r 2 ...(ii)
 1 + t 2  1 + t2 
and (3 − h ) 2 + (4 − k) 2 = r2 ...(iii)
On adding, we get
2 2 Now, from Eqs. (ii) and (iii), we get
 2 at   a(1 − t 2 )
x2 + y 2 =   +   (2 − h ) 2 + ( − 2 − k ) 2 = ( 3 − h ) 2 + ( 4 − k ) 2
 1 + t 2  1 + t2 
⇒ 2 h + 12 k = 17 ...(iv)
4a 2t 2 a 2(1 − t 2 )2 Also, given that centre ( h , k ) lies on x + y = 2. So, it will
= +
(1 + t )
2 2
(1 + t 2 ) 2 satisfy it.
a 2( 4t 2 + 1 + t 4 − 2 t 2 ) ∴ h + k =2 …(v)
=
(1 + t 2 ) 2 On solving Eqs. (iv) and (v), we get
a 2[ t 4 + 1 + 2 t 2 ] a 2( t 2 + 1)2 h = 0.7 and k = 1. 3
= = = a2 Now, from Eq. (ii), we get
(1 + t 2 ) 2 ( t 2 + 1) 2
r 2 = (2 − 0.7 )2 + ( −2 − 1. 3)2 = 1. 69 + 10. 89 = 12 . 58
⇒ x2 + y 2 = a 2
Now, put the values of h , k and r 2 in Eq. (i), we get the
7. Let P ( x , y ) be the point of intersection of the lines,
answer.
3x − 2 y = 1 …(i)
( x − 0.7 ) 2 + ( y − 1. 3) 2 = 12.58
and 4x + y = 27 …(ii)
which is the required equation of circle.
On solving Eqs. (i) and (ii), we get
CBSE Term II Mathematics XI 91

10. We know that, the equation of the circle having centre ( h , k ) 8


⇒ x 2 + 4x + 4 = − y+4
and radius r is 3
( x − h )2 + ( y − k )2 = r 2 …(i) 2 
⇒ ( x + 2 )2 = − 4  y − 1 …(i)
3 
Q Circle (i) passes through points ( 4, 1) and ( 6, 5).
∴ ( 4 − h ) 2 + (1 − k ) 2 = r 2 …(ii) Let X = x +2
2 
and ( 6 − h )2 + ( 5 − k )2 = r 2 …(iii) and Y =  y − 1
3 
From Eqs. (ii) and (iii), we have
∴ X 2 = − 4Y ...(ii)
( 4 − h ) 2 + (1 − k ) 2 = ( 6 − h ) 2 + ( 5 − k ) 2
Coordinates of focus of a parabola (ii) are
⇒ 16 + h 2 − 8h + 1 + k 2 − 2 k = 36 + h 2
X = 0 and Y = − 1
− 12 h + 25 + k 2 − 10k 2
⇒ x + 2 = 0 and y − 1 = − 1
⇒ −8h + 12 h − 2 k + 10k = 36 + 25 − 17 3
⇒ 4h + 8k = 44 …(iv) ⇒ x = − 2 and y = 0
Also, centre ( h , k ) lies on the line 4x + y = 16 ∴ Equation of directrix of parabola (ii) is Y = 1
∴ 4h + k = 16 …(v) 2 2
i.e. y −1=1 ⇒ y =2 ⇒ y = 3
Subtracting Eq. (v) from Eq. (iv), we have 3 3
( 4h + 8k ) − ( 4h + k ) = 44 − 16 Focus ≡ ( − 2 , 0) and directrix, y = 3
⇒ 7 k = 28 ⇒ k = 4 13. Given, equation of parabola is y 2 = 8x.
Substituting the value of k in Eq. (v), we get Here, a =2
4h + 4 = 16 ∴ Focus ≡ (2 , 0)
16 − 4 12 Now, focal distance = 4 units
h= = =3
4 4 We know that, focal distance is a distance of any point P( x , y )
Substituting the value of h and k in Eq. (iii), we have on the parabola from the focus F.
( 6 − 3) 2 + ( 5 − 4) 2 = r 2 ∴ PF = Focal distance
⇒ 32 + 12 = r 2 ⇒ r 2 = 10 ⇒ PF = ( x − a )2 + y 2
Hence, centre ( 3, 4) and radius 10 units [by distance formuala]
Hence, required equation of circle is
⇒ 4 = ( x − 2 ) + 8x
2
[Q y 2 = 8x]
( x − 3) + ( y − 4) = 10
2 2
⇒ 16 = x 2 − 4x + 4 + 8x
11. Given, centre of circle, O ≡ ( 3, − 1).
[on squaring both sides]
Let OM be the perpendicular distance from O to the line
2 x − 5y + 18 = 0. ⇒ 16 = ( x + 2 ) ⇒ x + 2 = 4
2

[taking positive square root on both sides]


2( 3) − 5( − 1) + 18 29
Then, OM = = = 29 ⇒ x =2
(2 ) 2 + ( 5 ) 2 29
Now, y2 = 8 × 2
In ∆OMP, OP 2 = OM 2 + PM 2 [by Pythagoras theorem]
⇒ y 2 = 16
 1 1 
⇒ OP 2 = ( 29 )2 + ( 3)2 Q PM = PQ = × 6 = 3 ⇒ y=±4
 2 2 
Hence, coordinates of a point are (2, 4) and (2, − 4).
⇒ OP 2 = 29 + 9 = 38 14. Here, focus is (4, 0) and directrix is x + 4 = 0.
P 3 M 3 Q 2x – 5y +18 = 0 Let P( x , y ) be any moving point, then draw PM ⊥ ZZ′ from
P to the directrix.
r Y
O
(3, –1)
x= – 4

M
P(x, y)

Hence, the equation of required circle is O


Z X
(0, 0) F(4, 0)
( x − 3) + ( y + 1) = 38
2 2

12. Given, 3x 2 + 12 x + 8y = 0
8 Z′
⇒ x 2 + 4x + y=0 [dividing by 3 both sides]
3 Y′
92 CBSE Term II Mathematics XI

Now, FP = PM [by definition of parabola] 17. Given, equation of parabola is x 2 = −36y ⇒ x 2 = −4( 9)y
⇒ FP 2 = PM 2 Here, a = 9
( x + 4) 2
⇒ ( x − 4) + ( y − 0) =
2 2
O
( (1 ) 2 )
⇒ x 2 − 8x + 16 + y 2 = x 2 + 8x + 16
⇒ y 2 = 16x A B
F(0, –9)
which is the required equation of parabola.
∴ Length of latusrectum = 16 units Focus ≡ ( 0, − a ) = ( 0, − 9)
15. Take a point P at one end of the major axis. Sum of the Let AB be the latusrectum.
distances of the point P to the foci is ∴ y = −9, then x 2 = −36 ( −9) ⇒ x = ± 18
1 + F2P = F1O + OP + F2P
FP
So, the coordinates of A and B are ( −18, − 9) and (18, 9),
(Qsince, F1 P = F1 O + OP)
respectively.
= c + a + a − c = 2a Area of ∆AOB = 2 × Area of ∆OFB
√b2 + c 2 √b2 + c 2 1 
Q = 2 ×  × 18 × 9 = 162 sq units
2 
a–c
b 18. Since, axis is vertical, so let arch of parabola is in the form
P P
F1 c O c F2 x 2 = 4ay …(i)
Given, OB = 10 m
a and AC = 5 m
5
Take a point Q at one end of the minor axis. ⇒ AB = m [QAC = 5 ]
2
Sum of the distances from the point Q to the foci is
5 
Hence, coordinate of A =  , 10 will satisfy Eq. (i).
∴ F1 Q + F2Q = b 2 + c 2 + b2 + c2 2 
2
= 2 b2 + c2  5
i.e.   = 4a × 10
 2
Since, both P and Q lie on the ellipse.
Y
By the definition of ellipse, we have
5/2
2 b 2 + c 2 = 2 a , i.e. a = b 2 + c 2 C A
B
or a = b + c , i.e. c = a − b
2 2 2 2 2
k 10m
16. Given, equation of line is R
P Q
X′ X
lx + my + n = 0. O
− lx − n Y′
⇒ y= …(i)
m 25
⇒ = 40a
and equation of parabola is y = 4ax
2
…(ii) 4
5
From Eqs. (i) and (ii), we get ⇒ a=
2 32
 − lx − n  5
  = 4ax From Eq. (i), x2 = 4 × y
 m  32
⇒ l 2x 2 + 2 lxn + n 2 = 4m 2ax 5
⇒ x2 = y
8
⇒ l x + 2 lxn − 4am x + n = 0
2 2 2 2
k
⇒ l 2x 2 + x(2 ln − 4am 2 ) + n 2 = 0 …(iii) Now, let OR = 2 m and PQ = k ⇒ RP =
2
Since, the line lx + my + n touches the parabola. k 
Therefore, P =  , 2 will lie on parabola.
So, Eq. (iii) have equal roots. 2 
i.e. discriminant,  k
2
5  2 5 
D = 0 ⇒ b 2 − 4ac = 0 ∴   = ×2 Qx = y
2 8  8 
⇒ (2 ln − 4am 2 )2 − 4l 2n 2 = 0 k 2
5
⇒ =
⇒ 4l n − 16lnam 2 + 16a 2m 4 − 4l 2n 2 = 0
2 2
4 4
∴ ln = am 2 Hence proved. ⇒ k = 5 = 2.23 m (approx).
CBSE Term II Mathematics XI 93

19. Let the vertex be at the lowest point and the axis vertical. 1
Also, e=
Y 2
 1
12 m A ⇒ 4a = a 2  1 −  [Q b 2 = a 2 (1 − e2 )]
 2
1 cm
1
3 cm B ⇒ 4= a ⇒ a = 8 and b 2 = 4a = 4 × 8 = 32
Beam 2
2 cm Hence, the required equation of ellipse is
X
O x2 y 2
+ =1 [Q put a 2 = 64 and b 2 = 32]
The equation of the parabola takes the form x 2 = 4ay. 64 32
 3   3  22. We have, foci ( ± 4, 0) which lie on X-axis, so the equation of
 , we have ( 6) = 4a 
2
Since, it passes through  6 , 
 100  100 ellipse is
i.e. a = 36 ×
100
= 300 m. x2 y 2
+ = 1, a > b
12 a 2 b2
1 Since, foci ≡ ( ± c , 0) = ( ± 4, 0) ⇒ c = 4
Let AB be the deflection of the beam which is m.
100
 2 
Coordinates of B are  x , . c c 4  1
 100 We know that, e = ⇒a= = = 12 Qe=
a e 1/ 3  3 
2
Therefore, x 2 = 4 × 300 × = 24 ∴ a = 12
100
Now, c 2 = a 2 − b 2 ⇒ ( 4)2 = (12 )2 − ( b )2
i.e. x = 24 = 2 6 m
⇒ b 2 = 144 − 16 = 128
x2 y 2
20. Given, equation of ellipse is + = 1. ∴ a 2 = (12 )2 = 144 and b 2 = 128
4 25
x2 x2 y2
Since, the denominator of is smaller than the denominator of Hence, the equation of the ellipse is + = 1.
4 144 128
y2 23. Since, major axis is along X-axis. Hence, equation of ellipse
. will be of the form
25
x2 y 2
So, the major axis is along Y-axis. + =1 …(i)
x2 y 2 a 2 b2
On comparing the given equation with + = 1, we get Given that, Eq. (i) passes through the points (4, 3) and (6, 2)
b2 a 2
i.e. they will satisfy it
b 2 = 4 and a 2 = 25
( 4) 2 ( 3) 2
⇒ b = 2 and a = 5 ∴ + 2 =1
a2 b
Here, a and b are lengths, so we take only positive sign. 16 9
⇒ + =1 …(ii)
Q c 2 = a 2 − b 2 = 25 − 4 = 21 a 2 b2
⇒ c = 21 ( 6 ) 2 (2 ) 2
and + 2 =1
a2 b
Here, the major axis is along Y-axis.
36 4
∴ Foci = ( 0, ± c ) = ( 0, ± 21 ) ⇒ + =1 …(iii)
a 2 b2
Vertices = ( 0, ± a ) = ( 0, ± 5) Multiply Eq. (ii) by 4 and Eq. (iii) by 9, then subtracting, we get
Length of major axis = 2 a = 2 × 5 = 10 units 64 324
− 2 = 4−9
Length of minor axis = 2 b = 2 × 2 = 4 units a2 a
c 21 260
Eccentricity, e = = ⇒ − 2 =−5
a 5 a
2 b2 2 × 4 8 260
∴ Length of latusrectum = = = units. ⇒ a2 =
a 5 5 5
21. Let the equation of an ellipse be ⇒ a 2 = 52
x2 y 2 From Eq. (ii), we get
+ = 1, a > b
a 2 b2 9 16
=1−
2 b2 b 2
52
Given, latusrectum = = 8 units
a 9 52 − 16
⇒ =
∴ b 2 = 4a …(i) b2 52
94 CBSE Term II Mathematics XI


9
=
36 x2 y2
2 + =1
b 52 1 / 16 1 / 18
9 × 52 ⇒ 16x 2 + 18y 2 = 1
⇒ b2 = = 13
36 5 5
Put the values of a 2 = 52 and b 2 = 13 in Eq. (i), we get 26. Given that, eccentricity = , i.e. e =
8 8
x2 y 2 x2 y 2
+ =1 Let equation of the ellipse be + = 1,
52 13 a 2 b2
which is the required equation of ellipse. Since the foci of this ellipse is ( ± ae, 0).
24. We have, foci of ellipse at (± 3, 0) which are on X-axis. ∴ Distance between foci = ( ae + ae)2
Therefore, equation of the ellipse is of the form
⇒ 2 a 2e2 = 10
x2 y 2
+ = 1, a > b ...(i) [Q distance between its foci = 10 ]
a 2 b2
Its foci are (± c, 0) = (± 3, 0) ⇒ a e =5
2 2

∴ c = 3 ⇒ a 2e2 = 25
Now, c2 = a 2 − b2 25 × 64
⇒ a2 =
25
⇒ 9=a −b2 2
[Q ae = c = 3] ...(ii)
∴ a=8
Since, Eq. (i) passes through (4, 1). We know that,
16 1 16 1 ∴ b 2 = a 2 (1 − e 2 )
∴ + =1 ⇒ + =1 [from Eq. (ii)]
a 2 b2 9 + b2 b2  25 
⇒ b 2 = 64 1 − 
⇒ 16b 2 + 9 + b 2 = b 2( 9 + b 2 )  64
⇒ 17 b 2 + 9 = 9b 2 + b 4  64 − 25
⇒ b 2 = 64  
 64 
⇒ b 4 − 8b 2 − 9 = 0
b 2 = 39
⇒ b 4 − 9b 2 + b 2 − 9 = 0
2 b2
⇒ ( b 2 − 9) ( b 2 + 1) = 0 ∴ Length of latusrectum of ellipse =
a
⇒ b 2 = 9 or −1  39 39
= 2   = units.
But b2 ≠ − 1 ⇒ b2 = 9  8 4
From Eq. (ii), we get x2 y2
27. The equation of ellipse is + = 1.
a 2 = 9 + b2 ⇒ a 2 = 9 + 9 36 20
⇒ a 2 = 18 x2 y 2
On comparing this equation with + = 1, we get
On putting the values of a 2 and b 2 in Eq. (i), we get a 2 b2
x2 y 2 a = 6 and b = 2 5
+ =1
18 9 We know that, b 2 = a 2 (1 − e2 )
which is the required equation of ellipse. ⇒ 20 = 36 (1 − e2 )
25. We have, major axis of ellipse lies on the X-axis, so the 20
equation of ellipse is ⇒ = 1 − e2
36
x2 y 2
+ = 1, a > b 20 16
a 2 b2 ∴ e= 1− =
36 36
2 b2 4
∴ Length of its latusrectum = = unit 4 2
a 9 e= =
6 3
2 a 2(1 − e2 ) 4
⇒ = [Q b 2 = a 2(1 − e2 )]  a a
a 9 Now, directrices =  + , − 
 e e
 1 4  1
⇒ 2 a 1 −  = Qe= a 6 6×3
 9 9 
 3  ∴ = = =9
e  2 2
1 2 2 1 1  
⇒ a = and b 2 = a = ⋅ =  3
4 9 9 4 18
a
1 1 and −
=−9
∴ a =
2
and b = 2
e
16 18
Hence, required equation of ellipse is ∴ Distance between the directrices = 9 − ( − 9) = 18 units
CBSE Term II Mathematics XI 95

28. We have, distance between foci = 8 units According to the question, ellipse (i) passes through the
point (−3, 1).
∴ 2 ae = 8 …(i)
9 1
and distance between directrices = 18 units ∴ + =1 [from Eq. (i)] …(ii)
a 2 b2
2a
∴ = 18 …(ii) ∴ b 2 = a 2 (1 − e 2 )
e
 2 3
From Eqs. (i) and (ii), we get ∴ b 2 = a 2 1 −  ⇒ b 2 = a 2 …(iii)
 5 5
2
a = 6 and e = 3
3 On substituting b 2 = a 2 in Eq. (ii), we get
5
We know that,
9 5 32
( ae)2 = a 2 − b 2 ⇒ ( 4)2 = ( 6)2 − b 2 + = 1 ⇒ 3a 2 = 32 ⇒ a 2 =
a 2 3a 2 3
⇒ b 2 = 36 − 16 = 20 3 32 32
Now, b 2 = × = [from Eq. (iii)]
Hence, the required equation of ellipse is 5 3 5
x2 y 2 On substituting a 2 =
32
and b 2 =
32
+ =1 in Eq. (i), we get
36 20 3 5
29. Since, major axis is along Y-axis. x2 y2
+ =1
Hence, equation of ellipse will be of the form 32 32
x2 y 2 3 5
+ =1 …(i) 3x 2 5y 2
b2 a 2 ⇒ + =1
Given that, Eq. (i) passes through the points (3, 2) and (1, 6) 32 32
i.e. they will satisfy it which is the required equation of ellipse.
32 2 2 x2 y 2
∴ + =1 31. Consider the equation of the ellipse is 2 + 2 = 1.
b2 a 2 a b
9 4 ∴ Length of major axis = 2a
⇒ + =1 …(ii)
b2 a 2 Length of minor axis = 2b
12 62 2 b2
and + =1 and length of latusrectum =
b2 a 2 a
1 36 2 b2 2 b
⇒ + =1 …(iii) Given that, =
b2 a 2 a 2
Multiplying Eq. (ii) by 9 and then subtract the Eq. (iii) from ⇒ a = 2b ⇒ b = a / 2
it, we get
We know that, b 2 = a 2 (1 − e 2 )
80 80
= 8 ⇒ b2 = 2
b 2
8  a
⇒   = a (1 − e )
2 2
2
⇒ b 2 = 10
From Eq. (ii), we get a2
⇒ = a 2 (1 − e 2 )
9 4 4
+ =1 1
10 a 2 ⇒ 1 − e2 =
4 9 4
⇒ =1− 1
a2 10 ⇒ e =1−
2

4 1 4
⇒ 2
= ⇒ a 2 = 40 3 3
a 10 ∴ e= =
4 2
Put the values of a 2 = 40 and b 2 = 10 in Eq. (i), we get
32. Let AB be the rod of length 12 cm.
x2 y 2
+ =1 Let OA = a and OB = b
10 40
Using by Pythagoras theorem in ∆OAB.
which is the required equation of ellipse. Y
x2 y 2 (0, b) B
30. Let equation of ellipse is, 2 + 2 = 1, where a > b K (i)
a b
and eccentricity, b P
2 2
e= ⇒ e2 = A
5 5 O a (a, 0)
X
96 CBSE Term II Mathematics XI

(Base)2 + (Perpendicular)2 = (Hypotenuse)2 Here, in hyperbolic equation coefficient of y 2 is positive, so


⇒ a + b = (12 )
2 2 2 transverse axis is along Y-axis.
⇒ a 2 + b 2 = 144  2 14 
…(i) ∴ Foci = ( 0, ± c ) =  0, ± 
Let AP = 3 cm  5 
⇒ PB = AB − AP  6
Vertices = ( 0, ± a ) =  0, ± 
= 12 − 3 = 9 cm  5
⇒ AP : PB = 3 : 9 = 1 : 3 2 14
Let P ( h , k ) for which locus to be found. c 5 = 14
Eccentricity, e = =
1 3 a 6 3
A P B 5
(a, 0) (h, k) (0, b)
2 b2 2 × 4 8 5 4 5
Latusrectum = = = = units.
∴ If P ( h , k ) divides the points A and B internally in the ratio a 6 6 3
m : n , then 5
mx 2 + nx1
h= 34. We have, foci of the hyperbola lies on X-axis. So, the
m +n equation of hyperbola is
my 2 + ny1 x2 y 2
and k= − =1 …(i)
m +n a 2 b2
1×0+ 3×a
∴ h= Foci ≡ ( ± c , 0) = ( ± 2 , 0) ⇒ c = 2
1+ 3
3
3a Eccentricity of the hyperbola, e =
⇒ h= 2
4
4h 1×b+ 3×0 We know that, c = ae
⇒ a= and k =  3
3 1+ 3 ∴ 2 = a 
 2
b
⇒ k= ⇒ b=4k 4
4 ⇒ a=
4h 3
Put the values of a = and b = 4 k in Eq. (i), we get
3 Also, c2 = a 2 + b2
2
 4 h 16 h 2  4
2
20
 + ( 4 k ) = 144 ⇒ + 16 k 2 = 144
2
 ⇒ (2 ) 2 =   + b 2 ⇒ b 2 =
 3  9  3 9
h2 h 2 k2 16 20
⇒ + k2 = 9 ⇒ + =1 Thus, a =2
and b =
2
9 81 9 9 9
Hence, the equation of locus of a point P( h , k ) is Hence, the required equation of hyperbola is
x2 y 2 9x 2 9y 2
+ =1 − =1 [from Eq. (i)]
81 9 16 20
33. Given, equation is 5y 2 − 9x 2 = 36, divide it by 36, we get 35. We have, vertices of hyperbola lies on X-axis. So, the
equation of hyperbola is
5y 2 9x 2 36 y 2 x2
− = ⇒ − =1 x2 y 2
36 36 36 36 4 − =1 …(i)
5 a 2 b2
y 2 x2 y 2 x2 Vertices ≡ ( ± a , 0) = ( ± 7 , 0) ⇒ a = 7
On comparing − = 1 with 2 − 2 = 1, we get
36 4 a b b2 b2
Now, e= 1+ ⇒ e2 = 1 +
5 a2 a2
36
a2 = and b 2 = 4  4
2
b2 4
5 ⇒   = 1+ 2 [Q e = and a = 7]
6  3 7 3
⇒ a= and b = 2
5 b 2 16 343
⇒ = − 1 ⇒ b2 =
36 49 9 9
∴ c2 = a 2 + b2 = +4
5 343
Thus, a 2 = 49 and b 2 =
36 + 20 56 9
⇒ c2 = =
5 5 Hence, the required equation of hyperbola is
2 14 x 2 9y 2
⇒ c= − =1 [from Eq. (i)]
5 49 343
CBSE Term II Mathematics XI 97

x2 y 2 (i) Centre (0, 0)


36. Given hyperbola is − = 1.
a 2 b2 (ii) Transverse axis = 2 a = 2 × 3 = 6 units
The eccentricity e of this hyperbola is (iii) Conjugate axis = 2 b = 2 × 3 3 = 6 3 units
b2 a 2 + b2 1 a2 (iv) Value of c = a 2 + b 2 = 9 + 27 = 36 = 6
e2 = 1 + ⇒ e2 = ⇒ 2 = 2 …(i)
a 2
a 2
e a + b2 (v) Vertices = ( 0, ± a ) = ( 0, ± 3)
y 2 x2 a2 9 3
The equation of the conjugate hyperbola is 2 − 2 = 1 (vi) Directrices, y = ± =± =±
b a c 6 2
The eccentricity e′ of this hyperbola is (vii) Foci = ( 0, ± c ) = ( 0, ± 6)
a2 b2 + a 2 b2 c 6
e′ 2 = 1 + ⇒ e′ 2
= ⇒
1
= …(ii) (viii) Eccentricity, e = = = 2
b2 b2 e′ 2 a 2 + b 2 a 3
2 b 2 2 × 27
On adding Eqs. (i) and (ii), we get (ix) Length of latusrectum = = = 18 units.
a 3
1 1 a 2 + b2
+ 2 = 2 =1 39. Length of latusrectum of the hyperbola i.e.
e 2
e′ a + b2 2 b2
8= ⇒ b 2 = 4a ...(i)
1 1 a
∴ + =1 Hence proved.
e2 e′ 2 ∴ Distance between the foci = 2ae
37. We have, foci of the hyperbola lies on Y-axis. So, the Since, transverse axis be 2a and conjugate axis be 2b.
equation of hyperbola is 1
∴ (2 ae) = 2 b
y 2 x2 2
− =1 …(i)
a 2 b2 ⇒ ae = 2 b ...(ii)
Foci ≡ ( 0, ± c ) = ( 0, ± 12 ) ⇒ c = 12 ∴ b 2 = a 2 ( e 2 − 1) ...(iii)
2 b2 From Eqs. (i) and (ii),
∴ Length of its latusrectum =
a a 2e2
2 4a =
2b 4
⇒ = 36 ⇒ b 2 = 18a
a ⇒ 16 a = a 2e2
Now, c = a + b ⇒ (12 ) = a + 18a
2 2 2 2 2
16
⇒ 16 = a e2 ⇒ a = …(iv)
⇒ a + 18a − 144 = 0 ⇒ ( a + 24)( a − 6) = 0
2
e2
2
⇒ a=6 [Q a = − 24 is neglect] Put the value of b in Eq. (iii) from Eq. (i), we get
Thus, a 2 = 36 and b 2 = 18a = 18 × 6 = 108 Q 4 a = a 2 ( e 2 − 1)
4
Hence, the required equation of hyperbola is ⇒ = e2 − 1
2 2
a
y x 4 e2
− =1 [from Eq. (i)]  4
36 108 ⇒ = e2 − 1 ⇒ e2  1 −  = 1
16  16
2 2
y x
38. Given, equation of hyperbola is − =1  12   16 
9 27 ⇒ e2   = 1 ⇒ e2 =  
 16   12 
2 2
y x
On comparing with − = 1, we get ⇒ e2 =
4
⇒e =
2
a 2 b2 3 3
a 2 = 9 and b 2 = 27 or a = 3 and b = 3 3
40. As the vertices are on the X- axis and their mid-point is the
Shape of hyperbola is conjugate hyperbola, because x2 y 2
coefficient of x 2 is negative. origin, the equation is of the type 2 − 2 = 1.
a b
Y
Here, b 2 = a 2 ( e2 − 1), vertices are ( ± a, 0) and directrices are
a
given by x = ± .
(0, 3) e
a 3
Directrix Thus, a = 6, = 4 and so e = which gives
X′ X e 2
O
Directrix 9 
b 2 = 36  − 1 = 45
(0, –3) 4 
Consequently, the required equation of the hyperbola is
x2 y 2
Y′ − =1
36 45
98 CBSE Term II Mathematics XI

41. The given equation may be written as y1 − y 2 −2 + 3 1


Slope of AB = = =−
x2 y2 x2 y2 x1 − x 2 1−4 3
− = 1 or 2
− =1
32 / 3 8 (4 2 / 3) (2 2 ) 2 ∴ Slope of HL = 3 [Q HL ⊥ AB ]
x2
y 2 5  5
Comparing the given equation with − = 1, we get Then, equation of HL will be y + = 3  x − 
a 2 b2 2  2
4 2
2
4 2 ⇒ 2 y + 5 = 6x − 15
a2 =   or a = . ⇒ 6x − 2 y = 20
 3 3
∴ 3x − y = 10 …(ii)
∴ Length of transverse axis of a hyperbola
4 2 8 2 Centre H of the circle lies on lines (i) and (ii), therefore it is
= 2a = 2 × = units the point of intersection of lines (i) and (ii).
3 3
On multiplying Eq. (ii) by 4 and adding it to Eq. (i), we get
42. Given, radius, r = 5 units, point lies on circle = (2 , 3)
47
and centre of circle is on X-axis. 15x = 47 ⇒ x =
15
∴ Its y-coordinate will be zero.
(2, 3)
On subtracting Eq. (ii) from Eq. (i), we get
3
5 5y = − 3 ⇒ y = −
5
X′ X
 47 3
(h, 0) ∴ Centre, H ≡  ,− 
 15 5
2 2
Let centre of circle be ( h , 0)  47   3 
Now, radius of the circle = HA =  − 1 +  − + 2
 15   5 
Now, distance from the centre to a point on the circle
= Radius of circle  32 
2
 7
2
=   + 
∴ ( h − 2 ) + ( 0 − 3) = 5
2 2
[by distance formula]  15   5
⇒ h 2 + 4 − 4h + 9 = 5 [Q ( a − b )2 = a 2 + b 2 − 2 ab ] ( 32 )2 + (21)2 1465
= = units
⇒ h + 4 − 4h + 9 = 25
2
[squaring on both sides] (15) 2
15
⇒ h − 4h + 13 − 25 = 0 ⇒ h − 4h − 12 = 0
2 2
Hence, the required equation of required circle is
⇒ h 2 − 6h + 2 h − 12 = 0 ⇒ h ( h − 6) + 2( h − 6) = 0  47  
2
3 1465
2
x −  + y +  =
⇒ ( h − 6)( h + 2 ) = 0  15   5 152
⇒ h − 6 = 0 or h + 2 = 0 ⇒ h = 6 or h = − 2 44. Let ( h , k ) be the centre and r be the radius of the circle.
So, the centre of circle is (6, 0) or (− 2, 0).
∴ Equation of circle
When centre ( h , k ) = ( 6, 0) and radius, r = 5 units, then
the equation of circle is ( x − h )2 + ( y − k )2 = r 2 …(i)
( x − 6)2 + ( y − 0)2 = ( 5)2 [Q( x − h )2 + ( y − k )2 = r 2] Q Circle (i) passes through points (2 , 3) and ( −1, 1).
⇒ ( x − 6) 2 + y 2 = 5 2 ∴ (2 − h ) 2 + ( 3 − k ) 2 = r 2 …(ii)
When centre ( h , k ) = ( − 2 , 0) and radius, r = 5 units, then ( − 1 − h ) + (1 − k ) = r
2 2 2
…(iii)
the equation of circle is
From Eqs. (ii) and (iii), we have
( x + 2 )2 + ( y − 0)2 = ( 5)2 [Q( x − h )2 + ( y − k )2 = r 2]
(2 − h ) 2 + ( 3 − k ) 2 = ( − 1 − h ) 2 + (1 − k ) 2
⇒ ( x + 2 )2 + y 2 = 52
⇒ 4 + h 2 − 4h + 9 + k 2 − 6k = 1 + h 2 + 2 h + 1 + k 2 − 2 k
Hence, the required equations of circle are
( x − 6)2 + y 2 = 52 and ( x + 2 )2 + y 2 = 52. ⇒ − 4h − 2 h − 6k + 2 k = 2 − 4 − 9
⇒ −6h − 4k = − 11
43. Let points A ≡ (1, − 2 ) and B ≡ ( 4, − 3)
⇒ 6h + 4k = 11 …(iv)
Given line is 3x + 4y = 7 …(i)
Also, the centre of circle lies on the line
Again, let L is the mid-point of AB.
 1 + 4 −2 + ( −3)  5 5 x − 3y = 11
∴ Mid-piont L ≡  , = ,−  ∴ h − 3k = 11 …(v)
 2 2   2 2
Multiplying Eq. (v) by 6 and subtracting from Eq. (iv), we have
( 6h + 4k ) − ( 6h − 18k ) = 11 − 66
H ⇒ 22 k = − 55
55 5
⇒ k=− =−
90° 22 2
A L B Substituting the value of k in Eq. (v), we get
(1, –2) (4, –3)
CBSE Term II Mathematics XI 99

 5 Y
h − 3 ×  −  = 11
 2 (50, 24) Q
15 15 7 k)
⇒ h+
= 11 ⇒ h = 11 − = P(18
, 24 m

30 m
2 2 2 k
X′ X
Now, substituting the value of h and k in Eq. (ii), we get O 18 m R
2 2 6m
 7  5
2 −  +  3 +  = r
2
 S
2  2 Y′
2 2 100 m
 3  11
⇒ −  +   = r
2
 2 2 Focus is at the middle of the cable and shortest and longest
9 121 130 vertical supports are 6 m and 30 m and roadway in 100 m
⇒ + = r2 ⇒ = r2 long.
4 4 4
∴ Required equation of circle Clearly, the coordinates of Q( 50, 24) will satisfy Eq. (i).
2 2 2500
 7  5 130 ∴ ( 50)2 = 4a × 24 ⇒ 2500 = 96a ⇒ a =
x −  + y +  =
 2  2 4 96
2 2 2500
 a  b Hence, from Eq. (i), x = 4 ×
2
y
On comparing to  x +  +  y +  = d, we get 96
 2  2
2500
130 65 ⇒ x2 = y
a = − 7 , b = 5 and d = = 24
4 2
Let PR = k m
45. Given, equation of parabola is
Then, point P ( 18, k ) will satisfy the equation of parabola.
9y 2 − 16x − 12 y − 57 = 0
2500
⇒ ( 3y )2 − 2 × 2 × 3y + 2 2 = 2 2 + 57 + 16x ∴ From Eq. (i), (18)2 = ×k
24
⇒ ( 3y − 2 )2 = 61 + 16x 2500
⇒ 324 = k
2 24
 2  61
⇒ 9  y −  = 16  x +  324 × 24
 3  16  ⇒ k=
2
2500
 2 16  61 324 × 6
⇒ y −  = x +  …(i) =
 3 9  16  625
2 61 1944
Let Y=y− and X = x + …(ii) =
3 16 625
16 ⇒ k = 3.11
∴ Y2 = X [from Eqs. (i) and (ii)] …(iii)
9 ∴ Required length = 6 + k = 6 + 3.11 = 9.11 m (approx.)
Now, the equation of axis of parabola (iii) is Y = 0 x2 y 2
2 2 47. Given, equation of ellipse is + = 1.
∴ y− =0 ⇒ y= 36 16
3 3
x2 y2
Coordinates of vertices of parabola (iii) are Since, denominator of is greater than denominator of .
36 16
X = 0 and Y = 0
∴ Major axis is along X-axis.
2 61 2
i.e. y− =0 ⇒ x=− and y = Y
3 16 3
(0,4)
On comparing Eq. (iii) with y 2 = 4ax, we get
16 4
4a = ⇒ a= X′ X
9 9 (−6,0 ) O (6,0)
∴ Equation of directrix of parabola (iii) is (0,−4)
X=−a
Y′
61 4 613
i.e. x+ =− ⇒ x=−
16 9 144 x2 y 2
On comparing the above equation with + = 1, we get
4 16 a 2 b2
and length of latusrectum = |4a| = 4 × = units.
9 9 a = 6 and b = 4.
46. Here, wires are vertical. ∴ Length of major axis = 2 a = 2 × 6 = 12 units
Let equation of the parabola be in the form Length of minor axis = 2 b = 2 × 4 = 8 units
x 2 = 4ay …(i) Value of c = a 2 − b 2 = ( 6)2 − ( 4)2
100 CBSE Term II Mathematics XI

= 36 − 16 = 20 = 2 5 Put the values of a and b in Eq. (i), we get


foci = ( ± c , 0) x2 y 2
+ =1
= ( ± 2 5 , 0) 16 4
∴ Vertices are ( 6,0) and ( −6,0) Given, AP = 1.5 m
2 2 ⇒ OP = OA − AP = 4 − 1.5
 b  4 ⇒ OP = 2.5 m
Eccentricity, e = 1 −   = 1 −  
 a  6
Let PQ = k
36 − 16 20 20 Then, the coordinate Q (2.5, k ) will satisfy the equation of
= = =
36 36 6 ellipse.
a 6 36 (2.5)2 k 2 6.25 k 2
Directrices are x = ± = ± =± i.e. + =1 ⇒ + =1
e 20 / 6 20 16 4 16 4
2 b 2 2( 4)2 16 k 2 1 6.25 16 − 6.25
and length of latusrectum = = = units. ⇒ = − =
a 6 3 4 1 16 16
2
48. Given, equation of ellipse is k 9.75 9.75
⇒ = ⇒ k2 =
x2 y2 4 16 4
+ =1 ⇒ k 2 = 2.4375
100 400
x2 y2 ∴ k = 1.56 m (approx.)
QDenominator of is smaller than the denominator of .
100 400 50. We have, equation of hyperbola is 5y 2 − 9x 2 = 36
∴The major axis is along Y-axis. It can be written as
x2 y 2 y2 x2
On comparing given equation with 2 + 2 = 1, we get, 2
− =1
b a  6 (2 ) 2
b 2 = 100 and a 2 = 400  
 5
⇒ b = 10 and a = 20 y2 x2
Here, a and b are lengths, so we take only positive sign. On comparing with 2
− 2 = 1, we get
a b
Now, c 2 = a 2 − b 2 = 400 − 100 = 300 2
 6
⇒ c = 10 3 a2 =  
 5
Here, major axis is along Y-axis. So,
6
∴ Foci = ( 0, ± c ) = ( 0, ± 10 3 ) and b2 = 2 2 ⇒ a = and b = 2
5
Vertices = ( 0, ± a ) = ( 0, ± 20) (i) Shape of hyperbola is conjugate hyperbola, because
Length of major axis = 2 a = 2 × 20 = 40 units coefficient of x 2 is negative.
Length of minor axis = 2 b = 2 × 10 = 20 units Y
) 6
c 10 3
Eccentricity, e = =
a 20
=
2
3 0,
√5 )
Directrix
2 b 2 2 × 100 X′ X
∴ Length of latusrectum = = = 10 units O
a 20 ) 6 Directrix
49. Clearly, equation of ellipse takes the form 0, −
√5 )
x2 y 2 Y′
+ =1 …(i)
a 2 b2 (ii) Centre ( 0, 0)
Here, it is given that 2 a = 8 and b = 2 6 12
(iii) Length of transverse axis = 2 a = 2 × = units
⇒ a = 4 and b = 2 5 5
Y (iv) Length of conjugate axis = 2 b = 2 × 2 = 4 units
36 56 2 14
B (v) Value of c = a 2 + b 2 = +4= =
Q 5 5 5
2m k  6
(vi) Vertices = ( 0, ± a ) =  0, ± 
P  5
X′ X
A′ O 1.5 m A
4m a2 36 × 5 18
8m (vii) Directrices, y = ± =± =±
c 5 × 2 14 70
B′  2 14 
Y′ (viii) Foci = ( 0, ± c ) =  0, ± 
 5 
CBSE Term II Mathematics XI 101

14 Q c2 = a 2 + b2
2
c 5 = 14 ∴ ( 3 5 ) 2 = a 2 + ( 4a )
(ix) Eccentricity, e = =
a 6 3
⇒ 9 × 5 = a 2 + 4a
5
⇒ a + 4a − 45 = 0
2
2 b2 2 × 4 4 5
(x) Length of latusrectum, = = units. On splitting the middle term
a 6 3
⇒ a 2 + 9a − 5a − 45 = 0
5
⇒ a( a + 9) − 5( a + 9) = 0
51. Since, foci ( ± 3 5 , 0) lies on X-axis, as y-coordinate is zero.
⇒ ( a − 5) ( a + 9) = 0
Hence, equation of hyperbola will be of the form
⇒ a − 5 = 0, a + 9 = 0
x2 y 2 ⇒ a = 5, − 9
− =1 …(i)
a 2 b2 So, a = 5 ⇒ b 2 = 4 × 5 = 20 [Q a can’t be negative]
where, it is given that foci ( ± 3 5 , 0) = ( ± c , 0) Put a 2 = 25 and b 2 = 20 in Eq. (i), we get
c=3 5 x2 y 2
− =1
and length of latusrectum 25 20
2 b2 which is the required equation of hyperbola.
= 8 ⇒ b 2 = 4a
a
Chapter Test Short Answer Type Questions
l

5. Find the centre and radius of each of the


following circle.
l
Multiple Choice Questions (i) x 2 + ( y + 2) 2 = 9 (ii) x 2 + y 2 + 6 x − 4 y + 4 = 0

1. The equation of the circle which passes 6. Find the equation of the circle passing through
through the point (4, − 5) and has its centre two points on Y -axis at distances 3 units from
(− 2, − 2), is the origin and having radius 5 units.
(a) ( x − 2)2 + ( y − 2)2 = 13 7. Find the equation of hyperbola, the length of
(b) ( x + 2)2 + ( y + 2)2 = 85 3
whose latusrectum is 8 units, eccentricity is
(c) Both (a) and (b) 5
(d) Neither (a) nor (b)
and whose transverse and conjugate axes are
2. The area of the circle centred at (2, 3) and along the X and Y -axes respectively.
passing through the point (5, 7) is
(a) 6 π sq units (b) 12π sq units l
Long Answer Type Questions
(c) 25 π sq units (d) 30 π sq units
8. In each of the following questions, find the
3. If the equation of the parabola is x 2 = − 8 y , then coordinates of the focus, axis of the parabola,
(a) focus is (0, –2) the equation of the directrix and the length of
(b) directrix is y = 2 the latusrectum.
(c) length of latusrectum is 8 units
(i) y 2 = 12x (ii) y 2 = − 8 x
(d) All of the above
(iii) y 2 = 10x (iv) x 2 = − 9 y
l
Case Based MCQs
9. Find the equation of the hyperbola with
4. A rod AB of length 15 cm rests in between two
(i) vertices (± 5, 0) and foci (± 7, 0)
coordinate axes in such a way the end point A
7
lies on X-axis and end point B lies on Y -axis. A (ii) vertices (0, ± 7) and e = .
point P(x , y) is taken on the rod in such a way 3
that AP = 6 cm. (iii) foci (0, ± 10) and passing through (2, 3).
Y
Answers For Detailed Solutions
B Scan the code
1. (d) 2. (c) 3. (d)

9 cm 4. (i) (a) (ii) (d) (iii) (b) (iv) (b) (v) (d)
5. (i) centre = (0, − 2); radius = 3 units
Q θ P(x, y) (ii) centre = (− 3, 2); radius = 3 units
x
y θ 6 cm x2 y2
X 6. (x + 4)2 + y 2 = 52 or (x − 4)2 + y 2 = 52 7. − =1
O R A 25 20
8. (i) Focus = (3, 0)
Let the rod AB making an angle θ with OX as
Axis = X-axis; Directrix, x = − 3
shown in figure.
Length of latusrectum = 12 units
Based on above information answer the
(ii) Focus = (− 2, 0)
following questions.
Axis = X-axis; Directrix, x = 2
(i) The value of cosθ is equal to Length of latusrectum = 8 units
(iii) Focus =  , 0
x y 9 6 5
(a) (b) (c) (d)
9 6 x y 2 
5
(ii) The value of sinθ is equal to Axis = X-axis; Directrix, x = −
2
6 9 x y
(a) (b) (c) (d) Length of latusrectum = 10 units
y x 9 6
(iv) Focus =  0, − 
9
(iii) The locus of P is  4
(a) parabola (b) an ellipse 9
Axis = Y-axis; Directrix, y =
(c) hyperbola (d) None of these 4
(iv) The length of major axis is equal to Length of latusrectum = 9 units
(a) 9 units (b) 18 units (c) 12 units (d) 81 units x2 y2
9. (i) − =1
(v) The length of minor axis is equal to 25 24
(a) 18 units (b) 6 units (ii) 9x 2 − 7 y 2 = − 343
(c) 36 units (d) 12 units (iii) y 2 − x 2 = 5
CBSE Term II Mathematics XI 103

CHAPTER 05

Introduction to
Three-Dimensional
Geometry

In this Chapter...
l Coordinate Axes
l Distance Formula
l Section Formulae

Coordinate Axes Coordinates of a Point in Space


Let XOX′, YOY′ and ZOZ′ be the three mutually perpendicular lines, Let P be a point in space. Through P, draw three
intersecting at O. The point O( 0 , 0 , 0 ) is called the origin and the lines planes parallel to the coordinate axes to meet the
( XOX′ , YOY ′ and ZOZ′ ) are called rectangular coordinate axes say axes in A, B and C, respectively.
X, Y and Z, respectively. Thus in the given figure, X′ OX is called the Let OA = x, OB = y and OC = z. These three
X-axis, Y ′OY is called the Y-axis, and Z′ OZ is called the Z-axis. numbers taken in order are called coordinates of a
point P and written as P ( x, y , z).
Coordinate Planes
Z
The three coordinate axes define three mutually perpendicular planes
XOY , YOZ and ZOX (or XY , YZ and ZX) called coordinate planes
which divide the space into eight parts called octants. C
Z
X′ z)
y,
C A′ (x,
P

e z
lan Y
-P YZ - Plane x O B
XZ O x
Y′ Y (0, 0, 0)
B′ B
XY - Plane y
A

X
A C′
X
Z′
104 CBSE Term II Mathematics XI

Sign of Coordinates For Internal Ratio


The sign of coordinates of the points in the octants in which the space is If C divides AB internally in the ratio m1 : m 2 , then
divided are given in the following table the coordinates of C are
m1 m2
Octants I II III IV V VI VII VIII
A C B
Coordinates OXYZ OX ′ YZ OX ′ Y ′Z OXY ′Z OXYZ′ OX ′ Y Z ′ OX ′ Y ′Z ′ OXY ′ Z ′
x + − − + + − − +  m 1 x 2 + m 2 x1 m 1 y 2 + m 2 y 1 m 1 z 2 + m 2 z 1 
 , , 
y + + − − + + − −  m1 + m 2 m1 + m 2 m1 + m 2 
z + + + + − − − −
For External Ratio
Coordinates of a Point on Coordinate Axes If C divides AB externally in the ratio m1 : m 2 , then
Let P ( x, y , z) be any point on a space. the coordinates of C are
Then, coordinates of a point on  m1 x 2 + ( − m 2 )x1 m1 y 2 + ( − m 2 )y 1
 , ,
(i) X axis will be of the form ( x, 0 , 0 )  m1 + ( − m 2 ) m1 + ( − m 2 )
(ii) Y-axis will be of the form ( 0 , y , 0 ) m1 z 2 + ( − m 2 )z1 

(iii) Z-axis will be of the form ( 0 , 0 , z). m1 + ( − m 2 ) 
Coordinates of a Point in Coordinate Planes  m x − m 2 x1 m 1 y 2 − m 2 y 1 m 1 z 2 − m 2 z 1 
= 1 2 , , 
Let P( x, y , z) be any point on a space and we draw three perpendicular  m1 − m 2 m1 − m 2 m1 − m 2 
lines from P to the coordinate planes XY , YZ and ZX, respectively.
Z Applications of section formulae
P(x, y, z) Coordinates of the Mid-Point
If C is the mid-point of AB, then m1 : m 2 = 1 : 1
so that
x + x2 y + y2 z + z2
Y x= 1 ,y = 1 and z = 1
O 2 2 2
These are the coordinates of the mid-point of the
X
line segment joining A( x1 , y 1 , z1 ) and B( x 2 , y 2 , z 2 ).
Then, coordinates of a point in
(i) XY-plane will be of the form ( x, y , 0 ). Coordinates of the Point which
[Qin XY-plane, the z-coordinate of a point is zero] Trisect the Line Segment
(ii) YZ-plane will be of the form ( 0 , y , z) and If R 1 and R 2 trisect the line segment AB, then
(iii) ZX-plane will be of the form ( x, 0 , z). AR 1 = R 1R 2 = R 2B

Distance Formula A R1 R2 B

The distance between two points P ( x1 , y 1 , z1 ) and Q ( x 2 , y 2 , z 2 ) is Thus, for finding the coordinates of R 1 , take ratio
given by 1 : 2 and for finding the coordinates of R 2 take ratio
PQ = ( x 2 − x1 ) 2 + ( y 2 − y 1 ) 2 + ( z 2 − z1 ) 2 2 : 1.
or PQ = ( x1 − x 2 ) 2 + ( y 1 − y 2 ) 2 + ( z1 − z 2 ) 2 Coordinates of centroid of a triangle
If the vertices of a ∆ABC are A( x1 , y 1 , z1 ),
Condition for Collinearity
B ( x 2 , y 2 , z 2 ) and C( x 3 , y 3 , z 3 ), then the centroid of
Three points A, B and C are said to be collinear, if AB + BC = AC
a ∆ABC is
Section Formulae  x + x 2 + x 3 y 1 + y 2 + y 3 z1 + z 2 + z 3 
G 1 , , 
Let A ( x1 , y 1 , z1 ) and B ( x 2 , y 2 , z 2 ) be the end points of a line segment  3 3 3 
AB and C be any point on AB which divide AB in the ratio m1 : m 2 .
CBSE Term II Mathematics XI 105

Solved Examples
Example 1. Name the octants in which the following Coordinate of A = (3, 0, 0)
points lie. Coordinate of D = (0, 0, 3)
(2 , − 1 , 1 ), (3, 2, − 1), (6, − 3, − 5), ( − 2 , − 1 , 3), Coordinate of F = (3, 3, 3)
( − 4, 2 , 3), ( − 1 , − 2 , − 3), ( − 9, 4, − 5) (2, 8, 4). Coordinate of C = (0, 3, 3)
Sol. The sign of coordinates of the points in the octants in which
the space is divided are given in the following table Example 3. Let M1 , M 2 , M 3 be the feet of the
perpendiculars drawn from the point A (4, 6, 1)
I II III IV V VI VII VIII
Octants on the XY , YZ and ZX-planes, respectively. Find
Coordinates OXYZ OX ′ YZ OX ′ Y ′Z OXY ′Z OXYZ′ OX ′ Y Z ′OX ′ Y ′Z ′ OXY ′ Z ′ the distance of these points M1 , M 2 , M 3 from the
point A.
x + − − + + − − +
Sol. M1 is the foot of perpendicular drawn from the point
y + + − − + + − −
A (4, 6, 1) to the XY-plane.
z + + + + − − − − Y
The name of octants in which given points lie, are given in
the following table M1(4, 6, 0)

Point Octant Name M2(0, 6, 1) A(4, 6,1)


(2, −1, 1) IV OXY ′ Z X

(3, 2, − 1) V OXYZ′ M3(4, 0, 1)


(6, − 3, − 5) VIII OXY ′ Z′ Z
( − 2 , − 1, 3) III OX′ Y ′ Z On XY-plane, Z-coordinate will be zero. Therefore, the
( − 4, 2 , 3) II OX′ YZ coordinates of the point M1 are (4, 6, 0). The distance
between the points A (4, 6, 1) and M1 (4, 6, 0) is 1 unit.
( − 1, − 2 , − 3) VII OX′ Y ′ Z′ Similarly, the length of foot of perpendiculars on YZ and
ZX-planes i.e. the length of AM 2 and AM 3 are 4 and 6 units.
( − 9, 4, − 5) VI OX′ YZ′
Example 4. Find the value of y, if the distance between
(2, 8, 4) I OXYZ
two points (7 , y, − 1 ) and (2, 3, − 2) is 6.
Sol. Given points are (7 , y , − 1) and (2, 3, − 2) and distance
Example 2. What are the coordinates of the vertices of
between the points = 6
a cube whose edge is 3 units, one of whose vertices
coincides with the origin and three edges passing ∴ (2 − 7 ) 2 + ( 3 − y ) 2 + ( − 2 + 1 ) 2 = 6
through the origin coincides with the positive [Q distance = ( x 2 − x1 )2 + ( y 2 − y1 )2 + ( z 2 − z1 )2 ]
direction of the axes through the origin?
Sol. Given, edge of a cube is 3 units. ⇒ ( − 5) 2 + ( 3 − y ) 2 + ( − 1) 2 = 6
Y
⇒ 25 + ( 3 − y )2 + 1 = 36
G
B [squaring both sides]
3 ⇒ ( 3 − y ) = 10
2
C F
⇒ 3 − y = ± 10

3 A X [taking square root both sides]


3 O
⇒ y = 3 ± 10
D
E ∴ y = 3 − 10 or 3 + 10
Z
Coordinate of O = (0, 0, 0) Example 5. Verify that the points (1,− 1, 3), (2 , − 4, 5)
Coordinate of G = (0, 3, 0) and ( 5, − 13, 11 ) are collinear.
Coordinate of B = (3, 3, 0) Sol. Let A(1, − 1, 3), B(2 , − 4, 5) and C( 5, − 13, 11) be the given
Coordinate of E = (3, 0, 3) points.
106 CBSE Term II Mathematics XI

We know that distance = ( x 2 − x1 )2 + ( y 2 − y1 )2 + ( z 2 − z1 )2 Example 8. Show that the points (1, 2, 1), (1, − 2, 2),
(1,– 1, 3) (2,– 4, 5) (5, – 13, 11) ( 5, − 2 , 1 ) and ( 5, 2 , 2 ) are the vertices of a square.
A B C Sol. Let A (1, 2, 1), B (1, − 2, 2), C (5, − 2, 1) and D (5, 2, 2) be the
given points.
Then, AB = (2 − 1)2 + ( − 4 + 1)2 + ( 5 − 3)2 (5, 2, 2) D C (5, –2, 1)
= 1 + 9 + 4 = 14 units

BC = ( 5 − 2 )2 + ( − 13 + 4)2 + (11 − 5)2


= 9 + 81 + 36 = 126 = 3 14 units (1, 2, 1) A B (1, – 2, 2)

and CA = (1 − 5)2 + ( − 1 + 13)2 + ( 3 − 11)2 We know, distance = ( x 2 − x1 )2 + ( y 2 − y1 )2 + ( z 2 − z1 )2


= 16 + 144 + 64 = 224 = 4 14 units ∴ AB = (1 − 1)2 + ( − 2 − 2 )2 + (2 − 1)2
We observe that, = 0 + 16 + 1 = 17 units
AB + BC = 14 + 3 14
BC = ( 5 − 1)2 + ( − 2 + 2 )2 + (1 − 2 )2
= 4 14 = CA
= 16 + 0 + 1 = 17 units
Hence, A ,B and C are collinear.
CD = ( 5 − 5)2 + (2 + 2 )2 + (2 − 1)2
Example 6. Prove that the triangle formed by joining = 0 + 16 + 1 = 17 units
the three points whose coordinates are A(1 , 2 , 3),
B(2 , 3, 1 ) and C( 3, 1 , 2 ) is an equilateral triangle. DA = (1 − 5)2 + (2 − 2 )2 + (1 − 2 )2
Sol. Given points, A(1, 2, 3), B (2, 3, 1) and C (3, 1, 2). = 16 + 0 + 1 = 17 units
Q Distance = ( x 2 − x1 ) + ( y 2 − y1 ) + ( z 2 − z1 )
2 2 2
Here, AB = BC = CD = DA.
So, ABCD is a square or a rhombus.
Then, AB = (2 − 1) + ( 3 − 2 ) + (1 − 3)
2 2 2

Now, AC = ( 5 − 1)2 + ( − 2 − 2 )2 + (1 − 1)2


= 1 + 1 + 4 = 6 units
= 16 + 16 + 0 = 2 × 16 = 4 2 units
BC = ( 3 − 2 )2 + (1 − 3)2 + (2 − 1)2
and BD = ( 5 − 1)2 + (2 + 2 )2 + (2 − 2 )2
= 1 + 4 + 1 = 6 units
= 16 + 16 + 0 = 2 × 16 = 4 2 units
and AC = ( 3 − 1)2 + (1 − 2 )2 + (2 − 3)2
Since, diagonal AC = diagonal BD
= 4 + 1 + 1 = 6 units Hence, ABCD is a square.
Since, AB = BC = AC Example 9. Find the coordinates of the point which
Hence, ∆ABC is an equilateral triangle. divides the line segment joining the points
A(2 , 1 , − 3) and B( 5, − 8 , 3) in the ratio 2 : 1
Example 7. Are the points A(3, 6, 9), B (10, 20, 30) and internally.
C (25, − 41, 5), the vertices of a right angled Sol. Let C( x , y , z ) be any point which divides the line segment
triangle? joining points A(2 , 1, − 3) and B( 5, − 8, 3) in the ratio 2 : 1
Sol. Given points, A (3, 6, 9), B (10, 20, 30) and C (25, − 41, 5). internally.
2:1
We know, distance = ( x 2 − x1 )2 + ( y 2 − y1 )2 + ( z 2 − z1 )2
A C B
∴ AB = (10 − 3)2 + (20 − 6)2 + ( 30 − 9)2 (2, 1, – 3) (5, – 8, 3)

= 49 + 196 + 441 = 686 units Here, the ratio is 2 : 1.


m 1 = 2, m 2 = 1
BC = (25 − 10) + ( − 41 − 20) + ( 5 − 30)
2 2 2
Then, coordinates of point C
= 225 + 3721 + 625 = 4571 units  m x + m 2x1 m1y 2 + m 2y1 m1z 2 + m 2z1 
= 1 2 , , 
and CA = ( 3 − 25)2 + ( 6 + 41)2 + ( 9 − 5)2  m1 + m 2 m1 + m 2 m1 + m 2 
= 484 + 2209 + 16 = 2709 units  2( 5) + 1(2 ) 2( − 8) + 1(1) 2( 3) + 1( −3)
= , , 
AB 2 + CA 2 = 686 + 2709  2+1 2+1 2+1 

= 3395 ≠ BC 2  10 + 2 − 16 + 1 6 − 3
= , , 
 3 3 3 
Hence, ∆ABC is not a right angled triangle.
= ( 4, − 5, 1)
CBSE Term II Mathematics XI 107

Example 10. Find the coordinates of the point which Sol. Let the coordinates of the fourth vertex of a parallelogram be
S ( x, y , z)
divides the line segment joining the points P(2 , − 1 , 5) (x, y, z)
and Q( 3, − 6, − 5) in the ratio 3 : 4 externally. S R (3, – 1, 2)
Sol. Let R ( x , y , z ) be any point which divides the line segment P
joining points P(2 , − 1, 5) and Q( 3, − 6, − 5) in the ratio 3 : 4
externally.
3:4 P Q
(– 1, 1, 2) (1, 2, – 4)
P R Q
(2, – 1, 5) (x, y, z) (3, – 6, – 5) We know that the diagonals of a parallelogram bisect each
other.
Here, the ratio is 3 : 4
∴ Mid-point of PR = mid-point of QS
m1 = 3, m 2 = 4 − + 3 1 − 1 2 + 2   1 + x 2 + y − 4 + z
 1
⇒ , ,  = , , 
Then, coordinates of point R  2 2 2   2 2 2 
 m x − m 2x1 m1y 2 − m 2y1 m1z 2 − m 2z1   x + 1 y + 2 z − 4
= 1 2 , ,  ⇒ (1, 0, 2) =  , , 
 m1 − m 2 m1 − m 2 m1 − m 2   2 2 2 
 3( 3) − 4(2 ) 3( − 6) − 4( − 1) 3( −5) − 4( 5) On comparing the coordinates both sides, we get
= , , 
 3−4 3−4 3−4  x+1 y+2 z−4
= 1, = 0, and =2
 9 − 8 − 18 + 4 −15 − 20 2 2 2
= , ,  x + 1 = 2, y + 2 = 0 and z − 4 = 4
 −1 −1 −1 
x = 1, y = − 2 and z = 8
= ( − 1, 14, 35)
Hence, the coordinates of the fourth vertex S is (1, − 2 , 8).
Example 11. Find the ratio in which the line segment
joining the points (3, 5, − 4) and (2, 4, 5) is divided Example 13. Find the coordinates of the points which
by the XY-plane. trisect the line segment joining the points
A( 5, − 8 , 8 ) and B( 4, 2 , − 3).
Sol. Let XY-plane divides the line segment joining the points
A( 3, 5, − 4) and B(2 , 4, 5) at P( x , y , z ) in the ratio λ : 1. Sol. Let C1 and C 2 be two points which trisect the joining
λ:1 of A and B. Then, AC1 = C1C 2 = C 2B
1:2 2:1
A P B A B
(3, 5, – 4) (2, 4, 5) (5, – 8, 8) C1 C2 (4, 2, – 3)
Then, the coordinates of
Point C1 divides the join of AB in the ratio 1 : 2
 λ (2 ) + 1( 3) λ ( 4) + 1( 5) λ ( 5) + 1( −4)
P= , ,  ∴Coordinates of C1
 λ+1 λ+1 λ+1 
 1( 4) + 2( 5) 1(2 ) + 2( −8) 1( − 3) + 2( 8)
= , , 
[Q coordinates of internal ratio,  1+2 1+2 1+2 
m1x 2 + m 2x1 m1y 2 + m 2y1 m1z 2 + m 2z1 
, ,  4 + 10 2 − 16 −3 + 16
m1 + m 2 m1 + m 2 m1 + m 2  =
 3
, , 
3 3 
 2 λ + 3 4λ + 5 5λ − 4  14 −14 13
= , ,  = , , 
 λ +1 λ+1 λ +1  3 3 3
Since, P lies on the XY-plane, so its Z-coordinate is zero. Also, C 2 divides the join of AB in the ratio 2 : 1.
5λ − 4 ∴Coordinates of C 2
∴ =0
λ+1  2( 4) + 1( 5) 2(2 ) + 1 ( −8) 2( − 3) + 1( 8)
= , , 
⇒ 5λ − 4 = 0  2+1 2+1 2+1 
4
⇒ λ=  8 + 5 4 − 8 −6 + 8
5 = , , 
 3 3 3 
Therefore, XY-plane divides AB internally in the ratio 4 : 5.
 13 −4 2 
= , , 
Example 12. Three consecutive vertices of a  3 3 3
parallelogram PQRS are P( − 1 , 1 , 2 ), Q(1 , 2 , − 4) and  14 −14 13  13 −4 2 
R( 3, − 1 , 2 ). Find the coordinates of the fourth vertex. Hence, required points are  ,
 3 3 3
,  and  , , .
 3 3 3
108 CBSE Term II Mathematics XI

Example 14. If (2, 3, 4) is the centroid of the ∆ABC Sol. Let P(2 , − 1, 1) divides the join of A( − 1, 2 , − 8) and
with vertices A( 3a, 1 , 6), B( − 2 , 2 b , − 8 ) and B( 3, − 2 , 4) in the ratio λ : 1.
C( 6, 16, c ), then find the values of a, b and c. λ:1
Sol. Given vertices of ∆ABC are A( 3a , 1, 6), B( − 2 , 2 b , − 8) and A P B
C( 6, 16, c ). (– 1, 2, – 8) (3, – 2, 4)
Centroid of triangle is Then, coordinates of P are
 x1 + x 2 + x 3 y1 + y 2 + y 3 z1 + z 2 + z 3   λ ( 3) + 1( − 1) λ ( − 2 ) + 1(2 ) λ ( 4) + 1( −8)
 , , 
 3 3 3   , , 
 λ+1 λ+1 λ+1 
 3a − 2 + 6 1 + 2 b + 16 6 − 8 + c 
= , ,  [using internal ratio formula]
 3 3 3 
 3λ − 1 −2 λ + 2 4λ − 8
 3a + 4 2 b + 17 c − 2  = , ,  … (i)
= , ,   λ+1 λ+1 λ+1
 3 3 3 
But coordinates of P are (2 , − 1, 1) [given] … (ii)
Also, given that centroid of triangle is (2, 3, 4).
3a + 4 On comparing Eqs. (i) and (ii), we get
∴ =2 3λ − 1
3 =2
⇒ 3a + 4 = 6 λ+1

⇒ a=
2 ⇒ 3λ − 1 = 2 λ + 2
3 ⇒ λ=3
2 b + 17 −2 λ + 2
=3 = −1
3 λ+1
⇒ 2 b + 17 = 9
⇒ − 2λ + 2 = − λ − 1
⇒ b=−4
⇒ λ=3
c −2
=4 4λ − 8
3 =1
λ+1
⇒ c − 2 = 12
⇒ 4λ − 8 = λ + 1
⇒ c = 14
2 ⇒ λ=3
Hence, a = , b = − 4 and c = 14 From each of these equations, we get
3
λ=3
Example 15. Using section formula, show that the
Since, from each equations, we get same value of λ.
points ( − 1 , 2 , − 8 ), (2, − 1 , 1) and( 3, − 2 , 4) are
Therefore, the given points are collinear and P divide AB
collinear.
internally in the ratio 3 : 1.
CBSE Term II Mathematics XI 109

Chapter
Practice
PART 1
Objective Questions
l
Multiple Choice Questions 10. Points P (2, 4, 6), Q(–2, –2, –2) and R(6, 10, 14) are
(a) vertices of a triangle (b) collinear
1. If a point is on the ZX-plane, then its coordinates
(c) non-collinear (d) Both (a) and (b)
will be
(a) ( x , y , 0) (b) ( 0, y , z ) 11. The point on Y-axis which is at a distance 10 from
(c) ( x , 0, z ) (d) ( x , y , z ) the point (1, 2, 3), is
(a) (0, 2, 0) (b) (0, 0, 2)
2. X-axis is the intersection of two planes
(c) (0, 0, 3) (d) None of these
(a) XY and XZ (b) YZ and ZX
(c) XY and YZ (d) None of these 12. If the distance between the points (a, 0, 1) and
3. L is the foot of the perpendicular drawn from a (0, 1, 2) is 27, then the value of a is
point P (6, 7, 8) on the XY-plane, then the (a) 5 (b) ± 5
coordinates of point L are (c) − 5 (d) None of these
(a) (6, 0, 0) (b) (6, 7, 0) 13. If P( − 2 , 3, 5), Q(1 , 2 , 3) and R(7 , 0, − 1 ), then
(c) (6, 0, 8) (d) None of these
(a) P , Q and R are collinear
4. Which of the following statement is correct? (b) P , Q and R are non-collinear
(a) The X-axis and Y-axis taken together determine a plane (c) PR = 4PQ
known as XY-plane
(d) PQ + QR = 2PR
(b) The coordinates of a point in the XY-plane are of the form
(0, 0, z) 14. If the point A (3, 2, 2) and B (5, 5, 4) are equidistant
(c) Coordinate planes divide the space into six octants from P, which is on X-axis, then the coordinates of P
(d) All the above are correct are
5. The point (− 2, −3, − 4) lies in the  39
(a) 
  49
, 2 , 0 (b) 
  39
, 2 , 0 (c) 
  49
, 0 , 0 (d) 

, 0 , 0
 4   4   4   4 
(a) first octant (b) seventh octant
(c) second octant (d) eight octant 15. The points (5, –1, 1), (7, – 4, 7), (1, – 6, 10) and
6. The distance between the points P(1 , − 3, 4) and ( −1 , − 3, 4) are
Q( − 4, 1 , 2 ) is (a) the vertices of a rectangle (b) the vertices of a square
(c) the vertices of a rhombus (d) None of these
(a) 5 (b) 2 5 (c) 3 5 (d) 48
16. The coordinates of the point R, which divides the
7. Distance of the point (3, 4, 5) from the origin is segment joining P( x 1 , y1 , z1 ) and Q ( x 2 , y 2 , z 2 )
(a) 50 (b) 3 (c) 4 (d) 5 internally in the ratio k : 1, are
8. The distance of point P(3, 4, 5) from the YZ-plane is  kx − x ky − y1 kz 2 − z1 
(a)  2 1 , 2 , 
(a) 3 units (b) 4 units  1−k 1−k 1−k 
(c) 5 units (d) 550 units  kx + x1 ky 2 + y1 kz 2 + z1 
(b)  2 , , 
9. What is the length of foot of perpendicular drawn  1+ k 1+ k 1+ k 
from the point P (3, 4, 5) on Y-axis?  kx + x1 ky 2 + y1 kz 2 + z1 
(c)  2 , , 
(a) 41 (b) 34  1−k 1−k 1−k 
(c) 5 (d) None of these (d) None of the above
110 CBSE Term II Mathematics XI

17. The coordinate of the point P which divides the line Using above information, answer the following.
joining the points A ( −2 , 0, 6) and B(10, − 6, − 12 ) (i) The coordinates of the centre is
internally in the ratio 5 : 1.  7 7  7 7 
(a)  − , , 0 (b)  , , 0
(a) (8, 5, 9) (b) (–8, 5, 9)  2 2  2 2 
(c) (8, –5, –9) (d) None of these  7 −7   7 −7 
(c)  , , 0 (d)  0, , 
18. The ratio in which the line joining (2, 4, 5) and 2 2   2 2 
(3, 5, –4) is divided by the YZ-plane, is (ii) The coordinates of the point which divides the line
(a) 2 : 3 (b) 3 : 2 AB into the ratio 3 : 1 internally, is
(c) –2 : 3 (d) 4 : –3  17 −23 3  17 −23 3
(a)  , ,  (b)  , , 
19. A point on XOZ-plane divides the join of  4 4 2 2 4 2
(5, –3, –2) and (1, 2, –2) at  17 −23 3  17 −23 1 
(c)  , ,  (d)  , , 
 13   13   4 4 4  4 4 2
(a)  , 0 − 2 (b)  , 0, 2
 5   5  (iii) The coordinates of the point which divides the line
(c) (5, 0, 2) (d) (5, 0, –2) AB into the ratio 3 : 2, is
20. The centroid of the triangle, if the mid-point of the  19 22 −3  19 −22 3
(a)  , ,  (b)  , , 
 5 5 5  5 5 5
sides of triangles are D(1, 2, –3), E(3, 0, 1) and
F(–1, 1, –4), is  19 −22 2   19 −21 3
(c)  , ,  (d)  , , 
(a) (1, 2, –1) (b) (1, 1, –2)  5 5 5  5 5 5
(c) (0, 1, –2) (d) None of these (iv) The value of the diameter is
(a) 2 14 (b) 14
l
Case Based MCQs (c) 129 (d) 3 14
21. Four students in traditional dresses represent four (v) The value of the radius is
states of India, standing at the points represented 14
(a) 14 (b)
by O(0, 0, 0), A(a, 0, 0), B ( 0, b , 0) C(0, 0, c). If a girl 2
representing BHARATMATA be placed in such a (c)
129
(d)
3 14
way that she is equidistant from the four students, 2 2
then answer the following questions which are
based on above it.
(i) x-coordinate of girl representing BHARATMATA is PART 2
a a a
(a) a (b)
2
(c)
3
(d)
4 Subjective Questions
(ii) y-coordinate of girl representing BHARATMATA is
b l
Short Answer Type Questions
(a) a (b) (c) 2b (d) 3b
2
(iii) z-coordinate of girl representing BHARATMATA is
1. Name the octants in which the following points lie;
c (2, 3, 8), (1, − 1, − 3), (−2 , 4, − 6), ( −3, − 3, 1 ),
(a) b (b) c (c) (d) 2c
2 ( − 3, − 2 , − 5), ( 6, − 1 , 4), (2 , 3, − 5) and ( − 6, 7 , 9).
(iv) Which concept is used for finding the coordinates 2. Locate the point (2 , 3, 4) in space.
of point?
(a) Distance formula (b) Section formula 3. In the given figure, if the coordinates of point P are
(c) Mid-point formula (d) All of these (a, b, c), then write the coordinates of A, D, B, C
(v) Which of the following is coordinates of origin and E.
point? Z
(a) (0, 0, 0) (b) (0, b, 0) E
(c) (a, 0, 0) (d) (0, 0, c) C
c F P (a, b, c)
22. Abhay and Birsen are roaming in a circular ground.
At a moment their positions are A(2 , 1 , − 3) and O b
B
Y
B( 5, − 8 , 3). And the points A and B are the vertices a
A D
of the diameter.
X
CBSE Term II Mathematics XI 111

4. If the vertices of a cube, whose edge is 2 units, one 19. Find the coordinates of the point which divides the
of whose vertices coincides with the origin and line segment joining the points ( − 4, 6, 8 ) and
there edges passing through the origin coincides (2 , − 2 , 4) in the ratio 1 : 3 internally.
with the positive direction of the axes through the 20. Find the coordinate of the point which divides the
origin, are O, A, G, D, B, F, E and C, then find the line segment joining the points (1, 2, 3) and (2, 4, 6)
coordinates of the vertices of the cube. in the ratio
5. What are the coordinates of the vertices of a cube (i) 1 : 2 internally
whose edge is 5 units, one of whose vertices (ii) 3 : 1 internally
coincides with the origin and three edges passing
through the origin coincides with the positive
21. Find the ratio in which the join of the points
P(2 , − 1 , 3) and Q( 4, 3, 1 ) is divided by the point
direction of the axes through the origin?
 20 5 15 
6. Show that the points ( 3, 2 , − 4), ( 5, 4, − 6) and R  , , .
7 7 7
( 9, 8 , − 10) are collinear.
7. Prove that the triangle formed by joining the three 22. Given that A ( − 2 , 3, 5), B (1, 2, 3) and C(7, 0, − 1) are
points whose coordinates are A(1 , 2 , 3), B(2 , 3, 1 ) and collinear. Find the ratio in which B divides AC.
C( 3, 1 , 2 ) is an equilateral triangle. 23. A point R with x-coordinate 4 lies on the line
8. Find the equation of the curve formed by the set of segment joining the points P(2 , − 3, 4) and
all points whose distances from the points ( 3, 4, − 5) Q(8 , 0, 10). Find the coordinates of the point R.
and ( − 2 , 1 , 4) are equal. 24. Given that P ( 3, 2 , − 4), Q( 5, 4, − 6) and R( 9, 8 , − 10)
9. Determine the point in YZ-plane which is are collinear. Find the ratio in which Q divides PR.
equidistant from three points A (2 ,0,3), B ( 0, 3, 2 )
25. Using section formula, show that the points
and C ( 0, 0, 1 ).
 1 
10. Show that D( − 1 , 4, − 3) is the circumcentre of (2 , − 3, 4), ( −1 , 2 , 1 ) and  0, , 2  are collinear.
 3 
∆ABC with vertices A( 3, 2 , − 5), B( − 3, 8 , − 5) and
C( − 3, 2 , 1 ). 26. Three vertices of a parallelogram PQRS are
11. Show that the points ( − 2 , 4, 1 ), ( − 1 , 5, 5), (1 , 1 , 1 ) P(2 , − 4, 3), Q( 3, 2 , − 6) and R( − 3, 1 , 4). Find the
and (2 , 2 , 5) are vertices of rectangle. coordinates of the fourth vertex.
12. Show that the points ( − 1 , − 6, 10), (1 , − 3, 4), 27. Let P( 3, 2 , − 5), Q(2 , 3, 4) and R( 4, − 7 , − 4) be the
( − 5, − 1 , 1 ) and ( − 7 , − 4, 7 ) are the vertices of a vertices of a triangle. The internal bisector of the
rhombus. angle P meets QR at the point M. Find the
13. If the points ( a − 1 , a, a + 1 ), ( a, a + 1 , a − 1 ) and coordinates of point M.
( a + 1 , a − 1 , a) be the vertices of a triangle, then find 28. Find the coordinate of the points which trisect
which type of triangle is formed with these vertices. the line segment joining the points A (2, 1, −3)
14. Find the length of the longest piece of a string that and B (5, − 8, 3).
can be stretched straight in a rectangular room 29. Find the coordinates of the points which trisect the
whose dimensions are 10, 13 and 8 units. line segment joining the points A( − 3, 1 , 2 ) and
15. Find the point in the XY-plane which is equidistant B( 3, − 8 , 5).
from the points (2, 0, 3), (0, 3, 2) and (0, 0, 1). 30. The vertices of a ∆ABC are A( a, 2 , 5), B( − 1 , b , − 4)
16. A triangle is given with vertices A( 0, 0, 6), B( 0, 4, 0) and C(2 , 6, c ). If the centroid of the triangle is
and C( 6, 0, 0). Find the length of median from point G (1, 1, 1), then find the values of a, b , c.
A of ∆ABC.
17. If the origin is the centroid of a ∆ABC having
l
Long Answer Type Questions
vertices A(a, 1, 3), B(– 2, b, – 5) and C(4, 7, c), then 31. Name the octants in which the following points lie;
find the values of a, b and c. ( − 2 , 1 , 3), (2, 2, − 2), (1, − 4, 1), ( − 3, − 2 , − 1 ),
18. Find the coordinates of the point which divides the ( − 5, − 2 , 2 ), ( − 4, 3, − 2 ), (2 , − 3, − 6) and (1, 4, 8).
line segment joining the points ( − 2 , 3, 5) and 32. Verify that the points ( 3, − 2 , 4), (1 , 0, − 2 ) and
(1 , − 4, 6) in the ratio 2 : 3 externally.
( −1 , 2 , − 8 ) are collinear.
112 CBSE Term II Mathematics XI

33. Show that the points (−2, 6, −2), (0, 4, −1), ( −2 , 3, 1 ) 44. Prove that the coordinates of the points which
and ( −4, 5, 0) are the vertices of a square. divide the lines joining the vertices of a tetrahedron
to the centroid of the opposite faces in the ratio
34. Find the equation of the set of points P, the sum of
3 : 1 are same.
whose distances from A( 4, 0, 0) and B( − 4, 0, 0) is
equal to 10. 45. The mid-points of the sides of a triangle are
(5, 7, 11), (0, 8, 5) and (2, 3, − 1). Find its vertices.
35. Find the locus of a point which moves such that the
sum of its distance from points A( 0, 0, − α) and l
Case Based Questions
B( 0, 0, α) is constant.
36. Verify that ( − 1 , 2 , 1 ), (1 , − 2 , 5), ( 4, − 7 , 8 ) and 46. The locations of the houses of three friends
is considered as A(6, 10, 14), B (2, 4, 6) and
(2 , − 3, 4) are the vertices of a parallelogram. If the C ( − 2 , − 2 , − 2 ).
given points are the vertices of a parallelogram,
then find the length of it’s sides. Using the above information, answer the following
(i) If one of the friend takes a walk as AB and AC, then
37. Three points A (3, 2, 0), B (5, 3, 2) and C ( −9, 6, − 3) in which case he has to walk more?
are forming a triangle. The bisector AD of ∠BAC (ii) What is the condition for three points to be
meets BC at D. Find the coordinates of D. collinear?
38. Verify the following. (iii) If given points A, B and C form a triangle, then
(i) ( 0 , 7 , − 10 ), (1 , 6 , − 6 ) and ( 4 , 9 , − 6 ) are the vertices name the type of the triangle.
of an isosceles triangle. 47. The 4 vertices of a parallelogram are P( − 2 , a),
(ii) ( 0 , 7 , 10 ), ( − 1 , 6 , 6 ) and ( − 4 , 9 , 6 ) are the vertices of Q( 5, 1 ), R (4, 3) and S( b , 7 ).
a right angled triangle.
S(b, 7) R(4, 3)
(iii) ( − 1 , 2 , 1 ), (1 , − 2 , 5 ), ( 4 , − 7 , 8 ) and (2 , − 3 , 4 ) are the
vertices of a parallelogram.
39. Find the coordinates of the point which divides the
line segment joining the points ( −2 , 3, 5) and
(1 , − 4, 6) in the ratio
(i) 2 : 3 internally (ii) 2 : 3 externally P(– 2, a) Q(5, 1)
40. A point C with y-coordinate ‘3’ lies on the line Using the above information answer the following.
segment joining the points A (3, 4, 2) and B (4, 1, 6).
(i) Find the value of a and b.
Find the coordinates of the point C.
(ii) If a point D1 divides the lines SR into the ratio
41. Find the length of the medians of the triangle with 2 : 1 internally, then find the coordinates of the
vertices A (0, 0, 6), B (0, 4, 0) and C (6, 0, 0). point D1 .
42. Find the length of the medians of the triangle with (iii) If a point D 2 divides the line PR into the ratio
vertices P (2, 0, 8), Q (4, 2, 0) and R(4, 0, 6). 3 : 2 externally, then find the coordinates of the
43. Two vertices of a triangle are A (3,4, 2) and point D 2 .
B (1, 3, 2). The medians of the triangle intersect at
(2, 4, 3). Find the remaining vertex C of the triangle.
CBSE Term II Mathematics XI 113

SOLUTIONS
Objective Questions 11. (a) Let P be the point on Y-axis. Then, it is of the form
1. (c) Coordinates of a point in ZX-plane will be ( x , 0, z ) P( 0, y , 0). Since, the point (1, 2 , 3) is at a distance 10 from
because in ZX-plane, y-coordinate is zero. ( 0, y , 0 ), therefore (1 − 0)2 + (2 − y )2 + ( 3 − 0)2 = 10
2. (a) We know that, on the XY and XZ-planes, the line of ⇒ y 2 − 4y + 4 = 0 ⇒ ( y − 2 ) 2 = 0 ⇒ y = 2
intersection is X-axis.
Hence, the required point is ( 0, 2 , 0).
Z
12. (b) Given, ( a − 0)2 + ( 0 − 1)2 + (1 − 2 )2 = 27
⇒ 27 = a 2 + 1 + 1 ⇒ a 2 = 25 ⇒ a = ± 5
O Y 13. (a) We know that points are said to be collinear, if they lie on
a line.
X Now, PQ = (1 + 2 )2 + (2 − 3)2 + ( 3 − 5)2
3. (b) Since, L is the foot of perpendicular from P on the = 9 + 1 + 4 = 14
XY-plane, z-coordinate will be zero. Hence, coordinates of
QR = (7 − 1)2 + ( 0 − 2 )2 + ( −1 − 3)2 = 36 + 4 + 16
L are (6, 7, 0).
4. (a) (i) The X-axis and Y-axis taken together determine a = 56 = 2 14
plane known as XY-plane.
and PR = (7 + 2 )2 + ( 0 − 3)2 + ( −1 − 5)2 = 81 + 9 + 36
(ii) The coordinates of a point in the XY-plane are of the
form ( x , y , 0). = 126 = 3 14
(iii) Coordinate planes divide the space into eight octants. Thus, PQ + QR = PR . Hence, P, Q and R are collinear.
5. (b) The point (− 2, − 3, − 4) lies in seventh octant. Also, PR ≠ 4PQ
6. (c) The distance between the points P (1, − 3, 4) and 14. (d) The point on the X-axis is of form P ( x , 0, 0). Since, the
Q( − 4, 1, 2 ) is points A and B are equidistant from P. Therefore,
PA 2 = PB 2, i.e. ( x − 3)2 + ( 0 − 2 )2 + ( 0 − 2 )2
PQ = ( − 4 − 1)2 + (1 + 3)2 + (2 − 4)2
= ( x − 5) 2 + ( 0 − 5) 2 + ( 0 − 4) 2
= 25 + 16 + 4 = 45 = 3 5 units
49
7. (a) Given, points P (3, 4, 5) and O (0, 0, 0), ⇒ 4x = 25 + 25 + 16 − 17 i. e. x =
4
PO = ( 0 − 3)2 + ( 0 − 4)2 + ( 0 − 5)2  49 
Thus, the point P on the X-axis is  , 0, 0 which is
= 9 + 16 + 25 = 50  4 
equidistant from A and B.
8. (a) Given, point is P (3, 4, 5).
Distance of P from YZ-plane, [Q YZ-plane, x = 0] 15. (c) Let A( 5, − 1, 1), B(7 , − 4, 7 ), C (1, − 6, 10) and
D ( − 1, − 3, 4) be the four points of a quadrilateral. Here,
d = ( 0 − 3) 2 + ( 4 − 4) 2 + ( 5 − 5) 2 = 3
AB = 4 + 9 + 36 = 7, BC = 36 + 4 + 9 = 7,
9. (b) We know that, on the Y-axis, x = 0 and z = 0.
CD = 4 + 9 + 36 = 7, DA = 36 + 4 + 9 = 7
∴ Point A (0, 4, 0),
AC = 16 + 25 + 81 = 122
PA = ( 0 − 3)2 + ( 4 − 4)2 + ( 0 − 5)2
BD = 64 + 1 + 9 = 74
= 9 + 0 + 25 = 34
Note that AB = BC = CD = DA and AC ≠ BD. Therefore,
10. (b) Clearly,
ABCD is a rhombus.
PQ = ( − 2 − 2 )2 + ( − 2 − 4)2 + ( − 2 − 6)2 16. (b) Substitute m1 = k and m 2 = 1 in the section formula for
= 16 + 36 + 64 = 116 = 2 29 internal division.
Section formula for internal division,
QR = ( 6 + 2 )2 + (10 + 2 )2 + (14 + 2 )2
 m x + m 2x1 m1y 2 + m 2y1 m1z 2 + m 2z1 
= 64 + 144 + 256 = 464 = 4 29 ( x, y , z) =  1 2 , , 
 m1 + m 2 m1 + m 2 m1 + m 2 
PR = ( 6 − 2 )2 + (10 − 4)2 + (14 − 6)2 Put m1 = k and m 2 = 1;
= 16 + 36 + 64 = 116 = 2 29  kx + x1 ky 2 + y1 kz 2 + z1 
= 2 , , 
 1+ k 1+ k 1+ k 
Since, QR = PQ + PR. Therefore, given points are collinear.
114 CBSE Term II Mathematics XI

17. (c) Let P( x , y , z ) be the required point. Then, P divides AB ⇒ x 2 + y 2 + z 2 = x 2 + a 2 − 2 ax + y 2 + z 2


in the ratio 5 : 1. So, ⇒ 0 = − 2 ax + a 2
 5 × 10 + 1 × −2 5 × −6 + 1 × 0 5 × −12 + 1 × 6 a
P( x , y , z ) =  , ,  ⇒ x=
 5+1 5+1 5+1  2
= ( 8, − 5, − 9) (ii) (b) Let A (a, 0, 0), B (0, b, 0), C (0, 0, c) and O (0, 0, 0) be
18. (c) Let the point R divides the line joining the points four points equidistant from the point P (x, y, z).
P (2 , 4, 5) and Q( 3, 5, − 4) in the ratio m : n . Then, the C (0,0,c)
coordinates of R are P (x, y, z)
 3m + 2 n 5m + 4n − 4m + 5n 
 , , . B (0, b, 0)
 m +n m +n m +n  O (0, 0, 0)
For YZ-plane, x-coordinates will be zero. A (a,0,0)
3m + 2 n m −2
∴ = 0⇒ = Then, PA = PB = PC = OP
m +n n 3
Now, OP = PB ⇒ OP 2 = PB 2
Hence, m : n = − 2 : 3
19. (a) Let point P( x , y , z ) divides the line joining the points A ⇒ x 2 + y 2 + z 2 = ( x − 0) 2 + ( y − b ) 2 + ( z − 0) 2
and B in the ratio m : 1. ⇒ x 2 + y 2 + z 2 = x 2 + y 2 + b 2 − 2 by + z 2
m 1 b
B
⇒ b 2 = 2 by ⇒ y =
A P 2
(5, –3, –2) (1, 2, –2)
(iii) (c) Let A (a, 0, 0), B (0, b, 0), C (0, 0, c) and O (0, 0, 0) be
Since, point P is on XOZ-plane. four points equidistant from the point P (x, y, z).
∴ y-coordinate = 0 Then, PA = PB = PC = OP
2m − 3 3 Now, OP = PC ⇒ OP 2 = PC 2
⇒ = 0⇒m =
m +1 2 ⇒ x 2 + y 2 + z 2 = ( x − 0 ) 2 + ( y − 0 ) 2 + ( z − c )2
3 + 2 × 5 13
Now, x= = ⇒ x 2 + y 2 + z 2 = x 2 + y 2 + z 2 + c 2 − 2 zc
3+2 5
⇒ 0 = − 2 zc + c 2
3 × ( −2 ) + 2 × ( −2 )
and z= = −2 c
5 ⇒ z=
2
 13 
∴ Required point is  , 0, − 2 . (iv) (d) All of these
 5 
(v) (a) Coordinates of origin is (0,0, 0)
20. (b) Centroid of ∆ABC = Centroid of ∆DEF
22. (i) (c) Given that the position of Abhay and Birsen are
A
A(2 , 1, − 3) and B( 5, − 8, 3).
Q AB is a diameter.
D E  2 + 5 1 − 8 −3 + 3
∴ Centre ( x , y , z ) =  , , 
 2 2 2 
B C {Q centre is the mid-point of the diameter}
F
 7 −7 
= , , 0
 1 + 3 − 1 2 + 0 + 1 − 3 + 1 − 4 2 2 
= , , 
 3 3 3 
(ii) (a)
= (1 , 1 , − 2 ) 3:1
21. (i) (b) Let A (a, 0, 0), B (0, b, 0), C (0, 0, c) and O (0, 0, 0) be A P B
(2, 1, – 3) (x1, y1, z1) (5, – 8, 3)
four points equidistant from the point P (x, y, z).
C (0,0,c) Using section formula,
P (x, y, z)  3 × 5 + 1 × 2 3( − 8) + 1(1) 3( 3) + 1( −3)
P( x1 , y1 , z1 ) =  , , 
 3+1 3+1 3+1 
B (0, b, 0)
O (0, 0, 0)  17 −23 3
= , , 
A (a,0,0)  4 4 2
Then, PA = PB = PC = OP (iii) (b)
3:2
Now, OP = PA ⇒ OP 2 = PA 2
⇒ x + y + z = ( x − a ) + ( y − 0) + ( z − 0)
2 2 2 2 2 2 A Q B
(2, 1, – 3) (x2, y2, z2) (5, – 8, 3)
[Q distance = ( x1 − x 2 )2 + ( y1 − y 2 )2 + ( z1 − z 2 )2 ]
CBSE Term II Mathematics XI 115

Using section formula, Point B lies on Y-axis, so its coordinates are ( 0, b , 0).
Q( x 2 , y 2 , z 2 ) Point C lies on Z-axis, so its coordinates are ( 0, 0, c ) and
 3 × 5 + 2 × 2 3( − 8) + 2(1) 3( 3) + 2( −3) point E lies in YZ-plane, so its coordinates are ( 0, b , c ).
= , ,  Hence, the coordinates of required points are
 3+2 3+2 3+2 
A (a, 0, 0), D (a, b, 0), B (0, b, 0), C ( 0, 0, c ) and E( 0, b , c ).
 19 −22 3
= , ,  4. Clearly, Point of O = (0, 0, 0)
 5 5 5
Point of A = (2, 0, 0) Point of G = (0, 2, 0)
(iv) (d) Using distance formula, Point of D = (0, 0, 2) Point of B = (2, 2, 0)
d = ( x 2 − x1 )2 + ( y 2 − y1 )2 + ( z 2 − z1 )2 Point of F = (2, 2, 2) Point of E = (2, 0, 2)
The diameter, Point of C = (0, 2, 2)
Y
AB = ( 5 − 2 )2 + ( − 8 − 1)2 + ( 3 + 3)2
G
= 9 + 81 + 36 B
2
= 126 = 3 14 units C F
(v) (d) We have, diameter AB = 3 14
X
diameter 3 14 2 O 2 A
∴ radius = =
2 2 D
E
Subjective Questions Z
1. The name of octants in which given points lie, are given in 5. Given, edge of a cube is 5 unit. It is clear that
the following table
Coordinate of O = ( 0, 0, 0) Coordinate of A = ( 5, 0, 0)
Point Octant Name Coordinate of G = ( 0, 5, 0) Coordinate of D = ( 0, 0, 5)
(2, 3, 8) I (all the coordinates are positive) XOYZ Coordinate of B = ( 5, 5, 0) Coordinate of F = ( 5, 5, 5)
(1, − 1, − 3) VIII (y and z-coordinates are negative) XOY′ Z′ Coordinate of E = ( 5, 0, 5) Coordinate of C = ( 0, 5, 5)
Y
(−2, 4, − 6) VI (x and z-coordinates are negative) X ′ OYZ′
G B
( −3, − 3, 1) III (x and y-coordinates are negative) X ′ OY′ Z
C
( −3, − 2, − 5) VII (all the coordinates are negative) X ′ OY′ Z′ F
5
( 6, − 1 , 4) IV (y-coordinate is negative) XOY′ Z
(2 , 3 , − 5 ) V (z-coordinate is negative) XOYZ′ O A
5 5 X
( − 6, 7 , 9) II (x-coordinate is negative) X ′ OYZ
D E
2. Given point is (2, 3, 4). Here, x-coordinate i.e. 2 is positive. Z
So, we take a point A in positive direction of X-axis at a
6. Let the given points are A( 3, 2 , − 4), B( 5, 4, − 6) and
distance 2 from O. Thus, point A be (2, 0, 0). From point A,
we draw a line perpendicular to X-axis which will be parallel to C( 9, 8, − 10).
Y-axis and take point B at a distance 3 from A in positive direction. Distance formula,
Z d = ( x 2 − x1 )2 + ( y 2 − y1 )2 + ( z 2 − z1 )2
C(2, 3, 4) AB = ( 5 − 3)2 + ( 4 − 2 )2 + ( − 6 + 4)2
= (2 ) 2 + (2 ) 2 + ( − 2 ) 2
O Y
= 4+ 4+ 4 = 4× 3 =2 3
B(2, 3, 0)
A(2, 0, 0) BC = ( 9 − 5)2 + ( 8 − 4)2 + ( − 10 + 6)2
X
= ( 4) 2 + ( 4) 2 + ( − 4) 2
Thus, point B be (2, 3, 0). Draw a line parallel to Z-axis from
point B, which is perpendicular to XY-plane and take point C at = 16 + 16 + 16 = 16 × 3 = 4 3
this line in positive direction at a distance 4 units from B. Thus, AC = ( 9 − 3)2 + ( 8 − 2 )2 + ( − 10 + 4)2
point C (2, 3, 4) is the required location of given point in space,
which is shown in the figure. = ( 6) 2 + ( 6) 2 + ( − 6) 2
3. Given, the coordinates of point P are ( a , b , c ). = 36 + 36 + 36 = 36 × 3 = 6 3
Which shows that, OA = a , OB = b and OC = c. Q AB + BC = AC or sum of two smaller distances is equal to
Now, point A lies on X-axis, so its coordinates are ( a , 0, 0). the larger distance.
Point D lies in XY-plane, so its coordinates are ( a , b , 0).
Hence, the given points are collinear.
116 CBSE Term II Mathematics XI

7. Given points, A(1, 2, 3), B (2, 3, 1) and C (3, 1, 2). We know ⇒ 9 − 6y + 4 − 4z = 1 − 2 z


the distance formula for P( x1 , y1 , z1 ) and Q( x 2 , y 2 , z 2 ) ⇒ 12 − 6y − 2 z = 0 … (ii)
d = ( x 2 − x1 )2 + ( y 2 − y1 )2 + ( z 2 − z1 )2 From Eq. (i) put 3y = z
12 − 2( z ) − 2 z = 0 ⇒ 12 = 4z ⇒ z = 3
AB = (2 − 1)2 + ( 3 − 2 )2 + (1 − 3)2
From Eq. (i), 3y = 3 ⇒ y = 1
= 1 + 1 + 4 = 6 units
∴ (0, 1, 3)
BC = ( 3 − 2 )2 + (1 − 3)2 + (2 − 1)2 10. Given points, A( 3, 2 , − 5), B( − 3, 8, − 5), C( − 3, 2 , 1) and
D( − 1, 4, − 3). Points A ,B and C forms ∆ABC.
= 1 + 4+ 1
To prove D, a circumcentre of ∆ABC with vertices A ,B and
= 6 units C, we need to prove AD = BD = CD.
CA = (1 − 3)2 + (2 − 1)2 + ( 3 − 2 )2 Using distance formula,
d = ( x 2 − x1 )2 + ( y 2 − y1 )2 + ( z 2 − z1 )2
= 4+1+1
= 6 units AD = ( − 1 − 3)2 + ( 4 − 2 )2 + ( − 3 + 5)2

We got that AB = BC = CA = 16 + 4 + 4 = 24 = 2 6 units


Q All three sides of the triangle are equal. Hence, the given BD = ( − 1 + 3)2 + ( 4 − 8)2 + ( − 3 + 5)2
points forms an equilateral triangle.
= 4 + 16 + 4 = 24 = 2 6 units
8. Let P( x , y , z ) be any point on the given curve and let
A( 3, 4 , − 5) and B( − 2 , 1, 4) be the given points. CD = ( − 1 + 3)2 + ( 4 − 2 )2 + ( − 3 − 1)2
Then, PA = PB = 4 + 4 + 16 = 24 = 2 6 units
Using distance formula, Q AD = BD = CD
d = ( x 2 − x1 )2 + ( y 2 − y1 )2 + ( z 2 − z1 )2 , we get Hence, D is the circumcentre of ∆ABC.
11. Let vertices of rectangle ABCD are
⇒ ( x − 3) 2 + ( y − 4) 2 + ( z + 5) 2
A ( − 2 , 4, 1), B ( − 1, 5, 5), C(1, 1, 1) and D (2 , 2 , 5).
= ( x + 2 ) 2 + ( y − 1) 2 + ( z − 4) 2
Now, AB = ( − 1 + 2 )2 + ( 5 − 4)2 + ( 5 − 1)2
On squaring both sides, we get
( x − 3) 2 + ( y − 4) 2 + ( z + 5) 2 = (1)2 + (1)2 + ( 4)2 = 1 + 1 + 16 = 18

= ( x + 2 ) 2 + ( y − 1) 2 + ( z − 4) 2 BC = (1 + 1)2 + (1 − 5)2 + (1 − 5)2


⇒ x 2 + 9 − 6x + y 2 + 16 − 8y + z 2 + 25 + 10z = (2 )2 + ( −4)2 + ( −4)2 = 4 + 16 + 16 = 36 = 6
= x + 4 + 4x + y + 1 − 2 y + z + 16 − 8z
2 2 2
CD = (2 − 1)2 + (2 − 1)2 + ( 5 − 1)2
⇒ − 6x − 8y + 10z + 50 = 4x − 2 y − 8z + 21
= (1)2 + (1)2 + ( 4)2 = 1 + 1 + 16 = 18
⇒ − 10x − 6y + 18z + 29 = 0
⇒ 10x + 6y − 18z − 29 = 0 and DA = ( − 2 − 2 )2 + ( 4 − 2 )2 + (1 − 5)2
Hence, the required curve is = ( −4)2 + (2 )2 + ( −4)2 = 16 + 4 + 16 = 36 = 6
10x + 6y − 18z − 29 = 0
∴ Diagonal AC = (1 + 2 )2 + (1 − 4)2 + (1 − 1)2
9. Let P( 0, y , z ) be a point on the YZ-plane such that
PA = PB = PC. = ( 3)2 + ( − 3)2 + 0 = 9 + 9 = 18
Now, PA = PB ⇒ ( 0 − 2 )2 + ( y − 0)2 + ( z − 3)2 and Diagonal BD = (2 + 1)2 + (2 − 5)2 + ( 5 − 5)2
= ( 0 − 0) + ( y − 3) + ( z − 2 )
2 2 2
= ( 3)2 + ( − 3)2 + 0 = 9 + 9 = 18
⇒ 4 + y + z + 9 − 6z = 0 + y + 9 − 6y + z + 4 − 4z
2 2 2 2
Here, AB = CD , BC = AD and diagonal AC = diagonal BD.
⇒ − 6z = − 6y − 4z Hence, ABCD is a rectangle.
⇒ 6y = 6z − 4z 12. Let the given vertices be A( − 1, − 6, 10), B(1, − 3, 4),
⇒ 6y = 2 z ⇒ 3y = z … (i) C( − 5, − 1, 1) and D( − 7 , − 4, 7 ).
and PB = PC To prove A ,B , C and D the vertices of a rhombus, we need to
show that AB = BC = CD = DA
⇒ ( 0 − 0) + ( y − 3) 2 + ( z − 2 ) 2
2
i.e. all sides equal andAC ≠ BD i.e. diagonals not equal.
= ( 0 − 0) 2 + ( y − 0) 2 + ( z − 1) 2 And the distance formula is
⇒ 0 + y + 9 − 6y + z 2 + 4 − 4z
2
d = ( x 2 − x1 )2 + ( y 2 − y1 )2 + ( z 2 − z1 )2
= 0 + y + z + 1 − 2z
2 2
Now, AB = (1 + 1)2 + ( − 3 + 6)2 + ( 4 − 10)2
CBSE Term II Mathematics XI 117

= 4 + 9 + 36 = 49 = 7 units OD = 100 + 169 = 269


BC = ( − 5 − 1) + ( − 1 + 3) + (1 − 4)
2 2 2 In ∆ODG;
OG 2 = OD 2 + DG 2
= 36 + 4 + 9 = 49 = 7 units
OG = 269 + 82 = 269 + 64 = 333 unit
CD = ( − 7 + 5)2 + ( − 4 + 1)2 + (7 − 1)2
15. Let the point in xy-plane be P( x1 , y1 , 0). Let the given points
= 4 + 9 + 36 = 49 = 7 units
are A(2 , 0, 3), B( 0, 3, 2 ) and C( 0, 0, 1).
DA = ( − 7 + 1)2 + ( − 4 + 6)2 + (7 − 10)2 According to the given condition,
= 36 + 4 + 9 = 49 = 7 units AP 2 = BP 2 = CP 2

AC = ( − 5 + 1)2 + ( − 1 + 6)2 + (1 − 10)2 ∴ ( x1 − 2 )2 + y12 + 9 = x12 + ( y1 − 3)2 + 4 = x12 + y12 + 1

= 16 + 25 + 81 = 122 units From 1st and 2nd terms,


x12 + 4 − 4x1 + y12 + 9 = x12 + y12 − 6y1 + 9 + 4
BD = ( − 7 − 1)2 + ( − 4 + 3)2 + (7 − 4)2
⇒ 4x1 − 6y1 = 0 …(i)
= 64 + 1 + 9 = 74 units From 2nd and 3rd terms,
We got that AB = BC = CD = DA but AC ≠ BD. x12 + y12 + 9 − 6y1 + 4 = x12 + y12 + 1 ⇒ 6y1 = 12 ⇒ y1 = 2
Hence A ,B , C and D forms a rhombus. On putting the value of y1 in Eq. (i), we get
13. Let the given vertices be A( a − 1, a , a + 1), B( a , a + 1, a − 1) x1 = 3
and C( a + 1, a − 1, a ). Hence, required point is ( 3, 2 , 0).
To know that type of the triangle, first we will obtain the 16. ABC is a triangle with vertices A( 0, 0, 6), B( 0, 4, 0) and
length of all three sides. C( 6, 0, 0).
And the distance formula is Let D be the mid-point of BC. So, AD will be the median of
d = ( x 2 − x1 )2 + ( y 2 − y1 )2 + ( z 2 − z1 )2 the triangle.
A(0,0,6)
Now, AB = ( a − a + 1)2 + ( a + 1 − a )2 + ( a − 1 − a − 1)2
= 1+1+ 4 = 6
F E
BC = ( a + 1 − a )2 + ( a − 1 − a − 1)2 + ( a − a + 1)2
= 1+ 4+1 = 6 B(0,4,0) C(6,0,0)
D
and CA = ( a − 1 − a − 1) + ( a − a + 1) + ( a + 1 − a )
2 2 2 (x, y, z)

= 4+1+1 = 6 ⇒ Coordinates of point D

∴ AB = BC = CA  0 + 6 4 + 0 0 + 0
= , ,  = ( 3, 2 , 0)
 2 2 2 
Hence, given points are vertices of an “equilateral triangle”
for any real value of a. Length of Median AD = Distance between A and D
14. Given dimensions are AD = ( 0 − 3)2 + ( 0 − 2 )2 + ( 6 − 0)2
a = 10, b = 13 and c = 8.
= 9 + 4 + 36 = 49 = 7
∴ Required length = a 2 + b 2 + c 2
17. For centroid of ∆ABC,
= 100 + 169 + 64 = 333 a −2 + 4 a + 2
x= =
Or 3 3
The longest piece of a string will be the length of the 1+ b+7 b+ 8
diagonal [ OG ]. y= =
3 3
C E 3− 5+ c c −2
and z= =
3 3
G
F But, given centroid is ( 0, 0, 0).
10 O 13 B a+2
8 ∴ = 0 ⇒ a = −2
3
A D
b+8
In ∆OAD; = 0 ⇒ b = −8
3
OD 2 = OA 2 + AD 2
c −2
= 0 ⇒ c =2
OD = 102 + 132 3
118 CBSE Term II Mathematics XI

18. The given points are A ( − 2 , 3, 5) and B (1, − 4, 6). The coordinates of point C
Let the point C divides the line externally in the ratio 2 : 3.  3(2 ) + 1(1) 3( 4) + 1(2 ) 3( 6) + 1( 3)
= , , 
3  3+1 3+1 3+1 
2
 7 7 21
= , , 
C A B 4 2 4
(–2,3, 5) (1, –4, 6)
21. Given points are P (2 , − 1, 3) and Q ( 4, 3, 1).
Here, the ratio is 2 : 3 .
Let the required ratio be λ : 1.
∴ m = 2, n = 3  4λ + 2 3λ − 1 λ + 3
The coordinates of point C Then, the coordinates of R are  , , 
 λ + 1 λ + 1 λ + 1
 m x 2 − nx1   my 2 − ny1   mz 2 − nz1   λ 1
=  , , 
 m − n   m − n   m − n   P R Q
(2, –1, 3) (4, 3, 1)
 2 × ( 1) − 3 × ( − 2 ) 2 × ( − 4) − 3 × 3 2 × 6 − 3 × 5 
= , ,   20 5 15
 (2 − 3 ) (2 − 3 ) (2 − 3 )  But, the coordinates of R are  , ,  . [given]
7 7 7
 2 + 6 − 8 − 9 12 − 15 4λ + 2 20
= , ,  = ( − 8, 17 , 3) ∴ = [Q x-coordinate of R]
 ( − 1) ( − 1) ( − 1)  λ+1 7
19. The given points are P( − 4, 6, 8) and Q(2 , − 2 , 4). ⇒ 28 λ + 14 = 20λ + 20
Let the point R divide the line internally in the ratio 1 : 3. 3
⇒ 8λ = 6 ⇒ λ =
1 3 4
3
P R Q So, the required ratio is : 1, i.e. 3 : 4.
(–4, 6, 8) (2, –2, 4) 4
Note Here, we can also find the ratio λ : 1 by taking
Here the ratio is 1 : 3
y-coordinate and z-coordinate.
∴ m1 = 1, m 2 = 3
22. Let B divides AC in the ratio λ : 1.
The coordinates of point R λ:1
 m x + m 2x1 m1y 2 + m 2y1 m1z 2 + m 2z1 
= 1 2 , ,  A B C
 m1 + m 2 m1 + m 2 m1 + m 2  (– 2, 3, 5) (1, 2, 3) (7, 0, – 1)

 1(2 ) + 3( − 4) 1( − 2 ) + 3( 6) 1( 4) + 3( 8) Here, the point B divides the line AC internally, so its
R = , ,  coordinates are
 1+ 3 1+ 3 1+ 3 
 mx 2 + nx1 my 2 + ny1 mz 2 + nz1 
 2 − 12 − 2 + 18 4 + 24  m +n , m +n , m +n 
= , ,   
 4 4 4 
 λ (7 ) + 1( − 2 ) λ(0) + 1( 3) λ( − 1) + 1( 5)
 − 10 16 28  5  ⇒ B= , , 
= , ,  =  − , 4, 7  λ+1 λ+1 λ+1 
 4 4 4  2 
7λ − 2 3 5 − λ
 5  = , , 
Hence, the required coordinates of the point is  − , 4, 7 .  λ + 1 λ + 1 λ + 1
 2 
20. The given points are A (1, 2, 3) and B (2, 4, 6). But B = (1, 2, 3)
(i) Let the point C divides the line internally in the ratio On comparing the corresponding coordinates
1 : 2. 7λ − 2 3 5−λ
= 1, = 2, =3
1:2 λ+1 λ+1 λ+1
A C B ⇒ 7 λ − 2 = λ + 1, 3 = 2 λ + 2, 5 − λ = 3λ + 3
(1, 2, 3) (2, 4, 6)
⇒ 6λ = 3, 1 = 2 λ, 2 = 4λ
The coordinates of point C 1
⇒ λ= ,
 1(2 ) + 2 (1) 1( 4) + 2(2 ) 1( 6) + 2 ( 3) 2
= , , 
 1+ 2 1+ 2 1+ 2  Hence, point B divides AC internally in the ratio 1 : 2.
4 8  23. Let the point R ( x , y , z ) divides PQ in the ratio k : 1.
=  , , 4
3 3  k 1
(ii) Let the point C divides the line internally in the ratio P R Q
3 : 1. (2, –3, 4) (4, y, z) (8, 0, 10)
3:1 Given, x-coordinate of R = 4.
k × 8+ 1×2 8k + 2
A C B ∴ =4 ⇒ =4
(1, 2, 3) (2, 4, 6) k+1 k+1
CBSE Term II Mathematics XI 119

⇒ 8k + 2 = 4k + 4 ⇒ 8k − 4k = 4 − 2 k+4
and = 2 ⇒ k + 4 = 2k + 2 ⇒ k = 2
2 1 k+1
⇒ 4k = 2 ⇒ k = =
4 2 From each of these equations, we get k = 2
1 Since, from each equations, we get same value of k.
So, k : 1 = : 1 or 1 : 2.
2 Therefore, the given points are collinear and C divides AB
Hence, the point R divides PQ internally in the ratio 1 : 2. internally in the ratio 2 : 1. Hence proved.
Therefore, y-coordinate of R 26. Let PQRS be a parallelogram and coordinates of the points S
 1 × 0 + 2 × ( − 3)  −6 be ( a , b , c ) and diagonals PR and QS intersect at θ.
=  =   = −2
 1+2   3 In a parallelogram diagonals bisect each other.
and z-coordinate of R ∴ Coordinates of mid-point of PR = coordinates of mid-point
 1 × 10 + 2 × 4  10 + 8 18 of QS
=  =  = =6 (a , b , c )
 1+2   3  3 S R (– 3, 1, 4)
Hence, coordiantes of point R are ( 4, − 2 , 6).
O
24. Let Q divides PR in the ratio k : 1.
k 1
P Q
P Q R (2, – 4, 3) (3, 2, – 6)
(3, 2, –4) (5, 4, –6) (9, 8, –10)
 2 − 3 −4 + 1 3 + 4  3 + a 2 + b −6 + c
Here, the point Q divides the line PR internally in the ratio ⇒ , ,  = , , 
 2 2 2   2 2 2 
k : 1, so its coordinates are
 1 3 7   3 + a 2 + b c − 6
 k × 9 + 1 × 3 k × 8 + 1 × 2 k( − 10) + 1 × ( −4) ⇒ − , − ,  =  , , 
 , ,   2 2 2  2 2 2 
 k+1 k+1 k+1 
[using internal ratio formula] On comparing corresponding coordinates, we get
 9k + 3 8k + 2 − 10k − 4 1 3+ a 3 2+b 7 c−6
− = ,− = , =
= , ,  2 2 2 2 2 2
 k+1 k+1 k+1 
⇒ − 1 = 3 + a, − 3 = 2 + b, 7 = c − 6
But given that Q = ( 5, 4, − 6)
⇒ − 4 = a, − 5 = b, 13 = c
On comparing the corresponding coordinates, we get
9k + 3 8k + 2 − 10 k − 4 Hence, the coordinates of the fourth vertex is ( − 4, − 5, 13).
= 5, = 4 and =−6
k+1 k+1 k+1 27. If PM is the internal bisector of angle P. Then, we have
(by geometry)
9k + 3 = 5k + 5, 8k + 2 = 4k + 4 and − 10k −4 =− 6k − 6 QM PQ
1 =
⇒ 4k = 2 ⇒ k = MR PR
2
P(3,2,–5)
Hence, point Q divides PR internally in the ratio 1 : 2.
 1 
25. Let C  0, , 2 divides the join of A (2 , − 3, 4) and
 3 
B ( −1, 2 , 1) in the ratio k : 1.
(2,–3,4) (–1,2,1)
A C B Q(2,3,4) R(4,–7,–4)
1 M
k
(2 − 3 ) 2 + ( 3 − 2 ) 2 + ( 4 + 5 ) 2
=
( 4 − 3) 2 + ( − 7 − 2 ) 2 + ( − 4 + 5) 2
Then, coordinates of C are
 − k + 2 2 k − 3 k + 4 1 + 1 + 81 1
 , ,  …(i) = =
 k + 1 k + 1 k + 1 1 + 81 + 1 1
[using internal ratio formula] QM : MR = 1 : 1
 1  Therefore, M is the mid-point of QR.
But coordinates of C are  0, , 2 . [given] …(ii)
 3   2 + 4 3 − 7 4 − 4  6 4 0
M= , ,  = ,− , 
On comparing Eqs. (i) and (ii), we get  2 2 2   2 2 2
−k + 2
= 0 ⇒ −k + 2 = 0 ⇒ k = 2 = ( 3, − 2 , 0)
k+1
28. Let the P ( x1 , y1 , z1 ) and Q ( x 2 , y 2 , z 2 ) trisect line segment AB.
2k − 3 1
= ⇒ 6k − 9 = k + 1 ⇒ 5k = 10 ⇒ k = 2
k+1 3 A P Q B
(2, 1, –3)(x1, y1, z1) (x2, y2, z2) (5,–8, 3)
120 CBSE Term II Mathematics XI

Since, the point P divides the line AB in the ratio 1 : 2 Centroid G


internally, then a − 1 +2
2 ×2 + 1× 5 9 1= ⇒3 = a + 1⇒a =2
x1 = = =3 3
1+2 3 2+b+6
1= ⇒3 = b + 8⇒b = − 5
2 × 1 + 1 × ( − 8) − 6 3
y1 = = = −2
3 3 5−4+ c
1= ⇒3 =1 + c⇒c =2
2 × ( − 3) + 1 × 3 − 6 + 3 − 3 3
z1 = = = = −1
3 3 3 ∴ a = 2, b = − 5and c = 2
Since, the point Q divide the line segment AB in 2 : 1, then 31. The sign of coordinates of the points in the octants in which
1×2 + 2 × 5 the space is divided are given in the following table
x2 = =4
3
I II III IV V VI VII VIII
1 × 1 + (− 8 × 2) Octants
y2 = =−5 ′ ′ ′
3 Coordinates OXYZ OX ′ YZ OX ′ Y ′Z OXY ′Z OXYZ′ OX ′ Y Z ′ OX Y Z OXY ′ Z ′
1 × ( − 3) + 2 × 3
z2 = =1 x + − − + + − − +
3
y + + − − + + − −
So, the coordinates of P are (3, − 2, − 1) and the coordinates
z + + + + − − − −
of Q are (4, − 5, 1).
The name of octants in which given points i.e., are given in
29. Let the point C1 trisects the line AB i.e., it divides the line in
the following table.
the ratio 1 : 2.
1:2 Point Octant Name
A C1 B
(– 3, 1, 2) (3, – 8, 5) (− 2 1, 3) II OX′ YZ

 1( 3) + 2( − 3) 1( − 8) + 2(1) 1( 5) + 2 (2 ) (2, 2, − 2) V OXYZ′


⇒ C1 =  , , 
 1+2 1+ 2 1+2  (1, − 4, 1) IV OXY ′ Z
 3 − 6 −8 + 2 5 + 4 ( − 3, − 2 , − 1) VII OX′ Y ′ Z′
= , , 
 3 3 3 
( − 5, − 2 , 2 ) III OX′ Y ′ Z
 −3 −6 9
= , ,  = ( − 1, − 2 , 3) ( − 4, 3, − 2 ) VI OX′ YZ′
 3 3 3
Again let the point C 2 divides AB internally in the ratio 2 : 1. (2 , − 3 , − 6 ) VIII OXY ′ Z′
Then, (1, 4, 8) I OXYZ
2:1
A C2 B 32. Let A ( 3, − 2 , 4), B (1, 0, − 2 ) and C ( −1, 2 , − 8) be the
(– 3, 1, 2) (3, – 8, 5)
given points.
 2( 3) + 1( − 3) 2( − 8) + 1(1) 2( 5) + 1(2 ) (3,– 2,4) (1,0, – 2) (–1,2, – 8)
⇒ C2 =  , , 
 2+1 2+1 2+1  A B C
 6 − 3 −16 + 1 10 + 2 
= , ,  Then, AB = (1 − 3)2 + ( 0 + 2 )2 + ( −2 − 4)2
 3 3 3 
[using distance formula]
 3 −15 12 
= , ,  = 4 + 4 + 36 = 44 = 2 11 units
3 3 3
= (1 , − 5 , 4 ) BC = ( −1 − 1)2 + (2 − 0)2 + ( −8 + 2 )2
Hence, required points are ( − 1, − 2 , 3) and (1, − 5, 4). [using distance formula]
30. Given that G(1, 1, 1) is the centroid of the ∆ABC. = 4 + 4 + 36 = 44 = 2 11 units
A(a,2,5)
and CA = ( 3 + 1)2 + ( −2 − 2 )2 + ( 4 + 8)2
(1,1,1)

[using distance formula]


F E
G = 16 + 16 + 144 = 176 = 4 11 units
We observe that,
B(–1,b,–4) C(2,6,c) AB + BC = 2 11 + 2 11
D
= 4 11 = CA
Hence, A, B and C are collinear.
CBSE Term II Mathematics XI 121

33. Let A ( −2 , 6, − 2 ), B ( 0, 4, − 1), C ( −2 , 3, 1) and D ( −4, 5, 0) be ⇒ 4x + 25 = 5 ( x + 4)2 + y 2 + z 2


the given points. [divide both sides by − 4]
(–4, 5, 0) (–2, 3, 1) Again squaring both sides, we get
D C
( 4x + 25)2 = 25 [( x + 4)2 + y 2 + z 2 ]
⇒ 16x 2 + 625 + 200x = 25 [( x + 4)2 + y 2 + z 2 ]
⇒ 16x 2 + 625 + 200x = 25 [ x 2 + 16 + 8x + y 2 + z 2 ]
A B ⇒ 16x 2 + 625 + 200x = 25x 2 + 400 + 200x + 25y 2 + 25z 2
(–2, 6, –2) (0, 4, –1)
⇒ 9x 2 + 25y 2 + 25z 2 − 225 = 0
∴ AB = ( 0 + 2 )2 + ( 4 − 6)2 + ( −1 + 2 )2 which is the required equation.
[using the distance formula] 35. Let P( x , y , z ) be the required point.
= 4+ 4+1 And we are given A( 0, 0, − α) and B( 0, 0, α).
According to the question,
= 9 = 3 units
AP + BP = K , where K is any arbitrary constant.
BC = ( −2 − 0)2 + ( 3 − 4)2 + (1 + 1)2 ⇒ ( x − 0) 2 + ( y − 0) 2 + ( z + α ) 2 +
= 4+1+ 4
( x − 0) 2 + ( y − 0) 2 + ( z − α ) 2 = K
= 9 = 3 units
⇒ x2 + y 2 + z2 + α 2 + 2 zα +
CD = ( −4 + 2 ) + ( 5 − 3) + ( 0 − 1)
2 2 2
x 2 + y 2 + z 2 + α 2 − 2 zα = K
= 4+ 4+1
⇒ x 2 + y 2 + z 2 + α 2 + 2 zα − K
= 9 = 3 units
= − x 2 + y 2 + z 2 + α 2 − 2 zα
AD = ( −4 + 2 )2 + ( 5 − 6)2 + ( 0 + 2 )2
On squaring both sides, we get
= 4+1+ 4 ( x 2 + y 2 + z 2 + α 2 + 2 zα − K ) 2
= 9 = 3 units
= ( x 2 + y 2 + z 2 + α 2 − 2 zα ) 2
Here, AB = BC = CD = DA.
⇒ x 2 + y 2 + z 2 + α 2 + 2 zα + K 2
So, ABCD is a square or a rhombus.
− 2 K x 2 + y 2 + z 2 + α 2 + 2 zα
Now, AC = ( −2 + 2 )2 + ( 3 − 6)2 + (1 + 2 )2
= x 2 + y 2 + z 2 + α 2 − 2 zα
[Q distance = ( x 2 − x1 )2 + ( y 2 − y1 )2 + ( z 2 − z1 )2 ]
⇒ 4 zα + K 2 = 2 K x 2 + y 2 + z 2 + α 2 + 2 zα
= 0 + 9 + 9 = 18 units
On squaring again, we get
and BD = ( −4 − 0)2 + ( 5 − 4)2 + ( 0 + 1)2
⇒ ( 4zα + K 2 )2 = 4K 2 ( x 2 + y 2 + z 2 + α 2 + 2 zα )
= 16 + 1 + 1 = 18 units
⇒ 16z 2α 2 + K 4 + 2 ⋅( 4zα )(K 2 )
Since, diagonal AC = diagonal BD
= 4K 2x 2 + 4K 2y 2 + 4K 2z 2 + 4K 2α 2 + 4K 2 ⋅ 2 zα
Hence, ABCD is a square.
34. Let the point be P( x , y , z ), then it is given PA + PB = 10 ⇒ 16z 2α 2 + K 4 + 8zαK 2
= 4K 2x 2 + 4K 2y 2 + 4K 2z 2 + 4K 2α 2 + 8zαK 2
⇒ ( x − 4) 2 + ( y − 0) 2 + ( z − 0) 2
⇒ 0 = 4K 2x 2 + 4K 2y 2 + 4K 2z 2
+ ( x + 4)2 + ( y − 0)2 + ( z − 0)2 = 10 − 16z 2α 2 + 4K 2α 2 − K 4
⇒ ( x − 4)2 + y 2 + z 2 = 10 − ( x + 4)2 + y 2 + z 2 ⇒ 4K 2x 2 + 4K 2y 2 + 4z 2(K 2 − 4α 2 ) + K 2( 4α 2 − K 2 ) = 0
On squaring both sides, we get Hence, the required locus of a point is
( x − 4)2 + y 2 + z 2 = 100 + ( x + 4)2 + y 2 + z 2 4K 2x 2 + 4K 2y 2 + 4z 2(K 2 − 4α 2 ) + K 2( 4α 2 − K 2 ) = 0
− 20 ( x + 4)2 + y 2 + z 2 36. Let A( − 1, 2 , 1), B(1, − 2 , 5), C( 4, − 7 , 8) and D(2 , − 3, 4) be
the vertices of a quadrilateral ABCD.
⇒ x + 16 − 8x = 100 + x + 16 + 8x
2 2
D(2, –3, 4) C(4, – 7, 8)
− 20 ( x + 4)2 + y 2 + z 2
⇒ − 8x − 8x − 100 = − 20 ( x + 4)2 + y 2 + z 2 P
⇒ − 16x − 100 = − 20 ( x + 4) + y + z 2 2 2

A(– 1, 2, 1) B(1, – 2, 5)
122 CBSE Term II Mathematics XI

Then, mid-point of AC 38. (i) Let A( 0, 7 , − 10), B(1, 6, − 6) and C ( 4, 9, − 6) are the
 −1 + 4 2 − 7 1 + 8  3 −5 9 vertices of a triangle. Then,
= , ,  = , , 
 2 2 2   2 2 2 Side AB = distance between points A and B
Q coordinates of mid- point  = ( 0 − 1)2 + (7 − 6)2 + ( − 10 + 6)2
 x + x  y + y  z + z 
=  1 2  = 1 + 1 + 16 = 18 = 3 2
2
, 1 2
 , 1 
  2   2   2  and side BC = distance between points B and C
 1 + 2 − 2 − 3 5 + 4  3 −5 9 = (1 − 4 ) 2 + ( 6 − 9 ) 2 + ( − 6 + 6 ) 2
and mid-point of BD =  , ,  = , , 
 2 2 2   2 2 2
= 9 + 9 + 0 = 18 = 3 2
Here, mid-points of both the diagonals are same i.e. they Clearly, AB = BC. Hence, triangle is an isosceles
bisect each other. Hence, ABCD is a parallelogram. triangle.
Now, the distance formula, (ii) Let A ( 0, 7 , 10), B ( − 1, 6, 6) and C( − 4, 9, 6) are the
d = ( x 2 − x1 )2 + ( y 2 − y1 )2 + ( z 2 − z1 )2 vertices of a triangle. Then,
AB = (1 + 1)2 + ( − 2 − 2 )2 + ( 5 − 1)2 Side AB = ( 0 + 1)2 + (7 − 6)2 + (10 − 6)2
= 4 + 16 + 16 = 36 = 6 ⇒ AB = 1 + 1 + 16 = 18 = 3 2

BC = ( 4 − 1) + ( − 7 + 2 ) + ( 8 − 5)
2 2 2 Side BC = ( − 1 + 4)2 + ( 6 − 9)2 + ( 6 − 6)2
= 9 + 25 + 9 = 43 ⇒ BC = 9 + 9 + 0 = 18 = 3 2
Q ABCD is a parallelogram and side CA = ( − 4 − 0)2 + ( 9 − 7 )2 + ( 6 − 10)2
∴ AB = CD and BC = AD ⇒ CA = 16 + 4 + 16 = 36 = 6
Hence, the length of sides of the parallelogram are 6 units,
Now, AB 2 + BC 2 = ( 3 2 )2 + ( 3 2 )2 = 62 = CA 2
43 units, 6 units and 43 units respectively.
∴ ∆ABC is right angled triangle at B.
37. Since, AD is the bisector of ∠BAC.
(iii) Let A( − 1, 2 , 1), B (1, − 2 , 5), C( 4, − 7 , 8) and D(2 , − 3, 4)
A(3,2,0) are the vertices of a quadrilateral ABCD.
D C

P
B C
D (–9,6,–3) A B
(5,3,2)
BD AB  − 1 + 4 2 − 7 1 + 8
⇒ = [by angle bisector theorem] ...(i) Then, mid-point of AC =  , , 
DC AC  2 2 2 
AB = ( 5 − 3)2 + ( 3 − 2 )2 + (2 − 0)2  3 −5 9
Now, = , , 
 2 2 2
[Q distance = ( x 2 − x1 )2 + ( y 2 − y1 )2 + ( z 2 − z1 )2 ]
 1 + 2 − 2 − 3 5 + 4
Mid-point of BD =  , , 
= 2 2 + 12 + 2 2  2 2 2 
= 4 + 1 + 4 = 9 = 3 units  3 −5 9
= , , 
 2 2 2
and AC = ( −9 − 3)2 + ( 6 − 2 )2 + ( −3 − 0)2
Mid-points of both the diagonals are same (i.e., they
= ( −12 )2 + ( 4)2 + ( −3)2 bisect each other.)
= 144 + 16 + 9 = 169 = 13 units Hence, ABCD is a parallelogram.
BD 3 39. (i) Let P (x, y, z) be any point which divides the line
Then, from Eq. (i), =
DC 13 segment joining points A ( −2 , 3, 5) and B (1, − 4, 6) in
Hence, D divides BC in the ratio 3 : 13. Then, coordinates the ratio 2 : 3 internally.
of D are 2 3
 3( −9) + 13( 5) 3( 6) + 13( 3) 3( −3) + 13(2 ) 
 , ,  A P B
 3 + 13 3 + 13 3 + 13  (– 2, 3, 5) (1,− 4,6)
[using internal ratio formula]
Here, the ratio is 2 : 3.
 −27 + 65 18 + 39 −9 + 26
= , ,  ∴ m1 = 2 , m 2 = 3
 16 16 16 
Then, coordinates of point P
 38 57 17   19 57 17 
= , ,  = , ,   m x + m 2x1 m1y 2 + m 2y1 m1z 2 + m 2z1 
 16 16 16   8 16 16  = 1 2 , ,
 m1 + m 2 m1 + m 2 m1 + m 2 
CBSE Term II Mathematics XI 123

2 × 1 + 3 × ( −2 ) 2( −4) + 3( 3) 2( 6) + 3( 5)   0 + 6 4 + 0 0 + 0
= , , Coordinates of point D =  , ,  = ( 3, 2 , 0)
 2+3 2+3 2 + 3   2 2 2 
Q coordinates of mid - point 
 2 − 6 −8 + 9 12 + 15  −4 1 27 
= , ,  = , ,   x + x y + y z + z  
 5 5 5   5 5 5  1 2
, 1 2
, 1 2
 
(ii) Let P (x, y, z) be any point which divides the line  2 2 2  
segment joining points A ( −2 , 3, 5) and B (1, − 4, 6) in  0 + 6 0 + 0 6 + 0
Coordinates of point E =  , ,  = ( 3, 0, 3)
the ratio 2 : 3 externally.  2 2 2 
Here, the ratio is 2 : 3.  0 + 0 0 + 4 6 + 0
and coordinates of point F =  , ,  = ( 0, 2 , 3)
∴ m1 = 2 , m 2 = 3  2 2 2 
Then coordinates of point P
Now, length of median AD
 m x − m 2x1 m1y 2 − m 2y1 m1z 2 − m 2z1   = Distance between points A and D
=  1 2 , , 
  m1 − m 2 m1 − m 2 m1 − m 2  
AD = ( 0 − 3)2 + ( 0 − 2 )2 + ( 6 − 0)2
3
2 [Q distance = ( x1 − x 2 )2 + ( y1 − y 2 )2 + ( z1 − z 2 )2 ]
P A B = 9 + 4 + 36 = 49 = 7
(– 2, 3, 5) (1,− 4,6)
Similarly, BE = ( 0 − 3)2 + ( 4 − 0)2 + ( 0 − 3)2
2(1) − ( 3)( −2 ) 2( −4) − ( 3) × 3 2( 6) − ( 3)( 5) 
= , , = 9 + 16 + 9 = 34
 2 − ( 3) 2 − ( 3) 2 − (3) 
and CF = ( 6 − 0)2 + ( 0 − 2 )2 + ( 0 − 3)2
 2 + 6 −8 − 9 12 − 15  8 −17 −3
= , ,  = , ,  = ( −8, 17 , 3) = 36 + 4 + 9
 2 − 3 2 − 3 2 − 3   −1 −1 −1
= 49 = 7
40. Let the point C( x , y , z ) divides AB in the ratio λ : 1.
λ:1
Hence, length of the medians are 7, 34 and 7.
42. PQR is a triangle with vertices P (2, 0, 8), Q (4, 2, 0) and
A C B
(3, 4, 2) (x, 3, z) (4, 1, 6) R (4, 0, 6). Let points M1 , M 2 and M 3 be the mid-points of
QR, PR and PQ, respectively. So, PM1 , QM 2 and RM 3 will be
Given, y-coordinate of C = 3 the medians of the triangle.
λ (1) + 1( 4) P(2, 0, 8)
=3
λ+1
⇒ λ + 4 = 3λ + 3
M3 M2
⇒ 1 = 2λ
1 λ 1
⇒ λ= ⇒ =
2 1 2 Q(4, 2, 0) R(4, 0, 6)
∴ The ratio λ : 1 = 1 : 2 M1
1:2  4 + 4 2 + 0 0 + 6
Coordinates of point M1 =  , ,  = ( 4,1, 3)
A C B  2 2 2 
(3, 4, 2) (x, 3, z) (4, 1, 6) x + x 2 y1 + y 2 z1 + z 2 
[Q coordinates of mid-point is 1 , ,
 1( 4) + 2( 3)  4 + 6 10 2 2 2 
x-coordinate of C =   =  =
 1+ 2   3  3  4 + 2 0 + 0 6 + 8
Coordinates of point M 2 =  , ,  = ( 3, 0,7 )
 1( 6) + 2(2 )  6 + 4 10  2 2 2 
and z-coordinate of C =   =  =  4 + 2 2 + 0 0 + 8
 1+2   3  3 and coordinates of point M 3 =  , ,  = ( 3, 1, 4)
 2 2 2 
 10 10
Hence, the coordinates of point C is  , 3,  . Now, length of median PM1 = Distance between
 3 3
points P and M1.
41. ABC is a triangle with vertices A (0, 0, 6), B (0, 4, 0) and
PM1 = ( 4 − 2 )2 + (1 − 0)2 + ( 3 − 8)2
C (6, 0, 0). Let points D, E and F be the mid-points of BC,
AC and AB, respectively. So, AD, BE and CF will be the [Q distance = ( x 2 − x1 )2 + ( y 2 − y1 )2 + ( z 2 − z1 )2 ]
medians of the triangle. = 4 + 1 + 25 = 30
A(0,0,6)
Similarly, QM 2 = ( 3 − 4)2 + ( 0 − 2 )2 + (7 − 0)2
= 1 + 4 + 49 = 54
F E
and RM 3 = ( 3 − 4)2 + (1 − 0)2 + ( 4 − 6)2 = 1 + 1 + 4 = 6
Hence, length of medians are 30, 54 and 6.
B(0,4,0) C(6,0,0)
D
124 CBSE Term II Mathematics XI

43. Given, two vertices of a triangle are A (3, 4, 2) and B (1, 3, 2). 45. Let vertices of the ∆ABC be A ( x1 , y1 , z1 ), B ( x 2 , y 2 , z 2 ) and
And two medians intersect at (2, 4, 3). Let third vertex of C ( x 3 , y 3 , z 3 ) and mid-points of the sides BC , CA and AB are
∆ABC be C( x 3 , y 3 , z 3 ). D( 5, 7 , 11), E( 0, 8, 5) and F(2 , 3, − 1), respectively.
We know that intersection of medians of triangle is known as A (x1, y1, z1)
centroid of a triangle.
 x + x 2 + x 3 y1 + y 2 + y 3 z1 + z 2 + z 3 
And G( x , y , z ) =  1 , , 
 3 3 3  (2, 3,–1) F E (0, 8, 5)

 3 + 1 + x3 4 + 3 + y 3 2 + 2 + z3
∴ (2, 4, 3) =  , , 
 3 3 3  (x2, y2, z2)B C (x3, y3, z3)
D
 4 + x3 7 + y 3 4 + z3
(2, 4, 3) =  , ,  (5, 7, 11)
 3 3 3 
4 + x3 Now, D is the mid-point of BC.
On comparing, 2 = x + x3
3 ∴ 5= 2 ⇒ x 2 + x 3 = 10 ...(i)
2
⇒ 6 = 4 + x3 ⇒ x3 = 2 y + y3
7 + y3 7= 2 ⇒ y 2 + y 3 = 14 ...(ii)
4= ⇒ 12 = 7 + y 3 ⇒ y 3 = 5 2
3 z + z3
4 + z3 and 11 = 2 ⇒ z 2 + z 3 = 22 ...(iii)
3= ⇒ 9 = 4 + z3 ⇒ z3 = 5 2
3 Similarly, for the sides AB and AC,
Hence, the remaining vertex is C(2 , 5, 5). x + x2
2= 1 ⇒ x1 + x 2 = 4 ...(iv)
44. Let ABCD be a tetrahedron, such that the coordinates of its 2
vertices are A ( x1 , y1 , z1 ), B ( x 2 , y 2 , z 2 ), C ( x 3 , y 3 , z 3 ) and y + y2
3= 1 ⇒ y1 + y 2 = 6 ...(v)
D ( x 4 , y 4 , z 4 ), respectively. 2
z + z2
A(x1, y1, z1) −1= 1 ⇒ z1 + z 2 = − 2 ...(vi)
2
x + x3
0= 1 ⇒ x1 + x 3 = 0 ...(vii)
D(x4, y4, z4) 2
y + y3
8= 1 ⇒ y1 + y 3 = 16 ...(viii)
G1 2
z + z3
B(x2, y2, z2) C(x3, y3, z3) 5= 1 ⇒ z1 + z 3 = 10 ...(ix)
2
Let the centroid of the faces BCD, ACD, ABD, ABC be G1 , On adding Eqs. (i) and (iv), we get
G 2 , G 3 and G 4 respectively. Then, centroid G1 of face BCD x1 + 2 x 2 + x 3 = 14
 x + x3 + x4 y 2 + y 3 + y 4 z2 + z3 + z4 ⇒ 2 x 2 + 0 = 14 [from Eq. (vii)]
= 2 , , 
 3 3 3  ⇒ x2 = 7
On putting x 2 = 7 in Eqs. (i) and Eqs. (iv), we get
Now, coordinates of point G dividing AG1 in the ratio
3 : 1 are x 3 = 3 and x1 = − 3
Thus, x1 = − 3, x 2 = 7 and x 3 = 3
  x2 + x3 + x4  y + y 3 + y 4
1 ⋅ x1 + 3 3
 1 ⋅ y1 + 3 2
  3

 On adding Eqs. (ii) and (v), we get
 , , y1 + 2 y 2 + y 3 = 20
 1+ 3 1+ 3
 ⇒ 2 y 2 + 16 = 20 [from Eq. (viii)]
⇒ 2y2 = 4 ⇒ y2 = 2
 z + z3 + z4 
1 ⋅ z1 + 3 2  On putting y 2 = 2 in Eqs. (ii) and Eqs. (v), we get
 3 
 y 3 = 12 and y1 = 4
1+ 3  Thus, y1 = 4, y 2 = 2 and y 3 = 12

On adding Eqs. (iii) and (vi), we get
 x + x 2 + x 3 + x 4 y1 + y 2 + y 3 + y 4 z1 + z 2 + z 3 + z 4  z1 + 2 z 2 + z 3 = 20
= 1 , , 
 4 4 4 
⇒ 2 z 2 + 10 = 20 [from Eq. (ix)]
Similarly, the point dividing BG 2, CG 3 and DG 4 in the ratio ⇒ 2 z 2 = 10 ⇒ z 2 = 5
3 : 1 has same coordinates. On putting z 2 = 5 in Eqs. (iii) and (vi), we get
Thus, the point z 3 = 17 and z1 = − 7
 x + x 2 + x 3 + x 4 y1 + y 2 + y 3 + y 4 z1 + z 2 + z 3 + z 4  Thus, z1 = − 7 , z 2 = 5 and z 3 = 17
G 1 , , 
 4 4 4  Hence, the vertices are A ( − 3, 4, − 7 ), B( 7 , 2 , 5) and
C( 3, 12 , 17 ).
is common to AG1, BG 2 CG 3 and DG 4.
CBSE Term II Mathematics XI 125

46. (i) Given points are A (6, 10, 14), B (2, 4, 6) and C ∴ The coordinates of the mid-points of PR and QS will
( − 2 , − 2 , − 2 ). be same.
Distance formula is Therefore, using midpoint section formula,
d = ( x 2 − x1 )2 + ( y 2 − y1 )2 + ( z 2 − z1 )2  x1 + x 2 y1 + y 2 
 , 
 2 2 
AB = (2 − 6)2 + ( 4 − 10)2 + ( 6 − 14)2
−2 + 4 5 + b
= 16 + 36 + 64 = 116 = 2 29 units x-coordinate ⇒ = ⇒2 = 5 + b ⇒ b = − 3
2 2
AC = ( − 2 − 6)2 + ( − 2 − 10)2 + ( − 2 − 14)2 a + 3 1+7
y-coordinate ⇒ = ⇒a + 3 = 8⇒a = 5
= 64 + 144 + 256 = 464 = 4 29 units 2 2
Hence, a = 5 and b = − 3
Here, AC > AB
(ii)
∴In the walk as AB and AC, the person walks more 2:1
when he goes from house A to house C. S(– 3, 7) D1 R(4, 3)
(ii) Three point will be collinear, if the sum of the two Using section formula (internal division)
small distances is equal to the largest distance.  m1x 2 + m 2x1 m1y 2 + m 2y1 
i.e. AB and BC are the small and AC is the greatest  , 
 m1 + m 2 m1 + m 2 
distance obtained by three points A , B and C then,
these points will be collinear, if AB + BC = AC  2( 4) + 1( − 3) 2( 3) + 1(7 )
D1( x , y ) =  , 
(iii) We have, AB = 2 29 units  2+1 2+1 
and AC = 4 29 [obtained in part (i)]  8 − 3 6 + 7   5 13
= ,  = , 
 3 3  3 3
Now, BC = ( − 2 − 2 )2 + ( − 2 − 4)2 + ( − 2 − 6)2
 5 13
= 16 + 36 + 64 = 116 = 2 29 units Hence, coordinates of D1 is  , 
3 3
Since, AC = AB + BC. Therefore, given points are
collinear. (iii) 3:2
Hence, no triangle is formed with the given points. P(– 2, 5) D1 R(4, 3) D2
47. (i) Given vertices are P( − 2 , a ), Q( 5, 1), R( 4, 3) and S( b , 7 ).
Using section formula (external division)
S(b, 7) R(4, 3)
 m1x 2 − m 2x1 m1y 2 − m 2y1 
 , 
 m1 − m 2 m1 − m 2 
I
 3( 4) − 2( − 2 ) 3( 3) − 2( 5)
D 2( x′ , y′ ) =  , 
P(– 2, a) Q(5, 1)  3−2 3−2 
 12 + 4 9 − 10
We know that the diagonals of a parallelogram bisect = ,  = (16, − 1)
 1 1 
each other.
Let I be the point at which diagonals intersect. Hence, coordinates of D 2 is (16, − 1).
Chapter Test
(iii) The length of ribbon CA is
l
Multiple Choice Questions (a) 3 (b) 4
1. The octants in which the points (–3, 1, 2) and (c) 5 (d) 6
(−3, 1, − 2) lie, are respectively (iv) Which of the following is distance formula?
(a) II and VI (b) III and V
(a) ( x2 − x 1)2 − ( y 1 − y 2)2 − ( z 1 − z2)2
(c) I and IV (d) II and V
(b) ( x2 − x 1)2 + ( y 2 − y 1)2 + ( z2 − z 1)2
2. Which of the following statements is incorrect?
(a) The equation of the plane z = 6 represents a plane (c) ( x 1 + x2)2 + ( y 1 + y 2)2 + ( z 1 + z2)2
perpendicular to the XY -plane, having a (d) None of the above
z-intercept of 6 units.
(b) The equation of the plane x = 0 represents the
(v) Ribbons form the sides of a
(a) Isosceles triangle
YZ-plane.
(b) Equilateral triangle
(c) The point on the X-axis with x-coordinate equal to (c) Right angled triangle
x 0 is written as (x 0 , 0, 0). (d) Isosceles right angled triangle
(d) x = x 0 represents a plane parallel to the YZ-plane.

3. What is the locus of a point for which y = 0 and l


Short Answer Type Questions
z = 0? 7. If L, M and N are the feet of perpendiculars
(a) Equation of X -axis (b) Equation of Y -axis drawn from the point P (3, 4, 5) on the XY, YZ
(c) Equation of Z -axis (d) None of these
and ZX-planes respectively, then find the
4. The length of the foot of perpendicular drawn distance of L, M and N from the point P.
from the point P(3, 4, 5) on Y-axis is 8. Name the octants in which the following
(a) 10 (b) 34
points lie.
(c) 113 (d) 5 2
(1, 2, 3), (4, − 2, 3), (4, − 2, − 5), (4, 2, − 5), (−4, 2, − 5),
5. The coordinates of a point P equidistant from (−4, 2, 5), (−3, − 1, 6), (2, − 4, − 7)
the four points O(0, 0, 0), A(l, 0, 0), B(0, m, 0) and
C(0, 0, n) are
9. Show that the points (0, 7, 10), (−1, 6, 6) and
(−4, 9, 6) are the vertices of a right angled
 l m n  l m n
(a)  , ,  (b)  , ,  isosceles triangle.
 3 3 3  4 4 4
 l m n 10. Three consecutive vertices of a parallelogram
(c)  , ,  (d) (l, m, n)
 2 2 2 ABCD are A (6, − 2, 4), B(2, 4, − 8) and C(−2, 2, 4).
Find the coordinates of the fourth vertex.
l
Case Based MCQs 11. Find the coordinates of the points which trisect
the line segment joining the points P(4, 2, − 6)
6. Three students are standing in a park with sign and Q (10, − 16, 6).
boards ‘‘SAVE ENVIRONMENT’’, ‘‘Don’t LITTER’’,
‘‘KEEP YOUR PLACE CLEAN’’. Their positions are
marked by the points A(0, 7, 10),
l
Long Answer Type Questions
B(−1, 6, 6) and C(−4, 9, 6). The three students are 12. If a parallelopiped is formed by planes drawn
holding GREEN colored ribbons together. through the points (5, 8, 10) and (3, 6, 8) parallel to
Answer the following questions which are the coordinate planes, then find the length of
based on above it. edges and diagonal of the parallelopiped by
(i) The length of ribbon AB is using distance formula.
(a) 2 3 (b) 3 2 (c) 4 3 (d) 5 4 13. Using section formula, show that the points
(ii) The length of ribbon BC is  2 
P(4, − 6, 8), Q(− 2, 4, 2) and R0, , 4 are collinear.
(a) 3 2 (b) 2 3 (c) 4 3 (d) 6 5  3 

Answers
1. (a) 2. (a) 3. (a) 4. (b) 5. (c) 6. (i) (b) (ii) (a) (iii) (d) (iv) (b) (v) (d)
7. 5 units, 3units and 4 units For Detailed Solutions
8. XOYZ , XOY ′ Z , XOY ′ Z ′ , XOYZ ′ , X ′ OYZ ′ , X ′ OYZ , X ′ OY ′ Z , XOY ′ Z ′ Scan the code
10. (2, − 4, 16) 11. (6, − 4, − 2) and (8, − 10, 2) 12. 2 units and 2 3 units
CBSE Term II Mathematics XI 127

CHAPTER 06

Derivatives

In this Chapter...
l Derivative at a Point
l First Principle of Derivative
l Algebra of Derivative of Functions
l Derivative of Trigonometric Functions

Derivative at a Point Sometimes f ′( x) is denoted by


d
[ f ( x)] or if y = f ( x), then it is
Suppose f is a real valued function and a is a point in dx
dy
its domain. denoted by and referred to as derivative of f ( x) or y with
dx
Then, derivative of f at a is defined by respect to x. It is also denoted by D[ f ( x)].
f (a + h ) − f (a)
lim Algebra of Derivative of Functions
h→0 h
provided this limit exists. Let f and g be two functions such that their derivatives are defined
in a common domain. Then,
The derivative of f ( x) at a is denoted by f ′ ( a ).
(i) Derivative of sum of two functions is sum of the derivatives
of the functions.
First Principle of Derivative d d d
[ f ( x) + g( x)] = f ( x) + g( x)
Suppose f is a real valued function, the function dx dx dx
defined by (ii) Derivative of difference of two functions is difference of the
f ( x + h ) − f ( x) derivatives of the functions.
lim ,
h→0 h d d d
[ f ( x) − g( x)] = f ( x) − g( x)
wherever the limit exists, is defined to be the derivative dx dx dx
of f at x and is denoted by f ′ ( x). (iii) Derivative of product of two functions is given by the
following product rule.
This definition of derivative is called the first principle
d d d
of derivative. [ f ( x) ⋅ g ( x)] = f ( x) g( x) + g( x) f ( x)
f ( x + h ) − f ( x) dx dx dx
Thus, f ′ ( x) = lim
h→0 h This is also known as Leibnitz product rule of derivative.
128 CBSE Term II Mathematics XI

d
(iv) Derivative of quotient of two functions is given by the [ f ( x)] = na n x n −1 + ( n − 1 )a n −1 x n − 2 + . . . + 2 a 2 x + a 1
following quotient rule. dx
d d
g( x) f ( x) −f ( x) g( x) Derivative of Trigonometric Functions
d  f ( x) dx dx
 = , g( x) ≠ 0
To find the derivative of trigonometric functions, we use the
dx  g( x) [ g( x)] 2
algebra of derivative and the following formulae
d d
Note [ c ⋅ f ( x)] = c f ( x) d
dx dx (i) (sin x) = cos x
dx
Some Important Theorems d
(ii) (cos x) = − sin x
Theorem 1 Derivative of f ( x) = x n is nx n −1 for any positive dx
integer n. d
(iii) (tan x) = sec 2 x
Note The above theorem is true for all powers of x i.e. n can dx
be any real number. d
(iv) (sec x) = sec x ⋅ tan x
Theorem 2 Derivative of Polynomial Functions dx
Let f ( x) = a n x n + a n −1 x n −1 + . . . + a 1 x + a 0 d
(v) (cot x) = − cosec 2 x
be a polynomial function, where a i ’s are all real numbers and dx
a n ≠ 0.
d
Then, the derivative function is given by (vi) ( cosec x) = − cosec x ⋅ cot x
dx

Solved Examples
Example 1. Find the derivative of (− x )−1 using first Example 2. Find the derivative using first principle of
principle. given function f ( x ) = sec x .
Sol. Let f( x ) = ( − x )−1 sec x + h − sec x
Sol. f′ ( x ) = lim
1 h→0 h
⇒ f( x ) = −
x cos x − cos x + h
f( x + h ) − f( x ) = lim
We have, f′ ( x ) = lim [by first principle] h→0 h cos x cos x + h
h→0 h
1 1  x + x + h  x − x + h
− + −2 sin   sin  
x+h x  1  2   2 
⇒ f′ ( x ) = lim Q f( x ) = = lim
h→0 h  x  h→ 0 {( x + h ) − x}cos x cos x + h
1 1  x + x + h

x x+h −2 sin  
⇒ f′ ( x ) = lim  2 
h→0 h = lim
x+h−x
h→ 0 ( x + h + x )cos x cos x + h
= lim
h → 0 x( x + h )h  x − x + h
sin  
= lim
h  2 
h → 0 x( x + h )h
× lim
h→ 0 ( x + h − x)
1 1
= = 2 sec x tan x
x( x + 0) x =
2 x
CBSE Term II Mathematics XI 129

Example 3. Find the derivative of f (x ) = 4 x − 2. Example 7. Find the derivative of


Sol. Let y = 4 x −2 x n + ax n − 1 + a 2 x n − 2 + K + a n − 1 x + a n
⇒ y = 4x1/ 2 − 2 for some fixed real number a.
Differentiating y w.r.t. x, we get Sol. Let f( x ) = x n + ax n −1 + a 2x n − 2 + K + a n −1x + a n
1 −1
dy 1 −1 2
= 4 × x2 − 0 = 2 x 2 = Differentiating f( x ) w.r.t. x, we get
dx 2 x
f′ ( x ) = nx n −1 + a( n − 1)x n −1 −1 + a 2( n − 2 )x n − 2 −1
Example 4. If y = e 3x x 3 , then find + K + a n −1(1) + 0
dy 1 [Q f ( x ) = ax n ⇒ f′ ( x ) = anx n −1 ]
at x = .
dx 3 ⇒ f′ ( x ) = nx n −1 + a( n − 1)x n − 2 + a 2 ( n − 2 )x n − 3 + K + a n −1
Sol. Given, y = e x3x 3
Example 8. Find the derivative of function
On differentiating both sides, w.r.t. x, we get
dy d 3x 3 f ( x ) = x − [ x ]; x ∈R, then find f ′ (1 / 2 ).
= (e x ) Sol. Given, f( x ) = x − [ x ]
dx dx
d d Differentiating f( x ) w.r.t. x, we get
= e3x ( x )3 + x 3 ( e3x )
dx dx f′ ( x ) = 1 − 0 ⇒ f′ ( x ) = 1
= 3x 2e3x + 3x 3e3x [Q [ ⋅ ] greatest integer function is constant function]
= 3x 2e3x(1 + x ) 1  1
At x = , f′   = 1
 dy  1  1 2  2
Now,   = 3 ⋅ ⋅ e 1 + 
 dx  x = 1 9  3 Example 9. For function f (x ) = 2 x 2 + 3x + 17, show that
3
4e f ′ ( 0) + 3f ′ ( −1 ) = 0.
=
9 Sol. Given, f( x ) = 2 x 2 + 3x + 17

Example 5. Find the derivative of ⇒ f ′ ( x ) = 4x + 3


Now, for f′( x ) value at x = 0
f ( x ) = ( ax + b ) ( cx + d) .
n m
⇒ f′ ( 0) = 4 × 0 + 3 = 3
Sol. Let y = ( ax + b )n ( cx + d)m
for x = − 1, f′( x ) is
Differentiating y w.r.t. x, we get ⇒ f′ ( −1) = 4( −1) + 3
dy d d = − 4 + 3= −1
= ( ax + b )n ( cx + d)m + ( cx + d)m ( ax + b )n
dx dx dx f ′ ( 0) + 3f ′ ( − 1) = 0
d
= ( ax + b )n m ( cx + d) m − 1 ( cx + d) 3 + 3 ( − 1) = 0
dx
3−3=0
d
+ ( cx + d)m n ( ax + b )n − 1 ( ax + b ) 0=0 Hence proved.
dx
= m ( ax + b )n ( cx + d)m − 1 ( c × 1 + 0) Example 10. If f (x ) = λx 2 + µx + 12, f ′ (4) = 15
+ n ( cx + d)m ( ax + b )n − 1 ( a × 1 + 0) f ′ (2 ) = 11, then find λ and µ.
Sol. f( x ) = λx 2 + µx + 12
= m ( ax + b )n ( cx + d)m − 1 c + n
Now, differentiating f( x ) w.r.t. x, we get
( cx + d)m ( ax + b )n −1 a
f′ ( x ) = 2λx + µ
dy
⇒ = ( ax + b )n − 1 ( cx + d)m − 1 According to the question,
dx [ mc ( ax + b ) + n a ( cx + d)] f′ ( 4) = 15
Example 6. Find the derivative of f′ (2 ) = 11
Put x = 4 and x = 2
f ( x ) = ( ax + b )n .
f′ ( 4) = 2 λ × 4 + µ = 15
Sol. Let y = ( ax + b )n f′ (2 ) = 2 λ × 2 + µ = 11
Differentiating y w.r.t. x, we get 8λ + µ = 15 …(i)
dy d 4λ + µ = 11 …(ii)
⇒ = n( ax + b )n − 1 ( ax + b )
dx dx Subtracting Eq. (ii) from Eq. (i)
= n ( ax + b )n − 1 × a 4λ = 4
dy λ =1
⇒ = na ( ax + b )n − 1
dx
130 CBSE Term II Mathematics XI

Putting the value of λ in Eq. (ii), we get dy d


= x 4 ( 5sin x − 3 cos x )
4(1) + µ = 11 dx dx
µ = 11 − 4 d 4
+ ( 5sin x − 3 cos x ) (x )
µ =7 dx
[by product formula]
∴ λ = 1 and µ = 7
= x 4( 5 cos x + 3sin x ) + ( 5sin x − 3 cos x ) 4x 3
4x 2 − 1
Example 11. Differentiate f (x ) = and find = x 3[ x( 5 cos x + 3sin x ) + 4( 5sin x − 3 cos x )]
(5 − 2 x )3
= x 3[ 5x cos x + 3x sin x + 20sin x − 12 cos x ]
value of f ′ ( x ) at x = 2.
dy
4x 2 − 1 Example 14. Find , when y = 5 sin x − 6 cos x + 7.
Sol. f( x ) = dx
( 5 − 2 x )3
Sol. Given, y = 5sin x − 6 cos x + 7
d d
( 5 − 2 x )3. ( 4x 2 − 1) − [( 5 − 2 x )3 ] ( 4x 2 − 1)
dx dx Differentiating y w.r.t. x, we get
f′ ( x ) =
[( 5 − 2 x )3 ]2 dy
= 5 cos x − 6( − sin x )+ 0 = 5 cos x + 6sin x
( 5 − 2 x )3( 8x − 0) − ( 4x 2 − 1) × 3( 5 − 2 x )2( − 2 ) dx
=
( 5 − 2 x )6 dy x
Example 15. Find of f ( x ) = .
=
( 5 − 2 x )2( 5 − 2 x ) × 8x + 6( 4x 2 − 1) ( 5 − 2 x )2 dx sin n x
( 5 − 2 x )6 Sol. Let y =
x
8x 2 + 40x − 6 sin n x
f′ ( x ) = Differentiating y w.r.t x, we get
( 5 − 2 x )4
Hence, at x = 2 d d
sin n x ( x) − x (sin n x )
dy dx dx
8 × 2 2 + 40 × 2 − 6 = [by quotient formula]
f′ (2 ) = dx (sin n x )2
( 5 − 2 × 2 )4
32 + 80 − 6 d
= = 106 sin n x × 1 − x (sin x )n
14 = dx
sin 2n x
Example 12. Find the derivative of Differentiating (sin x )n by chain rule,
y = ( 5x 3 + 3x − 1 )( x + 1 ) at x =1. d
sin n x − nx (sin x )n −1 (sin x )
Sol. Given, y = ( 5x 3 + 3x − 1) ( x + 1) = dx
Differentiating y w.r.t x, we get sin 2n x
dy d d sin x − nx sin n −1 x cos x
n
= ( 5x 3 + 3x − 1) ( x + 1) + ( x + 1) ( 5x 3 + 3x − 1) =
dx dx dx sin 2n x
= ( 5 x 3 + 3 x − 1 ) (1 + 0 ) + ( x + 1 ) ( 5 × 3 x 2 + 3 × 1 − 0 ) 2 sin 2 x + 3 cos x − 1
= ( 5x + 3x − 1) + ( x + 1) (15x + 3)
3 2 Example 16. If y = ,
sin x
= 5x 3 + 3x − 1 + 15x 3 + 3x + 15x 2 + 3 dy
then find .
= 20x + 15x + 6x + 2
3 2 dx
 dy  2 sin 2 x + 3 cos x − 1
  = 20(1)3 + 15(1)2 + 6(1) + 2 Sol. Given, y =
 dx  x =1 sin x
2 sin 2 x cos x 1
= 20 − 15 + 6 + 2 = +3 −
= 13 sin x sin x sin x
⇒ y = 2 sin x + 3 cot x − cosec x
Example 13. Find the derivative of On differentiating both sides w.r.t. x, we get
f ( x ) = x 4 ( 5 sin x − 3 cos x ). dy
= 2 cos x + 3 ( − cosec 2 x ) − ( − cosec x cot x )
Sol. Let y = x 4( 5sin x − 3 cos x ) dx
Differentiating y w.r.t. x, we get = 2 cos x − 3cosec 2 x + cosecx ⋅ cot x
CBSE Term II Mathematics XI 131

Chapter
Practice
PART 1
Objective Questions
l
Multiple Choice Questions x2 + x + 1 dy
10. If y = ,then at x = 1 is equal to
1. The derivative of function 6x 100
−x 55
+ x is x dx
3 1 4
(a) 600x 100
− 55x 55
+x (b) 600x 99
− 55x 54
+1 (a) (b) 1 (c) (d)
2 2 5
(c) 99x 99 − 54x 54 + 1 (d) 99x 99 − 54x 54
dy
2. The f ′ ( x ) of f ( x ) = x cos x is 11. If y = x −4 ( 3 − 4x −5 ), and at x = 1 is equal to 24K,
dx
(a) cos x + x sin x (b) cos x − x sin x then K is equal to
(c) x sin x − cos x (d) x cos x + sin x (a) 0 (b) 1 (c) 2 (d) 3
ax + b −5 −2
12. If f ( x ) = x ( 6 − 3x ). Then, f ′ (1 ) is equal to
3. The derivative of is
cx + d (a) − 8 (b) 10 (c) 9 (d) − 9
ad − bc ad − dc dy
(a) (b) 13. If y = ( 5x 3 + 3x − 1 )( x − 1 ), then at x = 1 is equal to
( cx + d)2 ( dx + c )2
dx
ab + bc bc − da
(c) (d) (a) 7 (b) 0 (c) 1 (d) 8
( cx + d)2 ( cx + d)2
dy 1
4. Find the derivative of 14. If y = x + , then at x = 1 is equal to
x dx
f ( x ) = 1 + x + x 2 + x 3 + . . . + x 50 at x = 1. 1 1
(a) 1 (b) (c) (d) 0
(a) 1275 (b) 2550 (c) 1276 (d) 675 2 2
dy x−4
5. If y = 2 x 3 − 4x 2 + 6x + 8, then at x = 1 is equal to 15. If f ( x ) = , then f ′ (1 ) is equal to
dx 2 x
(a) 4 (b) 12 (c) − 8 (d) 6 5 4
(a) (b) (c) 1 (d) 0
dy 4 5
6. If y = 2 x 3 + 8 x 2 + 5x, then at x = − 1 is equal to
dx 16. If a, b are fixed non-zero constants, then the
(a) 5 (b) − 5 (c) 1 (d) −1 a b
derivative of 4 − 2 is ma + nb, where
x x2 x3 xn x x
7. If y = 1 + + + +K + , then the value of −2 −4 2
1 ! 2 ! 3! n! (a) m = 4x 3 and n = (b) m = and n = 3
dy xn x3 x 5
x
−y+ is −4 −2 2
dx n! (c) m = 5 and n = 3 (d) m = 4x and n = 3
3
x x x
(a) 0 (b) n
1
(c) 2n (d) None of these 1+
3 x is
 1 dy 17. The derivative of
8. If y =  x +  , then at x = − 1 is equal to 1
 x dx 1−
x
(a) 3 (b) −1 (c) 1 (d) 0 2 −2
(a) (b)
x + x + x +1
4 3
dy 2 (1 + x ) 2 (1 − x ) 2
9. If y = , then at x = 1 is equal to −1 3
x dx (c) (d)
(a) 1 (b) 4 (c) 5 (d) 6 (1 − x ) 2 (1 − x ) 2
132 CBSE Term II Mathematics XI

dy π Based on above information, answer the following


18. If y = sin 2 x, then at x = is equal to
dx 4 questions
(a) 0 (b) 2 (c) 1 (d) 3 ex dy
(i) If y = , then at x = 1 is equal to
log e sin x dy π x dx
19. If y = e , then at x = is equal to (a) 0 (b) 1 (c) − 1 (d) 2
dx 2
(a) 1 (b) 0 (c) 2 (d) 3 2x + 3 dy
(ii) If y = 2 , then at x = 1 is equal to
π ( x − 5) dx
20. If f ( x ) = x sin x , then f ′   is equal to 9 9 3 3
2 (a) (b) − (c) (d) −
8 8 2 2
(a) 0 (b) 1 (c) − 1 (d) 1/2
log x dy
(iii) If y = , then is equal to
l
Case Based Questions x dx
1 + logx 1 − logx
21. Sam was learning ‘‘Algebra of Derivative (a) (b)
x2 x
of Functions’’ from his Tutor Rajesh. 1 + logx 1 − logx
(c) (d)
Derivative of product of two functions is given by x x2
the following product rule.
ex dy
d d d (iv) If y = , then is equal to
[ f ( x ) ⋅ g ( x )] = f ( x ) g( x ) + g( x ) f (x ) (1 + x) dx
dx dx dx
xex xex
This is also known as Leibnitz product rule of (a) (b)
(1 + x ) 2 (1 + x )
derivative.
xex
Based on above information, answer the following (c) (d) None of these
questions. (1 − x ) 2
dy sin x dy π
(i) If y = ( x + 1 )e x , then at x = 0 is equal to (v) If y = , then at x = is equal to
dx 1 + cos x dx 2
(a) 0 (b) 1 (c) 2 (d) 3 (a) − 1 (b) 0 (c) 1 (d) 2
dy π
(ii) If y = x tan x, then at x = is equal to
dx 4
(a)
π
2
+1 (b)
π
2
(c)
π
2
−1 (d) π PART 2
(iii) If y = x 3 log x, then
dy
is equal to Subjective Questions
dx
(a) x(1 + 3 log x ) (b) x 2(1 + 3 log x ) l
Short Answer Type Questions
(c) x 2(1 − 3 log x ) (d) None of these
1
dy 1. Find the derivative of f ( x ) = from first principle.
(iv) If y = e ( x + x ), then
x 3
at x = 1 is equal to x
dx
11 11 e 2. Find the derivative of a x from first principle.
(a) e (b) 11e (c) e (d)
3 2 5 3. Find the derivative of f ( x ) = ax + b, where a and b
dy are non-zero constants, by first principle.
(v) If y = ( x 2 + 2 x − 3 ) ( x 2 + 7 x + 5 ), then at x = 0 is
dx 4. Find the derivative of ( x − 1 ) ( x − 2 ) from first
equal to
(a) − 5 (b) 32 (c) 11 (d) − 11
principle.
22. Two friends Raj and Shyam of class XI standard 5. Find the derivative of e x , using first principle.
2
were discussing on the topic derivative of quotient 6. Find the derivative of e x from first principle.
of two functions. x +1
Derivative of quotient of two functions is given by 7. Find the derivative of   from the first
x −1
the following quotient rule.
d d principle.
g( x ) f ( x ) −f ( x ) g( x )
d  f ( x )  dx dx 8. Find the derivative of log a x from first principle.
= , g( x ) ≠ 0
dx  g( x )  [ g( x )] 2
9. Find the derivative of e x
from first principle.
CBSE Term II Mathematics XI 133

10. Find the derivative of sin x, using first principle. sec x − 1 p


33. The derivative of is , find p.
11. Find the derivative of f ( x ) = tan( ax + b ), by first sec x + 1 (sec x + 1 )2
principle. sin ( x + 9) dy
34. If y = , then find at x = 0.
12. Find the derivative of cos x, by first principle. cos x dx
13. Find the derivative of x tan x, by first principle. 35. Find the derivative of cot x.
sin x dy π
14. Find the derivative of f ( x ) = , by first principle. 36. If y = (sec x − 1 )(sec x + 1 ), then find at x = .
x dx 4
15. Find the derivative of tan x, by first principle. 37. Find the derivative of ( x 2 + 1 )cos x.
dy
16. Evaluate for y = cos x, by first principle.
dx
l
Long Answer Type Questions
du 38. Find the derivative of ax + b , by first principle.
17. If u = 7 t 4 − 2 t 3 − 8 t − 5, then find at t = 2 .
dt
39. Find the derivative of f ( x ) = sec x, by first principle.
x
18. Find the derivative of . 40. Differentiate the function cos( x 2 + 1 ), by the first
1 + tan x
principle.
px 2 + qx + r
19. Differentiate the functions w.r.t. x. 41. Find the derivative of x sin x from first principle.
ax + b
42. By using first principle, find the derivative of
dy π
20. If y = sin 3 x cos 3 x, then find at x = . function f ( x ) = sin 2 x.
dx 4
tan x
21. If p, q, r and s are fixed non-zero constants, then 43. Differentiate e from first principle.
r  44. Find the derivative of sin x functions, by using
the derivative of ( px + q) + s  is
x  first principle.
x dy 45. Differentiate the function
22. If y = , then prove that x = y (1 − y).
x+5 dx f ( x ) = ( ax + b )( cx + d)2 .
23. If f ( x )= 1 − x + x 2 − x 3 +…−x 99 + x 100 , then find x 2 + 3x − 9
46. Differentiate w.r.t. x.
f ′ (1 ). x 2 − 9x + 3
( x − 1 )( x − 2 ) 1 − tan x  dy −2
24. Find the derivative of . 47. If y =   , then show that = .
( x − 3)( x − 4)  1 + tan x  dx 1 + sin 2 x
25. Find the derivative of f ( x ) = sin 2 x. sin x + cos x
48. Find the derivative of .
26. Differentiate ( x + sec x ) ⋅ ( x − tan x ) w.r.t. x. sin x − cos x

27. Differentiate ( ax 2 + sin x )( p + q cos x ) w.r.t. x. 1 − cos 2 x  π π 


49. If y = , x ∈  0,  ∪  , π  ,
1 + cos 2 x  2 2 
sin ( x + a)
28. Evaluate derivative of y = . dy
cos x then find .
dx
sec x + tan x
29. Find the derivative of .
sec x − tan x l
Case Based Questions
x 5 − cos x 50. Suresh was learning the concept of ‘‘First Principle
30. Find the derivative of f ( x ) = . of Derivative’’ from his class XI standard
sin x
mathematics book.
a + b sin x
31. Find the derivative of f ( x ) = . Suppose f is a real valued function, the function
c + d cos x
f (x + h ) − f (x )
sin x + cos x dy defined by lim , wherever the limit
h→0 h
32. If y = , then find at x = 0.
sin x − cos x dx
134 CBSE Term II Mathematics XI

exists, is defined to be the derivative of f at x and is 51. Derivative of product of two functions is given by
denoted by f ′ ( x ). This definition of derivative is the following product rule.
called the first principle of derivative. d d d
[ f ( x ) ⋅ g ( x )] = f ( x ) g( x ) + g( x ) f (x )
f (x + h ) − f (x ) dx dx dx
Thus, f ′ ( x ) = lim
h→0 h This is also known as Leibnitz product rule of
Based on above information, answer the following derivative.
questions. Based on above information, answer the following
(i) Find the derivative of the function log x, by using questions.
first principle. (i) Find the derivative of f ( x) = (cos 2 x − sin 2 x)
(ii) Find the derivative of function tan x, by using first (ii) Find the derivative of function x 2 sec x
principle. (iii) Find the derivative of function x 2 tan x
(iii) By using first principle, find the derivative of (iv) Find the derivative of x 2 sin x + cos 2 x
function f ( x) = sin 2 x. (v) Find the derivative of ( 3 x + 5 ) (1 + tan x)

SOLUTIONS
Objective Questions At x = 1,
1. (b) Given, 6x100 − x 55 + x. f′ (1) = 1 + 2 (1) + 3(1)2 + K + 50(1)49
On differentiating w.r.t. x, we get = 1 + 2 + 3 + K + 50
= 6 ⋅ 100 ⋅ x 99 − 55 ⋅ x 54 + 1 ( 50)( 51)  n ( n + 1) 
= = 1275 Q Σn =
= 600x − 55x + 1
99 54 2  2 
1 5. (a) Given, y = 2 x 3 − 4x 2 + 6x + 8
2. (b) f′ ( x ) = lim [( x + h )cos( x + h ) − x cos x ]
h→0 h On differentiating both sides w.r.t. x, we get
1 dy d
= lim [ x{cos ( x + h ) − cos x} + h cos( x + h )] = (2 x 3 − 4 x 2 + 6 x + 8 )
h→0 h dx dx
d d d d
= 2 ( x3 ) − 4 ( x2 ) + 6 ( x) + ( 8)
1   2x + h h  dx dx dx dx
= lim x − 2 sin   sin  + h cos( x + h )
h→0 h    2  2   d 
= 2 ( 3x 2 ) − 4(2 x ) + 6(1) + 0 Q ( x n ) = nx n −1
 h   dx 
sin
  h 2  = 6x 2 − 8x + 6
= lim − 2 x sin  x +  + cos( x + h )
h→0  2 h
  dy
  Now, at x = 1 = 6(1)2 − 8(1) + 6
dx
h
 h
sin = 12 − 8 = 4
= − 2 x lim sin  x +  lim 2 ⋅ 1 + lim cos( x + h )
h→ 0  2  h→ 0 h 2 h → 0 6. (b) Given, y = 2 x 3 + 8x 2 + 5x
2 On differentiating both sides w.r.t. x, we get
= cos x − x sin x dy d
= (2 x 3 + 8 x 2 + 5 x )
1  a( x + h ) + b ax + b  dx dx
3. (a) f′ ( x ) = lim −
h→0h   c( x + h ) + d cx + d 
 = 6x 2 + 16x + 5
1  dy 
= lim Now,   = 6 − 16 + 5 = 11 − 16 = − 5
h→0 h  dx  x =−1
( ax + ah + b )( cx + d) − ( ax + b ) ( cx + ch + d)  7. (a) We have,
 {c( x + h ) + d)}( cx + d) 
  x x2 x3 x4 x n −1 xn
y =1+ + + + +K + +
1  adh − bch  ad − bc 1! 2 ! 3! 4! ( n − 1)! n !
= lim {c( x + h ) + d}( cx + d)  = ( cx + d)2
h→0 h   On differentiating both sides w.r.t. x, we get
4. (a) Given, f ( x ) = 1 + x + x 2 + x 3 + ... + x 50 dy 1 2 x 3x 2 nx n − 1 xn
∴ =0+ + + +K+ =y−
On differentiating both sides w.r.t. x, we get dx 1! 2 ! 3! n! n!
f′ ( x ) = 0 + 1 + 2 x + 3x 2 + ... + 50x 49 dy xn
⇒ −y+ =0
dx n!
CBSE Term II Mathematics XI 135

 1
3
1 1 1 13. (a) We have, y = ( 5x 3 + 3x − 1) ( x − 1)
8. (d) Given, y =  x +  = x 3 + 3x × 2 + 3x 2 × + 3
 x x x x Differentiating y w.r.t. x, we get
[Q ( a + b ) = a + 3a b + 3ab + b ]
3 3 2 2 3 dy d d
= ( 5x 3 + 3x − 1) ( x − 1) + ( x − 1) ( 5x 3 + 3x − 1)
3 1 dx dx dx
= x 3 + + 3x + 3 = ( 5 x 3 + 3 x − 1 ) (1 − 0 ) + ( x − 1 )
x x
On differentiating both sides w.r.t. x, we get ( 5 × 3x 2 + 3 × 1 − 0)
dy d 3 d  3 d d  1 = ( 5x 3 + 3x − 1) + ( x − 1) (15x 2 + 3)
= (x )+   + ( 3x ) +  
dx dx dx  x  dx dx  x 3 
= 5x 3 + 3x − 1 + 15x 3 + 3x − 15x 2 − 3
d d
= 3x 2 + 3 ( x − 1 ) + 3 × 1 + ( x − 3 ) = 20x 3 − 15x 2 + 6x − 4
dx dx
−3 3  dy 
= 3x 2 + 2 + 3 − 4 [Q put x = − 1] Now,   = 20 − 15 + 6 − 4 = 7
x x  dx  x =1
 dy  1
Now,   =3−3+ 3−3 =0 14. (d) Given, y= x+
 dx  x = −1 x
x4 + x3 + x2 + 1 On differentiating both sides w.r.t. x, we get
9. (c) Given, y = dy 1 1
x = −
On differentiating both sides w.r.t. x, we get dx 2 x 2 x 3/ 2
d  x 4 + x 3 + x 2 + 1 d  3 1  dy  1 1
dy
=  = ∴   = − =0
 x + x + x + 
2
  dx  x =1 2 2
dx dx  x  dx x
x−4
= 3x 2 + 2 x + 1 −
1 15. (a) Given, f( x ) =
x2 2 x
 dy  On differentiating both sides w.r.t. x, we get
Now,   = 3+ 2 + 1−1= 6−1= 5
 dx  x =1 2 x − ( x − 4) ⋅ 2 ⋅
1
f′ ( x ) = 2 x
x2 + x + 1
10. (a) Given, y = 4x
x 2 x − ( x − 4) 2 x − x + 4
= =
On differentiating both sides w.r.t. x, we get 4x 3/ 2 4x 3/ 2
dy d  x 2 + x + 1 d 3/ 2 x+4
=   = ( x + x1/ 2 + x −1/ 2 ) = 3/ 2
dx dx  x  dx 4x
1+ 4 5
3 1/ 2 1 −1/ 2 1 −3/ 2 ∴ f′ (1) = =
= x + x − x 4 × (1 ) 3/ 2
4
2 2 2
a b
 dy  3 1 1 3 16. (b) Let y = 4 − 2
Now,   = + − = x x
 dx  x =1 2 2 2 2
⇒ y = a x −4 − bx −2
11. (b) Given, y = x −4( 3 − 4x −5 )
Differentiating y w.r.t. x, we get
On differentiating both sides w.r.t. x, we get dy d −4 d −2
dy d −4 =a (x ) − b (x )
= [ x ( 3 − 4x −5 )] dx dx dx
dx dx 4a 2 b  d n 
d =− 5 + 3 Q ( x ) = nx n −1
= ( 3x − 4 − 4x − 9 ) x x 
 dx 
dx
1
= − 12 x −5 + 36x −10 1+
17. (b) Let y = x ⇒ y=x+1
 dy  1 x −1
Now,   = − 12 + 36 = 24 1−
 dx  x =1 x
According to the question, On differentiating y w.r.t. x, we get
24K = 24 ⇒ K = 1 d d
( x − 1) ( x + 1) − ( x + 1) ( x − 1)
dy dx dx
12. (d) Given, f( x ) = x −5 ( 6 − 3x −2 ) =
dx ( x − 1) 2

or f ( x ) = 6x − 5 − 3x − 7 ( x − 1 ) (1 + 0 ) − ( x + 1 ) (1 − 0 ) x − 1 − x − 1
= =
∴ f′ ( x ) =
d
( 6x −5 − 3x −7 ) = − 30x −6 + 21x −8 ( x − 1) 2 ( x − 1) 2
dx dy −2 −2
⇒ = =
Now, f′ (1) = − 30 + 21 = − 9 dx ( x − 1)2 (1 − x )2
136 CBSE Term II Mathematics XI

18. (c) Given, y = sin 2 x (iv) (c) Given, y = ex( x 3 + x )


On differentiating both sides w.r.t. x, we get dy d x 3
∴ = [ e ( x + x )]
dy d dx dx
= (sin x sin x )
dx dx d d
= ex ( x 3 + x ) + ( x 3 + x ) ( ex )
d d dx dx
= (sin x ) (sin x ) + (sin x ) (sin x )
dx dx  1 
= e x  3x 2 +  + (x + x ) e
3 x
 2 x
[using product rule of derivative]
= (cos x )sin x + sin x (cos x )  dy   1 7 11e
Now,   = e 3 +  + (1 + 1 )e = e + 2 e =
= 2 sin x cos x = sin 2 x  dx  x =1  2  2 2
 dy  π (v) (d) Given, y = ( x 2 + 2 x − 3) ( x 2 + 7 x + 5)
Now,   = sin = 1
 dx  x = π 2 dy d
4 ∴ = ( x 2 + 2 x − 3) ( x 2 + 7 x + 5)
dx dx
19. (b) Given, y = elog e sin x d 2
+ ( x + 7 x + 5)
2
( x + 2 x − 3)
⇒ y = sin x [Q elog e f ( x) = f( x )] dx
On differentiating both sides w.r.t. x, we get = ( x 2 + 2 x − 3 ) (2 x + 7 ) + ( x 2 + 7 x + 5 ) (2 x + 2 )
dy
=
d
(sin x ) = cos x  dy 
Now,   = ( − 3) (7 ) + ( 5)(2 )
dx dx  dx  x = 0
 dy  π = − 21 + 10 = − 11
Now,   = cos = 0
 dx  x = π 2 ex
2 22. (i) (a) Given, y =
x
20. (b) Given, f( x ) = x sin x
d d
x ( ex ) − ex ( x )
On differentiating both sides w.r.t. x, we get dy d  ex  dx dx
∴ =   =
d dx dx  x  x2
f′ ( x ) = ( x sin x )
dx
xex − ex ex( x − 1)
d d = =
f′ ( x ) = x (sin x ) + sin x ( x) x2 x2
dx dx
 dy  e(1 − 1)
f′ ( x ) = x cos x + sin x Now,   = =0
 dx  x =1 1
 π π π π
∴ f′   = cos + sin = 1 2x + 3
2 2 2 2 (ii) (b) Given, y =
( x 2 − 5)
21. (i) (c) Given, y = ( x + 1)ex
dy d  2 x + 3
dy d d ∴ =  
∴ = ( x + 1 )e x = ( x + 1 ) ( e x ) dx dx  x 2 − 5 
dx dx dx
d d d
+ e x ( x + 1) = e x ( x + 1) + e x ( x 2 − 5) (2 x + 3 ) − (2 x + 3 ) ( x 2 − 5 )
dx = dx dx
 dy  ( x 2 − 5) 2
Now,   = e ( 0 + 1) + e 0 = 1 + 1 = 2
0
 dx  x = 0 ( x 2 − 5)(2 ) − (2 x + 3)(2 x )
=
(ii) (a) Given, y = x tan x ( x 2 − 5) 2
dy d d d 2 x 2 − 10 − 4x 2 − 6x − 2 x 2 − 6x − 10
∴ = ( x tan x ) = x (tan x ) + tan x ( x) = =
dx dx dx dx ( x 2 − 5) 2 ( x 2 − 5) 2
= x sec 2 x + tan x  dy  − 2 − 6 − 10 18 −9
Now,   = =− =
 dy  π π  dx  x =1 16 16 8
Now,   = (2 ) + 1 = + 1
 dx  x = π 4 2 log x
4 (iii) (d) Given, y =
x
(iii) (b) Given, y = x 3 log x dy d  log x 
∴ =  
dy d 3 d dx dx  x 
∴ = ( x log x ) = x 3 (log x )
dx dx dx d d
x (log x ) − log x ( x )
d x3 = dx dx
+ log x ( x 3 ) = + log x( 3x 2 )
dx x x2
= x 2 + 3x 2 log x x 
 − log x
x  (1 − log x )
= x 2(1 + 3 log x ) = 2
=
x x2
CBSE Term II Mathematics XI 137

ex a xa h − a x
(iv) (a) Given, y = = lim
(1 + x ) h→ 0 h
dy d  ex   a h
− 1
∴ =   ⇒ f′ ( x ) = a x lim  
dx dx  1 + x  h→ 0  h 

(1 + x )
d x d
( e ) − e x (1 + x )  ax − 1 
dx dx = a x loge a Q lim = loge a 
=  x→ 0 x 
(1 + x ) 2
3. We have, f( x ) = ax + b
(1 + x )ex − ex ex + xex − ex
= = By definition of first principle, we have
(1 + x ) 2 (1 + x ) 2
f( x + h ) − f( x )
xex f′ ( x ) = lim
= h→ 0 h
(1 + x ) 2 a( x + h ) + b − ( ax + b ) ah
sin x = lim = lim =a
(v) (c) Given, y = h→ 0 h h→ 0 h
1 + cos x 4. Let f( x ) = ( x − 1)( x − 2 ) = x 2 − 3x + 2
dy d  sin x  By using first principle of derivative, we have
∴ =  
dx dx  1 + cos x  f( x + h ) − f( x )
f′ ( x ) = lim
d d h→ 0 h
(1 + cos x ) (sin x ) − sin x (1 + cos x )
= dx dx [( x + h )2 − 3( x + h ) + 2 ] − [ x 2 − 3x + 2 ]
(1 + cos x )2 = lim
h→ 0 h
(1 + cos x )(cos x ) − sin x( − sin x )
= [ x 2 + h 2 + 2 xh − 3x − 3h + 2 ] − [ x 2 − 3x + 2 ]
(1 + cos x )2 = lim
h→ 0 h
cos x + cos 2 x + sin 2 x 2 hx + h 2 − 3h h (2 x + h − 3 )
= = lim = lim = 2x − 3
(1 + cos x )2 h→ 0 h h→ 0 h
(1 + cos x ) 1 5. Let f( x ) = ex
= =
(1 + cos x )2 1 + cos x By using first principle of derivative, we have
 dy  1 f( x + h ) − f( x ) ex + h − ex
Now,   = =1 f′ ( x ) = lim = lim
 dx  x = π 1 + 0
h→ 0 h h→ 0 h
2

Subjective Questions e x ( e h − 1) ( e h − 1)
= lim = ex lim
h→ 0 h h→ 0 h
1
1. We have, f( x ) =  h h2 h3  
x 1 + + + + K ∞ − 1
 1! 2 ! 3! 
By using first principle, = ex lim  
f( x + h ) − f( x ) h→ 0 h
f′( x ) = lim  
h→ 0 h h h2
h 1 + + +K ∞
2 ! 3 !
  = ex lim   = ex × 1 = ex
1 1 1
− Q f( x ) =
x+h x  x  h→ 0 h
∴ f′ ( x ) = lim  1  2
h→ 0 h ∴ f( x + h ) =  6. Let f( x ) = ex
 x + h
By using first principle of derivative, we have
1  x − (x + h ) 2 2
= lim f( x + h ) − f( x ) e( x + h ) − ex
h→0 h  x ( x + h )  f′ ( x ) = lim
h→ 0
= lim
h→ 0
h h
1  −h  ex
2
+ h 2 + 2hx
− ex
2
= lim  x( x + h )  = lim
h→0 h   h→ 0 h
 h ( h + 2x)
 −1  −1 e 2 − 1  ( h + 2 x)
= lim  = 2 = lim ex   × ( h + 2 x)
h → 0  x (x + h )
 x h→ 0 h
 
2. Let f( x ) = a x. 2 e h ( h + 2x )
− 1
= ex lim   × hlim ( h + 2 x)
h→ 0 h ( h + 2 x ) →0
By using first principle of derivative, we have  
f( x + h ) − f( x ) 2
∴ f′ ( x ) = lim = ex × 1 × ( 0 + 2 x )
h→ 0 h
 ey − 1 
ax + h − ax
2
⇒ f′ ( x ) = lim = 2 x ex Q lim = 1
y→ 0 y
h→ 0 h  
138 CBSE Term II Mathematics XI

x+1 10. Let f ( x ) = sin x


7. Let f( x ) =
x −1 By using first principle of derivative, we have
By using first principle of derivative, we have f ( x + h ) − f( x ) sin( x + h ) − sin x
( x + h ) + 1 x + 1  f′( x ) = lim = lim
h→ 0 h h→0 h
f( x + h ) − f( x ) (x + h ) − 1 − x − 1 
f′ ( x ) = lim = lim    x + h + x  x + h − x
2 cos   ⋅ sin  
h→ 0 h h→ 0 h  2   2 
( x + h + 1) ( x − 1) − ( x + 1)( x + h − 1) = lim
= lim h→ 0 h
h→ 0 h( x + h − 1)( x − 1)   C + D  C − D 
( x 2 − x + xh − h + x − 1) − ( x 2 + xh − x + x + h − 1) Q sin C − sin D = 2 cos  2  sin  2  
= lim  
h→ 0 h( x + h − 1)( x − 1)  h h
− 2h 2 cos  x +  ⋅ sin
= lim  2 2
h → 0 h ( x + h − 1)( x − 1)
= lim
h→ 0 h
−2 −2 h
= lim = sin
h → 0 ( x + h − 1)( x − 1) ( x − 1) 2  h 2 Q h → 0 ⇒ h → 0
= lim cos  x +  ⋅ lim
h→ 0  2  h h  2 
8. Let f( x ) = loga x 2
→0
2
By using first principle of derivative, we have sin θ
 h  
f( x + h ) − f( x ) = lim cos  x +  × 1 Q lim =1
f′ ( x ) = lim h→ 0  2  θ → 0 θ 
h→ 0 h
 x + h = cos ( x + 0) = cos x [putting h = 0]
loga   d
loga( x + h ) − loga x  x  ∴ (sin x ) = cos x
⇒ f′ ( x ) = lim = lim dx
h→0 h h→0 h
 h 11. We have, f( x ) = tan( ax + b )
loga 1 + 
 x By first principle of derivative, we have
⇒ f′ ( x ) = lim
h→0 h f( x + h ) − f( x )
f′ ( x ) = lim
 h h→0 h
loge 1 + 
 x  loge λ  tan[ a( x + h ) + b ] − tan( ax + b )
= lim Q loga λ = log a  = lim
h → 0 (loge a ) ⋅ h  e  h→ 0 h
 h sin( ax + ah + b ) sin( ax + b )
loge 1 +  −
1  x cos( ax + ah + b ) cos( ax + b )
⇒ f′ ( x ) = lim = lim
loge a h → 0  h h→ 0 h
x  sin( ax + ah + b )cos( ax + b ) − sin( ax + b )
 x
 cos( ax + ah + b ) 
  h  = lim  
 log 1 +  
1  x h→ 0 h cos( ax + b )cos( ax + ah + b )
= Q lim = 1
x loge a  h → 0 h  a sin( ah )
= lim
 x  h → 0 a ⋅ h cos( ax + b )cos( ax + ah + b )

9. Let f( x ) = e x [Q sin A cos B − cos A sin B = sin( A − B )]


a sin ah
By using first principle of derivative, we have = lim lim
h → 0 cos( ax + b )cos( ax + ah + b ) ah → 0 ah
f( x + h ) − f( x )
f′ ( x ) = lim [Q h → 0, ⇒ ah → 0]
h→ 0 h
a  sin θ 
e x+ h − e x = ×1 Q lim
 θ → 0 θ
=1

= lim cos 2( ax + b )
h→0 h
x+ h− x = a sec ( ax + b )
2
e x(e − 1)
= lim 12. Let f( x ) = cos x
h→ 0 (x + h ) − x
x+ h− x
By using first principle of derivative, we have
e x(e − 1) f( x + h ) − f( x )
= lim f′ ( x ) = lim
h → 0 ( x + h − x )( x + h + x) h→ 0 h
−1 e x+ h − x
1 cos( x + h ) − cos x
=e x
lim × lim = lim
h→ 0 h
x + h − x)
h→ 0 ( h → 0 ( x + h + x)
1 cos( x + h ) − cos x ( cos( x + h ) + cos x )
= e ×1×
x = lim ×
2 x h→ 0 h ( cos( x + h ) + cos x )
e x  ey − 1  cos( x + h ) − cos x
= = 1 = lim
Q lim
y→ 0 y h → 0 h ( cos( x + h ) + cos x )
2 x  
CBSE Term II Mathematics XI 139

2x + h  h 15. Let f( x ) = tan x


−2 sin   sin  
 2  2
= lim By using first principle of derivative, we have
h → 0 h ( cos( x + h ) + cos x )
sin x + h sin x
  C + D  C −D  −
cos x + h cos x
Q cos C − cos D = −2 sin  2  sin  2   f′ ( x ) = lim
  h→ 0 h
2x + h h sin x + h ⋅ cos x − sin x ⋅ cos x + h
sin   sin
 2  2 = lim
= − lim × lim h→ 0 h ⋅ cos x + h cos x
h → 0 ( cos( x + h ) + cos x ) h h
→0
2
2 sin( x + h − x ) x+h − x
= lim ×
 h  h→ 0 h cos x ⋅ cos x + h x+h − x
Qh→ 0⇒ → 0
 2  1 sin( x + h − x )
= lim × lim
 2 x h → 0 cos x ⋅ cos x + h h→0 x+h − x
sin  
2 x+h − x
=− ×1 × lim
cos( x ) + cos( x ) h→0 h
=−
sin x sec 2 x
=
2 cos x 2 x
13. Let f( x ) = x tan x 16. We have, y = cos x
By using first principle of derivative, we have By using first principle of derivative, we have
[( x + h )tan( x + h ) − x tan x ] cos x + h − cos x
f′ ( x ) = lim f′ ( x ) = lim
h→ 0 h h→ 0 h
x[tan( x + h ) − tan x ] + h tan( x + h )  x + h + x  x + h − x
= lim
h→ 0 h −2 sin   sin  
 2   2 
x[sin( x + h )cos x − cos( x + h )sin x ] = lim
= lim h→ 0 (x + h ) − x
h→ 0 h cos( x + h )cos x
+ lim tan( x + h )  x+h + x  x + h − x
h→ 0 −2 sin   sin  
1 sin h  2   2 
= x lim × lim + tan x = lim × lim
h → 0 cos( x + h )cos x h→ 0 h
h→0 ( x + h + x) h→0  x + h − x
= x sec x + tan x
2 2 
 2 
14. We have,
1 sin x
sin x =−
f( x ) = 2 x
x
17. We have, u = 7 t 4 − 2 t 3 − 8t − 5
By using first principle of derivative, we have
sin( x + h ) sin x On differentiating both sides w.r.t. t, we get
− du d
f′ ( x ) = lim x+h x = [7 t 4 − 2 t 3 − 8 t − 5 ]
h→0 h dt dt
 d n 
x[sin( x + h ) − sin x ] − h sin x = 7( 4 t 3 ) − 2( 3 t 2 ) − 8(1) − 0 Q ( x ) = nx n −1
= lim  dx 
h→0 h ⋅ x( x + h )
= 28 t 3 − 6t 2 − 8
  x + h + x  x + h − x   du 
x 2 ⋅ cos  ⋅ sin    − h sin x Now,   = 28 (2 )3 − 6 (2 )2 − 8
   
= lim   dt  t = 2
2 2
h→0 h ⋅ x( x + h )
= 224 − 24 − 8 = 192
 h  h  x
x 2 ⋅ sin ⋅ cos x +   − h sin x 18. Let y =
 2
= lim  1 + tan x
2
h→0 h ⋅ x( x + h ) Differentiating y w.r.t. x, we get
 h  h
sin   cos x +  d
(1 + tan x ) ( x ) − x
d
(1 + tan x )
2  2 sin x dy
= lim ⋅ lim − lim = dx dx
h
→0  h  h →0 ( x + h ) h → 0 x( x + h )
dx (1 + tan x )2
 
2
2
(1 + tan x ) (1) − x ( 0 + sec 2 x )
 h  =
Qh→ 0⇒ → 0 (1 + tan x )2
 2 
cos x sin x 1 + tan x − x sec 2 x
= − 2 =
x x (1 + tan x )2
140 CBSE Term II Mathematics XI

px 2 + qx + r 22. We have, y =
x
19. Let y = …(i)
( ax + b ) x+5
On differentiating both sides w.r.t. x, we get On differentiating both sides of Eq. (i) w.r.t. x, we get
 d  d d
( ax + b ) ( px 2 + qx + r ) ( x + 5) x − x ( x + 5)
  dy d  x  dx dx
dx =  =
  dx dx  x + 5 ( x + 5) 2
 − ( px 2 + qx + r ) d ( ax + b )
dy  dx   du dv 
= [by quotient rule] v −u
dx ( ax + b )2  d  u dx dx 
Q dx  v  = v2 
( ax + b ) (2 px + q + 0) − ( px 2 + qx + r ) ( a × 1 + 0)  
=  
( ax + b )2 ( x + 5 ) (1 ) − x (1 + 0 ) x + 5 − x
( ax + b ) (2 px + q) − ( px 2 + qx + r ) a = =
= ( x + 5) 2 ( x + 5) 2
( ax + b )2 dy 5
⇒ =
(2 apx 2 + aqx + 2 bpx + bq) − ( apx 2 + aqx + ra ) dx ( x + 5)2
=
( ax + b )2 dy 5x
Now, LHS = x = ...(ii)
2 apx 2 + aqx + 2 bpx + bq − apx 2 − aqx − ra dx ( x + 5)2
=
( ax + b )2 x  x 
and RHS = y (1 − y ) = 1 − 
dy apx 2 + 2 bpx + bq − ra x+5  x + 5
=
dx ( ax + b )2 x  x + 5 − x 5x
=  = ...(iii)
20. Given, y = sin x cos x
3 3 x + 5  x + 5  ( x + 5) 2
On differentiating both sides w.r.t. x, we get From Eqs. (ii) and (iii), we get
dy
dy d d
= sin 3 x cos 3 x + cos 3 x sin 3 x [by product rule] x = y (1 − y ) Hence proved.
dx dx dx dx
23. We have, f( x ) = 1 − x + x 2 − x 3 + ...... − x 99 + x100
= sin 3 x ⋅ 3 cos 2 x ( − sin x ) + cos3 x ⋅ 3sin 2 x cos x
Differentiate f( x ) w.r.t. x, we get
[by chain rule]
f′ ( x ) = 0 − 1 + 2 x − 3x 2 +…− 99 x 98 + 100 x 99
= − 3 cos 2 x sin 4 x + 3sin 2 x cos4 x
= − 1 + 2 x − 3x 2 +…− 99x 98 + 100 x 99
= 3sin 2 x cos 2 x (cos 2 x − sin 2 x )
∴ f′ (1) = −1 + 2 − 3 +…− 99 + 100
= 3sin 2 x cos 2 x cos 2 x = ( − 1 − 3 − 5 −…− 99) + (2 + 4 +…+ 100)
3 3 50 50
= (2 sin x cos x )2 cos 2 x = sin 2 2 x cos 2 x = − [2 × 1 + ( 50 − 1)2 ] + [2 × 2 + ( 50 − 1)2 ] = 50
4 4 2 2
 dy  3 π π ( x − 1)( x − 2 ) x 2 − 3x + 2
Now,   = sin 2 cos = 0 24. Let y = =
 dx  x = π 4 2 2 ( x − 3)( x − 4) x 2 − 7 x + 12
4
r  On differentiating both sides w.r.t. x, we get
21. Let y = ( px + q)  + s
x   2 d 
( x − 7 x + 12 ) ( x 2 − 3x + 2 ) − ( x 2 − 3x + 2 )
Differentiating y w.r.t. x, we get  dx 
 
d r  r  d  d
dy
= ( px + q)  + s +  + s ( px + q) ( x 2 − 7 x + 12 )
dy  
dx dx  x  x  dx = dx
dx ( x 2 − 7 x + 12 )2
 r  r 
= ( px + q) − 2 + 0 +  + s ( p × 1 + 0)
 x   x 
=
− 4x 2 + 20x − 22
 d  1 d −1 1 ( x − 3)2( x − 4)2
−1 − 1
Q dx  x  = dx ( x ) = ( −1) x = − 2
25. We have, f( x ) = sin 2 x
 x 
 r  r  f( x ) = 2 sin x cos x
= ( px + q)  − 2  +  + s p
 x  x  On differentiating both sides w.r.t. x, we get
pxr qr pr  d d 
=− − 2 + + ps f′ ( x ) = 2 sin x (cos x ) + cos x (sin x )
 dx dx 
x2 x x
pr qr pr [using product rule of derivative]
=− − 2 + + ps = 2[sin x ⋅( − sin x ) + cos x ⋅ cos x ]
x x x
dy qr = 2 ( − sin 2 x + cos 2 x )
⇒ = ps − 2
dx x = 2 (cos 2 x − sin 2 x ) = 2 cos 2 x
CBSE Term II Mathematics XI 141

26. We have, y = ( x + sec x )( x − tan x ) sin x


d 5 d
( x − cos x ) − ( x 5 − cos x ) sin x
On differentiating both sides w.r.t. x, we get = dx dx
dy d (sin x )2
= [( x + sec x )( x − tan x )]
dx dx sin x( 5x 4 + sin x ) − ( x 5 − cos x )cos x
d d =
= ( x + sec x ) ⋅ ( x − tan x ) + ( x − tan x ) ( x + sec x ) sin 2 x
dx dx
5x 4 sin x − x 5 cos x + 1
[using product rule of derivative] =
sin 2 x
= ( x + sec x )(1 − sec 2 x )
a + b sin x
+ ( x − tan x )(1 + sec x ⋅ tan x ) 31. Let y = . Then,
27. Let y = ( ax 2 + sin x )( p + q cos x ) c + d cos x
 d 
On differentiating both sides w.r.t. x, we get ( c + d cos x ) ( a + b sin x ) − ( a + b sin x )
dy d d  dx 
= ( ax 2 + sin x ) ( p + q cos x ) + ( p + q cos x ) ( ax 2 + sin x )  
 d
dx dx dx ( c + d cos x ) 
dy  dx 
[using product rule of derivative] =
= ( ax 2 + sin x )( 0 − q sin x ) + ( p + q cos x )(2 ax + cos x ) dx ( c + d cos x )2
( c + d cos x )( b cos x ) − ( a + b sin x )( − dsin x )
= − q sin x ( ax 2 + sin x ) + ( p + q cos x ) (2ax + cos x ) =
( c + d cos x )2
sin ( x + a ) sin x cos a + cos x sin a
28. Let y = = bc cos x + adsin x + bd
cos x cos x =
[Qsin( A + B ) = sin A cos B + cos A sin B] ( c + d cos x )2

=
sin x cos a cos x sin a
+ = cos a tan x + sin a sin x + cos x
32. Given, y =
cos x cos x sin x − cos x
On differentiating both sides w.r.t. x, we get Differentiating w.r.t. x, we get
dy d d
= cos a (tan x ) + (sin a ) (sin x − cos x )(cos x − sin x ) − (sin x + cos x ) 
dy  (cos x + sin x ) 
dx dx dx
 d  =
= cos a sec 2 x + 0 Q ( constant) = 0 dx (sin x − cos x )2
 dx 
− (sin x − cos x )2 − (sin x + cos x )2
cos a  1  =
= Q sec x = (sin x − cos x )2
cos 2 x  cos x 
sec x + tan x  dy  − ( 0 − 1) 2 − ( 0 + 1) 2
29. Let y = Now,   =
sec x − tan x  dx  x = 0 ( 0 − 1) 2
1 sin x
+ −1−1
cos x cos x 1 + sin x =
= ⇒y = 1
1

sin x 1 − sin x
cos x cos x = −2
sec x − 1
On differentiating both sides w.r.t. x, we get 33. (a) Let y =
dy d  sec x + tan x  d  1 + sin x  sec x + 1
=   =  
dx dx  sec x − tan x  dx  1 − sin x  On differentiating both sides w.r.t. x, we get
d d
d d (sec x + 1) (sec x − 1) − (sec x − 1) (sec x + 1)
(1 − sin x ) (1 + sin x ) − (1 + sin x ) (1 − sin x ) dy dx dx
dx dx =
= dx (sec x + 1)2
(1 − sin x )2
2 sec x tan x
[using quotient rule of derivative] =
(1 − sin x ) ( 0 + cos x ) − (1 + sin x ) ( 0 − cos x ) (sec x + 1)2
= sin( x + 9)
(1 − sin x )2 34. (a) Given, y=
2 cos x cos x
= sin x cos 9 + cos x sin 9
(1 − sin x )2 =
cos x
30. We have, = tan x cos 9 + sin 9
x 5 − cos x On differentiating both sides w.r.t. x, we get
f( x ) =
sin x dy
= sec 2 x cos 9
On differentiating both sides w.r.t. x, we get dx
d  x 5 − cos x   dy 
f′ ( x ) =   Now,   = cos 9
dx  sin x   dx  x = 0
142 CBSE Term II Mathematics XI

35. (c) Let y = cot x   x + x + h (x − x − h ) 


 −2 sin  2
 ⋅ sin
 2 
On differentiating both sides w.r.t. x, we get = lim  
dy d d − cosec 2 x h→ 0
 h ⋅ cos x ⋅ cos ( x + h ) 
= (cot x ) = − cosec 2 x ( x) =  
dx dx dx 2 x
36. (c) Given, y = (sec x − 1)(sec x + 1)   C + D   C − D 
Q cos C − cos D = −2 sin  2  sin  2  
y = (sec 2 x − 1) [Q( a + b ) ( a − b ) = a 2 − b 2]  
= tan 2 x   h  h 
 − 2 sin  x + 2  ⋅  − sin 2  
On differentiating both sides w.r.t. x, we get = lim  
dy d h→ 0
 h ⋅ cos x cos ( x + h ) 
= 2 tan x ⋅ tan x = 2 tan x ⋅ sec 2 x [by chain rule]
dx dx  
 dy   π  π  h h
Now,   = 2 tan   sec 2   = 2(1)2 = 4 sin  x +  sin
 dx  x = π  4  4  2 2
4 = lim ⋅ lim
h → 0 cos ( x + h ) ⋅ cos x h → 0 h
37. (d) Let y = ( x 2 + 1)cos x, 2
Differentiating y w.r.t x, we get sin x  sin θ 
= ×1 Q lim =1
 θ → 0 θ 
dy d d
= ( x 2 + 1) (cos x ) + cos x ( x 2 + 1) cos 2 x
dx dx dx
sin x 1
[by product rule] = ⋅ = tan x sec x
cos x cos x
= ( x 2 + 1) ( − sin x ) + cos x(2 x )
40. Let f( x ) = cos( x 2 + 1)
= − x 2 sin x − sin x + 2 x cos x
We know by first principle,
38. Let f( x ) = ax + b. Then,
f( x + h ) − f( x )
f′ ( x ) = lim
f( x + h ) = a ( x + h ) + b = ( ax + b ) + ah h→0 h
d f( x + h ) − f ( x ) cos[( x + h )2 + 1] − cos( x 2 + 1)
∴ [ f( x )] = lim ∴ f′( x ) = lim
dx h→0 h h→0 h
d ( ax + b ) + ah − ax + b [Qf( x ) = cos ( x 2 + 1)]
⇒ [ f( x )] = lim
dx h→0 h ( x + h ) 2 + 1 + x 2 + 1 ( x + h ) 2 + 1 − ( x 2 + 1)
( ax + b ) + ah − ax + b −2 sin sin
d 2 2
⇒ [ f( x )] = lim = lim
dx h→0 h h→ 0 h
( ax + b ) + ah + ax + b   C + D  C − D 
× Qcos C − cos D = − 2 sin  2  sin  2  
( ax + b ) + ah + ax + b  
d ( ax + b ) + ah − ( ax + b ) ( x + h )2 + x 2 + 2 ( x + h )2 − x 2
⇒ [ f( x )] = lim − 2 sin sin
dx h → 0 h { ( ax + b ) + ah + ax + b } = lim 2 2
h→ 0 h
d ah
⇒ [ f( x )] = lim
dx h → 0 h { ax + b + ah + ax + b }
 ( x + h )2 + x 2 + 2  ( x + h )2 − x 2  ( x + h )2 − x 2 

d
[ f( x )] = lim
a −2 sin sin  
dx h → 0 ( ax + b ) + ah + ax + b  2  2  2 
= lim
d a h→ 0 ( x + h )2 − x 2
⇒ [ f( x )] = h×
dx ax + b + ax + b 2
d a ( x + h )2 + x 2 + 2 x 2 + h 2 + 2 xh − x 2
Hence, ( ax + b ) = − 2 sin ×
dx 2 ax + b = lim 2 2
h→0 h
39. Given, f ( x ) = sec x   ( x + h )2 − x 2  
By using first principle of derivative, we have  sin   
Q lim  2  
f( x + h ) − f ( x ) = 1
f′ ( x ) = lim  h → 0 ( x + h )2 − x 2 
h→ 0 h  
sec ( x + h ) − sec x  2 
∴ f′ ( x ) = lim
h→ 0 h  ( x + h )2 + x 2 + 2  h
1 1 −2 sin   (2 x + h )
−  2  2
cos ( x + h ) cos x = lim
= lim h→0 h
h→ 0 h
cos x − cos ( x + h )  2 x2 + 2  1
= lim = − 2 sin   × 2 x × = − 2 x sin ( x 2 + 1)
h → 0 h × cos x ⋅ cos ( x + h )  2  2
CBSE Term II Mathematics XI 143

41. We have, f( x ) = x sin x 2x + 0 2x + 0


= cos   × 1 × 2 sin   cos 0
By using first principle of derivative, we have  2   2 
f( x + h ) − f( x ) = cos x × 2 sin x × 1 = sin 2x
f′ ( x ) = lim
h→0 h 43. Let f( x ) = e tan x

( x + h )sin( x + h ) − x sin x
∴ f′ ( x ) = lim Then, f( x + h ) = e tan( x+ h )
h→0 h
d f( x + h ) − f( x )
( x + h )[sin x cos h + cos x sin h ] − x sin x Q ( f( x )) = lim
= lim dx h →0 h
h→ 0 h
d e tan( x+ h ) − e tan x
[Q sin( A + B ) = sin A cos B + cos A sin B ] ∴ ( f( x )) = lim
dx h→ 0 h
x sin x cos h + x cos x sin h + h sin x cos h 
  tan( x + h ) − tan x
− 1 
+ h cos x sin h − x sin x  d e
= lim  ⇒ ( f( x )) = lim e tan x  
h→ 0 h dx h→ 0  h 
x sin x (cos h − 1) + x cos x sin h   e tan( x+ h ) − tan x − 1 
 + +   
= lim  
h (sin x cos h cos x sin h ) d  tan( x + h ) − tan x 
h→ 0 h ⇒ ( f( x )) = e tan x lim  
dx h→ 0
 tan( x + h ) − tan x 
x sin x (cos h − 1) sin h × hlim 
= lim + lim x cos x  →0 h 
h→ 0 h h → 0 h
d tan( x + h ) − tan x
h (sin x ⋅ cos h + cos x ⋅ sin h ) ⇒ ( f( x )) = e tan x
× 1 × lim
+ lim dx h→ 0 h
h→ 0 h
1
 − (1 − cos h )  ×
= x sin x lim + x cos x(1) + sin x tan( x + h ) + tan x
h→ 0   h 
d sin h
h ⇒ ( f( x )) = e tan x
× lim
2 sin 2 dx h →0 h cos( x + h )cos x
= − x sin x ⋅ lim 2 × h + x cos x + sin x
h 4 1
h
→0 h × ×
2
4 tan( x + h ) + tan x
2 d 1 1
 h ⇒ ( f( x )) = e tan x
× ×
2  sin  dx cos 2 x 2 tan x
= − x ⋅ sin x ⋅ lim  2  × h + x cos x + sin x
4 h→0  h  d e tan x
2  2  ⇒ ( f( x )) = sec 2 x
dx 2 tan x
1  sin θ  44. Let f( x ) = sin x
= − x sin x ⋅ (1) × 0 + x ⋅ cos x + sin x Q lim =1
2 
 θ→ 0 θ 
By using first principle of derivative, we have
= x cos x + sin x f( x + h ) − f( x )
f′ ( x ) = lim
42. We have, h→ 0 h
f( x ) = sin 2x sin x + h − sin x
= lim
h→ 0 h
By using first principle of derivative,
f( x + h ) − f( x )  x + h + x  x + h − x
f′ ( x ) = lim 2 cos  sin  
h→0 h  2   2 
= lim
sin 2( x + h ) − sin 2 x h→ 0 (x + h ) − x
= lim
h→0 h   C + D  C − D 
[sin( x + h ) − sin x ] [sin( x + h ) + sin x ] Q sin C − sin D = 2 cos  2  sin  2  
= lim  
h→0 h  x+h + x  x + h − x
2 cos  sin  
 2x + h  h  2x + h  h   2   2 
2 cos 2  sin  2   2 sin  2  cos 2   = lim
= lim     h→ 0 ( x + h )2 − ( x )2
h→0 h
 x + h − x
 C+D C −D  sin  
Q sin C − sin D = 2 cos sin  2   x+h + x
 2 2  = lim × lim cos 
  h→ 0 x+h − x h→0  2 
and sin C + sin D = 2 sin C + D C − D 
cos
 2 2  2
h  1 
sin × lim  
2x + h 2 × lim 2 sin  2 x + h  cos h h→ 0  x + h + x 
= lim cos 
h→0  2  h h→0  2  2 1 1  sin θ 
= 1 × cos( x ) × = cos x Q lim =1
2 2 x 2 x  θ → 0 θ 
144 CBSE Term II Mathematics XI

45. We have, y = ( ax + b ) ( cx + d)2 (1 + tan x )( − sec 2 x ) − (1 − tan x )(sec 2 x )


=
On differentiating both sides w.r.t. x, we get (1 + tan x )2
dy d d −2 sec 2 x –2
= ( ax + b ) ( cx + d)2 + ( cx + d)2 ( ax + b ) = =
dx dx dx (1 + tan x ) 2
(cos x )(1 + tan 2 x + 2 tan x )
2
[using product rule of derivative] −2 −2
d 2 2 = =
= ( ax + b ) ( c x + d2 + 2 cxd) + ( cx + d)2 ( a × 1 + 0)  sin 2 x 2 sin x  (1 + sin 2 x )
dx (cos x ) 1 +
2
2
+ 
[Q ( A + B )2 = A 2 + 2 AB + B 2 ]  cos x cos x 
= ( ax + b ) ( c 2 2 x + 0 + 2 c × 1 × d) + ( cx + d)2 × a Hence proved.
sin x + cos x
= ( ax + b ) (2 c 2x + 2 cd) + a ( cx + d)2 48. Let f( x ) =
sin x − cos x
= ( ax + b ) 2 c ( cx + d) + a ( cx + d)2
On differentiating both sides w.r.t. x, we get
= ( cx + d) [2 c ( ax + b ) + a ( cx + d)]
 d 
= ( cx + d) (2 acx + 2 bc + acx + ad) (sin x − cos x ) (sin x + cos x )
 dx 
= ( cx + d) ( 3acx + 2 bc + ad)  
− (sin x + cos x ) d
x 2 + 3x − 9 (sin x − cos x )
46. Let y = 2
x − 9x + 3 f′( x ) =  dx 
(sin x − cos x )2
On differentiating both sides w.r.t. x, we get
 d d 
d  x 2 + 3x − 9  g( x ) f( x ) − f( x ) g( x )
dy
=  d  f( x )  dx dx 
dx dx  x 2 − 9x + 3  Q dx  g( x ) = [ g( x )]2 
 
 2 d 2    
( x − 9x + 3) dx ( x + 3x − 9) − ( x + 3x − 9) 
2
  d d  
  (sin x − cos x )  dx (sin x ) + dx (cos x ) 
 d 2   
 ( x − 9 x + 3 )   d 
= dx d
− (sin x + cos x ) (sin x ) − (cos x )
( x 2 − 9x + 3) 2   dx 
=
dx
[using quotient rule of derivative] (sin x − cos x )2
( x − 9x + 3)(2 x + 3 − 0) − ( x 2 + 3x − 9) 
2
(sin x − cos x) (cos x − sin x) − (sin x + cos x)
 
× (2 x − 9 + 0 ) (cos x + sin x)
=  =
( x 2 − 9x + 3) 2 (sin x − cos x)2
[( x 2 − 9x + 3) (2 x + 3) − ( x 2 + 3x − 9) (2 x − 9)]  d d 
= Q (sin x ) = cos x and (cos x ) = − sin x
( x 2 − 9x + 3) 2  dx dx 

2 x( x 2 − 9x + 3) + 3( x 2 − 9x + 3) − 2 x( x 2 + 3x − 9)  −(sin x − cos x ) − (sin x + cos x )


2 2
=
  (sin x − cos x )2
+ 9( x 2 + 3x − 9)
= 
(sin x − cos x )2 + (sin x + cos x )2 
( x 2 − 9x + 3) 2 =− 
 (sin x − cos x )2 
2 x( x 2 − 9x + 3 − x 2 − 3x + 9) + ( 3x 2 − 27 x + 9
  − (sin 2 x + cos 2 x − 2 sin x cos x + sin 2 x 
 + 9x 2 + 27 x − 81)   
=
+ cos 2 x + 2 sin x cos x )
(x 2 − 9x + 3)2 = 
2 x( − 12 x + 12 ) + (12 x 2 − 72 ) (sin x − cos x )2
=
( x 2 − 9x + 3) 2 − 2 (sin 2 x + cos 2 x ) −2
= =
− 24x 2 + 24x + 12 x 2 − 72 − 12 x 2 + 24x − 72 (sin x − cos x ) 2
(sin x − cos x )2
= =
( x 2 − 9x + 3) 2 ( x 2 − 9x + 3) 2 [Q sin 2 θ + cos 2 θ = 1]
− 12( x − 2 x + 6)
2
49. We have,
=
( x 2 − 9x + 3) 2 1 − cos 2 x 2 sin 2 x
1 − tan x y= = = tan 2 x
47. We have, y = 1 + cos 2 x 2 cos 2 x
1 + tan x
Q cos 2θ = 1 − 2 sin 2 θ ⇒ 2 sin 2 θ = 1 − cos 2θ and 
On differentiating both sides w.r.t. x, we get  
cos 2θ = 2 cos θ −1 ⇒ 2 cos θ = 1 + cos 2θ
2 2

d d
(1 + tan x ) ⋅ (1 − tan x ) − (1 − tan x ) ⋅ (1 + tan x )  π π 
dy dx dx ⇒ y = |tan x|, where x ∈  0,  ∪  , π 
=  2 2 
dx (1 + tan x )2
[using quotient rule of derivative]
CBSE Term II Mathematics XI 145

  π h
 tan x , x ∈  0, 2 
sin
2x + h 2
Now, y= = lim cos 
π h→0  2  h
− tan x , x ∈  , π 
 2  2
2x + h h
  π × lim 2 sin   cos
sec 2 x , if x ∈  0,  h→0  2  2
dy   2
∴ =  2 x + 0  2 x + 0
dx  π  = cos   × 1 × 2 sin   cos 0
− sec x , if x ∈  , π 
2
 2   2 
 2 
= cos x × 2 sin x × 1 = sin 2x
50. (i) Let f( x ) = log x
51. (i) We have, f( x ) = (cos 2 x − sin 2 x )
By using first principle of derivative, we have
f( x ) = (cos x − sin x )(cos x + sin x )
f( x + h ) − f( x ) log( x + h ) − log x
f′ ( x ) = lim = lim On differentiating both sides w.r.t. x, we get
h→ 0 h h→ 0 h d
 x + h  h f′ ( x ) = (cos x − sin x ) (cos x + sin x ) + (cos x + sin x )
log  log1 +  dx
 x   x 1 d
= lim = lim × (cos x − sin x )
h→ 0 h h→ 0 h x dx
x [using product rule of derivative]
1 1  1 + y  = (cos x − sin x ) ( − sin x + cos x ) + (cos x + sin x )
=1× = Q lim log  = 1
x x  y→ 0  y   ( − sin x − cos x )
= (cos x − sin x )(cos x − sin x ) − (cos x + sin x )
(ii) Let f( x ) = tan x
(cos x + sin x )
Then, by first principle of derivative, we get = (cos x − sin x )2 − (cos x + sin x )2
f( x + h ) − f( x )
f′ ( x ) = lim = (cos 2 x + sin 2 x − 2 sin x cos x ) − (cos2 x + sin 2 x
h→ 0 h
tan( x + h ) − tan x + 2sin x cos x )
= lim = − 2 sin x cos x − 2 sin x cos x
h→ 0 h
= − sin 2 x − sin 2 x = − 2 sin 2 x
1  sin( x + h ) sin x 
= lim  − (ii) Let y = x 2 sec x
h → 0 h  cos( x + h ) cos x 
On differentiating both sides w.r.t. x, we get
1 sin( x + h )cos x − sin x cos( x + h )  dy d d 2
= lim = x 2 (sec x ) + secx
h   (x )
h→ 0 cos( x + h )cos x  dx dx dx
1  sin( x + h − x )  [using product rule of derivative]
= lim
h→ 0 h  cos( x + h )cos x  = x 2 ⋅sec x tan x + secx(2 x )
[Q sin A cos B − cos A sin B = sin( A − B )] (iii) Let y = x 2 tan x
sin h 1 On differentiating both sides w.r.t. x, we get
= lim . lim dy d d 2
h→ 0 h h → 0 cos( x + h )cos x
= x 2 (tan x ) + tan x (x )
1  sin θ  dx dx dx
= 1⋅ Q lim =1
cos( x + 0)cos x  θ → 0 θ  [using product rule of derivative]
1 = x 2sec 2x + tan x (2 x )
= = sec 2 x (iv) Let y = x 2 sin x + cos 2 x
cos 2 x
d On differentiating both sides, w.r.t. x, we get
Hence, f′( x ) or (tan x ) = sec 2 x
dx dy d 2 d 2 d
= ( x sin x + cos 2 x ) = ( x sin x ) + (cos 2 x )
(iii) We have, f( x ) = sin 2x dx dx dx dx
d 2 d
By using first principle of derivative, we have = ( x sin x ) + (2 cos 2 x − 1)
dx dx
f( x + h ) − f( x ) sin 2( x + h ) − sin 2 x d 2 d
f′ ( x ) = lim = lim = ( x sin x ) + (2 cos x cos x − 1)
h→0 h h→0 h dx dx
[sin( x + h ) − sin x ] [sin( x + h ) + sin x ] = x 2 ⋅ cos x + sin x 2 x
= lim
h→0 h   d d  d 
+ 2  cos x (cos x ) + cos x (cos x ) − (1)
  dx dx  dx 
 2x + h  h  2x + h  h 
2 cos 2  sin  2   2 sin  2  cos 2   = x 2 cos x + 2 x sin x − 2 sin 2 x
= lim    
(v) Let y = ( 3x + 5)(1 + tan x )
h→0 h dy d
 C+D C −D  Then, = [( 3x + 5)(1 + tan x )]
Q sin C − sin D = 2 cos sin dx dx
 2 2  d d
  = ( 3x + 5) (1 + tan x ) + (1 + tan x ) ( 3x + 5)
and sin C + sin D = 2 sin C + D C − D  dx dx
cos
 2 2  ( 3x + 5) (sec 2 x ) + (1 + tan x ) ( 3)
= 3x sec 2 x + 5sec 2 x + 3 tan x + 3
Chapter Test
l
Multiple Choice Questions Based on above information, answer the
following questions
dy dy
1. Evaluate , if y = sin(x 2 + 1). (i) If y = x (2sin x + 3 cos x), then at x = 0 is equal
dx dx
(a) 2 x cos ( x + 1) to
(b) 2 x ( x2 + 1) (a) 1 (b) 2
(c) 3 (d) − 1
(c) 2 x sin( x + 1)
2

dy π
(d) 2 x cos ( x2 + 1) (ii) If y = tan2 x , then at x = is equal to
dx 4
 π (a) 4 (b) 3
x 2 cos 
 4 (c) 5 (d) 1
2. The derivative of is
sin x dy
(iii) If y = (x 3 + x 2 + 2)(sin x), then at x = 0 is equal
 π dx
x sin   [2sin x + x cos x]
 4 to
(a)
sin2 x (a) 4 (b) 3
 π (c) 1 (d) 2
x cos   [2sin x − x cos x]
 4 dy
(b) (iv) If y = (x + 2) (x 2 + 2x + 3), then at x = − 1 is
sin2 x dx
 π
x sin   [2sin x − x cos x] equal to
 4 (a) 3 (b) 1
(c)
cos2 x (c) 2 (d) 0
 π dy
x cos   [2sin x + x cos x] (v) If y = xe x + log x , then at x = 1 is equal to
 4
(d) dx
cos2 x (a) 2e + 1 (b) 2e − 1
(c) 2e (d) − 1
3. If the derivative of (x + cos x) (x − tan x) is
A(x + cos x) + B (x − tan x), then A and B l
Short Answer Type Questions
respectively are
(a) tan2 x and 1 − sin x
6. Find the first order derivative, using first
principle of
(b) − tan2 x and 1 − sin x
 π
(c) tan2 x and sin x − 1 f (x) = cos  x − 
 8
(d) − tan2 x and sin x − 1

4. If f is a function given by f (x) = 2x 2 + 3 x − 5 and 7. Evaluate differentiation, by first order principle


f ′ (0) = mf ′ (− 1), then m is equal to 1
f (x) =
(a) − 1 (b) − 2 x3
(c) − 3 (d) − 4 l
Long Answer Type Questions
l
Case Based MCQs 8. For the function
5. Pankaj was preparing for his maths test on the x 100 x 99 x2
f (x) = + +K+ + x + 1.
topic derivative of product of two functions. 100 99 2
Derivative of product of two functions is given Prove that f ′ (1) = 100f ′ (0)
by the following product rule. 4 x + 5sin x
9. Find the derivative of .
d
[f (x) ⋅ g (x)] = f (x)
d
g(x) + g(x)
d
f (x) 3 x + 7 cos x
dx dx dx
This is also known as Leibnitz product rule of
derivative.
Answers
For Detailed Solutions
1. (d) 2. (b) 3. (b) 4. (c) 5. (i) (c) (ii) (a) (iii) (d) (iv) (c) (v) (a)
Scan the code
π −3 35 + 28x sin x + 15x cos x − 15sin x + 28cos x
6. − sin  x −  7. 9.
 8 x4 (3x + 7cos x )2
CBSE Term II Mathematics XI 147

CHAPTER 07

Probability

In this Chapter...
l Outcomes and Sample Space
l Event
l Algebra of Events
l Probability Equally Likely Outcomes
l Addition Rule of Probability

Outcomes and Sample Space Sure Event The whole sample space S is also a subset of S, so it
represents an event. Since, every outcome of an experiment
A possible result of a random experiment is called its
carried out a member of S, therefore the event represented by S is
outcome. The set of all possible outcomes in a random
called a sure or a certain event.
experiment is called a sample space and it is generally
denoted by S, i.e. e.g.
Sample space (S) = {All possible outcomes} (i) Sun rises in the East, is a sure event.
Each element of a sample space is called a sample point (ii) On tossing a coin, either head or tail will occur, is a sure
or an event point. event.
(iii) On throwing a die, we have sample space
Event S = {1 , 2 , 3 , 4 , 5 , 6 }.
Any subset E of a sample space S is called an event. Let E = Event of getting a number less than 7.
e.g. On throwing a die, we have the sample space Then, E = {1 , 2 , 3 , 4 , 5 , 6}.
S = {1 , 2 , 3 , 4 , 5 , 6} So, E is a sure event.
Then, E = {2 , 3 , 4} is an event. [QE ⊆ S] Impossible Event The empty set φ is also a subset of the sample
Occurrence of an Event space (S) so it represents an event. Since there is no element in the
empty set, therefore it can not occurs.
An event associated with a random experiment is said to
occur, if any one of the elementary events associated to That’s why event φ is called an impossible event.
it is an outcome of the experiment. e.g. On throwing a die, we have the sample space
e.g. Suppose a die is thrown and let A be an event of S = {1 , 2 , 3 , 4 , 5 , 6}
getting an odd number. Let E = Event of getting a number less than 1.
Then, A = {1 , 3 , 5} Then, E = φ.
Types of Events So, E is called an impossible event.
Events can be classified into various types on the basis Simple Event If an event has only one sample point then it is
of their elements. called a simple or elementary event.
148 CBSE Term II Mathematics XI

e.g. Let a die is thrown, then sample space The Event ‘A and B’
S = {1 , 2 , 3 , 4 , 5 , 6} The event ‘A and B ’ is same as the event ‘A ∩ B’ and it
Again, let A = Event of getting 4 = {4} contains all those elements which are common to both A and
B = Event of getting 1 = {1} B. Thus, A and B = A ∩ B = {ω : ω ∈ A and ω ∈B}
Here, A and B are simple events.
Compound Event If an event has more than one sample The Event ‘A but not B’
point, then it is called a compound event. The events ‘A but not B ’ is same as the event A − B = (A ∩ B′ )
e.g. On rolling a die, we have sample space and it contains all those elements which are in A but not in B.
S = {1 , 2 , 3 , 4 , 5 , 6} Thus, A but not B = A − B = {ω : ω ∈ A and ω ∉B}
Then, the events Note For any event E, associated with a sample space S,
E : Getting an even number E′ = not E = S − E = {ω : ω ∈ S and ω ∉ E}.
F : Getting an odd number
G : Getting a multiple of 3
Mutually Exclusive Events
are all compound events. The subsets of S associated with Two events are said to be mutually exclusive, if the
these events are occurrence of any one of them excludes the occurrence of
other event, i.e. if they cannot occur simultaneously.
E = {2 , 4 , 6},
Thus, two events E 1 and E 2 are said to be mutually exclusive,
F = {1 , 3 , 5}, G = {3 , 6}
if E 1 ∩ E 2 = φ
Each of the above subsets contain more than one sample
e.g. In throwing a die, we have sample space
point, therefore they are all compound events.
S = {1 , 2 , 3 , 4 , 5 , 6}
Algebra of Events Let E 1 = Event of getting even numbers
We know that how to combine two or more sets by using the = {2 , 4 , 6}
operations on sets union (∪), intersection (∩)and difference E 2 = Event of getting odd numbers = {1 , 3 , 5}
(−). Like-wise we, can combine two or more events by using
the analogous set notations. Let A and B are two events Here, E 1 ∩ E 2 = φ.
associated with a sample space S, then So, E 1 and E 2 are mutually exclusive events.
Complementary Event In general, events E 1 , E 2 ,… , E n are said to be mutually
exclusive, if they are pair-wise disjoint, i.e. if E i ∩ E j = φ for
For every event E, there corresponds another event E′ called all i ≠ j.
the complementary event of E, which consist of those
outcomes that do not correspond to the occurrence of E. E′ is Exhaustive Events
also called the event ‘not E’.
For a random experiment, a set of events is said to be
e.g. In tossing three coins, the sample space is exhaustive, if one of them necessarily occurs whenever the
S = {HHH, HHT , HTH, THH, experiment is performed. Let E 1 , E 2 , E 3 ,…, E n be n
HTT , THT , TTH, TTT} subsets of a sample space S. Then, events E 1 , E 2 , E 3 ,…, E n
Let E = {THT , TTH, HTT} are called exhaustive events, if
= the event of getting only one head. E 1 ∪ E 2 ∪ E 3 ∪ . . . ∪E n = S
Clearly, for outcome HHT, event E has not occurred e.g. Consider the experiment of throwing a die. We have,
∴ HHT ∈ E ′ S = {1 , 2 , 3 , 4 , 5 , 6}. Let us define the following events
Similarly, for the outcomes HTH, THH, TTT and HHH, E 1 : a number less than 4 appears.
event E has not occurred. E 2 : a number greater than 2 but less than 5 appears.
∴ E ′ = {HHT, HTH, THH, TTT, HHH} E 3 : a number greater than 4 appears.
It can be seen that E′ contains only those outcomes which are Then, E 1 = {1 , 2 , 3}, E 2 = {3 , 4}, E 3 = {5 , 6}.
not in E.
We observe that
The Event ‘A or B’ E 1 ∪ E 2 ∪ E 3 = {1 , 2 , 3} ∪ {3 , 4} ∪ {5 , 6} = S
The event ‘A or B ’ is same as the event A ∪ B and it contains Thus, the events E 1 , E 2 and E 3 are exhaustive events.
all those elements which are either in event A or in events B
or in both. Thus, A or B = A ∪ B = {ω : ω ∈ A or ω ∈B}
CBSE Term II Mathematics XI 149

Probability of Equally Likely Outcomes Probability of the Event A or B or C


The outcomes of a random experiment are said to be equally If A, B and C are three events associated with a random
likely, if the chance of occurrence of each outcome is same, experiment, then
i.e. chances of each outcome is same. P( A ∪ B ∪ C ) = P( A ) + P( B) + P(C ) − P( A ∩ B)
Let the sample space of an experiment is − P( B ∩ C ) − P(C ∩ A ) + P( A ∩ B ∩ C )
S = {s 1 , s 2 , K , s n } If A, B and C are mutually exclusive events,
Also, let all the outcomes are equally likely. i.e. A ∩ B = φ,B ∩ C = φ, A ∩ C = φ, A ∩ B ∩ C = φ.
i. e. P(s i ) = p, for all s i ∈ S, where 0 ≤ p ≤ 1 Then, P( A ∪ B ∪ C) = P( A) + P(B) + P( C)
Probability of Complementary Event
Addition Rule of Probability
Let E be any event and E (or E′) be its complementary event.
If A and B are two events associated with a random
experiment, then Then, P(E ) = 1 − P(E )
P( A ∪ B) = P( A) + P(B) − P( A ∩ B) Important Results
i.e. P( A or B) = P( A) + P(B) − P( A and B) 1. For any two events A and B, A ⊆ B ⇒P( A) ≤ P(B)
It is known as addition law of probability for two events.
2. For any two events A and B,
Also,
(i) P( A − B) = P( A) − P( A ∩ B)
(i) when events A and B are mutually exclusive, then or P ( A ∩ B ) = P ( A) − P ( A ∩ B)
P ( A ∪ B) = P ( A) + P (B)
(ii) P( A ∩ B) = P( A ∪ B)
(ii) when events A and B are mutually exclusive and
exhaustive, then P ( A ∪ B) = P ( A) + P (B) = 1. 3. P ( A ∩ B) ≤ P( A) and P ( A ∩ B) ≤ P(B)
3 6 [If these conditions hold true, then events
e.g. If P( A) = , P(B) =
5 15 are said to be consistently defined]
150 CBSE Term II Mathematics XI

Solved Examples
Example 1. Two dice are thrown once. ∴ S = {( X , B1 ), ( X , B 2 ), ( X , G1 ), ( X , G 2 ),
The events A , B are as follows. ( Y , b1 ), ( Y , g1 ), ( Y , g2 ), ( Y , g3 )}
A : Getting an even number on first die. (ii) Given, defective bulb denoted by D and non-defective
B : Getting the sum of numbers on the dice ≥ 10. bulb denoted by N.
Describe the events There are 2 3 = 8 possible outcomes.
D N
(i) A ′ (ii) B ′
Sol. On throwing of two dice, we have sample space, DN ND NN
DD
(1,1), (1,2 ), (1, 3), (1, 4), (1, 5), (1, 6), 
(2 ,1), (2 ,2 ), (2 , 3), (2 , 4), (2 , 5), (2 , 6), DDD DDN DND DNN NDD NDN NND NNN
 
( 3,1), ( 3,2 ), ( 3, 3), ( 3, 4), ( 3, 5), ( 3, 6), Hence, the required sample space
S= 
( 4,1), ( 4,2 ), ( 4, 3), ( 4, 4), ( 4, 5), ( 4, 6), = {DDD , DDN , DND , DNN , NDD , NDN,
( 5,1), ( 5,2 ), ( 5, 3), ( 5, 4), ( 5, 5), ( 5, 6), NND , NNN}
 
( 6,1), ( 6,2 ), ( 6, 3), ( 6, 4), ( 6, 5), ( 6, 6)  Example 3. A pair of dice is rolled. If the outcome is
(i) A : Getting an even number on first die. a doublet, a coin is tossed. Then, the total number
⇒ A′ : Getting an odd number on first die. of outcomes for this experiment is …K… . What is
(1,1), (1,2 ), (1, 3), (1, 4), (1, 5), (1, 6),  K here?
 
= ( 3,1), ( 3,2 ), ( 3, 3), ( 3, 4), ( 3, 5), ( 3, 6), Sol. The sample space associated with the given random
( 5,1), ( 5,2 ), ( 5, 3), ( 5, 4), ( 5,5), ( 5, 6)  experiment is given by
 
S = {(1, 2), (1, 3), (1, 4), (1, 5), (1, 6), (2, 1), (2, 3), (2, 4),
(ii) B : Getting the sum of the numbers on the dice ≥ 10.
(2, 5), (2, 6), (3, 1), (3, 2), (3, 4), (3, 5), (3, 6), (4, 1), (4, 2),
⇒ B′ : Getting the sum of the numbers on the dice < 10.
(4, 3), (4, 5), (4, 6), (5, 1), (5, 2), (5, 3), (5, 4), (5, 6), (6, 1),
(1,1), (1,2 ), (1, 3), (1, 4), (1, 5), (1, 6),(2 ,1), (2 ,2 ),  (6, 2), (6, 3), (6, 4), (6, 5)}
(2 , 3), (2 , 4), (2 , 5), (2 , 6), ( 3,1), ( 3,2 ), ( 3, 3), 
=  Now, when outcomes is doublet, then coin is tossed.
( 3, 4), ( 3,5), ( 3, 6), ( 4,1), ( 4,2 ), ( 4, 3), ( 4, 4), ( 4, 5), So, sample space
( 5,1), ( 5,2 ), ( 5, 3), ( 5, 4), ( 6,1), ( 6,2 ), ( 6, 3)  = {(11, H) (11, T)

Example 2. Solve the following sample space problems. (22, H) (22, T)


(33, H) (33, T)
(i) 2 boys and 2 girls are in room X and 1 boy and
(44, H) (44, T)
3 girls are in room Y. Specify the sample space for
(55, H) (55, T)
the experiment in which a room is selected and
then a person. (66, H) (66, T)}
The total number of sample points = 42
(ii) Suppose 3 bulbs are selected at random from a lot.
Each bulb is tested and classified as defective Hence, the value of K is 42.
(D) or non-defective (N). Write the sample space Example 4. A coin is tossed two times, consider the
for this experiment. following events.
Sol. (i) Let the boys and girls in room X be denoted by P : No head appears.
B1 , B 2 , G1 , G 2 and those in room Y be denoted by
b1 , g1 , g2 , g3. A room out of the two can be selected in Q : Exactly one head appears.
two ways, either the room X is selected or the room Y is and R : Two heads appears.
selected. Corresponding to either case, there are four Do they form a set of mutually exclusive and
ways of selecting a person.
exhaustive events?
X Y Sol. When a coin is tossed two times, then the sample space is
S = {HH , HT , TH , TT}
Given, P = event of getting no head
∴ P = {TT}
B1 B2 G1 G2 b1 g1 g2 g3 Q = event of getting exactly one head
∴ Q = {HT , TH}
CBSE Term II Mathematics XI 151

and R = event of getting two heads. Thus, in this case, number of four digits numbers divisible
∴ R = {HH} by 5 = 3 × 2 × 1 = 6
Clearly, P ∩ Q = φ, P ∩ R = φ and Q ∩ R = φ Case II When unit’s place is filled by 5.
So, we can say that P , Q and R are mutually exclusive event. 5
[Q P , Q and R are pairwise disjoint]
Also, P ∪ Q ∪ R = {HH , HT , TH , TT} = S In this case, thousand’s place can be filled up in 2 ways.
∴ P , Q and R are exhaustive events. hundred’s place can be filled up in 2 ways and ten’s place
Hence, P , Q and R form a set of mutually exclusive and can be filled up in 1 way.
exhaustive events. Thus, in this case, number of four digits numbers divisible
by 5 = 2 × 2 × 1 = 4. From cases I and II,
Example 5. If a letter is chosen at random from the
Total number of 4-digits numbers divisible by 5 = 6 + 4 = 10
English alphabet, then find the probability that the
⇒ Number of favourable outcome = 10
letter is
10 5
(i) a vowel (ii) a consonant Hence, required probability = =
18 9
Sol. There are 26 letters in English alphabet, out of which 5 are
vowels and 21 are consonants. Example 7. The letters of the word ‘CLIFTON’ are
So, total number of possible outcomes = 26 arranged at random in a row. What is the chance
Let E be the event of getting a vowel and F be the event of that two vowels come together?
getting a consonant. Sol. There are 7 letters in the word ‘CLIFTON’.
Then, n (E ) = Number of outcomes favourable to E = 5 These 7 letters can be arranged in a row in 7! ways.
and n (F ) = Number of outcomes favourable to F = 21 ⇒ Total number of possible outcomes = 7!
5 Now, if vowels (I, O) come together, then we have 6 letters
(i) Now, Probability of getting a vowel, P(E ) =
26 (considering vowel as one letter i.e. CLFTN IO) which can
21 be arranged in 6! ways.
(ii) Probability of getting a consonant, P(F ) =
26 But two vowels can be put together in 2! ways.
Example 6. Without repetition of the number, four digit ∴The total number of arrangements in which two vowels
come together is 6! × 2 ! .
numbers are formed with the numbers 0, 2, 3, 5.
⇒ Number of favourable outcomes = 6! × 2 !
Find the probability of such numbers which are
6! × 2 ! 2
divisible by 5. Hence, required probability = =
7! 7
Sol. In a four digit number, 0 cannot put in thousand’s place.
So, the number of ways of filling up the thousand’s place = 3. Example 8. A typical PIN (personal identification
number) is a sequence of any four symbols chosen
from the 26 letters in the alphabet and the ten
Thousand’s Hundred’s Ten’s digits. If all PINs are equally likely, then find the
Unit’s
place place place place
probability that a randomly chosen PIN contains a
repeated symbol.
Now, we left with three digits (including 0) and 3 places. Sol. A PIN is a sequence of four symbols selected from 36
Note that, hundred’s place can be filled up in 3 ways. (26 letters + 10 digits) symbols.
Ten’s place can be filled up in 2 ways and unit’s place can be By the fundamental principle of counting, there are
filled up in 1 ways. 36 × 36 × 36 × 36 = 364 = 1679616 PINs in all. When
So, the number of 4-digits numbers = 3 × 3 × 2 × 1 = 18 repetition is not allowed, the multiplication rule can be
⇒ Total number of possible outcomes = 18 applied to conclude that there are
Now, if the number is divisible by 5, then its unit’s place is 36 × 35 × 34 × 33 = 1413720 different PINs.
filled by either 5 or 0. The number of PINs that contain atleast one repeated
Case I When units place filled by 0. symbol = 1679616 − 1413720
= 265896
0
Thus, the probability that a randomly chosen PIN contains a
In this case, thousand’s place can be filled up in 3 ways, repeated symbol is
265896
hundred’s place can be filled up in 2 ways and ten’s place = 0.1583
can be filled up in 1 ways. 1679616
152 CBSE Term II Mathematics XI

Example 9. A die has two faces each with number ‘1’, Number of ways in which 2 tickets without prizes can
three faces each with number ‘2’ and one face with be selected out of 9990 tickets = 9990
C2
number ‘3’. If die is rolled once, then find the C 9990
followings. ∴ Required probability = 10000 2
C2
(i) P (2) (ii) P (1 or 3)
(iii) Number of ways in which 10 tickets can be selected out
(iii) P (not 3)
of 10000 tickets = 10000 C10
Sol. (i) If E be the event getting a number 2, then n(E ) = 3
[Q there are three faces each with number 2] Number of ways in which 10 tickets without prizes can
∴ Probability of getting 2 be selected out of 9990 tickets = 9990
C10
Number of favourable outcomes
= 9990
C10
Total number of outcomes ∴ Required probability = 10000
n (E ) 3 1 C10
= = =
n (S ) 6 2 Example 11. The accompanying venn diagram shows
[in case of die total number of outcomes is 6.] three events A , B and C and also the probabilities of
(ii) If E be the event getting a number 1 or 3, then the various intersections [for instance
n (E ) = 2 + 1 = 3 P( A ∩ B ) = 0. 07]. Determine
n (E ) 3 1 (i) P( A) (ii) P(B ∩ C)
∴Probability of getting 1 or 3. = = =
n (S ) 6 2
(iii) P( A ∪ B) (iv) P( A ∩ B )
[Q there are two faces each with number 1 and
one face with number 3] (v) P(B ∩ C)
(iii) If E be the event getting a number not 3, then (vi) Probability of exactly one of the three event occurs.
n (E ) = 2 + 3 = 5
[not 3 means there will be the number on die other A B
0.10
than 3 i.e. 1 and 2. There are two faces with 0.13 0.07
0.15
number 1 and three faces with number 3]
n (E ) 5
∴ Probability of getting (not 3) P = =
n (S ) 6 0.28
C
Example 10. In a certain lottery, 10000 tickets are sold
and ten equal prizes are awarded. Then, the Sol. (i) P ( A ) = 0.13 + 0.07 = 0.20
probabilities of not getting a prize, if you buy (ii) P (B ∩ C ) = P (B ) − P (B ∩ C )
(i) one ticket (ii) two tickets = (0.07 + 0.10 + 0.15) − (0.15) = 0.17
(iii) 10 tickets, are respectively. (iii) P ( A ∪ B ) = P ( A ) + P (B ) − P ( A ∩ B )
Sol. = 0.20 + (0.07 + 0.10 + 0.15) − 0.07
Total tickets = 0.20 + 0.25 = 0.45
10000
(iv) P ( A ∩ B ) = P( A ) − P( A ∩ B ) = 0.20 − 0.07 = 0.13
(v) P (B ∩ C ) = 0.15
10 have 9990 are (vi) P (exactly one of the three events occurs)
prizes blanks = P( A only) + P(B only) + P(C only)
(i) 1 ticket out of 10000 ticket can be selected in = 10000 C1 = 0.13 + 0.10 + 0.28 = 0.51
ways. Example 12. A card is drawn from an ordinary pack and
Number of ways in which 1 ticket without prize is a gambler bets that it is spade or an ace.
obtained = 9990 C1 What are the odds against the winning his bet?
∴ Required probability P Sol. Let S be the sample space associated with the given random
Number of favourable outcomes experiment. Then, n (S)= 52.
=
Total number of outcomes Now, let A be the event of getting a spade and B be the
9990
C 9990 event of getting an ace. A ∩ B denote the event of getting an
= 10000 1 = [Q n C1 = n ]
C1 10000 ace of spade.
999 Then, n ( A ) = 13 and n (B ) = 4
=
1000 n ( A ∩ B) = 1
13 4 1
(ii) Number of ways in which 2 tickets can be selected out Thus, we have P ( A ) = , P (B ) = and P ( A ∩ B ) =
52 52 52
of 10000 tickets = 10000 C 2
CBSE Term II Mathematics XI 153

Now, P (winning the bet) = P (getting a spade or an ace) Sol. Let E1 ,E 2 ,E 3 and E 4 denote respectively the events that a
= P ( A ∪ B ) = P ( A ) + P (B ) − P ( A ∩ B ) student will receive A , B , C and D grades. Then, we have
13 4 1 16 4 P(E1 ) = 0.40, P(E 2 ) = 0.35, P(E 3 ) = 015
. and P(E 4 ) = 010
= + − = = .
52 52 52 52 13 (i) P (getting B or C grade) = P (E2 ∪ E 3 ) = P(E2 ) + P(E3 )
4 9
⇒ P (not winning the bet) = 1 − = = 0.35 + 015
. = 0.5
13 13
[QE2 and E3 are mutually exclusive events]
Hence, odds against the winning his bet (ii) P (getting atleast C grade) = P (getting C or D grade)
9
= P (E3 ∪ E4 ) = P (E3 ) + P (E4 )
P (not winning the bet) 13 9
= = = = 9: 4 = 015
. + 010
. = 025
.
P (winning the bet) 4 4
13 Example 15. Out of 100 students, two sections of
Example 13. In a class of 60 students, 30 opted for 40 and 60 are formed. If you and your friend are
NCC, 32 opted for NSS and 24 opted for both NCC among the 100 students.
and NSS. If one of these students is selected at (i) Find the probability that you both enter the same
random, then find the probability that section.
(i) the student opted for NCC or NSS. (ii) Find the probability that you both enter the
(ii) the student has opted for neither NCC nor NSS. different sections.
Sol. Let there be two sections A and B of 40 and 60 students,
(iii) the student has opted NSS but not NCC. respectively.
Sol. Let A and B denote the events that, the selected students (i) Let both students enter the section A.
opted NCC and NSS, respectively.
So, 38 students are to selected out of 98, since two
Given, n ( A ) = 30, n (B ) = 32 particular students are already in section A.
n ( A ∩ B ) = 24, n (S) = 60 ∴Number of ways of selecting 38 students out of
[Q 24 students opted for both NCC and 98 = 98C 38 ways
NSS i.e. they are common in both] Number of exhaustive cases of selecting 40 students out
n ( A ) 30 n (B ) 32 of 100 = 100 C 40 ways
∴ P(A ) = = , P (B ) = =
n (S) 60 n (S) 60
∴ P (both students enter section A)
n ( A ∩ B ) 24 98
and P( A ∩ B ) = = C
= 100 38 =
98!
÷
100!
n (S ) 60 C 40 38! 60! 40! 60!
(i) P(student opted for NCC or NSS) 98! 40! 60! 40 × 39 26
= P ( A ∪ B ) = P ( A ) + P (B ) − P ( A ∩ B ) = × = =
38! 60! 100! 100 × 99 165
30 32 24
= + − If both students enter the section B. Then, the number
60 60 60 of ways of selecting 58 students out of 98 = 98C 58 ways.
30 + 32 − 24 62 − 24 38 19
= = = = Total number of ways of selecting 60 students out of
60 60 60 30
100 = 100 C 60 ways.
(ii) P(student opted neither NCC nor NSS)
= P(A ∩ B ) = P(A ∪ B) ∴ Probability that two students enter section B
98
C 98! 100!  n n! 
19 30 − 19 11 = 100 58 = ÷
= 1 − P (A ∪ B) = 1 − = = Q C r = r !( n − r )! 
30 30 30 C 60 58! 40! 60! 40!  
(iii) P(student has opted NSS but not NCC) 98! 60! 40! 60 × 59 59
= × = =
= P(B ∩ A ) = P(B ) − P( A ∩ B ) 58! 40! 100! 100 × 99 165

=
32 24
− =
8
=
2 ∴P (that two particular students enter either section A
60 60 60 15 or B)
26 59 85 17
Example 14. The probability that a student will receive = + = =
165 165 165 33
A, B, C or D grade are 0.40, 0.35, 0.15 and 0.10,
(ii) The probability that they enter different sections
respectively. Find the probability that a student will
= 1 − P (that two particular students enter either section
receive A or B)
(i) B or C grade. 17 16
=1− =
(ii) atleast C grade. 33 33
154 CBSE Term II Mathematics XI

Chapter
Practice
PART 1
Objective Questions
l
Multiple Choice Questions 5. A die is rolled. Let E be the event “die shows 4”
1. A bag contains one white and one red ball. A ball and F be the event “die shows even number”.
is drawn from the bag. If the ball drawn is white it Then, E and F are
is replaced in the bag and again a ball is drawn, (a) mutually exclusive
otherwise, a die is tossed. The number of sample (b) exhaustive
point in the sample space of above experiment is (c) mutually exclusive and exhaustive
(d) None of the above
(a) 6 (b) 7 (c) 8 (d) 9
2. A boy has a ` 1 coin, a ` 2 coin and a ` 5 coin in his 6. If a single letter is selected at random from the
pocket. He takes out two coins out of his pocket, word ‘PROBABILITY’, then the probability that it
one after the other. If Q denotes a ` 1 coin, H is a vowel, is
1 4 2 3
denotes ` 2 coins and R denotes ` 5 coins. (a) (b) (c) (d)
3 11 11 11
Then, the sample space for the experiment is
(a) S = ( QH , QR , HQ, HR , RH ,RQ) 7. In a non-leap year, the probability of having
(b) S = ( QH , QR , QQ) 53 Tuesday or 53 Wednesday is
(c) S = ( QH , QR , HQ, HR ) (a)
1
(b)
2
(d) S = ( QH , QR , HQ, HR , RQ) 7 7
3
3. A coin is tossed three times, consider the following (c) (d) None of these
7
events
A : no head appears. 8. A letter is chosen at random from the word
‘ASSASSINATION’. Then, the probability that
B : exactly one head appears.
letter is (i) a vowel (ii) a consonant, respectively are
C : atleast two heads appear. 6 6 7 6 6 7 7 7
(a) and (b) and (c) and (d) and
Then, 13 13 13 13 13 13 13 13
(a) A, B and C are mutually exclusive events
9. A coin is tossed twice. Then, the probability that
(b) A, B and C are exhaustive events
(c) Only (a)
atleast one tail occurs is
1 1 1 3
(d) Both (a) and (b) (a) (b) (c) (d)
4 2 3 4
4. A coin is tossed two times, consider the following
events? 10. A fair coin with 1 marked on one face and 6 on the
other and a fair die are both tossed. Then, probability
E1 : No head appears. that sum of numbers gives 4.
E 2 : Exactly one head appears. 1 1 1 1
(a) (b) (c) (d)
and E 3 : Atleast one head appears. 12 13 15 2
They form a set of 11. Seven persons are to be seated in a row. The
(a) mutually exclusive event probability that two particular persons sit next to
(b) exhaustive event each other, is
(c) mutually exclusive and exhaustive event 1 1 2 1
(a) (b) (c) (d)
(d) mutually exclusive but not exhaustive event 3 6 7 2
CBSE Term II Mathematics XI 155

12. While shuffling a pack of 52 playing cards, 2 are will have faulty breaks as well as badly worn tires,
accidentally dropped. The probability that the is ...K... . Here, K refers to
missing cards to be of different colours is (a) 0.04 (b) 0.07 (c) 0.06 (d) 0.09
25 26 27 28
(a) (b) (c) (d) 20. A box contains 6 nails and 10 nuts. Half of the nails
52 51 50 49
and half of the nuts are rusted. If one item is chosen
13. 4 cards are drawn from a well-shuffled deck of at random, then the probability that it is rusted or is
52 cards. Thus, the probability of obtaining a nail, is ...K... . Here, K refers to
3 diamonds and one spade is 3 5 11 14
(a) (b) (c) (d)
13
C1 × 12C1 13
C 3 × 13C1 16 16 16 16
(a) 52
(b) 52
C1 C4
13
C1 × 13C 2 13
C 3 × 13C 2
l
Case Based MCQs
(c) 52
(d) 51
C2 C4 21. There are 4 red, 5 blue and 3 green marbles in a
basket.
14. In a lottery, a person chooses six different natural
numbers at random from 1 to 20 and if these six Based on the above information, answer the
numbers match with the six numbers already fixed following.
by the lottery committee, he wins the prize. Then, (i) If two marbles are picked at randomly, then the
the probability of winning the prize in the game is probability that both are red marbles is
1 1 1 1 3 1 1 1
(a) (b) (c) (d) (a) (b) (c) (d)
38500 38800 38060 38760 7 2 11 6
15. If A , B and C are three mutually exclusive and (ii) If three marbles are picked at randomly, then the
exhaustive events of an experiment such that probability that all are green marbles is
1 1 1 1
3P( A ) = 2P(B ) = P(C ), then P( A ) is equal to K . (a) (b) (c) (d)
Here, K refers to 220 55 75 80
1 2 5 6 (iii) If two marbles are picked at randomly, then find
(a) (b) (c) (d)
11 11 11 11 the probability that both are not blue marbles.
3 1 6 5 7 9
16. Given, P( A ) =and P(B ) = . Then, P (A or B), if A (a) (b) (c) (d)
11 12 22 11
5 5
and B are mutually exclusive events, is (iv) If three marbles are picked at randomly, then the
2 3 4 probability that atleast one of them is blue, is
(a) (b) (c) (d) 1 7 37 5 1
5 5 5 (a) (b) (c) (d)
12 44 12 44
17. The probability of getting atmost two tails or atleast
two heads in a toss of three coins is (v) If three marbles are picked randomly, then the
3 5 7 3 probability that either all are red or all are green.
(a) (b) (c) (d) 7 7 5 1
4 8 8 8 (a) (b) (c) (d)
44 12 12 44
2
18. Probability that Ram passed in Mathematics is 22. Two students Anil and Vijay appeared in an
3
4 examination. The probability that Anil will qualify
and the probability that he passed in English is . the examination is 0.05 and that Vijay will qualify is
9
1 0.10. The probability that both will qualify is 0.02.
If the probability of passing in both subjects is , On the basis of above information, answer the
4
then the probability that Ram will pass in atleast following questions.
one of these subjects will be (i) Find the probability that Vijay will not qualify the
31 35 1 1 exam.
(a) (b) (c) (d)
36 36 2 4 (a) 0.9 (b) 0.5 (c) 0.8 (d) 0.2

19. Probability that a truck stopped at a roadblock will (ii) Find the probability that atleast one of them will
have faulty brakes and badly worn tires, are 0.23 qualify the exam.
(a) 0.09 (b) 0.13 (c) 0.25 (d) 0.19
and 0.24, respectively. Also, the probability is 0.38
that a truck stopped at the roadblock will have (iii) Find the probability that atleast one of them will
faulty brakes or badly worn tires. Then, the not qualify the exam.
probability that a truck stopped at this roadblock (a) 0.82 (b) 0.74 (c) 0.56 (d) 0.98
156 CBSE Term II Mathematics XI

(iv) Find the probability that both Anil and Vijay will (iv) The sum of 6 appears in a single toss of a pair of
not qualify the exam. fair dice.
(a) 0.43 (b) 0.67 (c) 0.87 (d) 0.91 7. A die is loaded in such a way that each odd number
(v) Find the probability that only one of them will is twice as likely to occur as each even number.
qualify the exam. Find P(G), where G is the event that a number
(a) 0.21 (b) 0.11 (c) 0.31 (d) 0.25 greater than 3 occurs on a single roll of the die.
8. Four cards are drawn at random from pack of
PART 2 52 playing cards. Find the probability of getting
(i) all face cards.
Subjective Questions (ii) two red cards and two black cards.
(iii) one card from each suit.
l
Short Answer Type Questions 9. Three of six vertices of a regular hexagon are
1. Describe the sample space for the experiment of chosen at random, what is the probability that the
tossing a coin four times. triangle with these vertices is equilateral?
2. Three coins are tossed once. Let A denotes the 10. In a relay race, there are five teams A, B, C, D
event ‘‘three heads show’’, B denotes the event and E.
‘‘two heads and one tail show’’, C denotes the event (i) What is the probability that A, B and C finish first,
‘‘three tails show’’ and D denotes the event ‘‘a head second and third, respectively.
shows on the first coin’’. Which pair of events are (ii) What is the probability that A, B and C are first
(i) mutually exclusive events. three to finish (in any order), assume that all
(ii) compound events. finishing orders are equally likely.
3. Suppose that each child born is equally likely to be 11. Six new employees, two of whom are married to
a boy or a girl. Consider a family with exactly three each other, are to be assigned six desks that are
children. lined up in a row. If the assignment of employees to
(i) List the eight elements in the sample space whose desks is made randomly, what is the probability that
outcomes are all possible genders of the three the married couple will not have adjacent desks?
children. 12. A bag contains 8 red and 5 white balls. Three balls
(ii) Write each of the following events as a set and find are drawn at random. Find the probability that
its probability. (i) all the three balls are white.
(a) The event that exactly one child is a girl. (ii) all the three balls are red.
(b) The event that atleast two children are girls. (iii) one ball is red and two balls are white.
(c) The event that no child is a girl. 13. A bag contains 9 red, 7 white and 4 black balls. A
4. A fair coin with 1 marked on one face and 6 marked ball is drawn at random. Find the probability that
on the other and a fair die are both tossed. Find the the ball drawn is
probability that the sum of numbers that turn up is (i) red
(i) 3 (ii) 12. (ii) white
5. A die is thrown. Find (iii) not black
(i) P (a prime number) (ii) P(a number ≥ 3) 14. A coin is tossed and a die is thrown. Find the
(iii) P(a number ≤ 1) probability that the outcome will be a head or
6. Determine the probability p, for each of the a number greater than 4.
following events. 15. An integer is chosen at random from first two
hundred natural numbers. What is the probability
(i) An odd number appears in a single toss of a fair that the integer chosen is divisible by 6 or 8?
die.
16. A basket contains 20 apples and 10 oranges out
(ii) Atleast one head appears in two tosses of a fair
of which 5 apples and 3 oranges are defective.
coin.
If a person takes out 2 at random, then what is
(iii) 4 king, 2 of hearts or 3 of spades appears in the probability that either both are apples or both
drawing a single card from a well shuffled ordinary are good?
deck of 52 cards.
CBSE Term II Mathematics XI 157

17. In an interview for a job in call center, 5 boys and 26. Three numbers are chosen from 1 to 20. Find the
3 girls appeared. If 4 persons are to be selected at probability that they are not consecutive is
random from this group, then find the probability
27. Let A, B and C be three events such that P ( A ) = 0. 3,
that 3 boys and 1 girl or 1 boy and 3 girls are
selected. P (B ) = 0. 4, P (C ) = 0.8, P ( A ∩ B ) = 0. 08,
P (B ∩ C ) = 0.28 and P ( A ∩ B ∩ C ) = 0. 09. If
18. Four cards are drawn at a time from a pack of P( A ∪ B ∪ C ) ≥ 0.8, then show that P ( A ∩ C ) lies in
52 playing cards. Find the probability of getting all
the interval [ 0.23, 0. 43].
the four cards of the same unit.
28. Let A, B and C be three events. If the probability
19. Find the probability that, when a hand of 7 cards is
of occuring exactly one event out of A and B is 1 − x,
drawn from a well-shuffled deck of 52 cards, it
contains out of B and C is 1 − 2 x, out of C and A is 1 − x and
(i) all kings. (ii) 3 kings. (iii) at least 3 kings. that of occuring three events simultaneously is x 2 ,
then prove that the probability that at least one out
20. The accompanying Venn diagram shows three
of A, B and C will occur is greater than 1/2.
events, A, B and C and also the probabilities of the
various intersections [for instance, P( A ∩ B ) = 0. 6]. 29. If A and B are mutually exclusive events, such that
P( A ) = 0. 35 and P(B ) = 0. 45, then find
A B
0.20
(i) P( A ′ )
0.16 0.06
0.10 (ii) P(B′ )
(iii) P( A ∪ B )
0.29
C (iv) P( A ∩ B )
(v) P( A ∩ B ′ )
(vi) P( A′∩ B′ )
Determine
30. A team of medical students doing their internship
(i) P (A) (ii) P (B ∩ C) have to assist during surgeries at a city hospital.
(iii) P (A ∪ B) (iv) P ( A ∩ B ) The probabilities of surgeries rated as very
(v) P (B ∩ C) complex, complex, routine, simple or very simple
(vi) Probability of exactly one of the three occurs. are respectively, 0.15, 0.20, 0.31, 0.26 and 0.08.
Find the probabilities that a particular surgery will
21. Two dice are thrown. Find
be rated
(i) odds in favour of getting the sum 5.
(i) complex or very complex.
(ii) the odd against getting the sum 6. (ii) neither very complex nor very simple.
22. A bag contains 5 white and 7 black balls and a man (iii) routine or complex.
draw 4 balls at random. What are the odds against (iv) routine or simple.
these being all black?
31. A sample space consists of 9 elementary outcomes
23. Two dice are thrown together. What is the
E1 , E 2 , ..., E 9 whose probabilities are
probability that sum of the numbers on the two
faces is neither divisible by 3 nor by 4? P(E1 ) = P(E 2 ) = 0. 08, P (E 3 ) = P(E 4 ) = P(E 5 ) = 0.1,
24. The probability that a student will pass the final
examination in both English and Hindi is 0.5 and P(E 6 ) = P(E7 ) = 0.2 and P(E 8 ) = P(E 9 ) = 0. 07.
the probability of passing neither is 0.1. If the Suppose A = {E1 , E 5 , E 8}, B = {E 2 , E 5 , E 8 , E 9}
probability of passing the English examination is
(i) Calculate P( A), P(B) and P( A ∩ B).
0.75, then what is the probability of passing the
Hindi examination? (ii) Using the addition law of probability, calculate
25. P and Q are two condidates seeking admission in P ( A ∪ B).
NEET. The probability that P is selected is 0.5 and (iii) List the composition of the event A ∪ B and
the probability that both P and Q are selected is at calculate P ( A ∪ B) by adding the probabilities of
most 0.3. Then, find the probability of Q being the elementary outcomes.
selected. (iv) Calculate P ( B ) from P (B), also calculate P ( B )
directly from the elementary outcomes of B.
158 CBSE Term II Mathematics XI

l
Long Answer Type Questions 37. If an integer from 1 through 1000 is chosen at
random, then find the probability that the integer is
32. A die is thrown. Describe the following events a multiple of 2 or a multiple of 9.
(i) A : a number less than 7. 38. A box contains 6 red, 4 white and 5 black balls. A
(ii) B : a number greater than 7. person draws 4 balls from the box at random. Find
(iii) C: a multiple of 3. the probability that among the balls drawn, there is
(iv) D : a number less than 4. atleast one ball of each colour.
(v) E : an even number greater than 4. 39. One of the four persons John, Rita, Aslam and
(vi) F : a number not less than 3. Gurpreet will be promoted next month.
Consequently the sample space consists of four
Also, find A ∪ B , A ∩ B ,B ∪ C ,E ∩ F ,D ∩ E, A − C ,
elementary outcomes S = {John promoted, Rita
D − E,F′ and E ∩ F ′.
promoted, Aslam promoted, Gurpreet promoted}.
33. Two dice are thrown. The events A, B and C are as You are told that the chances of John’s promotion is
follows. same as that of Gurpreet. Rita’s chances of
A : getting an even number on the first die. promotion are twice as likely as Johns. Aslam’s
B : getting an odd number on the first die. chances are four times that of John.
C : getting the sum of the numbers on the dice ≤ 5. (i) Determine
Decribe the events P (John promoted),
(i) A′ P (Rita promoted),
(ii) not B P (Aslam promoted),
(iii) A or B P (Gurpreet promoted).
(iv) A and B (ii) If A = {John promoted or Gurpreet promoted},
(v) A but not C find P ( A)
(vi) B or C 40. A number is chosen at random from the numbers
ranging from 1 to 50. What is the probability that
34. Three coins are tossed once. Find the probability of
the number chosen is a multiple of 2 or 3 or 10?
getting
(i) 3 heads. 41. A card is drawn from a well-shuffled deck of 52
cards. Find
(ii) atleast 2 heads.
(iii) atmost 2 heads. (i) the odds in favour of getting a face card.
(iv) exactly two tails. (ii) the odds against getting a spade.
(v) no tail. 42. Four candidates A, B, C and D have applied for the
35. On vacations, Veena visits four cities (A, B, C assignment to coach a school cricket team. If A is
and D) in a random order. What is the probability twice as likely to be selected as B and B and C are
that she visits given about the same chance of being selected,
(i) A before B?
while C is twice as likely to be selected as D, then
(ii) A before B and B before C? what are the probabilities that
(iii) A first and B last? (i) C will be selected?
(iv) A either first or second? (ii) A will not be selected?
(v) A just before B? 43. If the letters of the word ‘ASSASSINATION’ are
36. A bag contains 8 red, 3 white and 9 blackballs. arranged at random. Find the probability that
If three balls are drawn at random, determine the (i) four S’s come consecutively in the word.
probability that (ii) two I’s and two N’s come together.
(i) all the three balls are of blue colour. (iii) all A’s are not coming together.
(ii) all the balls are of different colours. (iv) no two A’s are coming together.
(iii) one is red and two are white.
CBSE Term II Mathematics XI 159

l
Case Based Questions
44. On her vacation, Sania visits four cities.
Delhi, Lucknow, Agra, Meerut in a
random order.

Meerut New Delhi

Agra Lucknow

On the basis of above information,


answer the following questions.
(i) What is the probability that she visits
Delhi before Lucknow?
(ii) What is the probability she visit Delhi
before Lucknow and Lucknow before
Agra?
(iii) What is the probability she visits Delhi Based on the above information, answer the following
either first or second? (i) Find the probability that the student opted for NCC or NSS.
45. In a class of 60 students, 30 opted for (ii) Find the probability that the student has opted for neither
NCC, 32 opted for NSS and 24 opted for NCC nor NSS.
both NCC and NSS. One of these (iii) Find the probability that the student has opted NSS but not
students is selected at random. NCC.
160 CBSE Term II Mathematics XI

SOLUTIONS
Objective Questions 7. (b) In a non-leap year, there are 365 days which have
1. (c) Let the 2 balls be taken as W for white and R for red. 52 weeks and 1 day. If this day is a Tuesday or Wednesday,
then the year will have 53 Tuesday or 53 Wednesday.
If white ball is drawn, then again another ball is drawn.
1 1 2
So, the possible outcomes are (W, W), (W, R). ∴ Required probability = + =
7 7 7
If the red ball is drawn, then a die is rolled. So, the possible
outcomes are 8. (c) Given word is A S S A S S I N A T I O N
(R, 1), (R, 2), (R, 3), (R, 4), (R, 5), (R, 6). Number of vowels = 3(A)+ 2 (I ) + 1( O) = 6
∴ Sample space is given by Number of consonant = 7
S = {(W, W), (W, R), (R, 1), (R, 2), (R, 3), (R, 4), (R, 5), (R, 6)} Total number of letters = 13
⇒ n (S ) = 8 6 7
(i) P (vowels) = (ii) P (consonant) =
2. (a) Let Q denotes a ` 1 coin, H denotes a ` 2 coin and R 13 13
denotes a ` 5 coin. The first coin, he takes out of his pocket 9. (d) The sample space is S = {HH , HT , TH , TT}
may be anyone of the three coins Q, H or R. Corresponding Let E be the event of getting atleast one tail
to Q, the second draw may be H or R. So, the result of two
E = {HT , TH , TT}
draws may be QH or QR. Similarly, corresponding to H, the
second draw may be Q or R. ∴ Required probability P
Number of favourable outcomes
Therefore, the outcomes may be HQ or HR. Lastly, =
corresponding to R, the second draw may be H or Q. Total number of outcomes
n (E ) 3
So, the outcomes may be RH or RQ. = =
Thus, the sample space is S = {QH, QR, HQ, HR, RH, RQ} n (S ) 4
3. (d) The sample space of the experiment is 10. (a)
S = {HHH, HHT, HTH, THH, HTT, THT, TTH, TTT} Coin
Dice
A = {TTT},
B = {HTT , THT , TTH}
and C = {HHT, HTH, THH, HHH} 1 6 1 2 3 4 5 6

Now, A ∪ B ∪ C = {TTT, HTT, THT, TTH, HHT, HTH, Total number of possible cases in coin = 2
THH, HHH} Number of possible cases in die = 6
=S The sum is 4 only, if 1 turns up on coin and 3 turns up on die
i.e. only possibility is (1, 3).
Therefore, A, B and C are exhaustive events.
Number of favourable outcomes
Also, A ∩ B = φ, A ∩ C = φ and B ∩ C = φ ∴ Required probability =
Total number of outcomes
Therefore, they are mutually exclusive. 1 1
Hence, A, B and C form a set of mutually exclusive and = =
2 × 6 12
exhaustive events.
4. (b) E1 = {( T , T )}, E 2 = {(H , T ), ( T , H )} 11. (c) Given, number of persons = 7

and E 3 = {(H , T ), ( T , H ), (H , H )} Total number of sitting arrangements = 7!


Here, E1 ∩ E 2 = φ,E1 ∩ E 3 = φ and E 2 ∩ E 3 ≠ φ Favourable number of arrangements = 6!
Also, E1 ∪ E 2 ∪ E 3 = {(H , T ), ( T , H ), ( T , T ), (H , H )} = S 2 persons can be arranged in two ways.
E1 , E 2 and E 3 are not mutually exclusive but they are Total number of favourable arrangments = 2( 6!)
exhaustive events. 2( 6!) 2
∴ Required probability = =
5. (d) Let E = The die shows 4 = {4} 7! 7
Let F = The die shows even number = {2 , 4, 6} 12. (b) Probability that the missing cards to be different colours
∴ E ∩ F = {4} ≠ φ
26
C1 × 26 C1
= 52
Also, E ∪ F ≠ {1, 2 , 3, 4, 5, 6} = S C2
Hence, E and F are neither mutually exclusive nor [Q in a pack of cards, there are two colours,
exhaustive. each of them contain 26 cards]
6. (b) Total number of alphabet in the word probability = 11 26 × 26 26
= =
Number of vowels = 4 52 × 51 51
P (letter is vowel) =
4 2 ×1
11
CBSE Term II Mathematics XI 161

13. (b) Total number of ways selecting 4 cards out of 52 cards 18. (a) Let M be the event that Ram passed in Mathematics,
= 52
C4 E be the event that Ram passed in English and M ∩ E be the
event that Ram passed in both subjects.
If E be the event obtaining 3 diamonds and 1 spade, then
Now, P (Ram pass in atleast one subject) = P(M ∪ E )
n(E ) = 13C 3 × 13C1
2 4 1
13
C 3 × 13C1 = P(M ) + P(E ) − P(M ∩ E ) = + −
∴ Required probability = 52
3 9 4
C4 24 + 16 − 9 31
= =
14. (d) 6 numbers out of 20 numbers can be choosed in 20 C 6 36 36
ways in which only one combination of numbers is correct. 19. (d) Let B be the event that a truck stopped at the roadblock
Let the probability of winning the prize be E. will have faulty brakes and T be the event that it will have
badly worn tires.
i.e. n(E ) = 1, [Q only one prize can be won]
We have P(B ) = 0.23, P( T ) = 0.24 and P(B ∪ T ) = 0. 38
Number of elements in sample space n(S) = 20 C 6
and P(B ∪ T ) = P(B ) + P( T ) − P(B ∩ T )
1
∴ Required probability = 20
So 0. 38 = 0.23 + 0.24 − P(B ∩ T )
C6
⇒ P(B ∩ T ) = 0. 09
1  n n!  20. (c) Given that, number of nails = 6
= Q Cr =
20!/ (20 − 6)! 6! ( n − r )! r !  Number of nuts = 10
6× 5× 4× 3×2 ×1 Total number of items = 6 + 10 = 16
=
20 × 19 × 18 × 17 × 16 × 15 and half of them are rusted.
1 ∴Total rusted items = 3 + 5 = 8
= 8 6 3 11
38760 P(the chosen item is rusted or nail) = + − =
P 16 16 16 16
15. (b) Let 3P( A ) = 2P(B ) = P( C ) = P, which gives P( A ) = ,
3 21. Total marbles = 4 + 5 + 3 = 12
P 4×3
P(B ) = and P( C ) = P
2 4
C 2 ×1 1
(i) (c) Required probability = 12 2 = =
Now, since A ,B and C are mutually exclusive and exhaustive C 2 12 × 11 11
events, we have 2 ×1
P( A ) + P(B ) + P( C ) = 1 3
C 1 1
P P (ii) (a) Required probability = 12 3 = =
+ + P =1 C 3 12 × 11 × 10 220
3 2
3×2
6
⇒ P= 7×6
11
P 2
7
C2 2 ×1 21 7
Hence, P( A ) = = (iii) (c) Required probability = 12 = = =
3 11 C2 12 × 11 66 22
16. (c)Q A and B are mutually exclusive events. 2 ×1
⇒ A ∩ B = φ ⇒ P( A ∩ B ) = 0 (iv) (b) Required probability = 1 − P (none is blue)
7 × 6×5
∴ P (A or B) = P (A ∪ B) = P( A ) + P(B ) − P( A ∩ B ) 7
C3 3×2 7 37
3 1 4 = 1 − 12 = 1 − =1− =
= + −0= C3 12 × 11 × 10 44 44
5 5 5
3×2
17. (c) Clearly, the sample space 3
C 3 + 4C 3 1 + 4
(v) (d) Required probability = 12
=
S ={HHH,HHT,HTH,THH, TTH,THT,HTT,TTT} C3 220
Let A be the event of getting atmost two tails and B be the 5 1
= =
event of getting atleast two heads. 220 44
Then, A ={HHH,HHT,HTH, THH,TTH,THT,HTT} 22. Let E1 and E 2 denotes the events that Anil and Vijay will
and B ={HHT,HTH,THH,HHH} respectively qualify the exam.
7 4 4 (i) (a) The probability that Vijay will not qualify the exam
P ( A )= , P (B )= and P ( A ∩ B ) =
8 8 8 = 1 − P(E 2 ) = 1 − 0.10 = 0.9
Now, required probability = P ( A ∪ B ) (ii) (b) P(E1 ∪ E 2 ) = P(E1 ) + P(E 2 ) − P(E1 ∩ E 2 )
= P( A ) + P (B ) − P ( A ∩ B ) = 0.05 + 0.10 − 0.02 = 0.13
7 4 4 7 (iii) (d) Probability of atleast one of them does not qualify
= + − =
8 8 8 8 = P(E′1 ∪ E′ 2 ) = P [(E1 ∩ E 2 )′ ]
162 CBSE Term II Mathematics XI

= 1 − P(E1 ∩ E 2 ) = 1 − 0.02 = 0.98 3. (i) Let B denotes a boy and G denotes a girl. Then, all
(iv) (c) Probability that both Anil and Vijay will not qualify possible genders are expressed as
the exam = P(E′1 ∩ E′ 2 ) = P((E1 ∪ E 2 )′ )
= 1 − P(E1 ∪ E 2 ) = 1 − 0.13 = 0. 87 S = {BBB, BBG, BGB, GBB, BGG,GBG, GGB, GGG}
(v) (b) Probability that only one of them will qualify the (ii) (a) Let E1 denotes the event that exactly one child is
exam a girl.
= P [(E1 − E 2 ) ∪ (E 2 − E1 )] Then, E1 = {BBG, BGB, GBB}
= P(E1 − E 2 ) + P(E 2 − E1 ) ⇒ P (E1) =
3
= P(E1 ∪ E 2 ) − P(E1 ∩ E 2 ) 8
= 0.13 − 0. 02 = 0.11 (b) Let E2 denotes the event that atleast two children
Subjective Questions are girls.
Then, E 2 = {BGG, GBG, GGB, GGG}
1. Let us denote head by H and tail by T, then the possibilities
4 1
are ⇒ P (E2 ) = =
(Fourth toss) 8 2
(Third toss) H, H, H, H (c) Let E 3 denotes the event that no child is a girl.
(Second toss) H, H, H H, H, H, T 1
H, H Then, E3 = {BBB} ⇒ P (E3 ) =
H, H, T H, H, T, H 8
(First toss)
H, H, T, T 4. Let S be a sample space associated with the given
H
H, T , H H, T, H, H experiment.
H, T H, T, H, T Then, S = {(1,1),(1,2 ),(1, 3),(1, 4),(1, 5),(1, 6),
H, T, T, H
H, T, T ( 6,1),( 6,2 ),( 6, 3),( 6, 4),( 6, 5),( 6, 6)} ⇒ n (S) = 12
H, T, T, T
(i) Let A be the event of getting the sum of numbers 3.
(Fourth toss)
Then, A = {(1,2 )}
(Third toss) T, H, H, H
(Second toss) T, H, H ⇒ n (A) = 1
T, H, H, T n (A) 1
T, H T, H, T, H ∴ P(A ) = =
(First toss) T, H, T n (S) 12
T, H, T, T
T
T, T, H T, T, H, H (ii) Let B be the event of getting the sum of numbers 12.
T, T T, T, H, T Then, B = {( 6, 6)}
T, T, T, H ⇒ n (B ) = 1
T, T, T
n (B ) 1
T, T, T, T ∴ P (B ) = =
n (S) 12
Thus, the sample space for this experiment is given by
[{HHHH}, {HHHT}, {HHTH}, {HHTT}, {HTHH}, 5. In throwing a die, the sample space will be
{HTHT}, {HTTH}, {HTTT}, {THHH}, {THHT}, {THTH}, S = {1, 2 , 3, 4, 5, 6}
{THTT}, {TTHH}, {TTHT}, {TTTH}, {TTTT}] ∴ n (S ) = 6
2. When three coins are tossed, then the sample space is (i) Let E1 be the event of getting a prime number.
S = {HHH , HHT , HTH , THH , HTT , THT , TTH , TTT} Then, E1 = {2 , 3, 5}
Given, A = event of getting three heads ∴ n (E1 ) = 3
∴ A = {HHH} Now, probability of getting a prime number,
B = event of getting two heads and one tail n (E1 ) 3 1
P(E1 ) = = =
∴ B = {HHT, HTH, THH} n (S ) 6 2
C = event of getting three tails (ii) Let E 2 be the event of getting a number ≥ 3.
∴ C = {TTT} Then, E 2 = {3, 4, 5, 6}
and D = event of getting a head on first coin. ∴ n (E 2 ) = 4
∴ D = {HHH, HTT, HHT, HTH} Now, probability of getting a number ≥ 3,
(i) Clearly, A ∩ B = φ, A ∩ C = φ, n (E 2 ) 4 2
P (E 2 ) = = =
A ∩ D = {HHH}, B ∩ C = φ, n (S ) 6 3
B ∩ D = {HHT, HTH} and C ∩ D = φ (iii) Let E 3 be the event of getting a number ≤ 1.
∴ We can say A and B, A and C, B and C, C and D are Then, E 3 = {1}
mutually exclusive events.
∴ n (E 3 ) = 1
(ii) B and D are compound event, since they have more
than one sample point. Now, probability of getting a number ≤ 1,
n (E 3 ) 1
P (E 3 ) = =
n (S ) 6
CBSE Term II Mathematics XI 163

6. (i) When a die is thrown the possible outcomes are 9. Let ABCDEF be a regular hexagon.
S = {1, 2, 3, 4, 5, 6} out of which 1, 3, 5 are odd. A
3 1
∴ Required probability = = F B
6 2
(ii) When a fair coin is tossed two times, the sample space is
S = {HH , HT , TH , TT} E C
In atleast one head favourable outcomes are D
HH , HT , TH.
3 Clearly, out of 6 vertices, 3 vertices can be chosen in 6 C 3
∴ Required probability = ways = 20 ways.
4
(iii) Total cards = 52 Thus, total number of triangles so formed = 20
Favourable = 4 king + 2 of heart + 3 of spade [Q no three points are collinear]
=4+1+1=6 Note that, out of these 20 triangles only
6 3 ∆ACE and ∆BDF are equilateral triangles.
∴ Required probability = =
52 26 2 1
∴ Required probability = =
(iv) When a pair of dice is rolled total sample parts are 36. 20 10
Out of which (1, 5), (5, 1), (2, 4), (4, 2) and (3, 3) 10. The sample space consisting of all finishing orders in the
represent the sum as 6. first three places, will have
5 5!
∴ Required probability = 5
P3 i.e. = 5 × 4 × 3 = 60 sample points, each with a
36 ( 5 − 3)!
7. Let E1 denote the event ‘getting number i on the upper face 1
probability of .
of the die’, i = 1, 2 , 3, 4, 5, 6. 60
Clearly, E i , i = 1, 2 , 3, 4, 5, 6 are mutually exclusive and (i) If A, B and C finish first, second and third, respectively,
exhaustive events. then there is only one finishing order for this i.e. ABC.
Let P(E 2 ) = P(E 4 ) = P(E 6 ) = P 1
=
Therefore, P(E1 ) = P(E 3 ) = P(E 5 ) = 2P 60
Now, E1 ∪ E 2 ∪ E 3 ∪ E 4 ∪ E 5 ∪ E 6 = S (ii) If A, B and C are the first three finishers, then there will
⇒ P(E1 ∪ E 2 ∪ E 3 ∪ E 4 ∪ E 5 ∪ E 6 ) = P(S) be 3! arrangements for A, B and C. Thus, the sample
points corresponding to this event will be 3! in number.
⇒ P(E1 ) + P(E 2 ) + P(E 3 ) + P(E 4 ) + P(E 5 ) + P(E 6 ) = 1
Hence, P(A, B and C are first three to finish)
⇒ 2P + P + 2P + P + 2P + P = 1
3! 1
1 = =
⇒ 9P = 1 ⇒ P = 60 10
9
Now, G = E 4 ∪ E 5 ∪ E 6 11. Let the couple occupied adjacent desks consider those two
as 1.
⇒ P( G ) = P(E 4 ∪ E 5 ∪ E 6 )
There are ( 4 + 1 ) i.e. 5 persons to be assigned.
⇒ P( G ) = P(E 4 ) + P(E 5 ) + P(E 6 )
[Q E 4 , E 5 and E 6 are mutually exclusive] ∴ Number of ways of assigning these five persons = 5! × 2 !
4 Total number of ways of assigning 6 persons = 6!
⇒ P( G ) = P + 2P + P = 4P = 5! × 2 !
9 ∴ Probability that the couple has adjacent desk =
6!
8. Total number of possible outcomes = 52
C4
2 1
(i) We know that, there are 12 face cards. = =
6 3
∴ Number of favourable outcomes = 12C 4 Probability that the married couple will have non-adjacent
12
C4 1 2
Hence, P (getting all face cards) = 52
desks = 1 − =
C4 3 3
(ii) We know that, there are 26 red cards and 26 black cards. 12. Q Number of red balls = 8
∴ Number of favourable outcomes = 26 C 2 × 26 C 2 and number of white balls = 5
5!
( 26 C )2
Hence, P (getting 2 red and 2 black cards) = 52 2 5
C3
C4 (i) P (all the three balls are white) = 13 = !2 !
3
C3 13!
(iii) We know that, there are 4 suits each having 13 cards. 3! 10!
∴ Number of favourable outcomes 5! 3! 10!
= ×
= 13C1 × 13C1 × 13C1 × 13C1 = (13)4 3! 2 ! 13!
(13)4 5 × 4 × 3 × 2! 10!
Hence, P (getting one card from each suit) = . = ×
52
C4 2! 13 × 12 × 11 × 10!
164 CBSE Term II Mathematics XI

5×4×3 5 5 15. Let S be the sample space associated with the given random
= = =
13 × 12 × 11 13 × 11 143 experiment.
(ii) P (all the three balls are red) Then, n (S)=200
8! Now, let A be the event that the chosen number is divisible by
8
C3 8! 3! 10! 6 andB be the event that the chosen number is divisible by 8.
= 13 = ! 5! =
3 ×
C3 13! 3! × 5! 13! Then, A = {6,12 ,18,… ,198}
3! 10! and B = {8,16,24 ,… ,192}
8 × 7 × 6 × 5! 10!
= × ⇒ n ( A ) = 33 and n (B )=24
5! 13 × 12 × 11 × 10!
[to find n ( A ) and n (B ), divide 198 by 6 and 192 by 8]
8 ×7 × 6 28
= = Also, A ∩ B = {24,48,72 ,… ,192}
13 × 12 × 11 143
[Q A ∩ B denotes the event that the chosen
(iii) P (one ball is red and two balls are white)
number is divisible by LCM (6,8)]
8
C1 × 5C 2 8 × 10 40
= 13 = = 33 24 8
C3 13 × 2 × 11 143 Now, P( A ) = ; P(B ) = and P ( A ∩ B ) =
200 200 200
13. Total number of balls = 9 red + 7 white + 4 black = 20 ∴ P (chosen integer is divisible by 6 or 8)
⇒ Total number of possible outcomes, n (S) = 20 = P ( A ∪ B ) = P ( A ) + P (B ) − P ( A ∩ B )
(i) Let E1 be the event of getting a red ball. 33 24 8 49
= + − =
Then, n (E1 ) = 9 200 200 200 200
n (E1 ) 9 16. Let S be the sample space associated with the given
Now, P (getting a red ball) = P(E1 ) = =
n (S) 20 experiment.
(ii) Let E 2 be the event of getting a white ball. Then, n (S) = 30 C 2. [Q total number of fruits = 20 + 10 = 30]
Then, n (E 2 ) = 7 Now, let A be the event that both are apples and B be the
n (E 2 ) 7 event that both are good fruits. A ∩ B denotes the event
Now, P (getting a white ball) = P (E 2 ) = =
n (S) 20 that both are good apples.
(iii) Let E 3 be the event of getting a non-black ball. Then, Then, n ( A ) = 20 C 2 and n (B ) = 22C 2.
n (E 3 ) = 9 + 7 = 16 [Q number of good fruits = 15 + 7 = 22]
Now, P (getting a non-black ball) n ( A ∩ B ) = 15C 2
n(E 3 ) 16 4
= P(E 3 ) = = = [Q number of good apples =15]
n(S) 20 5
20 22 15
C2 C2 C2
14. When a coin and a die are thrown, then sample space is ∴ P(A ) = 30
, P (B ) = 30
and P ( A ∩ B ) = 30
C2 C2 C2
S = {H1, H2 , H 3, H 4,H 5, H 6, T1, T2 , T 3, T 4, T 5, T 6}
∴ n (S) = 12 Now, required probability,
Let A : Event of getting a head. P ( A ∪ B ) = P( A ) + P(B ) − P( A ∩ B )
20 22
⇒ A = {H1, H2 , H 3, H 4,H 5, H 6} C C 15 C
= 30 2 + 30 2 − 30 2
∴ n (A) = 6 C2 C2 C2
n (A) 6 190 + 231 − 105 316
Then, P( A ) = = = 30
=
n (S) 12 C2 435
B : Event of getting a number greater than 4. 17. Given, number of boys = 5 and number of girls = 3
⇒ B = {H 5, H 6, T 5, T 6} Let A : Selection of 3 boys and 1 girl
∴ n (B ) = 4 B : Selection of 1 boy and 3 girls
5×4
Then, P(B ) =
n (B ) 4
= ×3
5
C 3 × C1
3
2 ×1 30 3
n (S) 12 ∴ P( A ) = = = =
8
C4 8 × 7 × 6 × 5 70 7
Also, A ∩ B = event of getting a head and a number greater
4× 3×2 ×1
than 4 = {H 5, H 6}
∴ n( A ∩ B ) = 2 C1 × 3C 3
5
5×1 5 1
and P(B ) = = = =
2
8
C4 8 × 7 × 6 × 5 70 14
Then, P( A ∩ B ) = 4× 3×2 ×1
12
Now, required probability, Hence, the required probability,
P( A ∪ B ) = P( A ) + P(B ) − P( A ∩ B ) P ( A ∪ B ) = P( A ) + P(B ) − P( A ∩ B )
3 1 7 1
=
6
+
4

2
=
8 2
= = P( A ) + P(B ) − 0 = + = =
12 12 12 12 3 7 14 14 2
CBSE Term II Mathematics XI 165

18. Let S be the sample space associated with the given random (iii) P( A ∪ B ) = P( A ) + P(B ) − P( A ∩ B )
experiment. Then, n (S) = 52C 4. = 0.22 + 0. 06 + 0.20 + 0.10 − 0. 06 = 0. 52
Now, let A be the event of getting all spade cards, (iv) P( A ∩ B ) = P( A ) − P( A ∩ B )
B be the event getting all club cards, = 0.16 + 0. 06 − 0. 06 = 0.16
C be the event getting all heart cards and (v) P(B ∩ C ) = 0.10
D be the event of getting all diamond cards. (vi) P(exactly one of the three occurs)
Then, n ( A ) = 13C 4, n (B ) = 13C 4, n ( C ) = 13C 4 = 0.16 + 0.20 + 0.29 = 0. 65
and n (D ) = 13C 4 21. Let S be the sample space associated with the given random
13 13 13 experiment. Then, n (S)= 36.
C4 C4 C4
⇒ P( A ) = , P(B ) = , P( C ) =
52
C4 52
C4 52
C4 (i) Let E be the event of getting the sum 5. Then,
13
C4 E ={(1, 4),( 4,1),(2 , 3),( 3,2 )}
and P (D ) = 52
. ⇒ n (E ) = 4
C4
n (E ) 4 1 1 8
Now, required probability = P ( A ∪ B ∪ C ∪ D ) ∴ P (E ) = = = ⇒ P (E ) = 1 − =
n(S) 36 9 9 9
[Q A, B, C and D are mutually exclusive events]
P(E )
13
C 44 Hence, odds in favour of getting the sum 5 =
= 4 × 52 4 = P( E )
C 4 4165 = 1: 8
19. Let S be the sample space associated with the given random (ii) Let F be the event of getting the sum 6.
experiment. Then, n (S) = 52C 7
Then, F ={(1, 5),( 5,1),(2 , 4),( 4,2 ),( 3, 3)} ⇒ n (F ) = 5
(i) Let A be the event that it contains all kings. n (F ) 5 5 31
∴ P (F ) = = ⇒ P (F ) = 1 − =
Then, n( A ) = 4C 4 × 48C 3 n(S) 36 36 36
48 × 47 × 46 P (F )
48
C3 3×2 ×1 Hence, odds against getting the sum 6 =
⇒ P ( A ) = 52 = P (F )
C 7 52 × 51 × 50 × 49 × 48 × 47 × 46 31
7×6× 5× 4× 3×2 ×1 31 36 31
1 = 36 = × =
= [Q 4 C 4 = 1] 5 36 5 5
7735 36
(ii) Let B be the event that it contains 3 kings. = 31 : 5
Then, n (B ) = 4C 3 × 48C 4 22. Given, total number of white balls = 5
48 × 47 × 46 × 45 and total number of black balls = 7

4 × 48C 4 4× 3×2 ×1 ∴Total number of balls = 12
⇒ P(B ) = 52 =
C7 52 × 51 × 50 × 49 × 48 × 47 × 46 Let S be the sample space associated with the given
7×6× 5× 4× 3×2 ×1 experiment. Then, n (S) = 12C 4.
9
= [Q 4 C 3 = 4C1 = 4] Now, let E be the event of getting black balls only.
1547
Then, n (E ) = 7 C 4
(iii) Let C be the event that it contains at least 3 kings.
Then, P ( C ) = P ( A ∪ B ) 7
C4 7× 6× 5× 4 7
⇒ P (E ) = 12 = =
= P ( A ) + P (B ) C 4 12 × 11 × 10 × 9 99
[Q A and B are mutually exclusive] 7 92
1 9 46 ⇒ P( E ) = 1 − P(E ) = 1 − =
= + = 99 99
7735 1547 7735
Hence, odds against getting black balls
20. Given, Ven diagram
92
B P( E ) 99 92
A = = = = 92 : 7
0.20 P(E ) 7 7
0.16 0.06
0.10 99
23. Let S be the sample space. Then, n (S) = 36.
0.29
C Also, let A be the event that sum of numbers of two faces is
divisible by 3 and B be the event that sum of numbers is
divisible by 4 i.e. A be the event of getting the sum 3, 6, 9, 12
(i) P( A ) = 0.16 + 0. 06 = 0.22 and B be the event of getting the sum 4, 8, 12.
(ii) P(B ∩ C ) = P(B ) − P(B ∩ C ) Then,
= 0. 06 + 0.20 + 0.10 − 0.10 A = {(1,2 ),(2 ,1),(2 , 4),( 4,2 ),( 3, 3),(1, 5), ( 5,1),( 4, 5),( 5, 4),
= 0. 06 + 0.20 = 0.26 ( 6, 3),( 3, 6),( 6, 6)}
166 CBSE Term II Mathematics XI

B = {(2 ,2 ),(1, 3),( 3,1),(2 , 6),( 6,2 ), ( 5, 3),( 3, 5),( 4, 4),( 6, 6)} ⇒ 0. 8 − 1. 23 ≤ −P ( A ∩ C ) ≤ 1 − 1. 23
and A ∩ B = {( 6, 6)} [subtracting 1.23 from each terms]
⇒ n ( A ) = 12, n (B ) = 9 and n ( A ∩ B ) = 1 ⇒ − 0. 43 ≤ − P ( A ∩ C ) ≤ −0.23

12
P( A ) = , P(B ) =
9
and P ( A ∩ B ) =
1 ⇒ 0.23 ≤ P ( A ∩ C ) ≤ 0. 43
36 36 36 [multiplying each term by ( −1)]
Now, P ( A ∪ B ) = P ( A ) + P (B ) − P ( A ∩ B) Hence, P ( A ∩ C ) lies in the interval [0.23, 0.43].
12 9 1 20 5 Hence proved.
= + − = =
36 36 36 36 9 28. We have, P( A ) + P(B ) − 2P( A ∩ B ) = 1 − x ...(i)
Now, required probability = P ( A ∩ B )
P(B ) + P( C ) − 2P(B ∩ C ) = 1 − 2 x ...(ii)
5 4
= P(A ∪ B) = 1 − P(A ∪B) = 1 − = P( C ) + P( A ) − 2P( C ∩ A ) = 1 − x ...(iii)
9 9
and P( A ∩ B ∩ C ) = x 2 ...(iv)
24. Let events H and E denote the students that passing Hindi
and English examination, respectively. On adding Eqs. (i), (ii) and (iii), we get
Given, probability of passing both examinations P( A ) + P(B ) + P( C ) − P( A ∩ B ) − P(B ∩ C )
= P(H ∩ E ) = 0. 5 3 − 4x
−P( C ∩ A ) =
and probability of passing neither examinations 2
= P(H′∩E′ ) = P(H ∪ E )′ = 01 3 − 4x 2 3
. Now, P( A ∪ B ∪ C ) = + x = x2 − 2 x +
Then, P(H ∪ E ) = 1 − P(H ∪ E )′ = 1 − 0.1 = 0.9 2 2
1 1
Also, it is given, that probability of passing English = ( x − 1) 2 + >
examination, 2 2
P(E ) = 0.75 29. We have, A and B are mutually exclusive events.
Now, consider P(H ∪ E ) = P(H ) + P(E ) − P(H ∩ E ) Also, it is given that P ( A ) = 0. 35 and P (B ) = 0. 45
⇒ 0. 9 = P(H ) + 0.75 − 0.5 (i) P ( A′ ) = 1 − P ( A ) = 1 − 0. 35 = 0. 65
⇒ 0. 9 = P(H ) + 0.25 (ii) P (B′ ) = 1 − P (B ) = 1 − 0. 45 = 0. 55
(iii) P ( A ∪ B ) = P ( A ) + P (B ) = 0. 35 + 0. 45 = 0. 8
⇒ P(H ) = 0.9 − 0.25 = 0.65
[Q A and B are mutually exclusive]
Hence, probability of passing Hindi examination is 0.65.
(iv) P ( A ∩ B ) = 0
25. Let A1 and A 2 be two events defined as follows
[Q A and B are mutually exclusive, therefore A ∩ B = φ]
A1 = P is selected, A 2 = Q is selected
We have, P( A1 ) = 0.5 and P( A1 ∩ A 2 ) ≤ 0.3 (v) P ( A ∩ B′ ) = P ( A ) − P ( A ∩ B )
= P ( A ) − 0 = P( A ) = 0. 35
Now, P( A1 ∪ A 2 ) ≤ 1
(vi) P ( A′ ∩ B′ ) = P [( A ∪ B )′ ] = 1 − P ( A ∪ B ) = 1 − 0. 8 = 0.2
⇒ P( A1 ) + P( A 2 ) − P( A1 ∩ A 2 ) ≤ 1
30. Let E1, E 2, E 3, E 4 and E 5 be the event that surgeries are
⇒ P( A 2 ) ≤ 0.5 + P( A1 ∩ A 2 )
rated as very complex, complex, routine, simple or very
⇒ P( A 2 ) ≤ 0. 8 simple, respectively.
26. Three numbers chosen from ∴ P (E1 ) = 0. 15, P (E 2 ) = 0. 20, P (E 3 ) = 0. 31,
20 × 19 × 18 × 17 !
1 to 20 = 20 C 3 = P (E 4 ) = 0. 26, P (E 5 ) = 0. 08
17 ! × 3 × 2 × 1
(i) P (complex or very complex) = P (E 2 or E1 )= P(E1 or E 2 )
= 20 × 19 × 3 = 1140
= P (E1 ∪ E 2 ) = P (E1 ) + P (E 2 ) − P (E1 ∩ E 2 )
Now, set of three consecutive numbers = 18
= 0.15 + 0.20 − 0 [P (E1 ∩ E 2 ) = 0]
18 3
P(they are consecutive) = = [because all events are independent]
1140 190
= 0. 35
3 187
P(they are not consecutive) = 1 − = (ii) P (neither very complex nor very simple),
190 190 P (E1′ ∩ E′5 ) = P (E1 ∪ E 5 )′
27. We have, P ( A ∪ B ∪ C ) ≥ 0. 8 = 1 − P (E1 ∪ E 5 )
∴ 0.8 ≤ P ( A ∪ B ∪ C ) ≤ 1 = 1 − [P (E1 ) + P (E 5 )]
[Q probability of occurrence of an event = 1 − (0.15 + 0.08)
is always less than or equal to 1] = 1 − 0.23
⇒ 0. 8 ≤ P ( A ) + P (B ) + P ( C ) − P( A ∩ B ) − P (B ∩ C ) = 0.77
−P ( A ∩ C) + P ( A ∩ B ∩ C) ≤ 1 (iii) P (routine or complex) = P (E 3 ∪ E 2 ) = P (E 3 ) + P (E 2 )
⇒ 0. 8 ≤ 0. 3 + 0. 4 + 0. 8 − 0. 08 − 0.28 = 0. 31 + 0. 20 = 0. 51
− P ( A ∩ C ) + 0. 09 ≤ 1 (iv) P (routine or simple) = P (E 3 ∪ E 4 ) = P (E 3 ) + P (E 4 )
⇒ 0. 8 ≤ 1.23 − P ( A ∩ C ) ≤ 1 = 0. 31 + 0. 26 = 0. 57
CBSE Term II Mathematics XI 167

31. Given, S = {E1, E 2, E 3, E 4,E 5, E 6, E 7, E 8, E 9} (2 ,1),(2 ,2 ),… ,(2 ,6),  (1,1),(1,2 ),… ,(1, 6), 
   
A = {E1, E 5, E 8}, B = {E 2, E 5, E 8, E 9 } 33. ( i) A = ( 4,1),( 4,2 ),… ,( 4, 6),, B = ( 3,1),( 3,2 ),… ,( 3, 6),
( 6,1),( 6,2 ),… ,( 6,6)  ( 5,1),( 5,2 ),… ,( 5, 6) 
P (E1 ) = P (E 2 ) = 0. 08    
P (E 3 ) = P (E 4) = P (E 5 ) = 0.1 and C = {( 1 , 1 ),( 1 , 2 ),( 2 , 1 ),( 1 ,3 ),( 3 , 1 ),
P (E 6 ) = P (E 7 ) = 0.2, P (E 8 ) = P (E 9 ) = 0. 07 (2 ,2 ),(2 , 3),( 3,2 ),(1,4),( 4,1)}
(i) P ( A ) = P (E1 ) + P (E 5 ) + P (E 8 ) (1,1),(1,2 ),… , (1, 6), 
 
= 0. 08 + 0. 1 + 0. 07 = 0. 25 (i) A′ = ( 3,1),( 3,2 ),… ,( 3, 6), =B
P (B ) = P (E 2 ) + P (E 5 ) + P (E 8 ) + P (E 9 ) ( 5,1),( 5,2 ),… ,( 5, 6) 
 
= 0. 08 + 0. 1 + 0. 07 + 0. 07 = 0. 32 (ii) B′ = {(2 ,1),(2 ,2 ),(2 ,3),(2 ,4),(2 ,5),
A ∩ B = {E 5 ,E 8} (2 ,6),( 4,1),( 4,2 ),( 4,3),( 4,4),( 4,5),( 4,6),
P ( A ∩ B ) = P (E 5 ) + P (E 8 ) = 0. 1 + 0. 07 = 0. 17 ( 6,1),( 6,2 ),( 6,3),( 6,4),( 6,5),( 6,6)} = A
(ii) P( A ∪ B ) = P( A ) + P(B ) − P( A ∩ B ) (iii) A ∪ B = {(1,1),(1,2 ),(1,3),(1,4),(1,5),
= 0.25 + 0. 32 − 0.17 = 0. 40 (1,6),( 3,1),( 3,2 ),( 3,3),( 3,4),( 3,5),( 3,6),
(iii) A ∪ B = {E1 ,E 2 , E 5 , E 8 , E 9 } ( 5,1),( 5,2 ),( 5,3),( 5,4),( 5,5),( 5,6),(2 ,1),
P ( A ∪ B ) = P (E1 ) + P (E 2 ) + P (E 5 ) + P (E 8 ) + P (E 9 ) (2 ,2 ),(2 ,3), (2 , 4), (2 ,5),(2 ,6),( 4,1),( 4,2 ),( 4,3),
( 4,4),( 4,5),( 4,6),( 6,1),( 6,2 ),( 6,3),( 6,4),( 6,5),( 6,6)} = S
= 0. 08 + 0. 08 + 0. 1 + 0. 07 + 0. 07 = 0. 40
(iv) A ∩ B = φ
(iv) Q P ( B ) = 1 − P (B ) = 1 − 0. 32 = 0. 68
(v) A − C = {(2 ,4),(2 ,5),(2 ,6),( 4,2 ),( 4,3),
and B = {E1 , E 3 , E 4 , E 6 , E 7 } ( 4,4),( 4,5),( 4,6),( 6,1),( 6,2 ),( 6,3),( 6,4),( 6,5),( 6,6)}
∴P ( B ) = P (E1 ) + P (E 3 ) + P (E 4 ) + P (E 6 ) + P (E 7 ) (vi) B ∪ C = {(1,1),(1,2 ),(1,3),(1,4),(1,5),(1,6),
= 0. 08 + 0. 1 + 0. 1 + 0. 2 + 0. 2 = 0. 68 (2 ,1),(2 ,2 ),(2 ,3),( 3,1),( 3,2 ),( 3,3),( 3,4),( 3,5),( 3,6),
32. When a die is thrown, then sample space ( 4,1),( 5,1),( 5,2 ), ( 5,3),( 5,4),( 5,5),( 5,6)}
S = {1, 2 , 3, 4, 5, 6} 34. In random experiment of tossing three coins, the sample
(i) A : a number less than 7 = {1, 2 , 3, 4, 5, 6} space is
(ii) B : a number greater than 7 = { } = φ S = {HHH , HHT , HTH , THH , HTT , THT , TTH , TTT}
(iii) C: a multiple of 3 = {3, 6} ⇒ n (S ) = 8
(iv) D : a number less than 4 = {1, 2 , 3} (i) Let E1 be the event of getting 3 heads. Then, outcomes
(v) E : an even number greater than 4 = {6} favourable to E1 is {HHH}.
(vi) F : a number not less than 3 = {3, 4, 5, 6} Thus, n (E1 ) = 1,
Now, A ∪ B = The elements which are in A or B or both n (E1 ) 1
∴ P (E1 ) = =
= {1, 2 , 3, 4, 5, 6} ∪ φ n (S ) 8
= {1, 2 , 3, 4, 5, 6} (ii) Let E 2 be the event of getting atleast 2 heads.
A ∩ B = The elements which are common in both Then, outcomes favourable to E 2 are {HHT, HTH,
A and B = {1, 2 , 3, 4, 5, 6} ∩ φ = φ THH and HHH}.
B ∪ C = The elements which are in B or C or both Thus, n (E 2 ) = 4
= { } ∪ {3, 6} = {3, 6} n (E 2 ) 4 1
∴ P(E 2 ) = = =
E ∩ F = The elements which are common in both n (S ) 8 2
E and F = {6} ∩ {3, 4, 5, 6} = {6} (iii) Let E 3 be the event of getting atmost 2 tails. Then,
D ∩ E = The elements which are common in both outcomes favourable to E 3 are {HHH, HHT, HTH,
THH, TTH, THT, HTT}.
D and E = {1, 2 , 3} ∩ {6} = φ
Thus, n (E 3 ) = 7
A − C = The elements which are in A but not in C
n (E 3 ) 7
= {1, 2 , 3, 4, 5, 6} − {3, 6} ∴ P (E 3 ) = =
n (S ) 8
= {1, 2 , 4, 5}
(iv) Let E 4 be the event of getting exactly two tails. Then,
D − E = The elements which are in D but not in E
outcomes favourable to E 4 are {TTH, HTT, THT}.
= {1, 2 , 3} − {6}
Thus, n (E 4 ) = 3
= {1, 2 , 3} n (E 4 ) 3
F′ = The elements which are not in F ∴ P (E 4 ) = =
n (S ) 8
= (S − F ) (v) Let E 5 be the event of getting no tail. Then, outcomes
= {1, 2 , 3, 4, 5, 6} − {3, 4, 5, 6} = {1, 2} favourable to E 5 is {HHH}.
and E ∩ F′ = E ∩ (S − F ) Thus, n (E 5 ) = 1
= E ∩ ({1, 2 , 3, 4, 5, 6} − {3, 4, 5, 6}) n (E 5 ) 1
∴ P(E 5 ) = =
= {6} ∩ {1, 2} = φ n (S ) 8
168 CBSE Term II Mathematics XI

35. Number of ways, in which Veena can visit four cities A, B, C (ii) Let E 2 be the event of getting all balls of different
and D, is 4! i.e. 24. colours. Then,
∴ n (S) = 24 n (E 2 ) = 8C1 × 3C1 × 9 C1
Clearly, sample space for this experiment is =8×3×9
ABCD, ABDC, ACBD, ACDB, ADBC, ADCB , 8×3×9
Hence, required probability =
BACD, BADC, BCAD, BCDA, BDAC, BDCA ,  20
C3
S= 
8×3×9×6
CABD, CADB, CBAD, CBDA, CDAB, CDBA,  =
DABC, DACB, DCAB, DCBA, DBAC, DBCA  20 × 19 × 18
3 × 6 18
(i) Let E1 be the event that Veena visits A before B. Then, = =
ABCD, ABDC, ACBD, ACDB, ADBC, ADCB , 95 95
E1 =   (iii) Let E 3 be the event of getting one red and two white
CABD, CADB, CDAB, DABC, DACB, DCAB 
balls. Then,
⇒ n (E1 ) = 12
n (E1 ) 12 1 n(E 3 ) = 8C1 × 3C 2 = 8 × 3
∴ P(she visits A before B) = P (E1 ) = = =
n (S) 24 2 Hence, required probability
8×3 8×3 2
(ii) Let E 2 be the event that she visits A before B and B = 20 = =
before C. C3 20 × 19 × 18 95
Then, E 2 = {ABCD, ABDC, ADBC, DABC} 3 × 2 ×1
⇒ n (E 2 ) = 4 37. Multiple of 2 from 1 to 1000 are 2, 4, 6, 8, ..., 1000
∴ P (she visits A before B and B before C) = P (E 2 ) Let n be the number of terms of above series.
n (E 2 )
= ∴ n th term = 1000
n (S )
⇒ 2 + ( n − 1) 2 = 1000
4 1
= = ⇒ 2 + 2 n − 2 = 1000
24 6
(iii) Let E 3 be the event that she visits A first and B last. ⇒ 2 n = 1000
Then, E 3 = {ACDB, ADCB} ⇒ n (E 3 ) = 2 ∴ n = 500
∴ P (she visits A first and B last) = P (E 3 ) Since, the number of multiple of 2 are 500.
n (E 3 ) 2 1 So, the multiple of 9 are 9, 18, 27, ..., 999
= = =
n (S) 24 12 Let m be the number of term in above series.
(iv) Let E 4 be the event that she visits A either first or ∴ m th term = 999
second. Then, ⇒ 9 + ( m − 1) 9 = 999
ABCD, ABDC, ACBD, ACDB, ADBC, ADCB , ⇒ 9 + 9 m − 9 = 999
E4 =  
BACD, BADC, CABD, CADB, DABC, DACB  ⇒ 9 m = 999
⇒ n (E 4 ) = 12 ∴ m = 111
Hence, P (she visits A either first or second) Since, the number of multiple of 9 are 111. So, the multiple
n (E 4 ) 12 1 of 2 and 9 both are 18, 36, ..., 990
= P (E 4 ) = = =
n (S) 24 2 Let p be the number of terms in above series.
(v) Let E 5 be the event that she visits A just before B. ∴ p th term = 990
Then, ⇒ 18 + ( p − 1)18 = 990
E 5 ={ABCD, ABDC, CABD, CDAB, DABC, DCAB} ⇒ 18 + 18p − 18 = 990
⇒ n (E 5 ) = 6 ⇒ 18p = 990
Hence, P (she visits A just before B) 990
n (E 5 ) 6 1 ∴ p= = 55
= P (E 5 ) = = = 18
n (S) 24 4
Since, the number of multiple of 2 and 9 are 55.
36. Total number of balls = 8 + 3 + 9 = 20 ∴ Number of multiple of 2 or 9 = 500 + 111 − 55 = 556
Number of ways of selecting 3 balls out of 20 balls = 20
C3 n (E ) 556
∴ Required probability = = = 0. 556
⇒ Total number of possible outcomes = 20
C3 n (S) 1000
(i) Let E1 be the event of getting all balls of blue colour. 38. Given, total number of red balls = 6
Then, n (E1 ) = 0 Total number of white balls = 4
Hence, required probability = 20
0
=0 and total number of black balls = 5
C3 ∴Total number of balls = 15
CBSE Term II Mathematics XI 169

Let S be the sample space associated with the random 1 1


= + −0 [QP (E1 ∩ E 4 ) = 0 ]
experiment. Then, n (S) = 15C 4 = 1365 8 8
2 1
Here, atleast one ball of each colour can be drawn in one of = =
the following ways 8 4
(i) 1 red, 1 white and 2 black balls 40. Let S be the sample space associated with the given random
(ii) 2 red, 1 white and 1 black balls experiment. Then, n (S)= 50.
(iii) 1 red, 2 white and 1 black balls Now, let A be the event that the chosen number is a multiple
of 2, B be the event that the chosen number is a multiple of
Now, let A be the event of getting 1 red, 1 white and 2 black
3 and C be the event that the chosen number is a multiple
balls, B be the event of getting 2 red, 1 white and 1 black
of 10.
balls and C be the event of getting 1 red, 2 white and 1 black
balls. Then, A = {2 , 4, 6 ,… , 48, 50},
Then, n ( A ) = 6 C1 × 4C1 × 5C 2 = 6 × 4 × 10 = 240 B = {3, 6, 9… , 48} and C = {10,20,… , 50}
A ∩ B denotes the event that chosen number is multiple of 6
n (B ) = 6 C 2 × 4C1 × 5C1 = 15 × 4 × 5 = 300
= LCM (2 , 3)
and n ( C ) = 6 C1 × 4C 2 × 5C1 = 6 × 6 × 5 = 180 B ∩ C denotes the event that chosen number is a multiple of
240 300 180 30 = LCM (3, 10)
∴ P(A ) = ; P (B ) = and P ( C ) =
1365 1365 1365 A ∩ C denotes the event that chosen number is a multiple of
Now, required probability 10 = LCM (2, 10)
= P ( A ∪ B ∪ C ) = P ( A ) + P (B ) + P ( C ) and A ∩ B ∩ C denotes the event that chosen number is a
multiple of 30 = LCM (2, 3, 10)
[Q A, B and C are mutually exclusive events]
∴ A ∩ B = {6,12 ,… , 48},
240 300 180 720 48
= + + = = B ∩ C = {30}, A ∩ C = {10, 20… , 50}
1365 1365 1365 1365 91
and A ∩ B ∩ C = {30}
39. Let E1 = John promoted
⇒ n ( A ) = 25, n (B ) = 16, n ( C ) = 5, n ( A ∩ B ) = 8,
E 2 = Rita promoted
n (B ∩ C ) = 1, n ( A ∩ C ) = 5 and n( A ∩ B ∩ C ) = 1
E 3 = Aslam promoted
n ( A ) 25
E 4 = Gurpreet promoted ∴ P(A ) = =
n (S) 50
Given, sample space, S = {John promoted, Rita promoted,
n (B ) 16
Aslam promoted, Gurpreet promoted} P (B ) = =
i.e. S = {E1 , E 2 , E 3 , E 4 } n (S) 50
n ( C) 5
It is given that, chances of John’s promotion is same as that P ( C) = =
of Gurpreet. n (S) 50
P (E1 ) = P (E 4 ) n (A ∩ B) 8
P(A ∩B) = = ,
Rita’s chances of promotion are twice as likely as John. n (S ) 50
P (E 2 ) = 2P (E1 ) n (B ∩ C ) 1
P (B ∩ C ) = = ,
And Aslam’s chances of promotion are four times that of n (S ) 50
John.
n ( A ∩ C) 5
P (E 3 ) = 4P (E1 ) P ( A ∩ C) = =
n (S ) 50
Now, P (E1 ) + P (E 2 ) + P (E 3 ) + P (E 4 ) = 1
n ( A ∩ B ∩ C) 1
⇒ P (E1 ) + 2P (E1 ) + 4P (E1 ) + P (E1 ) = 1 and P ( A ∩ B ∩ C ) = =
⇒ 8P (E1 ) = 1 n (S ) 50
1 Now, required probability
∴ P(E1 ) =
8 = P ( A ∪ B ∪ C)
1 = P ( A ) + P(B ) + P( C ) − P ( A ∩ B )
(i) P (John promoted) = P (E1 ) =
8 − P (B ∩ C ) − P( A ∩ C ) + P( A ∩ B ∩ C )
1 2 1
P (Rita promoted) = P (E 2 ) = 2P (E1 ) = 2 × = = 25 16 5 8 1 5 1 25 + 8 33
8 8 4 = + + − − − + = =
50 50 50 50 50 50 50 50 50
1 1
P (Aslam promoted) = P (E 3 ) = 4P (E1 ) = 4 × = 33
8 2 Hence, required probability is .
50
1
P (Gurpreet promoted) = P (E 4 ) = P (E1 ) = 41. Let S be the sample space associated with the given random
8
experiment. Then, n (S)= 52.
(ii) A = John promoted or Gurpreet promoted
(i) Let E be the event of getting of face card.
A = E1 ∪ E 4
Then, n (E ) = 12
P ( A ) = P (E1 ∪ E 4 ) = P (E1 ) + P (E 4 ) − P (E1 ∩ E 4 )
170 CBSE Term II Mathematics XI

n(E ) 12 3 S S S S A A A I I N N T O
∴ P (E ) = = =
n(S) 52 13
1 9
3 10
⇒ P ( E ) = 1 − P (E ) = 1 − =
13 13 10!
Number of words when all S’s are together =
Hence, odds in favour of getting a face card 3! 2 ! 2 !
3 Total number of word using letters of the word
P (E ) 13 3 13!
= = = = 3: 10 ‘ASSASSINATION’ =
P ( E ) 10 10 3! 4! 2 ! 2 !
13 10!
∴Required probability =
(ii) Let F be the event of getting a spade. Then, n (F )=13. 3! 2 ! 2 ! × 13!
n(F ) 13 1 3! 4! 2 ! 2 !
∴ P (F ) = = = 10! × 4! 4!
n(S) 52 4 = =
13! 13 × 12 × 11
1 3
⇒ P ( F ) = 1 − P(F ) = 1 − = 24 2
4 4 = =
1716 143
Hence, odds against of getting a spade
3 (ii) If 2 I’s and 2 N’s come together, then there as
P (F ) 4 3 10 alphabets.
= = = = 3:1 Number of word when 2 I’s and 2 N’s come together
P (F) 1 1
10! 4!
4 = ×
42. It is given that A is twice as likely to be selected as B. 3! 4! 2 ! 2 !
10! 4!
P ( A ) = 2 P (B )
P (A) ∴ Required probability = 4! 2 ! 2 !
3 !
⇒ = P (B ) 13!
2 3! 4! 2 ! 2 !
while C is twice as likely to be selected as D. 4! 10! 3! 4! 2 ! 2 !
= ×
P ( C ) = 2 P (D ) ⇒ P (B ) = 2 P (D ) 2 ! 2 ! 3! 4! 13!
[Q B and C are given about the same chance 4! 10! 4!
of being selected] = =
13! 13 × 12 × 11
P (A) P (A)
⇒ = 2 P (D ) ⇒ P (D ) = 24 2
2 4 = =
13 × 12 × 11 143
P (B ) = P ( C )
(iii) If all A’s are coming together, then there are
Now, sum of probability = 1 11 alphabets.
P ( A ) + P (B ) + P ( C ) + P (D ) = 1 Number of words when all A’s come together
P (A) P (A) P (A) 11!
P (A) + + + =1 =
2 2 4 4! 2 ! 2 !
4P ( A ) + 2P ( A ) + 2P ( A ) + P ( A ) Probability when all A’s come together
⇒ =1
4 11!
4
⇒ 9 P (A) = 4 ⇒ P (A) = = 4 ! 2 ! 2 ! = 11! × 4! 3! 2 ! 2 !
9 13! 4! 2 ! 2 ! 13!
P (A)
(i) P (C will be selected) = P ( C ) = P (B ) = 4! 3! 2 ! 2 !
2 11! × 3! 6 1
= = =
4  9 13! 13 × 12 26
= Q P (A) = 
9×2  4
Required probability when all A’s does not come
2 together
=
9 1 25
=1− =
4 5 26 26
(ii) P (A will not be selected) = P( A′ ) = 1 − P( A ) = 1 − =
9 9 (iv) If no two A’s are together, then first we arrange the
43. Total number of letters in the word ‘ASSASSINATION’ alphabets except A’s.
are 13.
S S S S I N T I O N
Out of which 3A’s, 4S’s, 2 I’s, 2 N’s, 1 T and 1O.
(i) If four S’s come consecutively in the word, then we 10!
All the alphabets except A’s are arranged in .
considers these 4 S’s as 1 group. 4! 2 ! 2 !
Now, the number of laters is 10. There are 11 vacant places between these alphabets.
CBSE Term II Mathematics XI 171

11! (iii) (c) Let E 4 be the event that she visits Delhi (D) either
So, 3 A’s can be place in 11 places in 11 C 3 ways =
3! 8! first or second. Then,
∴ Total number of words when no two A’s together
11! 10! DLAM, DLMA, DALM, DAML, DMLA, DMAL ,
= × E4 =  
3! 8! 4! 2 ! 2 ! LDAM, LDMA, ADLM, ADML, MDLA, MDAL 
11! × 10! 4! 3! 2 ! 2 !
Required probability = ×
3! 8! 4! 2 ! 2 ! 13! ⇒ n (E 4 ) = 12
10! Hence, P (she visits Delhi (D) either first or second)
=
8! × 13 × 12 n (E 4 ) 12 1
= P (E 4 ) = = =
10 × 9 90 15 n (S) 24 2
= = =
13 × 12 156 26 45. Let A and B denote the events that, the selected students
44. Let the Sania visits four cities Delhi, Lucknow, Agra, Meerut opted NCC and NSS, respectively.
are respectively D , L , A and M. Number of way’s in which Given, n ( A ) = 30, n (B ) = 32
Sania can visit four cities D , L , A and M is 4! i.e. 24. n ( A ∩ B ) = 24, n (S) = 60
∴ n(S) = 24 [Q 24 students opted for both NCC and
Clearly, sample space for this experiment is NSS i.e. they are common in both]
n ( A ) 30 n (B ) 32
∴ P(A ) = = , P (B ) = =
DLAM, DLMA, DALM, DAML, DMLA, DMAL , n (S) 60 n (S) 60
LDAM, LDMA, LADM, LAMD, LMDA, LMAD ,
S= n ( A ∩ B ) 24
 and P( A ∩ B ) = =
ADLM, ADML, ALDM, ALMD, AMDL, AMLD, n (S ) 60
MDLA, MDAL, MLDA, MLAD, MADL, MALD  (i) P(student opted for NCC or NSS)
(i) (a) Let E1 be the event that Sania visits Delhi (D ) = P ( A ∪ B ) = P ( A ) + P (B ) − P ( A ∩ B )
before Lucknow (L ). 30 32 24
= + −
Then, 60 60 60
DLAM, DLMA, DALM, DAML, DMLA, DMAL , 30 + 32 − 24 62 − 24
E1 =   = =
ADLM, ADML, AMDL, MDLA, MDAL, MADL  60 60
⇒ n (E1 ) = 12 38 19
= =
n (E1 ) 12 1 60 30
∴ P (she visits D before L) = P (E1 ) = = =
n (S) 24 2 (ii) P(student opted neither NCC nor NSS)
(ii) (c) Let E 2 be the event that she visits D before L and L = P(A ∩ B ) = P(A ∪ B)
before A. = 1 − P (A ∪ B)
Then, 19 30 − 19 11
=1− = =
E 2 = {DLAM , DLMA , DMLA , MDLA} 30 30 30
⇒ n (E 2 ) = 4 (iii) P(student has opted NSS but not NCC)
∴ P (she visits D before L and L before A) = P (E 2 ) = P(B ∩ A ) = P(B ) − P( A ∩ B )
n (E 2 ) 4 1 32 24 8 2
= = = = − = =
n (S) 24 6 60 60 60 15
Chapter Test (ii) What is the probability that Raju getting two
red cards and two black card.
( 13 C 2)2 26
C4 (26C 2) 2 (26C 4)2
(a) (b) (c) (d)
l
Multiple Choice Questions 52
C4 52
C4 52
C4 52
C4
1. One die of red colour(R), one of white colour(W) (iii) What is the probability that Raju getting one
and one of blue colour(B) are placed in a bag. card from each suit.
One die is selected at random and rolled, its 13
C4 (13) 4
colour and the number on its uppermost face is (a) (b)
52 52
C4 C4
noted. Then, the sample space is
R1. R 2, R 3, R 4, R 5, R6  ( 13 C 4)2
(c) (d) None of these
  52
C4
(a) S = W 1, W 2. W 3, W 4, W 5, W 6.
B1. B 2, B 3, B 4, B 5, B6  (iv) What is the probability that Raju getting all
 
R1, R 2 R 3, R 4, R 5 R6 king cards.
  1 2 4 6
(b) S = W 1, W 2, W 4, W 5  (a) (b) (c) (d)
B1. B 3, B 4, B 5 B6  52
C4 52
C4 52
C4 52
C4
 
(c) Both (a) and (b) (v) What is the probability that Raju getting two
(d) None of the above king and two Jack cards.
4
2. An experiment is called random experiment, if C2 36
(a) (b)
52 52
it …X… . Here, X refers to C4 C4
(a) has more than one possible outcome 6
(c) (d) None of these
(b) is not possible to predict the outcome in advance 52
C4
(c) Both (a) and (b)
(d) None of the above l
Short Answer Type Questions
3. 6 boys and 6 girls sit in a row randomly.
The probability that all 6 girls sit together is 5. The numbers 1, 2, 3 and 4 are written separately
1 1 1 1 on the four slips of paper, the slips are put in a
(a) (b) (c) (d) box and mixed thoroughly. A person draw two
130 131 132 62
slips from the box, one after the other, without
l
Case Based MCQs replacement. Describe the sample space for
the experiment.
4. Four friends Daksh, Yash, Sourabh, and Raju are
playing cards. Daksh, shuffling a cards and told 6. An experiment involves rolling a pair of dice
to Raju choose any four cards. and recording the numbers that comes up.
Describe the following events.
A : the sum is greater than 8.
B : 2 occurs on either die.
C : the sum is at least 7 and multiple of 3.
Which pairs of these events are mutually
exclusive?
l
Long Answer Type Questions
7. An urn contains 7 white, 5 black and 3 red balls.
Two balls are drawn at random. Find the
probability that
On the basis of above information, answer the
following questions. (i) both the balls are red.
(i) What is the probability that Raju getting all (ii) one ball is red, other is black.
face card. (iii) one ball is white.
12 16
(a)
C4
(b)
C4 8. A box contains 100 bolts and 50 nuts. It is
52 52
C4 C4 given that 50% bolts and 50% nuts are rusted.
( 12 C 2)2 Two objects are selected from the box at
(c) (d) None of these
52
C4 random. Find the probability that either both
are bolts or both are rusted.
Answers
1. (a) 2. (c) 3. (c) 4. (i) (a) (ii) (c) (iii) (b) (iv) (a) (v) (b) For Detailed Solutions
 (1, 2), (1, 3), (1, 4), (2, 1), (2, 3), (2, 4), Scan the code
5. S =   8. 0.58
 (3, 1), (3, 2), (3, 4), (4, 1), (4, 2), (4, 3) 
CBSE Term II Mathematics XI 173
Mathem atics
Class 11th ( Term II)

Practice Paper 1 *

(Solved)
Instructions Time : 2 Hr
Max. Marks : 40
1. The question paper contains three sections A, B and C.
2. Section A has 5 questions with 3 internal choices.
3. Section B has 4 questions with 3 internal choices.
4. Section C has 1 Case Based MCQs comprises of 5 MCQs.
5. There is no negative marking.
* As exact Blue-print and Pattern for CBSE Term II exams is not released yet. So the pattern of this
paper is designed by the author on the basis of trend of past CBSE Papers. Students are advised
not to consider the pattern of this paper as official, it is just for practice purpose.

Section A (3 Marks Each)


This section consists of 5 questions of Short Answer Type.
1. Prove that,
π 3π 5π 7π 3
cos 4 + cos 4 + cos 4 + cos 4 = .
8 8 8 8 2
Or If cos θ = cos α cos β, then prove that
θ +α θ −α β
tan tan = tan 2 .
2 2 2
x+6
2. Solve > 0 and draw the graph of the solution set.
x−4
Or A milkman has 80% milk in his stock of 800 L of adulterated milk. How much 100% pure milk is added to it,
so that purity is between 90% and 95%?
n
C × rC x n
3. Prove that n −rx = Cx.
C r −x
4. In a single throw of three dice, determine the probability of getting
(i) total of atmost 5. (ii) a total of atleast 5.
5. Find the equation of the circle passing through the points (4, 1) and (6, 5) whose centre is on the line
4x + y = 16.
Or A rod of length 12 cm moves with its ends always touching the coordinate axes. Determine the equation of
locus of point P on the rod, which is 3 cm from the end in contact with the X-axis.
174 CBSE Term II Mathematics XI

Section B (5 Marks Each)


This section consists of 4 questions of Long Answer Type.
6. Prove that sin 3A sin 3 A + cos 3A cos 3 A = cos 3 2 A.
Or Prove that tan θ tan ( 60° − θ )tan ( 60° + θ ) = tan 3θ.
7. The longest side of a triangle is twice the shortest side and third side is 2 cm longer than the shortest side. If the
perimeter of the triangle is more than 166 cm, then find the minimum length of shortest side.
Or Solve the following system of inequalities graphically x ≥ 4, y ≥ 3.
8. Find the ratio in which the points join A (2 , 1 , 5) and B (3, 4, 3) is divided by the plane 2 x + 2 y − 2 z = 1. Also, find
the coordinates of the point of division.
9. An urn contains 9 red, 7 white and 4 black balls. If two balls are drawn at random, find the probability that
(i) both the balls are red.
(ii) one is white and other is red.
(iii) the balls are of same colour.
Or A box contains 10 bulbs, of which just three are defective. If at random a sample of five bulbs is drawn, find the
probabilities that the sample contains
(i) exactly one defective bulb.
(ii) exactly two defective bulbs.
(iii) no defective bulbs.

Section C (1 Mark Each)


This section consists of 1 Case Based comprises of 5 MCQs.
10. On her vacation, Veena visits four cities. Delhi, Lucknow, Agra, Meerut. In a random order.

Meerut New Delhi

Agra Lucknow

(i) What is the probability that she visits Delhi before Lucknow.
1 1 1 1
(a) (b) (c) (d)
2 3 6 12
(ii) What is the probability she visit Delhi before Lucknow and Lucknow before Agra.
1 1 1 1
(a) (b) (c) (d)
2 4 6 12
(iii) What is the probability she visits Delhi first and Lucknow last.
1 1 1 1
(a) (b) (c) (d)
6 4 2 12
(iv) What is the probability she visits Delhi either first or second.
1 2 1 1
(a) (b) (c) (d)
3 3 2 6
(v) What is the probability she visits Delhi just before Lucknow.
1 1 2 1
(a) (b) (c) (d)
4 6 3 12
CBSE Term II Mathematics XI 175

Solutions
2
π  2 π Or
1. Here, cos 4 =  cos 
8  8 Given, cos θ = cos α cos β
2 β
 π 1 − tan 2
 1 + cos  cos θ 2 = cos θ
4 ⇒ cos β = ⇒
= [Qcos 2 x = 2 cos 2 x − 1 ] cos α 2β cos α
 2  1 + tan
2
 
β β
2 1 − tan 2 + 1 + tan 2
 1  2 2 cos θ + cos α
1 +   2 + 1
2 ⇒ =
 2  β β cos θ − cos α
= =  1 − tan 2 − 1 − tan 2
 2   2 2  2 2
 
  [applying componendo and dividendo]
2 +1+2 2 θ + α θ − α
= 2 cos   cos  
8 2  2   2 
⇒ =
3 2 β θ + α θ − α
= + − 2 tan 2 − 2 sin   sin  
8 4 2  2   2 
θ + α θ − α
2
3π  2 3π  1
cos 4 =  cos  ⇒ = cot   cot  
8  8 2β  2   2 
tan
2 2
 3π 
 1 + cos  2β θ + α θ − α
= 4  ⇒ tan = tan   tan   Hence proved.
2  2   2 
 2 
  2. If
a
> 0, then (a > 0 and b > 0) or (a < 0 and b < 0)
2 b
 π
 1 − cos   3π π ∴ (x + 6 > 0 and x − 4 > 0) or (x + 6 < 0 and x − 4 < 0)
= 4
Q cos 4 = − cos 4  Case I When x + 6 > 0 and x − 4 > 0 ⇒ x > −6 and x > 4
 2   
  ⇒x>4
 1 
2 Case II When x + 6 < 0 and x − 4 < 0 ⇒ x < −6 and x < 4 ⇒
1 −  2 x < −6
= 2  =  2 − 1
 2  ∴From both cases, the solution is x < −6 or x > 4
 2 2 
  ∴Solution set = {x ∈ R : x < −6 } ∪ {x ∈ R : x > 4}
 
2 + 1−2 2 3 2 = ( −∞ , − 6) ∪ ( 4, ∞ )
= = − The graph of the solution set is given below
8 8 4
5π  3π  3π











cos
8
= cos  π −
 8
 = − cos
8
– –6 –5 –4 –3 –2 –1 0 1 2 3 4

Or
4 5π 4 3π
∴ cos = cos Let x litres of 100% pure milk be added to 800 L of
8 8
adulterated milk for desired purity.
7π  π π
And cos = cos  π −  = − cos 90  80 100
8  8 8 Then, ( 800 + x ) × <  800 × +x× 
100  100 100
7 π π
∴ cos 4 = cos 4 95
8 8 < ( 800 + x ) ×
100
4 π 3π 5π 7π
Now, LHS = cos + cos 4 + cos4 + cos4 ⇒ ( 800 + x ) × 90 < ( 800 × 80 + 100x ) < ( 800 + x ) × 95
8 8 8 8
π 3π 3π 4 π
⇒ 72000 + 90x < 64000 + 100x < 76000 + 95 x
= cos 4 + cos 4 + cos4 + cos
8 8 8 8 ⇒ 72000 + 90x < 64000 + 100x
 π 3π  and 64000 + 100x < 76000 + 95x
= 2  cos 4 + cos 4 
 8 8 ⇒ 8000 < 10x and 5x < 12000
3 2 3 2  6 ⇒ x > 800 and x < 2400
=2  + + −  =2  
8 4 8 4  8 ∴ 800 < x < 2400
3 For desired purity of milk, the milkman is free to add any
= = RHS Hence proved. quantity from 800 L to 2400 L of 100% pure milk.
2
176 CBSE Term II Mathematics XI

Cr × r Cx
n
Or
3. Let LHS = n −x
Cr −x Let AB = 12 cm be the length of rod moves with its ends always
n! r! touching the coordinate axes as shown in the figure.
× Y
r !( n − r )! x ! ( r − x )!
=
( n − x )!
( n − x − r + x )! ( r − x )! B
n ! × ( n − r )! 9
= θ P(h, k)
( n − r )! × x ! × ( n − x )! N h
k θ 3
n! X′ X
= = n C x = RHS O M A
x ! ( n − x )!
Y′
Hence proved.
Let P( h , k ) be the point on the rod AB, such that P is at a
4. Total number of outcomes of throwing three dice distance of 3 cm from the end A on the X-axis and it makes an
simultaneously, n(S) = 6 × 6 × 6 = 63 = 216 angle θ.
(i) A total of atmost 5 can be obtained in anyone of the But AB = 12 cm
following ways
∴ PB = AB − PA = 12 − 3 = 9 cm
(1, 1, 1), (1, 1, 2), (1, 2, 1) (2, 1, 1), (1, 1, 3), (1, 3, 1),
(3, 1, 1), (1, 2, 2), (2, 1, 2), (2, 2, 1) Draw a perpendicular lines from P to the BO and OA which
meet at points N and M, respectively.
∴ Favourable outcomes, n( f2 ) = 10
n( f2 ) 10 5 Here, ∠BAO = ∠BPN = θ [corresponding angles]
Hence, required probability = = = PN h
n(S) 216 108 In ∆BNP, cosθ = ⇒ cosθ ...(i)
=
PB 9
(ii) Let A be the event ‘getting atleast 5’.
Similarly, in ∆AMP,
Then, P( A ) = 1 − P( A ) = 1 − P (getting a total of atmost 4) PM
A total of atmost 4 can be obtained in anyone of the sin θ =
PA
following ways (1, 1, 1), (1, 1, 2), (1, 2, 1), (2, 1, 1)
k
4 ⇒ sin θ = ...(ii)
∴ P( A ) = 3
216
On squaring Eqs. (i) and (ii) and then adding, we get
∴ P( A ) = 1 – P( A )
4 212 53 h 2 k2
=1− = = cos 2 θ + sin 2 θ = +
216 216 54 81 9
5. Let the required equation of the circle having centre ( h , k ) h 2 k2
⇒ 1= + ⇒ h 2 + 9k 2 = 81
be ( x − h )2 + ( y − k )2 = r 2. …(i) 81 9
Hence, the equation of a locus of a point P is x 2 + 9y 2 = 81.
Since, the circle passes through the points (4, 1) and
(6, 5), so we have 6. We know that,
( 4 − h ) 2 + (1 − k ) 2 = r 2 …(ii) sin 3A = 3 sin A – 4 sin 3 A
and ( 6 − h )2 + ( 5 − k )2 = r 2 ...(iii) 3 sin A – sin 3A
⇒ sin 3 A =
From Eqs. (ii) and (iii), we get 4
( 4 – h ) 2 + (1 − k ) 2 = ( 6 − h ) 2 + ( 5 − k ) 2 and cos 3A = 4 cos 3 A – 3 cos A
⇒ 16 + h 2 − 8 h + 1 + k 2 – 2 k = 36 + h 2 – 12 h + 25 3 cos A + cos 3A
⇒ cos 3 A =
4
+ k 2 − 10k
⇒ 4h + 8k = 44 Now, LHS = sin 3A sin 3 A + cos 3A cos 3 A
⇒ h + 2 k = 11 ...(iv)  3sin A – sin 3A 
= sin 3A  
Also, centre ( h , k ) lies on the line 4x + y = 16  4 
∴ 4 h + k = 16 ...(v)  3 cos A + cos 3A 
+ cos 3A  
On solving Eqs. (iv) and (v), we get  4 
h = 3 and k = 4 1
= [ 3sin A sin 3A + 3 cos A cos 3A – sin 2 3A + cos 2 3A ]
On substituting the values of h and k in Eq. (ii), we get 4
1
r 2 = ( 4 − 3 ) 2 + (1 − 4 ) 2 = [ 3 (cos A cos 3A + sin A sin 3A ) + (cos 2 3A – sin 2 3A )]
4
= 12 + (– 3)2 = 10 1
= [ 3 cos( 3A – A ) + cos 2 ( 3A )]
Now, substituting the values of h , k and r 2 in Eq. (i), we get 4
( x − 3)2 + ( y − 4)2 = 10 [ Q cos ( A – B ) = cos A cos B + sin A sin B ]
which is the required equation.
CBSE Term II Mathematics XI 177

=
1
[ 3 cos 2 A + cos 3 (2 A )] i.e. 0 ≥ 4 (which is false)
4 0 ≥ 3 (which is false)
1
= [ 3 cos 2 A + ( 4 cos 3 2 A – 3 cos 2 A )] Y
4
1
= ( 4 cos 3 2 A ) = cos 3 2 A = RHS Hence proved.
4 y=3
(0, 3) (4, 3)
Or
LHS = tan θ tan ( 60° − θ )tan ( 60° + θ )
X′ X
sin θ sin ( 60° − θ ) sin ( 60° + θ ) (4, 0)
= ⋅ ⋅
cos θ cos ( 60° − θ ) cos ( 60° + θ )
sin θ[2 sin ( 60° − θ )sin ( 60° + θ )] Y′
= x=4
cos θ [2 cos ( 60° − θ )cos ( 60° + θ)]
Both the graphs will be parallel and away from the origin.
sin θ (cos 2θ − cos 120° )
= Thus, common shaded region shows the solution of the
cos θ (cos 120° + cos 2θ )
inequalities.
Q 2 sin A sin B = cos ( A − B ) − cos ( A + B )  8. Suppose the plane 2 x + 2 y − 2 z = 1 divides the line joining
and 2 cos A cos B = cos ( A + B ) + cos ( A − B )
  the points A (2 , 1, 5) and B ( 3, 4, 3) at a point C in the ratio
 1 k : 1.
sin θ  cos 2θ +  Q cos 120° = cos(180° − 60° )
 2  Then, the coordinates of C are
= 1 
 1  = − cos 60° = −   3 k + 2 4 k + 1 3 k + 5
cos θ  cos 2θ −   2   , , . …(i)
 2  k+1 k+1 k+1
1 Since, point C lies on the plane 2 x + 2 y − 2 z = 1.
sin θ cos 2θ + sin θ
2 2 sin θ cos 2 θ + sin θ
= = ∴Coordinates of C must satisfy the equation of plane,
1
cos θ cos 2θ − cos θ 2 cos θ cos 2 θ − cos θ
 3k + 2  4 k + 1  3 k + 5
2 i.e. 2   +2  −2   =1
sin (θ + 2 θ ) + sin ( − θ ) + sin θ  k+1  k+1  k+1
=
cos (θ + 2 θ ) + cos ( − θ ) − cos θ 6 k + 4 + 8 k + 2 – 6 k − 10
⇒ =1
Q 2 sin A cos B = sin ( A + B ) + sin ( A − B )  k+1
and 2 cos A cos B = cos ( A + B ) + cos ( A − B )] ⇒ 8k − 4 = k + 1 ⇒ k = 5 / 7
 
5
sin 3θ − sin θ + sin θ Q sin ( −θ ) = − sin θ  So, the required ratio is : 1 or 5 : 7.
= and cos ( −θ ) = cos θ  7
cos 3θ + cos θ − cos θ   On substituting k = 5 / 7 in Eq. (i), we get
sin 3θ  5 5 5 
= = tan 3θ = RHS Hence proved.  3 × + 2 4 × + 1 3 × + 5
cos 3θ 7 7 7
Coordinates of C =  , , 
7. Let the length of shortest side be x cm.  5+1 5
+1
5
+1 
According to the question,  7 7 7 
Longest side = 2 × shortest side = 2x cm  29 9 25
= , , 
and third side = 2 + shortest side = (2 + x ) cm  12 4 6 
Perimeter of triangle = x + 2 x + ( x + 2 ) = 4x + 2 9. There are 20 balls in the bag out of which 2 balls can be
Since, perimeter > 166 cm drawn in 20 C 2 ways.
∴ 4x + 2 > 166 ⇒ 4x > 166 − 2
So, total number of outcomes = 20
C 2 = 190
164
⇒ 4x > 164 ⇒ x > = 41 cm (i) There are 9 red balls out of which 2 balls can be drawn
4
in 9 C 2 ways.
Hence, the minimum length of shortest side be 41 cm.
Or The given system of inequalities ∴ Favourable number of outcomes = 9 C 2 = 36
x≥4 ...(i) So, required probability =
36 18
=
and y≥3 ...(ii) 190 95
Step 1 Consider the inequations as strict equations (ii) Out of 7 white balls, one white ball can be drawn in 7 C1
i.e x = 4, y = 3 ways and out of 9 red balls, one red ball can be drawn
Step 2 Plot the graph of x = 4 will be a line parallel to Y-axis, in 9 C1 ways.
intersecting the X-axis at 4. The graph of y = 3 will be a line
∴ Favourable number of outcomes = 7 C1 × 9 C1 = 63
parallel to X-axis, intersecting the Y-axis at 3.
63
Step 3 Take a point (0, 0) and put it in the given inequations (i) So, required probability =
and (ii). 190
178 CBSE Term II Mathematics XI

(iii) Two balls are drawn of same colour mean that either
both are red or both are white or both are black, out of ABCD, ABDC, ACBD, ACDB, ADBC, ADCB ,
BACD, BADC, BCAD, BCDA, BDAC, BDCA , 
9 red balls two red balls are drawn in 9 C 2 ways.  
S= 
Similarly, two white balls can be drawn from 7 white  CABD, CADB, CBAD, CBDA, CDAB, CDBA, 
DABC, DACB, DCAB, DCBA, DBAC, DBCA 
balls, in 7 C 2 ways and two black balls drawn from
4 black balls in 4C 2 ways. (i) (a) Let E1 be the event that Veena visits A before B.
∴ Number of ways of drawing 2 balls of same colour Then,
ABCD, ABDC, ACBD, ACDB, ADBC, ADCB ,
= 9 C 2 + 7 C 2 + 4C 2 = 36 + 21 + 6 = 63 E1 =  
63 CABD, CADB, CDAB, DABC, DACB, DCAB 
So, required probability =
190 ⇒ n (E1 ) = 12
n (E1 ) 12 1
Or ∴ P(she visits A before B) = P (E1 ) = = =
Out of 10 bulbs 5 can be drawn in 10 C 5 ways. n (S) 24 2

So, total number of outcomes (ii) (c) Let E 2 be the event that she visits A before B and B
10! before C.
= 10 C 5 = = 252
5! 5! Then E 2 = {ABCD, ABDC, ADBC, DABC}
(i) There are 3 defective and 7 non-defective bulbs. The ⇒ n (E 2 ) = 4
number of ways of selecting one defective bulb out of 3 ∴ P (she visits A before B and B before C) = P (E 2 )
and 4 non-defective bulbs out of 7 is 3 C1 × 7 C 4 . n (E 2 ) 4 1
= = =
n (S) 24 6
∴ Favourable number of outcomes = C1 × C 4
3 7

3! 7! (iii) (d) Let E 3 be the event that she visits A first and B last.
= × = 3 × 35 = 105
1! 2 ! 3! 4! Then, E 3 = {ACDB, ADCB}
105 5 n (E 3 ) = 2
So, required probability = =
252 12 ∴ P (she visits A first and B last) = P (E 3 )
(ii) The number of ways of selecting 2 defective bulbs out n (E 3 ) 2 1
= = =
of 3 and 3 non-defective bulbs out of 7 is 3 C 2 × 7 C 3 . n (S) 24 12

∴ Favourable number of outcomes = 3C 2 × 7 C 3 (iv) (c) Let E 4 be the event that she visits A either first or
second. Then,
= 3 × 35 = 105 ABCD, ABDC, ACBD, ACDB, ADBC, ADCB ,
105 5 E4 =  
So, required probability = = BACD, BADC, CABD, CADB, DABC, DACB 
252 12
⇒ n (E 4 ) = 12
(iii) No defective bulb means all non-defective bulbs. Hence, P (she visits A either first or second)
The number of ways of selecting all 5 non-defective n (E 4 ) 12 1
bulbs out of 7 is 7 C 5. = P (E 4 ) = = =
n (S) 24 2
∴ Favourable number of outcomes = 7 C 5 = 21 (v) (a) Let E 5 be the event that she visits A just before B.
21 1
So, required probability = = Then,
252 12 E 5 = {ABCD, ABDC, CABD, CDAB, DABC, DCAB}
10. Let the Veena visits four cities Delhi, Lucknow, Agra, ⇒ n (E 5 ) = 6
Meerut A, B, C and D respectively Number of way’s in Hence, P (she visits A just before B)
which Veena can visit four cities A, B, C and D is 4! i.e. 24. n (E 5 ) 6 1
= P (E 5 ) = = =
∴ n(S) = 24 n (S) 24 4
Clearly, sample space for this experiment is
CBSE Term II Mathematics XI 179
Mathem atics
Class 11th ( Term II)

Practice Paper 2 *

(Unsolved)
Instructions Time : 2 Hr
Max. Marks : 40
1. The question paper contains three sections A, B and C.
2. Section A has 5 questions with 3 internal choices.
3. Section B has 4 questions with 3 internal choices.
4. Section C has 1 Case Based MCQs comprises of 5 MCQs.
5. There is no negative marking.
* As exact Blue-print and Pattern for CBSE Term II exams is not released yet. So the pattern of this
paper is designed by the author on the basis of trend of past CBSE Papers. Students are advised
not to consider the pattern of this paper as official, it is just for practice purpose.

Section A (3 Marks Each)


This section consists of 5 questions of Short Answer Type.
1 1 x
1. If + = , then find x.
3! 4! 6!
Or If 16 C r = 16C r + 2 , find r C 4 .
2 ab
2. If α and β are distinct roots of a cos θ + b sin θ = c, prove that sin (α + β) = .
a + b2
2

x 2 y2
3. Find the distance between the directrices of the ellipse + = 1.
36 20
Or Find the equation of the hyperbola, whose vertices are ( 0, ± 5) and foci ( 0, ± 8 ).
4. Find the equation of hyperbola whose foci are (0, ± 12) and the length of latusrectum is 36.
Or If y = 2 x is a chord of the circle x 2 + y 2 − 10x = 0, find the equation of the circle with this chord as a diameter.
5. A man wants to cut three lengths from a single piece of board of maximum length 91 cm. The second length
is to be 3 cm longer than the shortest and third length is to be twice as long as the shortest. What are the
possible lengths for the shortest board, if third piece is to be atleast 5 cm longer than the second? Also, show
the possible lengths on a graph paper in I quadrant.

Section B (5 Marks Each)


This section consists of 4 questions of Long Answer Type.
1
6. Prove that cos 20° cos 40° cos 60° cos 80° = .
16
180 CBSE Term II Mathematics XI

4 5 π
Or If cos (α + β) = , sin (α − β) = and α , β lie between 0 and , find tan 2α.
5 13 4
7. Differentiate xe x using first principle.
x
Or Differentiate using first principle.
sin x
|x| − 1
8. Solve the inequality ≥ 0, x ∈R and x ≠ ± 2.
|x| − 2
9. Find the equation of the circle passing through the points (1 , − 2 ) and (4, − 3) and whose centre lies on the line
3 x + 4 y = 7.
Or Prove that the centres of the three circles
x 2 + y 2 − 4x − 6y − 12 = 0, x 2 + y 2 + 2 x + 4y − 5 = 0 and x 2 + y 2 − 10x − 16y + 7 = 0 are collinear.

Section C (1 Mark Each)


This section consists of 1 Case Based comprises of 5 MCQs.
10. Five students Ajay, Shyam, Rahul, Ravi and Rashid are standing in a play ground in a line.

(i) Total number of ways of standing arrangement of five students is


(a) 120 (b) 60 (c) 24 (d) None of these
(ii) Total number of arrangement of standing, if Ajay and Ravi sit together is
(a) 60 (b) 48 (c) 72 (d) 120
(iii) Total number of arrangement Ravi and Rahul standing at extreme position is
(a) 24 (b) 36 (c) 48 (d) 12
(iv) Total number of arrangement, if shyam is standing in the middle is
(a) 24 (b) 12 (c) 6 (d) 36
(v) Total number of arrangement if Ravi and Rahul not stand together, is
(a) 72 (b) 120 (c) 60 (d) 144

Answers
y2 x2
1. 150 Or 35 3. 18 Or − =1 4. 3 y 2 − x 2 = 108 Or x 2 + y 2 − 2x − 4 y = 0 5. Required length ∈ [8, 22] in cm
25 39
2 2
sin x − x cos x
9.  x −
56 47   3 1465
6. Or 7. e x (x + 1) Or 8. Or (− ∞ , − 2) ∪ [−1, 1] ∪ (2, ∞ )  + y+  =
33 sin 2 x  15   5 225
10. (i) (a) (ii) (b) (iii) (d) (iv) (a) (v) (a)
CBSE Term II Mathematics XI 181
Mathem atics
Class 11th ( Term II)

Practice Paper 3 *

(Unsolved)
Instructions Time : 2 Hr
Max. Marks : 40
1. The question paper contains three sections A, B and C.
2. Section A has 5 questions with 3 internal choices.
3. Section B has 4 questions with 3 internal choices.
4. Section C has 1 Case Based MCQs comprises of 5 MCQs.
5. There is no negative marking.
* As exact Blue-print and Pattern for CBSE Term II exams is not released yet. So the pattern of this
paper is designed by the author on the basis of trend of past CBSE Papers. Students are advised
not to consider the pattern of this paper as official, it is just for practice purpose.

Section A (3 Marks Each)


This section consists of 5 questions of Short Answer Type.
− x −1
1. If tan α = (1 + 2 ) and tan β = (1 + 2 x +1 )−1 , then find α + β.
Or Prove that sin x + sin 3x + sin 5x + sin 7 x = 4 cos x cos 2 x sin 4x.
2. In how many ways one can select a cricket team of 11 players from 16 players, if
(i) there is no restriction (ii) one of them must be included
(iii) one of them, who is in bad form, must be excluded.
Or Out of 6 men and 3 women a committee of 5 persons is to formed so as to include atleast 2 women. In how
many ways can it be done?
3. What is the length of latusrectum of the ellipse 16x 2 + y 2 = 16?
Or Find the equation of the ellipse, if foci are ( ± 3, 0) and a = 4.
sin ( x + 5) dy
4. If y = , then find .
cos x dx
5. Show that the coordinates of the centroid of the triangle with vertices A(x 1 , y1 , z1 ), B(x 2 , y 2 , z 2 ) and
 x + x 2 + x 3 y1 + y 2 + y 3 z1 + z 2 + z 3 
C( x 3 , y 3 , z 3 ) is  1 , , .
 3 3 3 

Section B (5 Marks Each)


This section consists of 4 questions of Long Answer Type.
6. Solve the system of inequalities graphically x + y > 8; x − y < 0
182 CBSE Term II Mathematics XI

2x − 3 4x
Or Solve the system of linear inequalities 2(2 x + 3) − 10 < 6 ( x − 2 ), +6≥4+
4 3
7. If a cos 2θ + b sin 2θ = c has α and β as its roots, prove that
2b 2
tan α + tanβ = and tan α – tan β = b2 – c 2 + a2 .
a+c a+c
Or Prove that
 π  π 3
(i) 1 + cos 2 x + cos 4 x + cos 6 x = 4 cos x cos 2 x cos 3 x. (ii) cos 2 x + cos 2  x +  + cos 2  x −  = .
 3  3 2
8. Four persons are to be chosen at random from a group of 3 men, 2 women and
4 children. Find the probability of selecting
(i) 1 man, 1 woman and 2 children. (ii) exactly 2 children.
9. In how many different ways, the letters of the word ‘ALGEBRA’ can be arranged in a row, if
(i) the two A’s are together? (ii) the two A’s are not together?
Or Find the number of different words that can be formed from the letters of the word ‘INTERMEDIATE’ such
that two vowels never come together.

Section C (1 Mark Each)


This section consists of 1 Case Based comprises of 5 MCQs.
10. Four friends Sourabh, Shiva, Mayank, and Rahul playing cards, Daksh, shuffling a cards and told to Rahul
choose any four cards.

(i) What is the probability that Rahul getting all face card.
12 16
C4 C4 (12 C 2 )2
(a) 52
(b) 52
(c) 52
(d) None of these
C4 C4 C4
(ii) What is the probability that Rahul getting two red cards and two black card.
(13 C 2 )2 26
C4 ( 26 C 2 )2 ( 26 C 4 )2
(a) 52
(b) 52
(c) 52
(d) 52
C4 C4 C4 C4
(iii) What is the probability that Rahul getting one card from each suit.
13
C4 (13)4 (13 C 4 )2
(a) 52
(b) 52
(c) 52
(d) None of these
C4 C4 C4
(iv) What is the probability that Rahul getting all king cards.
1 2 4 6
(a) 52
(b) 52
(c) 52
(d) 52
C4 C4 C4 C4
(v) What is the probability that Rahul getting two king and two jack cards.
4
C2 36 6
(a) 52
(b) 52
(c) 52
(d) None of these
C4 C4 C4

Answers
π 1 x2 y2 2 10
1. 2. (i) 4368 ways (ii) 3003 ways (iii) 1365 ways Or 75 3. Or + =1 4. sec 2x cos 5 6. Or no solution 8. (i) (ii)
4 2 16 7 7 21
9. (i) 720 (ii) 1800 Or 151200 10. (i) (a) (ii) (c) (iii) (b) (iv) (a) (v) (b)
JEE NEET ADDA

This Pdf Is
Download From
www.jeeneetadda.in

Visit www.jeeneetadda.in for


More Premium Stuffs,Latest
Books,Test Papers,Lectures etc.
jeeneetadda
jeeneetadda_official
jeeneetadda

VISIT NOW !!

You might also like